Sunteți pe pagina 1din 338

University of San Jose-Recoletos

School of Law
LABOR LAW II
LABOR RELATIONS LAW
Weekday Class

COMPILATION OF DIGESTED CASES


 KINDS OF EMPLOYMENT
 POST-EMPLOYMENT
 TERMINATION

Presented To:
Atty. Mario Dennis Calvo
PHILIPPINE TRANSMARINE CARRIERS, INC v. FELICISIMO CARILLA,
G.R. NO. 157975 June 26, 2007

FACTS:

Respondent was hired by petitioner, a manning agent to work as Master on


board MV Handy-Cam Azobe for twelve months. Their approved POEA contract
provided that respondent would get a basic monthly pay, fixed monthly
overtime, master's allowance and leave with pay of six days per month.

On November 29, 1993, respondent boarded the vessel in Abidjan, Ivory Coast,
Africa. On June 6, 1994, while the vessel was in Bombay, India, respondent was
dismissed and repatriated to the Philippines.

Respondent filed with the Philippine Overseas and Employment Agency (POEA)
a complaint for illegal dismissal with claims for salaries and other benefits for the
unexpired portion of his contract as well as unremitted allotments and
damages. He alleged that: he was dismissed without notice and hearing and
without any valid reason.

Petitioner filed its Answer4 contending that: respondent's termination was for
cause; he failed to take the necessary steps to ensure the safety of the vessel
and its cargo while plying the waters of South Korea and Keelung port causing
petitioner to incur a huge amount of damages on cargo claims and vessel
repairs; respondent's incompetence is therefore penalized with dismissal; despite
the fact that respondent was warned of his lapses, he had not shown any
improvement which forced petitioner to dismiss and replace him with a
competent one.

The parties submitted their respective position papers. The case was
subsequently transferred to the Arbitration Branch of the NLRC pursuant to
Republic Act (RA) No. 8042, otherwise known as the "Migrant Workers and
Overseas Filipinos Act of 1995."5

The LA rendered a decision6 in favor of respondent. The LA found that


respondent's long experience as a seaman and his various recommendations
from his previous employers contradicted any finding of incompetence; that the
unauthenticated logbook extract submitted by petitioner lacked even an iota
of admissibility as the entries appearing therein had been merely copied from
the original logbook.

Aggrieved, petitioner filed its appeal with the NLRC who dismissed the appeal
finding that petitioner's evidence which consisted of a document dated June 1,
1994, entitled "Logs of Events During Respondent's Command" and the Senior
Officer Evaluation Reports, did not prove anything as these documents, besides
being unauthenticated, were self-serving and unreliable.Petitioner's MR was
subsequently denied.

Petitioner filed with the CA a Petition for Certiorari under Rule 65.The CA denied
the petition for lack of merit.Petitioner's MRwas .Hence, the instant Petition for
Review on Certiorari on the following grounds:

ISSUE:

(1) The CA committed a mistake of law when it upheld the ruling of the NLRC
that the documentary evidence presented by petitioner herein are "self-serving
and unreliable."

(2) It was not in accord with law and jurisprudence that respondent herein was
illegally dismissed and was thus entitled to the monetary value of the unserved
portion of his employment contract including pay for unserved overtime and
pay for unearned leave credits.

RULING: No. We find no merit in the petition.

To begin with, the question of whether respondent was dismissed for just cause is
a question of fact which is beyond the province of a Petition for Review
on Certiorari .The LA and the NLRC have ruled on the factual issues, and these
were affirmed by the CA. Thus, they are accorded not only great respect but
also finality, and are deemed binding upon us so long as they are supported by
substantial evidence.

In termination cases, the burden of proof rests upon the employer to show that
the dismissal of the employee is for just cause and failure to do so would mean
that the dismissal is not justified. A dismissed employee is not required to prove
his innocence of the charges leveled against him by his employer. The
determination of the existence and sufficiency of a just cause must be exercised
with fairness and in good faith and after observing due process.

Respondent was dismissed because of his alleged incompetence. To prove


respondent's incompetence while on board the vessel, petitioner presented a
piece of paper dated June 1, 1994 entitled "Logs of Events During CaptCarilla
(sic) Command," enumerating therein the alleged incidents where damages to
timber products and on the vessel occurred; and the Senior Officer Evaluation
Reports showing respondent's unsatisfactory performance, prepared by Chief
Officer R. Miu and Chief Engineer N.K. Jaggi, who allegedly had served with
respondent and had seen his work on board the vessel.
The court agree with the LA, NLRC and the CA in their finding that petitioner's
documents were not authenticated and, hence, were self-serving and
unreliable. It appears from the "Logs of Events During Capt. Carilla Command"
that it is merely a typewritten enumeration of several alleged incidents of
damages to the cargoes and to the vessel, but it does not state the source and
who prepared the same. While petitioner claims that it was prepared by the
vessel's technical superintendent, he was not identified at all. The log of events
did not also provide a detailed account of respondent's act of incompetence
which caused those alleged incidents. There is no way of verifying the truth of
these entries, and if they were actually recorded in the vessel logbook on the
dates the alleged incidents took place.

In Wallem Maritime Services, Inc. v. National Labor Relations Commission,21 we


rejected a typewritten collation of excerpts from what could be the logbook
and found that what should have been submitted as evidence was the logbook
itself or even authenticated copies of pertinent pages thereof.

In Abacast Shipping and Management Agency, Inc. v. National Labor Relations


Commission,22 we held:The log book is a respectable record that can be relied
upon to authenticate the charges filed and the procedure taken against the
employees prior to their dismissal.

It would have been a simple matter, considering the ease of reproducing the
same, to make photocopies of the pertinent pages of the log book to
substantiate the petitioner's contention. Why this was not done is something that
reasonably arouses the curiosity of this Court and suggests that there probably
were no entries in the log book at all that could have proved the alleged
offenses of the private respondents."

Petitioner's arguments are that respondent, being the person responsible for
accomplishing the vessel logbook by writing entries on the day-to-day events on
board, could not be expected to reflect any derogatory reports about his own
performance. Assuming the vessel logbook kept by respondent did not reflect
the different untoward incidents that occurred in the vessel, petitioner should
have presented other evidence to substantiate these incidents.

Petitioner's log of events purports to show that the timber products on the vessel
were damaged, and that the vessel was towed to a port for repair. It was also
alleged in petitioner's pleadings that it had incurred huge amounts for damages
on cargo claims. However, petitioner failed to present these cargo claims from
the shipper/consignees, and petitioner's payment thereof as well as its expenses
for the cost of the repair of the vessel.
Moreover, the two sets of Senior Officer Evaluation Reports allegedly prepared
by the officers next in rank to respondent did not help to justify respondent's
dismissal for incompetency. While the reports showed that respondent was
given an unsatisfactory performance rating and a recommendation for his
replacement, they failed to show the exact designations of the persons who
prepared the same, and neither do their signatures appear over the typewritten
names. In fact, these alleged officers did not even execute an affidavit to attest
to the truth of those reports. Thus, we agree with the LA and the NLRC that these
documents, being unauthenticated, have no probative value.

Respondent was terminated without having been given the opportunity to


defend himself. He was summarily dismissed and repatriated to the Philippines
without being informed of the charges against him; nor was he given the
chance to refute the charges.

WHEREFORE, the petition is PARTLY GRANTED. The Decision of the CA


is AFFIRMED with MODIFICATION that the monthly overtime as well as leave pay
included in the computation of the unexpired portion of the contract should be
deducted from the awarded amount.

ROYALE HOMES MARKETING CORPORATION, vs. FIDEL P. ALCANTARA


[deceased], substituted by his heirs,

Not every form of control that a hiring party imposes on the hired party is
indicative of employeeemployer relationship. Rules and regulations that merely
serve as guidelines towards the achievement of a mutually desired result without
dictating the means and methods of accomplishing it do not establish
employer-employee relationship.

FACTS:

Royale Homes, a corporation engaged in marketing real estates, appointed


Alcantara as its Marketing Director for a fixed period of one year. His work
consisted mainly of marketing Royale Homes’ real estate inventories on an
exclusive basis. Royale Homes reappointed him for several consecutive years.

Alcantara filed a Complaint for Illegal Dismissal against Royale. Alcantara


alleged that he is a regular employee of Royale Homes since he is performing
tasks that are necessary and desirable to its business and that the acts of the
executive officers of Royale Homes amounted to his dismissal from work without
any valid or just cause and in gross disregard of the proper procedure for
dismissing employees.
Royale Homes, on the other hand, vehemently denied that Alcantara is its
employee. It argued that the appointment paper of Alcantara is clear that it
engaged his services as an independent sales contractor for a fixed term of one
year only. He never received any salary, 13th month pay, overtime pay or
holiday pay from Royale Homes as he was paid purely on commission basis. In
addition, Royale Homes had no control on how Alcantara would accomplish his
tasks and responsibilities as he was free to solicit sales at any time and by any
manner which he may deem appropriate and necessary. According to Royale
Homes, Alcantara decided to leave the company after his wife, who was once
connected with it as a sales agent, had formed a brokerage company that
directly competed with its business, and even recruited some of its sales agents.
Two months after he relinquished his post, however, Alcantara appeared in
Royale Homes and submitted a letter claiming that he was illegally dismissed.
The Labor Arbiter rendered a Decision holding that Alcantara is an employee of
Royale Homes and that the pre-termination of his contract was against the law.
The NLRC rendered its Decision, ruling that Alcantara is not an employee but a
mere independent contractor of Royale Homes. It based its ruling mainly on his
employment contract. The CA promulgated its Decision granting Alcantara’s
Petition and reversing the NLRC’s Decision. Applying the four-fold and economic
reality tests, it held that Alcantara is an employee of Royale Homes.

ISSUE:

Whether or not Alcantara was an employee of Royale Homes.

RULING: NO.

The primary evidence of the nature of the parties’ relationship in this case is the
written contract that they signed and executed in pursuance of their mutual
agreement. While the existence of employer-employee relationship is a matter
of law, the characterization made by the parties in their contract as to the
nature of their juridical relationship cannot be simply ignored, particularly in this
case where the parties’ written contract unequivocally states their intention at
the time they entered into it. In this case, the contract, duly signed and not
disputed by the parties, conspicuously provides that "no employer-employee
relationship exists between" Royale Homes and Alcantara, as well as his sales
agents. It is clear that they did not want to be bound by employer-employee
relationship at the time of the signing of the contract.

In determining the existence of an employer-employee relationship, this Court


has generally relied on the four-fold test, to wit: (1) the selection and
engagement of the employee; (2) the payment of wages; (3) the power of
dismissal; and (4) the employer’s power to control the employee with respect to
the means and methods by which the work is to be accomplished.

However, not every form of control is indicative of employer-employee


relationship. A person who performs work for another and is subjected to its rules,
regulations, and code of ethics does not necessarily become an employee. As
long as the level of control does not interfere with the means and methods of
accomplishing the assigned tasks, the rules imposed by the hiring party on the
hired party do not amount to the labor law concept of control that is indicative
of employer-employee relationship. In Insular Life Assurance Co., Ltd. v. National
Labor Relations Commission it was pronounced that:

Logically, the line should be drawn between rules that merely serve as guidelines
towards the achievement of the mutually desired result without dictating the
means or methods to be employed in attaining it, and those that control or fix
the methodology and bind or restrict the party hired to the use of such means.
The first, which aim only to promote the result, create no employer-employee
relationship unlike the second, which address both the result and the means
used to achieve it.

Notably, Alcantara was not required to observe definite working hours. Except
for soliciting sales, Royale Homes did not assign other tasks to him. He had full
control over the means and methods of accomplishing his tasks as he can
"solicit sales at any time and by any manner which he may deem appropriate
and necessary." He performed his tasks on his own account free from the control
and direction of Royale Homes in all matters connected therewith, except as to
the results thereof. This Court is, therefore, convinced that Alcantara is not an
employee of Royale Homes, but a mere independent contractor.

Payment of Wages
The element of payment of wages is also absent in thiscase. As provided in the
contract, Alcantara’s remunerations consist only of commission override of 0.5%,
budget allocation, sales incentive and other forms of company support. There is
no proof that he received fixed monthly salary. No payslip or payroll was ever
presented and there is no proof that Royale Homes deducted from his supposed
salary withholding tax or that it registered him with the Social Security System,
Philippine Health Insurance Corporation, or Pag-Ibig Fund. In fact, his Complaint
merely states a ballpark figure of his alleged salary of P100,000.00, more or less.
All of these indicate an independent contractual relationship. Besides, if
Alcantara indeed consideredhimself an employee of Royale Homes, then he,
an experienced and professional broker, would have complained that he was
being denied statutorily mandated benefits. But for nine consecutive years, he
kept mum about it, signifying that he has agreed, consented, and accepted
the fact that he is not entitled tothose employee benefits because he is an
independent contractor.

The Court is, therefore,convinced that Alcantara is not an employee of Royale


Homes, but a mereindependent contractor.The NLRC is, therefore, correct in
concluding that the Labor Arbiter has no jurisdiction over the case and that the
same is cognizable by the regular courts.

WHEREFORE, the instant Petition is hereby GRANTED.

LEGEND HOTEL (MANILA), owned by TITANIUM CORPORATION, and/or, NELSON


NAPUD, in his capacity as the President of Petitioner Corporation, v. HERNANI S.
REALUYO, also known as JOEY ROA,

The law affords protection to an employee, and does not countenance any
attempt to subvert its spirit and intent. Any stipulation in writing can be ignored
when the employer utilizes the stipulation to deprive the employee of his security
of tenure. The inequality that characterizes employer-employee relationship
generally tips the scales in favor of the employer, such that the employee is
often scarcely provided real and better options.

FACTS:

This labor case for illegal dismissal involves a pianist employed to perform in the
restaurant of a hotel.

Realuyo, whose stage name was Joey R. Roa, filed a complaint for alleged
unfair labor practice, constructive illegal dismissal, and the
underpayment/nonpayment of his premium pay for holidays, separation pay,
service incentive leave pay, and 13 th month pay. He prayed for attorney’s fees
and damages. Roa averred that he had worked as a pianist at the Legend
Hotel’s Tanglaw Restaurant from September 1992 with an initial rate of
P400.00/night; and that it had increased to P750.00/night. During his
employment, he could not choose the time of performance, which had been
fixed from 7:00PM to 10:00pm for three to six times a week. On July 9, 1999, the
management had notified him that as a cost-cutting measure, his services as a
pianist would no longer be required effective July 30, 1999.
In its defense, petitioner denied the existence of an employer-employee
relationship with Roa, insisting that he had been only a talent engaged to
provide live music at Legend Hotel’s Madison Coffee Shop for three hours/day
on two days each week; and stated that the economic crisis that had hit the
country constrained management to dispense with his services.

The LA dismissed the complaint for lack of merit upon finding that the parties
had no employer- employee relationship, because Roa was receiving talent fee
and not salary, which was reinforced by the fact that Roa received his talent
fee nightly, unlike the regular employees of the hotel who are paid monthly.
NLRC affirmed the LA’s decision. However, CA set aside the decision of the
NLRC, saying CA failed to take into consideration that in petitioner s line of work,
he was supervised and controlled by respondent s restaurant manager who at
certain times would require him to perform only tagalog songs or music, or wear
barong tagalog to conform with Filipiniana motif of the place and the time of his
performance is fixed by the respondents from 7:00 pm to 10:00 pm, three to six
times a week. Petitioner could not choose the time of his performance.

ISSUE: (a) whether or not respondent was an employee of petitioner; and (b) if
respondent was petitioner s employee, whether he was validly terminated.

RULING:

YES. Employer-employee relationship existed between the parties.

Roa was undeniably employed as a pianist of the restaurant. The hotel wielded
the power of selection at the time it entered into the service contract dated
Sept. 1, 1992 with Roa. The hotel could not seek refuge behind the service
contract entered into with Roa. It is the law that defines and governs an
employment relationship, whose terms are not restricted to those fixed in the
written contract, for other factors, like the nature of the work the employee has
been called upon to perform, are also considered. The law affords protection to
an employee, and does not countenance any attempt to subvert its spirit and
intent. Any stipulation in writing can be ignored when the employer utilizes the
stipulation to deprive the employee of his security of tenure. The inequality that
characterizes employer-employee relationship generally tips the scales in favor
of the employer, such that the employee is often scarcely provided real and
better options.

The argument that Roa was receiving talent fee and not salary is baseless. There
is no denying that the remuneration denominated as talent fees was fixed on
the basis of his talent, skill, and the quality of music he played during the hours of
his performance. Roa’s remuneration, albeit denominated as talent fees, was
still considered as included in the term wage in the sense and context of the
Labor Code, regardless of how petitioner chose to designate the remuneration,
as per Article 97(f) of the Labor Code.

The power of the employer to control the work of the employee is considered
the most significant determinant of the existence of an employer-employee
relationship. This is the so-called control test, and is premised on whether the
person for whom the services are performed reserves the right to control both
the end achieved and the manner and means used to achieve that end.

A review of the records shows, however, that respondent performed his work as
a pianist under petitioner’s supervision and control. Specifically, petitioner s
control of both the end achieved and the manner and means used to achieve
that end was demonstrated by the following, to wit:lιbrαrÿ

A. He could not choose the time of his performance, which petitioners had fixed
from 7:00 pm to 10:00 pm, three to six times a week;

b. He could not choose the place of his performance;

c. The restaurant’s manager required him at certain times to perform only


Tagalog songs or music, or to wear barong Tagalog to conform to the Filipiniana
motif; andcralawlibrary

d. He was subjected to the rules on employees representation check and chits,


a privilege granted to other employees.

Lastly, petitioner claims that it had no power to dismiss respondent due to his not
being even subject to its Code of Discipline, and that the power to terminate
the working relationship was mutually vested in the parties, in that either party
might terminate at will, with or without cause. This claim is contrary to the
records. Indeed, the memorandum informing respondent of the
discountinuance of his service because of the financial condition of petitioner
showed the latter had the power to dismiss him from employment.

Whereby, we deny the petition.


ROLANDO Y. TAN, vs. LEOVIGILDO LAGRAMA and THE HONORABLE COURT OF
APPEALS,

The Bureau of Working Conditions classifies workers paid by results into two
groups, namely; (1) those whose time and performance is supervised by the
employer, and (2) those whose time and performance is unsupervised by the
employer. The first involves an element of control and supervision over the
manner the work is to be performed, while the second does not. If a piece
worker is supervised, there is an employer-employee relationship. However, such
an employee is not entitled to service incentive leave pay since, as pointed out
in Makati Haberdashery v. NLRC andMark RocheInternational v. NLRC, he is paid
a fixed amount for work done, regardless of the time he spent in accomplishing
such work.

FACTS:

Private respondent Lagrama is a painter, making ad billboards and murals for


the motion pictures shown at the Empress, Supreme, and Crown Theaters for
more than 10 years which is managed by Rolando Tan. He was dismissed for
urinating inside the drawing area. Lagrama denied the charge against him. He
claimed that he was not the only one who entered the drawing area and that,
even if the charge was true, it was a minor infraction to warrant his dismissal.

Lagrama filed a complaint with the Sub-Regional Arbitration Branch No. X of the
National Labor Relations Commission (NLRC) in Butuan City. He alleged that he
had been illegally dismissed and sought reinvestigation and payment of 13th
month pay, service incentive leave pay, salary differential, and damages.
Petitioner Tan denied that Lagrama was his employee. He asserted that
Lagrama was an independent contractor who did his work according to his
methods, while he (petitioner) was only interested in the result thereof. He cited
the admission of Lagrama during the conferences before the Labor Arbiter that
he was paid on a fixed piece-work basis, i.e., that he was paid for every painting
turned out as ad billboard or mural for the pictures shown in the three theaters,
on the basis of a no mural/billboard drawn, no pay policy. He submitted the
affidavits of other cinema owners, an amusement park owner, and those
supervising the construction of a church to prove that the services of Lagrama
were contracted by them. He denied having dismissed Lagrama and alleged
that it was the latter who refused to paint for him after he was scolded for his
habits. The LA awarded a total of 136,849.99 in benefits and damages.

Petitioner Rolando Tan appealed to the NLRC finding Lagrama to be an


independent contractor, and for this reason reversing the decision of the Labor
Arbiter. Lagrama filed a motion for reconsideration which was denied. He then
filed a petition for certiorari under Rule 65 before the Court of Appeals. The CA
reinstated the decision of the LA.

ISSUE:

Whether or not an employer-employee relationship existed, the private


respondent being a piece worker.

RULING: YES.

In determining whether there is an employer-employee relationship, we have


applied a four-fold test, to wit: (1) whether the alleged employer has the power
of selection and engagement of employees; (2) whether he has control of the
employee with respect to the means and methods by which work is to be
accomplished; (3) whether he has the power to dismiss; and (4) whether the
employee was paid wages. These elements of the employer-employee
relationship are present in this case.

The existence in this case of the first element is undisputed. It was petitioner who
engaged the services of Lagrama without the intervention of a third party. Of
the four elements of the employer-employee relationship, the control test is the
most important. Compared to an employee, an independent contractor is one
who carries on a distinct and independent business and undertakes to perform
the job, work, or service on its own account and under its own responsibility
according to its own manner and method, free from the control and direction of
the principal in all matters connected with the performance of the work except
as to the results thereof. Hence, while an independent contractor enjoys
independence and freedom from the control and supervision of his principal, an
employee is subject to the employers power to control the means and methods
by which the employees work is to be performed and accomplished.

In the case at bar, albeit petitioner Tan claims that private respondent Lagrama
was an independent contractor and never his employee, the evidence shows
that the latter performed his work as painter under the supervision and control of
petitioner. Lagrama worked in a designated work area inside the Crown Theater
of petitioner, for the use of which petitioner prescribed rules. The rules included
the observance of cleanliness and hygiene and a prohibition against urinating in
the work area and any place other than the toilet or the rest rooms. 9 Petitioner's
control over Lagrama's work extended not only to the use of the work area, but
also to the result of Lagrama's work, and the manner and means by which the
work was to be accomplished.
Moreover, it would appear that petitioner not only provided the workplace, but
supplied as well the materials used for the paintings, because he admitted that
he paid Lagrama only for the latter's services.10

Private respondent Lagrama claimed that he worked daily, from 8 o'clock in the
morning to 5 o'clock in the afternoon. Petitioner disputed this allegation and
maintained that he paid Lagrama P1,475.00 per week for the murals for the
three theaters which the latter usually finished in 3 to 4 days in one week. 11 Even
assuming this to be true, the fact that Lagrama worked for at least 3 to 4 days a
week proves regularity in his employment by petitioner.

Second. That petitioner’s right to hire and fire was admitted by him in his
position paper submitted to the NLRC. By stating that he had the right to fire
Lagrama, petitioner in effect acknowledged Lagrama to be his employee. For
the right to hire and fire is another important element of the employer-employee
relationship

Third.Payment of wages is one of the four factors to be considered in


determining the existence of employer-employee relation. That Lagrama
worked for Tan on a fixed piece-work basis is of no moment. Payment by result is
a method of compensation and does not define the essence of the relation. It is
a method of computing compensation, not a basis for determining the
existence or absence of employer-employee relationship. One may be paid on
the basis of results or time expended on the work, and may or may not acquire
an employment status, depending on whether the elements of an employer-
employee relationship are present or not.

WHEREFORE, based on the foregoing, the petition is DENIED.

GMA NETWORK, INC., Petitioner, vs. CARLOS P. PABRIGA, GEOFFREY F. ARIAS,


KIRBY N. CAMPO, ARNOLD L. LAGAHIT, and ARMANDO A. CATUBIG, Respondents.

FACTS: Private respondents were engaged by petitioner for the latters


operations in the Technical Operations Center as Transmitter/VTR men, as
Maintenance staff and as Cameramen On July 19 1999 due to the miserable
working conditions private respondents were forced to file a complaint against
petitioner before the NLRC Regional Arbitration Branch No. VII Cebu City.
Private respondents filed an amended complaint raising the following additional
issues of 1) Unfair Labor Practice; 2) Illegal dismissal; and 3) Damages and
Attorneys fees.
An amicable settlement between the parties was set but the same proved to
be futile.

The Labor Arbiter dismissed the complaint of respondents for illegal dismissal and
unfair labor practice, but held petitioner liable for 13th month pay.

The NLRC reversed the Decision of the Labor Arbiter, and held that

a) All complainants are regular employees with respect to the particular activity
to which they were assigned, until it ceased to exist. As such, they are entitled to
payment of separation pay computed at one (1) month salary for every year of
service;
b) They are not entitled to overtime pay and holiday pay; and
c) They are entitled to 13th month pay, night shift differential and service
incentive leave pay.

When Petitioner elevated the case to the CA via a Petition for Certiorari, it
rendered its Decision denying the petition for lack of merit. Hence, this present
Petition for Review on Certiorari.

ISSUES:
[1] Did the CA err in finding the respondents as regular employees of the
petitioner?

[2] Did the CA err in awarding separation pay to the respondents absent a
finding that respondents were illegally dismissed?

HELD: Respondents claim that they are regular employees of petitioner GMA
Network, Inc. The latter, on the other hand, interchangeably characterize
respondents employment as project and fixed period/fixed term employment.

ARTICLE 280. Regular and casual employment. The provisions of written


agreement to the contrary notwithstanding and regardless of the oral
agreement of the parties, an employment shall be deemed to be regular where
the employee has been engaged to perform activities which are usually
necessary or desirable in the usual business or trade of the employer, except
where the employment has been fixed for a specific project or undertaking the
completion or termination of which has been determined at the time of the
engagement of the employee or where the work or services to be performed is
seasonal in nature and employment is for the duration of the season.
An employment shall be deemed to be casual if it is not covered by the
preceding paragraph: Provided, That, any employee who has rendered at least
one year of service, whether such service is continuous or broken, shall be
considered a regular employee with respect to the activity in which he is
employed and his employment shall continue while such activity actually exist.

Pursuant to the above-quoted Article 280 of the Labor Code, employees


performing activities which are usually necessary or desirable in the employers
usual business or trade can either be regular, project or seasonal employees,
while, as a general rule, those performing activities not usually necessary or
desirable in the employers usual business or trade are casual employees. The
consequence of the distinction is found in Article 279 of the Labor Code, which
provides:

ARTICLE 279. Security of tenure. In cases of regular employment, the employer


shall not terminate the services of an employee except for a just cause or when
authorized by this Title. An employee who is unjustly dismissed from work shall be
entitled to reinstatement without loss of seniority rights and other privileges and
to his full backwages, inclusive of allowances, and to his other benefits or their
monetary equivalent computed from the time his compensation was withheld
from him up to the time of his actual reinstatement.

On the other hand, the activities of project employees may or may not be
usually necessary or desirable in the usual business or trade of the employer.

The term "project" could also refer to, secondly, a particular job or undertaking
that is not within the regular business of the corporation. Such a job or
undertaking must also be identifiably separate and distinct from the ordinary or
regular business operations of the employer. The job or undertaking also begins
and ends at determined or determinable times. ALU-TUCP v. National Labor
Relations Commission, G.R. No. 109902, August 2, 1994

The jobs and undertakings are clearly within the regular or usual business of the
employer company and are not identifiably distinct or separate from the other
undertakings of the company. There is no denying that the manning of the
operations center to air commercials, acting as transmitter/VTR men,
maintaining the equipment, and acting as cameramen are not undertakings
separate or distinct from the business of a broadcasting company.

In sum, we affirm the findings of the NLRC and the Court of Appeals that
respondents are regular employees of petitioner. As regular employees, they are
entitled to security of tenure and therefore their services may be terminated only
for just or authorized causes. Since petitioner failed to prove any just or
authorized cause for their termination, we are constrained to affirm the findings
of the NLRC and the Court of Appeals that they were illegally dismissed.

Since the respondents were illegally dismissed, they entitled to separation pay in
lieu of reinstatement.

As regards night shift differential, the Labor Code provides that every employee
shall be paid not less than ten percent (10%) of his regular wage for each hour
of work performed between ten o'clock in the evening and six o'clock in the
morning.

As employees of petitioner, respondents are entitled to the payment of this


benefit in accordance with the number of hours they worked from 10:00 p.m. to
6:00 a.m., if any.

The matter of attorney's fees cannot be touched once and only in the fallo of
the decision, else, the award should be thrown out for being speculative and
conjectural. In the absence of a stipulation, attorney's fees are ordinarily not
recoverable; otherwise a premium shall be placed on the right to litigate. They
are not awarded every time a party wins a suit.

In the case at bar, the factual basis for the award of attorney's fees was not
discussed in the text of NLRC Decision. Thus, the Court constrained to delete the
same.

Mylene Carvajal vs. Luzon Development Bank and/or Oscar Ramirez

FACTS:
Carvajal was employed as a trainee-teller by Luzon Development Bank (Bank)
under a six-month probationary employment contract. Ramirez is the President
and CEO of the Bank. A month into her employment, she was send a
Memorandum directing her to explain in writing why she should not be
subjected to disciplinary action for her eight tardiness on November 2003. A
second Memorandum was sent to her on January for her again chronic
tardiness on December 2003. She submitted her written explanations for both
events and manifested her acceptance of the consequences of her actions.
She was terminated for three days effective 21 January 2004. However, on 22
January, her termination was lifted but at the same time, her services were
terminated. In the respondents’ position paper to the LA, they explained that
the reasons for her absence are chronic tardiness, absenteeism and failure to
perform satisfactorily as a probationary employee.
LA Decision: The petitioner was illegally dismissed because she was not afforded
the notice in writing informing her of what the Bank would like to bring out to her
for the latter to answer in writing.
NLRC Decision: NLRC affirmed the decision of the LA.
CA Decision: The CA found that the petitioner was not entitled to backwages
because she was rightfully dismissed for failure to meet the employment
standards.

ISSUE:
Whether the petitioner can be considered a regular employee at the time of
her dismissal.

HELD:
No. Carvajal’s appointment letter reads that “Possible extension of this contract
will depend on the job requirements of the Bank and your overall performance.
Performance review will be conducted before possible renewal can take
effect.” Therefore, petitioner knew, at the time of her engagement, that she
must comply with the standards set forth by respondent and perform
satisfactorily in order to attain regular status. Even the NLRC upheld the
petitoner’s probationary status, stating that reinstatement is not synonymous to
regularization.

Although probationary employees also enjoy security of tenure, he may still be


terminated because of just and authorized causes of termination and the
additional ground under Article 281 of the Labor Code, i.e. the probationary
employee may also be terminated for failure to qualify as a regular employee in
accordance to the reasonable standards set by the employer. Punctuality is a
reasonable standard imposed on every employee, whether in government or
private sector. This, together with absenteeism, underperformance and mistake
in clearing a check are infractions that cannot be tantamount to satisfactory
standards.
In addition to the abovementioned, it has been previously held in PDI vs.
Magtibay, Jr., that the second requirement under Article 281 does not require
notice and hearing. Due process of law for this second ground consists of
making the reasonable standards expected of the employee during his
probationary period known to him at the time of his engagement. By the very
nature of probationary employment, the employee knows from the very start
that he will be under close observation and continuous scrutiny by his
supervisors. If termination is for cause, it may be done at anytime during the
probation.
San Miguel Corporation vs. Caroline del Rosario
G.R. No. 168194

Facts: On April 17, 2000, respondent was employed by petitioner as key account
specialist. On March 9, 2001, petitioner informed respondent that her
probationary employment will be severed at the close of the business hours of
March 12, 2001. On March 13, 2001, respondent was refused entry to petitioner's
premises. On June 24, 2002, respondent filed a complaint against petitioner for
illegal dismissal and underpayment/non-payment of monetary benefits.
Respondent alleged that petitioner feigned an excess in manpower because
after her dismissal, it hired new recruits and re-employed two of her batch
mates. On the other hand, petitioner claimed that respondent was a
probationary employee whose services were terminated as a result of the
excess manpower that could no longer be accommodated by the company.

The Labor Arbiter declared respondent a regular employee because her


employment exceeded six months and holding that she was illegally dismissed
as there was no authorized cause to terminate her employment. On appeal to
NLRC, it modified the previous decision.

Issue: Whether or not the respondent was an employee and was illegally
terminated. If so, is she entitled to monetary benefits?

Ruling: In termination cases, the burden of proving the circumstances that


would justify the employee's dismissal rests with the employer. The best proof that
petitioner should have presented to prove the probationary status of
respondent is her employment contract. None, having been presented, the
continuous employment of respondent as an account specialist for almost 11
months, from April 17, 2000 to March 12, 2001, means that she was a regular
employee and not a temporary reliever or a probationary employee. And while
it is true that by way of exception, the period of probationary employment may
exceed six months when the parties so agree, such as when the same is
established by company policy, or when it is required by the nature of the work,
none of these exceptional circumstance were proven in the present case. Thus,
respondent whose employment exceeded six months is undoubtedly a regular
employee of petitioner.

Her termination from employment must be for a just or authorized cause,


otherwise, her dismissal would be illegal. Petitioner tried to justify the dismissal of
respondent under the authorized cause of redundancy. It thus argued in the
alternative that even assuming that respondent qualified for regular
employment, her services still had to be terminated because there are no more
regular positions in the company. Undoubtedly, petitioner is invoking a
redundancy which allegedly resulted in the termination not only of the trainees,
probationers but also of some of its regular employees.

Redundancy, for purposes of the Labor Code, exists where the services of an
employee are in excess of what is reasonably demanded by the actual
requirements of the enterprise. Succinctly put, a position is redundant where it is
superfluous, and superfluity of a position or positions may be the outcome of a
number of factors, such as overhiring of workers, decreased volume of business,
or dropping of a particular product line or service activity previously
manufactured or undertaken by the enterprise. The criteria in implementing a
redundancy are: (a) less preferred status, e.g. temporary employee; (b)
efficiency; and (c) seniority. What further militated against the alleged
redundancy advanced by petitioner is their failure to refute respondent's
assertion that after her dismissal, it hired new recruits and re-employed two of
her batch mates. The Court finds that petitioner was not able to discharge the
burden of proving that the dismissal of respondent was valid.

Considering that respondent was illegally dismissed, she is entitled not only to
reinstatement but also to payment of full back wages, computed from the time
her compensation was actually withheld from her on March 13, 2001, up to her
actual reinstatement. She is likewise entitled to other benefits, i.e., service
incentive leave pay and 13th month pay computed from such date also up to
her actual reinstatement. Respondent is not entitled to holiday pay because the
records reveal that she is a monthly paid regular employee. Under Section 2,
Rule IV, Book III of the Omnibus Rules Implementing the Labor Code, employees
who are uniformly paid by the month, irrespective of the number of working
days therein, shall be presumed to be paid for all the days in the month whether
worked or not.
MACARTHUR MALICDEM AND HERMENIGILDO FLORES,Petitioners, v. MARULAS
INDUSTRIAL CORPORATION AND MIKE MANCILLA,Respondents.

FACTS:

Petitioners Malicdem and Flores were hired by respondent corporation as


extruder operators in 2006 They were responsible for the bagging of filament
yarn, the quality of pp yarn package and the cleanliness of the work place
area. Their employment contracts were for a period of one (1) year. Every year
thereafter, they would sign a Resignation/Quitclaim in favor of Marulas a day
after their contracts ended, and then sign another contract for one (1) year until
such time that they were told not to report to work anymore. They were asked to
sign a paper acknowledging the completion of their contractual status.
Claiming that they were illegally dismissed, the corporation countered that their
contracts showed that they were fixedterm employees for a specific
undertaking which was to work on a particular order of a customer for a specific
period. Their severance from employment then was due to the expiration of
their contracts.

ISSUE: Whether or not petitioners were illegally dismissed

HELD: Yes. CA affirming NLRC decision annulled and set aside

Labor Law: Effect of continuous re-hiring of a project employee for the same
tasks that are vital, necessary and indispensable to the usual trade or business of
the employer

Once a project or work pool employee has been: (1) continuously, as opposed
to intermittently, rehired by the same employer for the same tasks or nature of
tasks; and (2) these tasks are vital, necessary and indispensable to the usual
business or trade of the employer, then the employee must be deemed a
regular employee, pursuant to Article 280 of the Labor Code and jurisprudence.
To rule otherwise would allow circumvention of labor laws in industries not falling
within the ambit of Policy Instruction No. 20/Department Order No. 19, hence
allowing the prevention of acquisition of tenurial security by project or work pool
employees who have already gained the status of regular employees by the
employers conduct.

The test to determine whether employment is regular or not is the reasonable


connection between the particular activity performed by the employee in
relation to the usual business or trade of the employer. If the employee has been
performing the job for at least one year, even if the performance is not
continuous or merely intermittent, the law deems the repeated and continuing
need for its performance as sufficient evidence of the necessity, if not
indispensability of that activity to the business.

It is clear then that there was deliberate intent on the part of the employer to
prevent the regularization of petitioners. To begin with, there is no actual project.
The only stipulations in the contracts were the dates of their effectivity, the duties
and responsibilities of the petitioners as extruder operators, the rights and
obligations of the parties, and the petitioners compensation and allowances. As
there was no specific project or undertaking to speak of, the respondents
cannot invoke the exception in Article 280 of the Labor Code.This is a clear
attempt to frustrate the regularization of the petitioners and to circumvent the
law.

Even granting that petitioners were project employees, they can still be
considered as regular as they were continuously hired by the same employer for
the same position as extruder operators. Being responsible for the operation of
machines that produced sacks, their work was vital and indispensable the
business of the employer. The respondents cannot use the alleged expiration of
the employment contracts of the petitioners as a shield of their illegal acts. The
project employment contracts that the petitioners were made to sign every
year since the start of their employment were only a stratagem to violate their
security of tenure in the company.

The respondents invocation ofWilliam Uy Construction Corp. v. Trinidadis


misplaced because it is applicable only in cases involving the tenure of project
employees in the construction industry. It is widely known that in the construction
industry, a project employees work depends on the availability of projects,
necessarily the duration of his employment. It is not permanent but coterminous
with the work to which he is assigned.It would be extremely burdensome for the
employer, who depends on the availability of projects, to carry him as a
permanent employee and pay him wages even if there are no projects for him
to work on.The rationale behind this is that once the project is completed it
would be unjust to require the employer to maintain these employees in their
payroll.

Under Article 279 of the Labor Code, an employee who is unjustly dismissed from
work shall be entitled to reinstatement without loss of seniority rights and other
privileges and to his full backwages, inclusive of allowances, and to his other
benefits or their monetary equivalent computed from the time his compensation
was withheld from him up to the time of his actual reinstatemen
FVR Skills v. Seva

Facts:
Seva and 27 others were employed by FVR Skills, an independent
contractor engaged in the business of providing janitorial and other manpower
services to clients. As early as 1998 some of the respondents had already been
under FVR’s employ.
FVR entered into a contract of janitorial service with Robinsons Land Corp.
for a period of one year. Pursuant to this, respondents were deployed to
Robinsons. Halfway through the service contract, petitioner asked them to
execute individual contracts which stipulated that their respective employments
shall end on December 31, 2008, unless earlier terminated.
FVR and Robinsons no longer extended their contract of Janitorial
services. Petitioner dismissed the respondents as they were project employees
whose duration of employment was dependent on the petitioner’s service
contract with Robinsons.
They filed a complaint for illegal dismissal arguing that they were not
project but regular employees who may only be dismissed for just or authorized
causes.

Issue:
WON respondents are regular employees of the petitioner.

Ruling:
Yes. The primary standard in determining regular employment is the
reasonable connections between the particular activity performed by the
employee and the employer’s business or trade. This connection can be
ascertained by considering the nature of the work performed and its relation to
the scheme of the particular business, or the trade in its entirety.
In the case at bar, respondents had been working for FVR as early as
1998. Even before the service contract with Robinsons, they were already under
FVR’s employ. They had been doing the same type of work and occupying the
same positions from the time they were hired until they were dismissed. FVR did
not present any evidence to refute their claim that from that time of their hiring
until dismissal, there was no gap in between the projects where they were
assigned to.
Respondents’ work as janitors, service crews and sanitation aides, are
necessary or desirable to the petitioner’s business of providing janitorial and
manpower services to its clients as an independent contractor.
Under DO 18-02, the applicable labor issuance to the petitioner’s case,
the contractor or subcontractor is considered as the employer of the
contractual employee for purposes of enforcing the provisions of the LC and
other social legislation.
Although respondents were assigned as contractual employees to the
petitioner’s various clients, under the law, they remain to be the petitioner’s
employees, who are entitled to all the rights and benefits of regular
employment.

OMNI HAULING SERVICES Inc. vs BON

FACTS:

Petitioner Omni Hauling Services, Inc. (Omni), was awarded a one (1) year
service contract by the local government of Quezon City to provide garbage
hauling services. For this purpose, Omni hired respondents as garbage truck
drivers and paleros who were then paid on a per trip basis.

When the service contract was renewed for another year, Omni required each
of the respondents to sign employment contracts which provided that they will
be "re-hired" only for the duration of the same period. However, respondents
refused to sign the employment contracts, claiming that they were regular
employees since they were engaged to perform activities which were
necessary and desirable to Omni’s usual business or trade. Omni terminated
their employment.

Case for illegal dismissal, non-payment of ECOLA, 13th month plus damages

LA: Respondents were not illegally dismissed. They were informed that their
employment will be limited for a specific period of one year and was co-
terminus with the service contract with the Quezon City government.
Respondents were not regular but merely project employees whose hiring was
solely dependent on the aforesaid service contract.

NLRC: Affirmed the LA decision.

CA: Reversed. NLRC failed to consider the glaring fact that no contract of
employment exists to support petitioners’ allegation that respondents are fixed-
term (or properly speaking, project) employees.

ISSUE: Are they regular or project based employees?

HELD: Regular Employees.

Art. 280. Regular and casual employment. The provisions of written agreement
to the contrary notwithstanding and regardless of the oral agreement of the
parties, an employment shall be deemed to be regular where the employee
has been engaged to perform activities which are usually necessary or desirable
in the usual business or trade of the employer, except where the employment
has been fixed for a specific project or undertaking the completion or
termination of which has been determined at the time of the engagement of
the employee or where the work or service to be performed is seasonal in
nature and the employment is for the duration of the season. A project
employee is assigned to a project which begins and ends at determined or
determinable times. Unlike regular employees who may only be dismissed for just
and/or authorized causes under the Labor Code, the services of employees
who are hired as "project employees" may be lawfully terminated at the
completion of the project.

Principal test for determining whether particular employees are properly


characterized as "project employees" as distinguished from "regular employees,"
is whether or not the employees were assigned to carry out a "specific project or
undertaking," the duration (and scope) of which were specified at the time they
were engaged for that project.
The project could either be (1) a particular job or undertaking that is within the
regular or usual business of the employer company, but which is distinct and
separate, and identifiable as such, from the other undertakings of the company;
or (2) a particular job or undertaking that is not within the regular business of the
corporation.

Employers claiming that their workers are project employees should not only
prove that the duration and scope of the employment was specified at the time
they were engaged, but also that there was indeed a project. Even though the
absence of a written contract does not by itself grant regular status to
respondents, such a contract is evidence that respondents were informed of the
duration and scope of their work and their status as project employees.

In the case at bar, the logical conclusion is that respondents were not clearly
and knowingly informed of their employment status as mere project employees,
with the duration and scope of the project specified at the time they were
engaged. As such, the presumption of regular employment should be accorded
in their favor pursuant to Article 280 of the Labor Code which provides that
"[employees] who have rendered at least one year of service, whether such
service is continuous or broken [– as respondents in this case –] shall be
considered as [regular employees] with respect to the activity in which [they]
are employed and [their] employment shall continue while such activity actually
exists." Add to this the obvious fact that respondents have been engaged to
perform activities which are usually necessary or desirable in the usual business
or trade of Omni, i.e., garbage hauling, thereby confirming the strength of the
aforesaid conclusion.
Roy Asos V PNCC

FACTS:

Petitioner Roy D. Pasos started working for respondent petitioner was designated
as "Clerk II (Accounting)" and was assigned to the "NAIA – II Project." It was
likewise stated therein:

According to the contract he maybe terminated at anytime for cause as


provided for by law and/or existing Company Policy.

Petitioner’s employment, however, did not end on July 25, 1996 but was
extended until August 4, 1998, or more than two years . Based on PNCC’s
"Appointment for Project Employment" petitioner was rehired as "Accounting
Clerk (Reliever)" and assigned to the "PCSO – Q.I. Project." It was stated therein
that his employment shall end on February 11, 1999 and may be terminated for
cause or in accordance with the provisions of Article 282 of the Labor Code, as
amended. However, said employment did not actually end on February 11,
1999 but was extended until February 19, 1999 based on the "Personnel Action
Form-Project Employment" dated February 17, 1999.8

On February 23, 1999, petitioner was again hired by PNCC as "Accounting Clerk"
and was assigned to the "SM-Project" based on the "Appointment for Project
Employment 9 It did not specify the date when his employment will end but it
was stated therein that it will be "co-terminus with the completion of the project."
Said employment supposedly ended on August 19, 1999 per "Personnel Action
Form – Project Employment" However, it appears that said employment was
extended per "Appointment for Project employment" dated August 20, 1999 11 as
petitioner was again appointed as "Accounting Clerk" for "SM Project (Package
II)." It did not state a specific date up to when his extended employment will be,
but it provided that it will be "co-terminus with the x xx project." In "Personnel
Action Form – Project Employment" dated October 17, 2000,12 it appears that
such extension would eventually end on October 19, 2000.

Despite the termination of his employment on October 19, 2000, petitioner


claims that his superior instructed him to report for work the following day,
intimating to him that he will again be employed for the succeeding SM
projects. For purposes of reemployment, he then underwent a medical
examination which allegedly revealed that he had pneumonitis. Petitioner was
advised by PNCC’s physician, Dr. Arthur C. Obena, to take a 14-day sick leave.

On November 27, 2000, after serving his sick leave, petitioner claims that he was
again referred for medical examination where it was revealed that he
contracted Koch’s disease. He was then required to take a 60-day leave of
absence.13 The following day, he submitted his application for sick leave but
PNCC’s Project Personnel Officer, Mr. R.S. Sanchez, told him that he was not
entitled to sick leave because he was not a regular employee.

Petitioner still served a 60-day sick leave and underwent another medical
examination on February 16, 2001. He was then given a clean bill of health and
was given a medical clearance by Dr. Obena that he was fit to work.

Petitioner claims that after he presented his medical clearance to the Project
Personnel Officer on even date, he was informed that his services were already
terminated on October 19, 2000 and he was already replaced due to expiration
of his contract. This prompted petitioner on February 18, 2003 to file a
complaint14 for illegal dismissal against PNCC he argued that he is deemed a
regular employee of PNCC due to his prolonged employment as a project
employee as well as the failure on the part of PNCC to report his termination
every time a project is completed.

PNCC countered that petitioner was hired as a project employee in several


projects with specific dates of engagement and termination and had full
knowledge and consent that his appointment was only for the duration of each
project. It further contended that it had sufficiently complied with the reportorial
requirements to the Department of Labor and Employment (DOLE). The
submission of termination reports by PNCC was however disputed by petitioner
based on the verifications18 issued by the DOLE NCR office that he was not
among the affected employees listed in the reports filed by PNCC

The Labor Arbiter ruled that petitioner attained regular employment status with
the repeated hiring and rehiring of his services more so when the services he
was made to render were usual and necessary to PNCC’s business. The Labor
Arbiter likewise found that from the time petitioner was hired in 1996 until he was
terminated, he was hired and rehired by PNCC and made to work not only in
the project he had signed to work on but on other projects as well, indicating
that he is in fact a regular employee. He also noted petitioner’s subsequent
contracts did not anymore indicate the date of completion of the contract and
the fact that his first contract was extended way beyond the supposed
completion date.

The NLRC reversed and set aside.

ISSUE: Whether petitioner is a regular employee and not a mere project


employee and thus can be dismissed for cause?
Rulling:

Duration of project employment should be determined at the time of hiring

In the instant case, the appointments issued to petitioner indicated that he was
hired for specific projects. This Court is convinced however that although he
started as a project employee, he eventually became a regular employee of
PNCC.

Under Article 280 of the Labor Code, as amended, a project employee is one
whose "employment has been fixed for a specific project or undertaking the
completion or termination of which has been determined at the time of the
engagement of the employee or where the work or services to be performed is
seasonal in nature and the employment is for the duration of the season." Thus,
the principal test used to determine whether employees are project employees
is whether or not the employees were assigned to carry out a specific project or
undertaking, the duration or scope of which was specified at the time the
employees were engaged for that project.33

In the case at bar, petitioner worked continuously for more than two years after
the supposed three-month duration of his project employment for the NAIA II
Project. While his appointment for said project allowed such extension since it
specifically provided that in case his "services are still needed beyond the
validity of the contract, the Company shall extend his services," there was no
subsequent contract or appointment that specified a particular duration for the
extension. His services were just extended indefinitely until "Personnel Action
Form – Project Employment" dated July 7, 1998 was issued to him which
provided that his employment will end a few weeks later or on August 4, 1998.
While for first three months, petitioner can be considered a project employee of
PNCC, his employment thereafter, when his services were extended without any
specification of as to the duration, made him a regular employee of PNCC. And
his status as a regular employee was not affected by the fact that he was
assigned to several other projects and there were intervals in between said
projects since he enjoys security of tenure.

Failure of an employer to file termination reports after every project completion


proves that an employee is not a project employee

In this case, records clearly show that PNCC did not report the termination of
petitioner’s supposed project employment for the NAIA II Project to the DOLE.
Department Order No. 19, or the "Guidelines Governing the Employment of
Workers in the Construction Industry," requires employers to submit a report of an
employee’s termination to the nearest public employment office every time an
employee’s employment is terminated due to a completion of a project. PNCC
submitted as evidence of its compliance with the requirement supposed
photocopies of its termination reports, each listing petitioner as among the
employees affected. Unfortunately, none of the reports submitted pertain to the
NAIA II Project.

Policy Instruction No. 20 is explicit that employers of project employees are


exempted from the clearance requirement but not from the submission of
termination report. We have consistently held that failure of the employer to file
termination reports after every project completion proves that the employees
are not project employees.

Universal Robina Milling Corporation vs. Acibo

Facts:
 Petitioner is a domestic corporation engaged in the sugar cane milling
business; Cabati is petitioner’s Business Unit General Manager
 Complainants were employees of Petitioner
o Hired on various dates as drivers, crane operators, bucket hookers,
welders, mechanics, laboratory attendants and aides, steel workers,
laborers, carpenters, masons.
o At the start of their engagements, the complainants signed
contracts of employment for a period of 1 month or for a given
season
o Petitioner repeatedly hired the complainants to perform the same
duties and for every engagement, required the latter to sign new
employment contracts for the same duration of 1 month or a given
season
 Complainants filed before the LA complaints for regularization,
entitlement to the benefits under the existing CBA and attorney’s fees.
 LA: Dismissed the case holding that they were seasonal or project workers,
not regular employees. They could not be regularized since their
respective employments were coterminous with the phase of the work or
special project, ending upon completion. As such, they were not entitled
to benefits under the CBA which covered only regular employees.
 NLRC: Reversed LA’s decision. Complainants are regular employees.
complainants performed activities which were usually necessary and
desirable in the usual trade or business of petitioner, and had been
repeatedly hired for the same undertaking every season.
 In this regard, the CA held that the various activities that the complainants
were tasked to do were necessary, if not indispensable, to the nature of
URSUMCO’s business. As the complainants had been performing their
respective tasks for at least one year, the CA held that this repeated and
continuing need for the complainants’ performance of these same tasks,
regardless of whether the performance was continuous or intermittent,
constitutes sufficient evidence of the necessity, if not indispensability, of
the activity to URSUMCO’s business.

Issue:
Whether the complainants are regular employees? No. Complainants are
regular seasonal workers.

Held:
o Regular employment refers to that arrangement where the employee has
been engaged to perform activities necessary or desirable in the usual
trade or business of the employer. Primary standard that determines
regular employment is the reasonable connection between the particular
activity performed by the employee and the usual trade or business of the
employer, emphasizing on the necessity or desirability of the employee’s
activity.
o Casual employment is when the engagement lasts at least one year,
regardless of continuity. The controlling test in this arrangement is the
length of time during which the employee is engaged.
o Project employment is an arrangement where the employment has been
fixed for a specific project or undertaking whose completion or
termination has been determined at the time of the engagement of the
employee. The services of the project employees are legally and
automatically terminated upon the end or completion of the project as
the employee’s services are coterminous with the project.
o Two requirements are necessary to defeat the presumption of regularity of
employment:
o Designation of a specific project or undertaking for which the
employee is hired; and,
o Clear determination of the completion or termination of the project
at the time of the employee’s engagement.
o Seasonal employment is similar to project employment, lasting for the
duration of the season. To exclude seasonal employees from being
classified as regular employees, the employer must show:
o The employee performed work or services seasonal in nature; and,
o He had been employed for the duration of the season.
o When seasonal employees are continuously and repeatedly hired to
perform the same tasks for several seasons or even after the cessation of
the season, this length of time may serve as a badge of regular
employment. If these workers classified as seasonal are called to work
from time to time and merely temporarily laid off during the off-season,
they are not considered as separated from service, but simply on leave
until re-employed. The indispensability or desirability of the activity
performed by the employee will not preclude the parties from entering
into a valid fixed term employment agreement.
o Here, the employees were made to perform various tasks that did not
pertain to any specific phase of the milling operations that would cease
upon completion of a particular phase in the milling of the sugar. They
performed duties regularly and habitually needed by the company during
the milling season. Loader operators, hookers, crane operators and drivers
hauled and transported sugarcane from the plantation to the mill; lab
attendants, workers and laborers milled the sugar; and welders,
carpenters and utility workers ensured smooth and continuous operation
of the mill for the season.
o They were also regularly and repeatedly hired for the same tasks year
after year. Such regular and repeated hiring of the same workers for two
separate seasons in two different sets, set a system of regular seasonal
employment in the sugar industry and others with a similar nature of
operations. Plantation workers or mill employees did not work continuously
for a whole year, but only for the duration of the growing of the
sugarcane, or for the milling season. The Court has previously settled that
seasonal workers, called to work from time to time and temporarily laid off
during the off-season are not separated from service, but considered on
leave until re-employment. They are regular seasonal employees. As such,
they cannot be lumped together with the regular employees such as the
administrative or office personnel performing their tasks the whole year,
regardless of season due to difference in the nature of their duties and the
duration of their work vis-à-vis the company’s operations.
JAIME N. GAPAYAO vs. ROSARIO FULO, SOCIAL SECURITY SYSTEM and SOCIAL
SECURITY COMMISSION, G.R. No. 193493, June 13, 2013

FACTS:

Jaime Fulo died of "acute renal failure secondary to 1st degree burn 70%
secondary electrocution" while doing repairs at the residence and business
establishment of Gapayao. Gapayao extended some financial assistance to
Rosario Fulo, the wife of the deceased and the latter executed an Affidavit of
Desistance stating that she was not holding them liable for the death of her
late husband.

Thereafter, private respondent filed a claim for social security benefits with the
Social Security System (SSS)Sorosogon Branch. However, upon verification and
evaluation, it was discovered that the deceased was not a registered member
of the SSS.

Upon Rosario's insistence that her late husband had been employed by
petitioner from January 1983 up to his untimely death on 4 November 1997,
the SSS conducted a field investigation to clarify his status of employment.

The findings revealed that Mr. Jaime Fulo was an employee of Jaime
Gapayao as farm laborer from 1983 to 1997 and that Mr. Jaime Fulo receives
compensation on a daily basis ranging from P5.00 to P60.00 from 1983 to 1997.
As per interview, Mrs. Estela Gapayao contends that Jaime Fulo is an
employee of Mr. & Mrs. Jaime Gapayao on an extra basis.

the SSS demanded that petitioner remit the social security contributions of the
deceased. Gapayao denied that the deceased was his employee but was
rather an independent contractor whose tasks were not subject to his control
and supervision. Assuming arguendo that the deceased was his employee, he
was still not entitled to be paid his SSS premiums for the intervening period
when he was not at work, as he was an "intermittent worker who was only
summoned every now and then as the need arose." Hence, Gapayao insisted
that he was under no obligation to report the formers demise to the SSS for
social security coverage.

Rosario alleges that her late husband had been in the employ of petitioner for
14 years, from 1983 to 1997. During that period, he was made to work as a
laborer in the agricultural landholdings, a harvester in the abaca plantation,
and a re pairman/utility worker in several business establishments owned by
petitioner. The considerable length of time during which [the deceased] was
given diverse tasks by Gapayao was a clear indication of the necessity and
indispensability of her late husband’s services to Gapayao's business.

ISSUE:

Whether or not there exists between the deceased Jaime Fulo and Gapayao
an employer- employee relationship that would merit an award of benefits in
favor of Rosario Fulo under social security laws.

RULING:

Yes. Farm workers may be considered regular seasonal employees. Farm


workers generally fall under the definition of seasonal employees. Court held
that seasonal employees may be considered as regular employees. Regular
seasonal employees are those called to work from time to time. The nature of
their relationship with the employer is such that during the off season, they are
temporarily laid off; but reemployed during the summer season or when their
services may be
needed. They are in regular employment because of the nature of their job, and
not because of the length of time they have worked. The other tasks allegedly
done by the deceased outside his usual farm work only bolster the existence of
an employer-employee relationship. It only proves that even during the off
season, the deceased was still in the employ of Gapayao. The most telling indicia
of this relationship is the Compromise Agreement executed by Gapayao and
Rosario. Gapayao entered into the agreement with full knowledge that he was
described as the employer of the deceased.

Pakyaw workers are considered employees for as long as their employers


exercise control over them. In this case, Gapayao wielded control over the
deceased in the discharge of his functions.

The right of an employee to be covered by the Social Security Act is premised on


the existence of an employer-employee relationship. That having been
established, the Court ruled in favor of Rosario.

Tan vs. Lagrama, G.R. No. 151228, August 15, 2002

Facts

Lagrama works for Tan as painter of billboards and murals for the motion
pictures shown at the theaters managed by Tan for more than 10years.
Lagrama was dismissed for having urinated in his working area. Lagrama filed a
complaint for illegal dismissal and non-payment of benefits. Tan asserted that
Lagrama was an independent contractor as he was paid in piece-work basis

Issue

W/N Lagrama is an independent contractor or an employee of Tan?

Ruling
Lagrama is an employee not an independent

contractor. Applying Four-Fold Test

A. Power of Control - Evidence shows that the Lagrama performed his work as
painter and under the supervision and control of Tan.

Lagrama worked in a designated work area inside the theater of Tan for the use
of which petitioner prescribed rules, which rules included the observance of
cleanliness and hygeine and prohibition against urinating in the work area and
any other place other than rest rooms. Tan's control over Lagrama's work
extended not only the use of work area but also the result of Lagrama;s work
and the manner and means by which the work was to be accomplished.
Lagrama is not an independent contractor because he did not enjoy
independence and freedom from the control and supervision of Tan and he
was subjected to Tan's control over the means and methods by which his work is
to be performed and accomplished

B. Payment of Wages

Lagrama worked for Tan on a fixed piece work basis is of no moment. Payment
by result is a method of compensation and does not define the essence of the
relation. Tan Lagrama was not reported as an employee to the SSS is not
conclusive, on the question whether he was an employee, otherwise Tan would
be rewarded for his failure or even neglect to perform his obligation.
C. Power of Dismissal – by Tan stating that he had the right to fire Lagrama, Tan
in effect acknowledged Lagrama to be his employee.

D. Power of Selection and Engagement of Employees – Tan engaged the


services of Lagrama without the intervention of third party.

FUJI TELEVISION NETWORK, INC. VS. ARLENE S. ESPIRITU,


G.R. NO. 204944-45

FACTS:

Arlene S. Espiritu (Arlene) was engaged by Fuji Television Network, Inc. (Fuji) as a
news correspondent/producer tasked to report Philippine news to Fuji through its
Manila Bureau field office. The employment contract was initially for one year,
but was successively renewed on a yearly basis with salary adjustments upon
every renewal.

In January 2009, Arlene was diagnosed with lung cancer. She informed Fuji
about her condition, and the Chief of News Agency of Fuji, Yoshiki Aoki,
informed the former that the company had a problem with renewing her
contract considering her condition. Arlene insisted she was still fit to work as
certified by her attending physician.

After a series of verbal and written communications, Arlene and Fuji signed a
non-renewal contract. In consideration thereof, Arlene acknowledged the
receipt of the total amount of her salary from March-May 2009, year-end bonus,
mid-year bonus and separation pay. However, Arlene executed the non-
renewal contract under protest.

Arlene filed a complaint for illegal dismissal with the NCR Arbitration Branch of
the NLRC, alleging that she was forced to sign the non-renewal contract after
Fuji came to know of her illness. She also alleged that Fuji withheld her salaries
and other benefits when she refused to sign, and that she was left with no other
recourse but to sign the non-renewal contract to get her salaries.

Labor Arbiter dismissed the complaint and held that Arlene was not a regular
employee but an independent contractor.

The NLRC reversed the Labor Arbiter’s decision and ruled that Arlene was a
regular employee since she continuously rendered services that were necessary
and desirable to Fuji’s business.

The Court of Appeals affirmed that NLRC ruling with modification that Fuji
immediately reinstate Arlene to her position without loss of seniority rights and that
she be paid her backwages and other emoluments withheld from her. The Court
of Appeals agreed with the NLRC that Arlene was a regular employee,
engaged to perform work that was necessary or desirable in the business of Fuji,
and the successive renewals of her fixed-term contract resulted in regular
employment. The case of Sonza does not apply in the case because Arlene was
not contracted on account of a special talent or skill. Arlene was illegally
dismissed because Fuji failed to comply with the requirements of substantive
and procedural due process. Arlene, in fact, signed the non-renewal contract
under protest as she was left without a choice.

Fuji filed a petition for review on certiorari under Rule 45 before the Supreme
Court, alleging that Arlene was hired as an independent contractor; that Fuji
had no control over her work; that the employment contracts were renewed
upon Arlene’s insistence; that there was no illegal dismissal because she freely
agreed not to renew her fixed-term contract as evidenced by her email
correspondences.
Arlene filed a manifestation stating that the SC could not take jurisdiction over
the case since Fuji failed to authorize Corazon Acerden, the assigned attorney-
in-fact for Fuji, to sign the verification.

ISSUES: Was Arlene an independent contractor? Was Arlene a regular employee?

RULING:

Arlene was not an independent contractor.

Fuji alleged that Arlene was an independent contractor citing the Sonza case.
She was hired because of her skills. Her salary was higher than the normal rate.
She had the power to bargain with her employer. Her contract was for a fixed
term. It also stated that Arlene was not forced to sign the non-renewal
agreement, considering that she sent an email with another version of her non-
renewal agreement.

Arlene argued (1) that she was a regular employee because Fuji had control and
supervision over her work; (2) that she based her work on instructions from Fuji;
(3) that the successive renewal of her contracts for four years indicated that her
work was necessary and desirable; (4) that the payment of separation pay
indicated that she was a regular employee; (5) that the Sonza case is not
applicable because she was a plain reporter for Fuji; (6) that her illness was not a
ground for her dismissal; (7) that she signed the non-renewal agreement
because she was not in a position to reject the same.

The level of protection to labor should vary from case to case. When a
prospective employee, on account of special skills or market forces, is in a
position to make demands upon the prospective employer, such prospective
employee needs less protection than the ordinary worker.

The level of protection to labor must be determined on the basis of the nature of
the work, qualifications of the employee, and other relevant circumstances such
as but not limited to educational attainment and other special qualifications.

Fuji’s argument that Arlene was an independent contractor under a fixed-term


contract is contradictory. Employees under fixed-term contracts cannot be
independent contractors because in fixed-term contracts, an employer-
employee relationship exists. The test in this kind of contract is not the necessity
and desirability of the employee’s activities, “but the day certain agreed upon
by the parties for the commencement and termination of the employment
relationship.” For regular employees, the necessity and desirability of their work in
the usual course of the employer’s business are the determining factors. On the
other hand, independent contractors do not have employer-employee
relationships with their principals.

To determine the status of employment, the existence of employer-employee


relationship must first be settled with the use of the four-fold test, especially the
qualifications for the power to control.

The distinction is in this guise:


Rules that merely serve as guidelines towards the achievement of a mutually
desired result without dictating the means or methods to be employed creates
no employer-employee relationship; whereas those that control or fix the
methodology and bind or restrict the party hired to the use of such means
creates the relationship.
In appliacation, Arlene was hired by Fuji as a news producer, but there was no
evidence that she was hired for her unique skills that would distinguish her from
ordinary employees. Her monthly salary appeared to be a substantial sum. Fuji
had the power to dismiss Arlene, as provided for in her employment contract.
The contract also indicated that Fuji had control over her work as she was
rquired to report for 8 hours from Monday to Friday. Fuji gave her instructions on
what to report and even her mode of transportation in carrying out her functions
was controlled.

Therefore, Arlene could not be an independent contractor.

Arlene was a regular employee with a fixed-term contract.

In determining whether an employment should be considered regular or non-


regular, the applicable test is the reasonable connection between the
particular activity performed by the employee in relation to the usual business or
trade of the employer. The standard, supplied by the law itself, is whether the
work undertaken is necessary or desirable in the usual business or trade of the
employer, a fact that can be assessed by looking into the nature of the services
rendered and its relation to the general scheme under which the business or
trade is pursued in the usual course. It is distinguished from a specific undertaking
that is divorced from the normal activities required in carrying on the particular
business or trade.

However, there may be a situation where an employee’s work is necessary but is


not always desirable in the usual course of business of the employer. In this
situation, there is no regular employment.

Fuji’s Manila Bureau Office is a small unit213 and has a few employees. Arlene had
to do all activities related to news gathering.

A news producer “plans and supervises newscast [and] works with reporters in
the field planning and gathering information, including monitoring and getting
news stories, rporting interviewing subjects in front of a video camera, submission
of news and current events reports pertaining to the Philippines, and traveling to
the regional office in Thailand.” She also had to report for work in Fuji’s office in
Manila from Mondays to Fridays, eight per day. She had no equipment and had
to use the facilities of Fuji to accomplish her tasks.

The successive renewals of her contract indicated the necessity and desirability
of her work in the usual course of Fuji’s business. Because of this, Arlene had
become a regular employee with the right to security of tenure.

Arlene’s contract indicating a fixed term did not automatically mean that she
could never be a regular employee. For as long as it was the employee who
requested, or bargained, that the contract have a “definite date of
termination,” or that the fixed-term contract be freely entered into by the
employer and the employee, then the validity of the fixed-term contract will be
upheld.

Alilin vs Petron, GR No. 177592,

FACTS:

Alilin, et al. are laborers hired by Romualdo D. Gindang Contractor and RDG to
work in the premises of Petron's bulk plant. Their dates of hiring range from 1968
to 1993. In 2000, Petron and RDG entered into a Contract of Services for the
period June 1, 2000 to May 31, 2002 whereby RDG undertook to provide Petron
with janitorial, maintenance, tanker receiving, packaging and other utility
services in its Mandaue Bulk Plant. This contract was extended on July 31, 2002
and further
extended until September 30, 2002. Upon expiration, no further extension was
made. Thus, on October 16, 2002, Alilin, et al. were barred from continuing their
services with Petron.

Hence, the filing of a complaint for illegal dismissal, etc. against Petron, claiming
to be the latter's regular employees. Petron, on the other hand, alleges that they
are employees of RDG, an independent contractor. It presented the following
pieces of evidence: (1) RDG's Certificate of Registration of Business Name issued
by DTI; (2) RDG's Certificate of Registration issued by DOLE;
(3) Contractor's Pre-Qualification Statement; (4) Conflict of Interest Statement
signed by Romeo Gindang as manager of RDG; (5) RDG's Audited Financial
Statements for the years 1998, 1999 and 2000; (6) RDG's Mayor's Permit for the
years 2000 and 2001; (7) RDG's Certificate of Accreditation issued by DTI; (8)
performance bond and insurance policy; (9) SSS Online Inquiry System Employee
Contributions and Employee Static Information; and (10) Romeo's affidavit
stating that he had paid the salaries of his employees assigned to Petron.

LA found against Petron and ruled that Alilin, et al. are its regular employees
because their jobs were directly related to Petron's business operations; they
worked under the supervision of Petron's foreman; they were using Petron's tools
and equipment in the performance of their works. NLRC affirmed the ruling.
However, CA reversed the ruling and found RDG to be a legitimate contractor.

ISSUE: Whether or not RDG is a legitimate contractor

HELD:

Petron failed to discharge the burden of proving that RDG is a legitimate


contractor. Hence, the presumption that RDG is a labor-only contractor stands.

The audited financial statements and other financial documents of RDG for the
years 1999 to 2001 establish that it does have sufficient working capital to meet
the requirements of its service contract. In fact, the financial evaluation
conducted by Petron of RDG's financial statements for years 1998-2000 showed
RDG to have a maximum financial capability of Php4.807 Million as of
December 1998, and PHp1.611 Million as of December 2000. Petron was able to
establish RDG's sufficient capitalization when it entered into the service contract
in 2000. The Court stresses though that this determination of RDG's status as an
independent contractor is only with respect to its financial capability for the
period covered by the financial and other documents presented. In other
words, the evidence adduced merely proves that RDG was financially qualified
as a legitimate contractor but only with respect to its last service contract with
Petron in the year 2000.

As may be recalled, petitioners have rendered work for Petron for a long period
of time even before the service contract was executed in 2000. The respective
dates on which petitioners claim to have started working for Petron, as well as the
fact that they have rendered continuous service to it until October 16, 2002,
when they were prevented from entering the premises of Petron's Mandaue Bulk
Plant, were not at all disputed by Petron. In fact, Petron even recognized that
some of the petitioners were initially fielded by Romualdo Gindang, the father of
Romeo, through RDG's precursor, Romualdo D. Gindang Contractor, while the
others were provided by Romeo himself when he took over the business of his
father in 1989. Hence, while Petron was able to establish that RDG was financially
capable as a legitimate contractor at the time of the execution of the service
contract in 2000, it nevertheless failed to establish the financial capability of RDG
at the time when petitioners actually started to work for Petron in 1968, 1979,
1981, 1987, 1990, 1992 and 1993.

Petron's power of control over petitioners exists in this case


The facts that petitioners were hired by Romeo or his father and that
their salaries were paid by them do not detract from the conclusion
that there exists an employer-employee relationship between the
parties due to Petron's power of control over petitioners. One
manifestation of the power of control is the power to transfer
employees from one work assignment to another. Here, Petron could
order petitioners to work outside of their regular "maintenance/utility"
job. Also, petitioners were required to report for work every day at the
bulk plant, observe an 8:00 a.m. to 5:00 p.m. daily work schedule, and
wear proper uniform and safety helmets as prescribed by the safety
and security measures being implemented within the bulk plant. All
these imply control. In an industry where safety is of paramount
concern, control and supervision over sensitive operations, such as
those performed by the petitioners, are inevitable if not at all necessary.
Indeed, Petron deals with commodities that are highly volatile and
flammable which, if mishandled or not properly attended to, may cause
serious injuries and damage to property and the environment. Naturally,
supervision by Petron is essential in every aspect of its product handling in
order not to compromise the integrity, quality and safety of the products
that it distributes to the consuming public.

Petitioners already attained regular status as employees of Petron

Petitioners were given various work assignments such as tanker


receiving, barge loading, sounding, gauging, warehousing, mixing,
painting, carpentry, driving, gasul filling and other utility works. Petron
refers to these work assignments as menial works which could be
performed by any able-bodied individual. The Court finds, however, that
while the jobs performed by petitioners may be menial and mechanical,
they are nevertheless necessary and related to Petron's business
operations. If not for these tasks, Petron's products will not reach the
consumers in their proper state. Indeed, petitioners' roles were vital
inasmuch as they involve the preparation of the products that Petron will
distribute to its consumers.

Furthermore, while it may be true that any able-bodied individual can


perform the tasks assigned to petitioners, the Court notes the
undisputed fact that for many years, it was the same able- bodied
individuals (petitioners) who performed the tasks for Petron. The
engagement of petitioners for the same works for a long period of time
is a strong indication that such works were indeed necessary to Petron's
business. In view of these, and considering further that petitioners' length
of service entitles them to become regular employees under the Labor
Code, petitioners are deemed by law to have already attained the
status as Petitioner's regular employees. As such, Petron could not
terminate their services on the pretext that the service contract it
entered with RDG has already lapsed.

First Philippine Industrial Corporation v. Calimbas


G.R. No. 179256

FACTS:

Private respondent First Philippine Industrial Corporation (FPIC) is a


domestic corporation primarily engaged in the transportation of
petroleum products by pipeline. Petitioners Raquel Calimbas and Luis
Mahilom were engaged by De Guzman Manpower Services (DGMS) to
perform secretarial and clerical jobs to FPIC. Petitioner Calimbas was
assigned as a department secretary at the Technical Services
Department, while petitioner Mahilom served as a clerk at the Money
Movement Section of the Finance Division.

On June 21, 2001, FPIC, through its Human Resources Manager, Lorna
Young, informed Calimbas and Mahilom that their services to the
company would no longer be needed by July 30, 2001. On August 3,
2001, Calimbas and Mahilom signed quitclaims, releasing and
discharging DGMS from whatever claims that they might have against it.

On August 16, 2001, Calimbas and Mahilom still filed a complaint against
FPIC for illegal dismissal. The Labor Arbiter rendered a decision holding
that respondents were regular employees of petitioner, and that they
were illegally dismissed. The NLRC upheld the decision of the LA. In a
resolution, NLRC reversed its decision. The CA reversed the decision of the
NLRC.

ISSUE: Whether or not respondents are regular employees?

HELD: Respondents are petitioners employees.

First, in Vinoya v. National Labor Relations Commission, categorically


stated that the actual paid-in capital of P75,000.00 could not be
considered as substantial capital. Thus, DGMSs actual paid-in capital in
the amount of P75,000.00 does not constitute substantial capital essential
to carry out its business as an independent job contractor. In spite of its
bare assertion that the Vinoya case does not apply in the present case,
DGMS has not shown any serious and cogent reason to disregard the
ruling in the aforementioned case. Records likewise reveal that DGMS has
no substantial equipment in the form of tools, equipment and machinery.
As a matter of fact, respondents were using office equipment and
materials owned by petitioner while they were rendering their services at
its offices.

Second, petitioner exercised the power of control and supervision over


the respondents. The fact that DGMS did not assign representatives to
supervise over respondents work in petitioners company tends to disprove
the independence of DGMS. It is axiomatic that the test to determine the
existence of independent contractorship is whether one claiming to be
an independent contractor has contracted to do the work according to
his own methods and without being subjected to the control of the
employer, except only to the results of the work. Obviously, on this score
alone, petitioner cannot rightly claim that DGMS was an independent job
contractor inasmuch as respondents were subjected to the control and
supervision of petitioner while they were performing their jobs.

Third, also worth stressing are the points highlighted by respondents: (1)
Respondents worked only at petitioners offices for an uninterrupted
period of five years, occupying the same position at the same
department under the supervision of company officials; (2) Three weeks
ahead of the termination letters issued by DGMS, petitioners HR Manager
Lorna Young notified respondents, in a closed-door meeting, that their
services to the company would be terminated by July 31, 2001; (3) In the
termination letters prepared by DGMS, it was even stressed that the said
termination letters will formalize the verbal notice given by petitioners HR
Administration personnel; (4) The direct superiors of respondents were
managerial employees of petitioner, and had direct control over all the
work-related activities of the latter. This control included the supervision of
respondents performance of their work and their compliance with
petitioners company policies and procedures. DGMS, on the other hand,
never maintained any representative at the petitioners office to oversee
the work of respondents.
An employer-employee relationship exists between petitioner and
respondents. And having served for almost five years at petitioners
company, respondents had already attained the status of regular
employees.
Aliviado vs. Procter and Gamble
G.R. No. 160506

Facts:
Petitioners worked as merchandisers of P&G from various dates and they
all individually signed employment contracts with either Promm-Gem or
SAPS for periods of more or less five months at a time. They were assigned
at different outlets, supermarkets and stores where they handled all the
products of P&G.They received their wages from Promm-Gem or SAPS.
SAPS and Promm-Gem imposed disciplinary measures on erring
merchandisers for reasons such as habitual absenteeism, dishonesty or
changing day-off without prior notice.

Petitioners filed a complaint against P&G for regularization, service


incentive leave pay and other benefits with damages.The complaint was
later amended to include the matter of their subsequent dismissal.

The Labor Arbiter dismissed the complaint for lack of merit and ruled that
there was no employer-employee relationship between petitioners and
P&G.He found that the selection and engagement of the petitioners, the
payment of their wages, the power of dismissal and control with respect
to the means and methods by which their work was accomplished, were
all done and exercised by Promm-Gem/SAPS. He further found that
Promm-Gem and SAPS were legitimate independent job contractors. The
NLRC and the CA both affirmed the ruling of the Labor Arbiter.

Issue: WON P&G is the employer of petitioners.

Held:

In order to resolve the issue of whether P&G is the employer of petitioners,


it is necessary to first determine whether Promm-Gem and SAPS are labor-
only contractors or legitimate job contractors.Clearly, the law and its
implementing rules allow contracting arrangements for the performance
of specific jobs, works or services. However, in order for such outsourcing
to be valid, it must be made to an independent contractor because the
current labor rules expressly prohibit labor-only contracting.
To emphasize, there is labor-only contracting when the contractor or sub-
contractor merely recruits, supplies or places workers to perform a job,
work or service for a principal and any of the following elements are
present:

- The contractor or subcontractor does not have substantial capital


or investment which relates to the job, work or service to be
performed andthe employees recruited, supplied or placed by
such contractor or subcontractor are performing activities which
are directly related to the main business of the principal; or
- The contractor does not exercise the right to control over the
performance of the work of the contractual

Under the circumstances, Promm-Gem cannot be considered as a labor-


only contractor. We find that it is a legitimate independent contractor.

Considering that SAPS has no substantial capital or investment and the


workers it recruited are performing activities which are directly related to
the principal business of P&G, we find that the former is engaged in
“labor-only contracting”.Where labor-only contracting exists, the Labor
Code itself establishes an employer-employee relationship between the
employer and the employees of the labor-only contractor. The statute
establishes this relationship for a comprehensive purpose: to prevent a
circumvention of labor laws. The contractor is considered merely an
agent of the principal employer and the latter is responsible to the
employees of the labor-only contractor as if such employees had been
directly employed by the principal employer.

Javier v. Fly Ace Corporation &Florelyn Castillo


G.R. No. 192558

FACTS: Javier an employee of Fly Ace performing various work for the
latter filed a complaint before the NLRC for underpayment of salaries and
other labor standard benefits.He alleged that he reported for work from
Monday to Saturday from 7:00 oclock in the morning to 5:00 o'clock in the
afternoon; that during his employment, he was not issued an
identification card and pay slips by the company; that he reported for
work but he was no longer allowed to enter the company premises by
the security guard upon the instruction of Ruben Ong (Mr. Ong), his
superior; that after several minutes of begging to the guard to allow him
to enter, he saw Ong whom he approached and asked why he was
being barred from entering the premises; that Ong replied by saying,
Tanunginmoanakmo;that he discovered that Ong had been courting his
daughter Annalyn after the two met at a fiesta celebration in Malabon
City; that Annalyn tried to talk to Ong and convince him to spare her
father from trouble but he refused to accede; that thereafter, Javier was
terminated from his employment without notice; and that he was neither
given the opportunity to refute the cause/s of his dismissal from work.

Fly Ace denied the existence of employer-employee relationship


between them and Javier as the latter was only called roughly 5 to 6
times only in a month whenever the vehicle of its contracted hauler,
Milmar Hauling Services, was not available. Labor Arbiter dismissed the
complaint ruling that respondent Fly Ace is not engaged in trucking
business but in the importation and sales of groceries. Since there is a
regular hauler to deliver its products, we give credence to Respondents
claim that complainant was contracted on pakiaobasis.

On appeal, NLRC reversed the decisin of the LA. It was of the view that a
pakyaw-basis arrangement did not preclude the existence of employer-
employee relationship. Payment by result is a method of compensation
and does not define the essence of the relation. It is a mere method of
computing compensation, not a basis for determining the existence or
absence of an employer-employee relationship. The NLRC further averred
that it did not follow that a worker was a job contractor and not an
employee, just because the work he was doing was not directly related
to the employers trade or business or the work may be considered as
extra helper as in this case; and that the relationship of an employer and
an employee was determined by law and the same would prevail
whatever the parties may call it. Finding Javier to be a regular employee,
the NLRC ruled that he was entitled to a security of tenure. For failing to
present proof of a valid cause for his termination, Fly Ace was found to be
liable for illegal dismissal of Javier who was likewise entitled to backwages
and separation pay in lieu of reinstatement. However, on appeal, CA
reversed the ruling of NLRC.The CA ruled that Javier's failure to present
salary vouchers, payslips, or other pieces of evidence to bolster his
contention, pointed to the inescapable conclusion that he was not an
employee of Fly Ace. Further, it found that Javiers work was not necessary
and desirable to the business or trade of the company, as it was only
when there were scheduled deliveries, which a regular hauling service
could not deliver, that Fly Ace would contract the services of Javier as an
extra helper. Lastly, the CA declared that the facts alleged by Javier did
not pass the control test.

He contracted work outside the company premises; he was not required


to observe definite hours of work; he was not required to report daily; and
he was free to accept other work elsewhere as there was no exclusivity of
his contracted service to the company, the same being co-terminous
with the trip only. Since no substantial evidence was presented to
establish an employer-employee relationship, the case for illegal dismissal
could not prosper. Hence, this appeal.

ISSUE: Does an employer-employee relationship exist between Javier and


Fly Ace, thereby holding the latter guilty of illegal dismissal?

HELD: The LA and the CA found Javier's claim of employment with Fly Ace
as wanting and deficient. The Court is constrained to agree. Labor
officials are enjoined to use reasonable means to ascertain the facts
speedily and objectively with little regard to technicalities or formalities
but nowhere in the rules are they provided a license to completely
discount evidence, or the lack of it. The quantum of proof required,
however, must still be satisfied. Hence, when confronted with conflicting
versions on factual matters, it is for them in the exercise of discretion to
determine which party deserves credence on the basis of evidence
received, subject only to the requirement that their decision must be
supported by substantial evidence.Accordingly, the petitioner needs to
show by substantial evidence that he was indeed an employee of the
company against which he claims illegal dismissal. In sum, the rule of
thumb remains: the onus probandi falls on petitioner to establish or
substantiate such claim by the requisite quantum of evidence. Whoever
claims entitlement to the benefits provided by law should establish his or
her right thereto. Sadly, Javier failed to adduce substantial evidence as
basis for the grant of relief.

By way of evidence on this point, all that Javier presented were his self-
serving statements purportedly showing his activities as an employee of
Fly Ace. Clearly, Javier failed to pass the substantiality requirement to
support his claim.

While Javier remains firm in his position that as an employed stevedore of


Fly Ace, he was made to work in the company premises during weekdays
arranging and cleaning grocery items for delivery to clients, no other
proof was submitted to fortify his claim. The lone affidavit executed by
one Bengie Valenzuela was unsuccessful in strengthening Javiers cause.

The Court is of the considerable view that on Javier lies the burden to
pass the well-settled tests to determine the existence of an employer-
employee relationship.

G.R. No. 157633 September 10, 2014

NORTHWEST AIRLINES, INC., vs. MA. CONCEPCION M. DEL ROSARIO,

Facts:

Petitioner Northwest Airlines, Inc. employed respondent Ma. Concepcion


M. Del Rosario (Del Rosario) on December 10, 1994 as one of its Manila-
based flight attendants. On May 18, 1998, Del Rosario was assigned at the
Business Class Section of Northwest Flight NW 26 bound for Japan. During
the boarding preparations, Del Rosario and another flight attendant
Kathleen Gamboa (Gamboa) had a heated argument because of the
former’s offensive remarks. Theconfrontation between the two prompted
their fellow attendants to get Maria Rosario D. Morales, the Assistant Base
Manager, to pacify them.

The parties differed on what happened thereafter. Del Rosario claimed


that only an animated discussion had transpired between her and
Gamboa, but Morales insisted that it was more than an animated
discussion, recalling that Del Rosario had even challenged Gamboa to a
brawl. This led to dismissal of Del Rosario from the service pending an
investigation of the fighting incident between her and Gamboa. Del
Rosario was subsequently informed of her termination from the service.
Northwest stated that based on the results of the investigation, Del
Rosario and Gamboa had engaged in a fight on board the aircraft, even
if there had been no actual physical contact between them; and that
because fighting was strictly prohibited by Northwest to the point that
fighting could entail dismissal from the service even if committed for the
first time, Northwest considered her dismissal from the service justified and
in accordance with the Rules of Conduct for Employees.

Del Rosario subsequently filed her complaint for illegal dismissal against
Northwest.

Labor Arbiter Teresita D. Castillon-Lora ruled in favor of Northwest, holding


that the dismissal of Del Rosario had been justified and valid because
Northwest is entitled protect its business interests by putting up an
impeccable image to the public.
Upon appeal, the NLRC reversed the decision of the Labor Arbiter, and
ruled in favor of Del Rosario, declaring that the incident between her and
Gamboacould not be held similar to the fightthat Northwest penalized
under its Rules of Conduct.

Aggrieved, Northwest elevated the adverse decision of the NLRC to the


CA on certiorari, averring that the NLRC thereby committed grave abuse
of discretion in reversing the decision of the Labor Arbiter, and submitting
that Del Rosario’s dismissal from the service had been for a just cause.
However, the CA sustained the NLRC observing that Northwest did not
discharge its burden to prove not merely reversible error but grave abuse
of discretion amounting to lack or excess of jurisdiction on the part of the
NLRC; and that, indeed, the NLRC had correctly held that Del Rosario’s
conduct did not constitute serious misconduct, because the NLRC, in
determining the usual, ordinary and commonly understood meaning of
the word fighting, had resorted to authoritative lexicons that supported its
conclusion that the exchange of words between Del Rosario and
Gamboadid not come within the definition of the word fighting. Thus, this
present petition.

Issue:

Whether or not Del Rosario’s dismissal from the service was valid

Ruling:

No. Del Rosario’s dismissal from the service is invalid. The Court affirms the
decision of the CA. Northwest argues that Del Rosario was dismissed on
the grounds of serious misconduct and willful disobedience. But
misconduct or improper behavior, to be a just cause for termination of
employment, must: (a) be serious; (b) relate to the performance of the
employee’s duties; and (c) show that the employee has become unfit to
continue working for the employer.

There is no doubt that the last two elements of misconduct were present
in the case of Del Rosario. The cause of her dismissal related to the
performance of her duties as a flight attendant, and she became unfit to
continue working for Northwest. However, with respect to the first
element, it cannot be said that the fight between Del Rosario and
Gamboa is serious as to warrant the termination of her employment even
if it was her first offense. Based on the foregoing, the incident involving Del
Rosario and Gamboa could not be justly considered as akin to the fight
contemplated by Northwest. In the eyes of the NLRC, Del Rosario and
Gamboa were arguing but not fighting.

Under the circumstances, therefore, the CA properly ruled that the NLRC
did not gravely abuse its discretion amounting to lack or excess of
jurisdiction by declaring Del Rosario's dismissal unjustified. Alas, Northwest
did not show how the NLRC could have abused its discretion, let alone
gravely, in ruling adversely against it.

WHEREFORE, the Court AFFIRMS the decision of the Court of Appeals


promulgated on June 21, 2002; and ORDERS the petitioner to pay the
costs of suit.
G.R. No. L-101438 October 13, 1992

CATHEDRAL SCHOOL OF TECHNOLOGY and SR. APOLINARIA TAMBIEN,


RVM, petitioners,
vs.
NATIONAL LABOR RELATIONS COMMISSION and TERESITA
VALLEJERA, respondents.

Facts:

Private respondent TeresitaVallejera (Vallejera) sought admission as an


aspirant to the Congregation of the Religious of Virgin Mary (RVM), upon
the recommendation of Archbishop Patrick Cronin. In order to observe
the life of a religious, she came to live with the sisters of the congregation
and received free board and lodging at the house of the nuns. During
the period of her aspirancy and in return for her accommodations, she
volunteered to assist as a library aide in the library section of the
Cathedral School of Technology, an educational institution run by the
RVM sisters. Later, private respondent had a change of heart and
confessed that she was no longer interested in becoming a nun. She
pleaded, however, to be allowed to continue living with the sisters for she
had no other place to stay in, to which request the sisters acceded and,
in exchange therefor, she voluntarily continued to assist in the school
library.

On January 29, 1988, Vallejera formally applied for and was appointed to
the position of library aide with a monthly salary of P1,171.00. It was at
around this time, however, that trouble developed. The sisters began
receiving complaints from students and employees about private
respondent’s difficult personality and sour disposition at work.Vallejera
was summoned to the Office of the Directress by herein petitioner Sister
ApolinariaTambien, RVM, shortly after the resignation of the school’s Chief
Librarian, Heraclea Nebria, on account of irreconcilable differences with
said respondent, for the purpose of clarifying the matter. Petitioner also
informed private respondent of the negative reports received by her
office regarding the latter’s frictional working relationship with co-workers
and students and reminded private respondent about the proper
attitude and behavior that should be observed in the interest of peace
and harmony in the school library

Private respondent resented the observations about her actuations and


was completely unreceptive to the advice given by her superior. On
separate occasions thereafter, petitioners sent at least three persons to
talk to and convince private respondent to settle her differences with the
former. Private respondent, however, remained adamant in her refusal to
submit to authority. On June 15, 1989, Sister Apolinaria sent a letter 2
formally informing private respondent that she had a month from said
date or until July 15, 1989 to look for another job as the school had
decided to accept her resignation. Vallejera then filed for a complaint for
illegal deduction and underpayment of salary, overtime pay and service
incentive pay which was later amended to include illegal dismissal.

The labor arbiter rendered a decision in favor of Vallejera holding that she
was illegally dismissed for lack of clue process, in that she was summarily
dismissed without a hearing being conducted in order to afford her an
opportunity to present her side.

On appeal, the NLRC affirmed the labor arbiter’s decision, with


modifications, on the rationale that while petitioners had valid reasons to
terminate the services of private respondent, the dismissal was
nonetheless illegal for lack of due process, hence the award of
backwages, separation pay and attorney’s fees. Thus, this present
petition.

Issue:

Whether or not NLRC committed abuse of discretion in ordering the


payment of said monetary claims where the dismissal is illegal for denial
of due process but there is a finding of a valid ground for termination

Ruling:

No. The NLRC did not commit an abuse of discretion. An evaluative


review of this case supports a just cause for termination. The reason for
which Vallejera’s services were terminated, namely, her unreasonable
behavior and unpleasant department in dealing with the people she
closely works with in the course of her employment, is analogous to the
other "just causes" enumerated under the Labor Code. In fact, as earlier
stated, her overbearing personality caused the chief librarian to resign,
Furthermore, the complaints about her objectionable behavior were
confirmed by her reproachable actuations during her meeting with the
petitioner directress. To make matters worse, she ignored the persons sent
by petitioners on separate occasions to intervene in an effort to bring the
matter to a peaceful resolution.

On the matter of illegal dismissal, the Court find no merit in NLRC’s


ratiocination. Its ruling that private respondent was illegally dismissed was
premised solely on the fact of alleged lack of procedural due process,
without regard to whether or not there was lawful cause for such
dismissal, which latter aspect constitutes the element of substantive due
process. It stands to reason that the separation of private respondent
from the service is justified as borne out by the circumstances of this case.
Verily, Vallejera is not entitled to the payment of separation pay and the
recovery of attorney’s fees.

IN VIEW OF ALL THE FOREGOING, the dispositions of public respondent in


its resolution dated June 24, 1992 are hereby ANNULLED and SET ASIDE.
Petitioners are, however, ordered to INDEMNIFY private respondent in the
amount of P1,000.00 concordant with the current jurisprudential norm.
G.R. No. 159302 February 6, 2008

CITIBANK, N.A., petitioner,


vs.
NATIONAL LABOR RELATIONS COMMISSION and ROSITA TAN
PARAGAS, respondents.

Facts:

Rosita Tan Paragas (Paragas) was found by Labor Arbiter


GeobelBartolabac to be an employee of petitioner Citibank, N.A. for
around eighteen (18) years from August 8, 1979 to September 4, 1997. At
the time her employment was terminated by petitioner for serious
misconduct, willful disobedience, gross and habitual neglect of duties
and gross inefficiency, she was occupying the position of filing clerk.

Sometime in the early part of 1993, as a result of the reorganization,


respondent bank declared certain officers and employees, or their
positions/functions, redundant. Among these affected was complainant
Paragas. However, to accommodate the union officers’ request,
complainant’s employment was not terminated but was assigned to
Records Management Unit of the Quality Assurance Division as bank
statement retriever. Beginning 21 February 1995, Paragas’ job in the bank
was to file and retrieve UAOFs.

On 11 December 1996, complainant was assigned to undertake the


special project of reorganizing the UAOF’s. On 10 January 1997, AVP
NarcisoFerrera issued a Memo to complainant calling Paragas’ attention
on the various misfiling on the reorganized UAOF file. Again, sometime in
April 1997, Paragas received another memo from AVP Ferrera regarding
her persistent failure to submit weekly report on the progress of her work
under the Special Project. By the end of June 1997, Paragas’
accomplishment was only 30% of the total work to be done.

On 25 July 1997, AVP Ferrera directed Paragasto explain in writing why her
employment should not be terminated. Correspondingly, Paragaswas
placed under Preventive suspension and thereafter submitted her written
explanation on 31 July 1997. Finally, on 4 September 1997, Citibank, N.A.
terminated her employment on the ground of serious misconduct, willful
disobedience, gross and habitual neglect of duties and gross inefficiency.

Respondent filed a complaint for illegal dismissal, praying


for reinstatement, backwages, damages and attorney’s fees. The labor
arbiter dismissed the complaint for lack of merit, finding that her dismissal
on the ground of work inefficiency was valid.

On appeal, the NLRC, by Resolution of October 24, 2000, affirmed the


decision of the labor arbiter with the modification that respondent should
be paid separation pay "as a form of equitable relief" in view of her length
of service with petitioner.

Respondent filed a MOTION FOR PARTIAL RECONSIDERATION of the NLRC


Resolution. She no longer challenged her dismissal on the ground of work
inefficiency, but prayed that petitioner be ordered to pay her the
"Provident Fund" benefits under its retirement plan for which she claimed
to be qualified pursuant to petitioner’s "Working Together" Manual which
states that an employee who resigned or to be discharged shall be
entitled to retirement benefits for reasons other than misconduct.

Finding that respondent’s dismissal was "for causes other than


misconduct," the NLRC, by the granted respondent’s motion for partial
reconsideration which was affirmed in toto by the appellate court.

Issues:

(1) Whether or not Paragas’ dismissal was valid


(2) Whether or not the alleged acts of respondent Paragas constituted
serious misconduct as to properly deny her claim for retirement
benefits

Ruling:

Yes. The dismissal of Paragas was valid and her acts constituted serious
misconduct, not only work inefficiency. The performance appraisals of
respondent from July to December 1994, from January to June 1995, and
from July to December 1996, did not merely show that respondent was
not able to meet performance targets. More relevantly, they
also consistently noted significant behavioral and attitudinal problems in
respondent. In particular, respondent was found to be very
argumentative;she had difficulty working with others; she was hard to
deal with; and she never ceased being the subject of complaints from
co-workers.

Moreover, beyond the documents referred to in the labor arbiter’s


decision, there are other pieces of evidence on record which further
establish that respondent was validly dismissed not only for work
inefficiency but for serious misconduct. The Court sees no reason why
these should not be accorded credibility along with those cited by the
labor arbiter. The assessment of respondent’s performance by Randy
Uson, another superior of respondent, was given weight. Significantly,
Uson later commented on respondent’s behavior as follows:

"Less tangible but none the less real,


are the common concerns raised by her peers and supervisor, on
the stress and tension created when Rose is around. The conscious
effort to ‘get out of her way’ and avoid conflict, hinders productivity
and efficiency and has adversely affected the morale of the entire
unit. x xx" (Emphasis and underscoring supplied)

When an employee, despite repeated warnings from the employer,


obstinately refuses to curtail a bellicose inclination such that it erodes the
morale of co-employees, the same may be a ground for dismissal for
serious misconduct.

It is respondent’s obstinate refusal to reform herself which ultimately


persuades this Court to find that her dismissal on the ground of serious
misconduct was valid. Having been validly dismissed on the ground of
serious misconduct, respondent is thus disqualified from receiving her
retirement benefits pursuant to the provision of petitioner’s "Working
Together" Manual.

WHEREFORE, the petition is GRANTED. The Court of Appeals Decision of


January 24, 2003 and Resolution of July 29, 2003 are SET ASIDE. The NLRC
Resolution dated October 24, 2001 granting private respondent’s MOTION
FOR PARTIAL RECONSIDERATION is thus VACATED.
G.R. No. 157376 October 2, 2007

CORAZON C. SIM, petitioners,


vs.
NATIONAL LABOR RELATIONS COMMISSION and EQUITABLE PCI-
BANK, respondents.

Facts:

Corazon Sim (petitioner) filed a case for illegal dismissal with the Labor
Arbiter, alleging that she was initially employed by Equitable PCI-Bank
(respondent) in 1990 as Italian Remittance Marketing Consultant to the
Frankfurt Representative Office. Eventually, she was promoted to
Manager position, until September 1999, when she received a letter from
Remegio David -- the Senior Officer, European Head of PCIBank, and
Managing Director of PCIB- Europe -- informing her that she was being
dismissed due to loss of trust and confidence based on alleged
mismanagement and misappropriation of funds.

The Labor Arbiter dismissed the case for want of jurisdiction and/or lack of
merit. Accordingly so, the labor relations system in the Philippines has no
extra-territorial jurisdiction. It is limited to the relationship between labor
and capital within the Philippines. Assuming for the sake of argument that
this Office has jurisdiction over this case, still, this Office is inclined to rule in
favor of the respondent. Complainant, as General Manager is an
employee whom the respondent company reposed its trust and
confidence. In other words, she held a position of trust. It is well-settled
doctrine that the basic premise for dismissal on the ground of loss of
confidence is that the employee concerned holds a position of trust and
confidence.

On appeal, the National Labor Relations Commission (NLRC) affirmed the


Labor Arbiter's Decision and dismissed petitioner's appeal for lack of merit.
Without filing a motion for reconsideration with the NLRC, petitioner went
to the Court of Appeals (CA) via a petition for certiorari under Rule 65 of
the Rules of Court.

The CA dismissed the petition due to petitioner's non-filing of a motion for


reconsideration with the NLRC. Petitioner filed a motion for
reconsideration but it was nonetheless denied by the CA. Hence, the
present recourse under Rule 45 of the Rules of Court.
Issue:

Whether or not the dismissal of petitioner Sim was valid

Ruling:

Yes. The dismissal of petitioner was valid. Petitioner does not deny having
withdrawn the amount of P3,000,000.00 lire from the bank's account.
What petitioner submits is that she used said amount for the Radio
Pilipinassa Roma radio program of the company. Respondent, however,
countered that at the time she withdrew said amount, the radio program
was already off the air. Respondent is a managerial employee. Thus, loss
of trust and confidence is a valid ground for her dismissal. The mere
existence of a basis for believing that a managerial employee has
breached the trust of the employer would suffice for his/her dismissal.

Moreover, under Rule 65, the remedy of filing a special civil action
for certiorari is available only when there is no appeal; or any plain,
speedy, and adequate remedy in the ordinary course of law. A "plain"
and "adequate remedy" is a motion for reconsideration of the assailed
order or resolution, the filing of which is an indispensable condition to the
filing of a special civil action for certiorari.This is to give the lower court the
opportunity to correct itself. Here, petitioner failed to allege any reason
why in her case a motion for consideration is to be dispensed with.

The Court notes, however, a palpable error in the Labor Arbiter's


disposition of the case, which was affirmed by the NLRC, with regard to
the issue on jurisdiction. It was wrong for the Labor Arbiter to rule that
"labor relations system in the Philippines has no extra-territorial jurisdiction.
Under Article 217 of the Labor Code, R.A. 8042 and its Implementing
Omnibus Rules and Regulations, it is clear that labor arbiters have original
and exclusive jurisdiction over claims arising from employer-employee
relations, including termination disputes involving all workers, among
whom are overseas Filipino workers.

In any event, since the CA did not commit any error in dismissing the
petition before it for failure to file a prior motion for reconsideration with
the NLRC, and considering that the Labor Arbiter and the NLRC's factual
findings as regards the validity of petitioner's dismissal are accorded great
weight and respect and even finality when the same are supported by
substantial evidence, the Court finds no compelling reason to relax the
rule on the filing of a motion for reconsideration prior to the filing of a
petition for certiorari. WHEREFORE, the petition is DENIED.
TIRAZONA vs. PHILIPPINE EDS TECHNO-SERVICE INC. (PET, Inc.)
G.R. No. 169712, January 20 2009

Principle: Loss of Trust and Confidence; Due Process; Dismissals have two
facets—first, the legality of the act of dismissal, which constitutes
substantive due process, and second, the legality in the manner of
dismissal, which constitutes procedural due process; Under Article 282(c)
of the Labor Code, loss of trust and confidence is one of the just causes
for dismissing an employee; An employee holds a position of trust and
confidence where he or she is entrusted with confidence on delicate
matters, such as care and protection, handling or custody of the
employer’s property.

FACTS: Ma. Wenelita Tirazona, the Administrative Manager of the


respondent company called her attention by the officers and directors of
PET Inc. because of her improper handling of a situation involving a rank-
and-file employee. Claiming she was denied due process, she
demanded Php 2M indemnity from PET and its officers/directors. She also
admitted to reading a confidential letter addressed to PET
officers/directors containing the legal opinion of the counsel of PET
regarding her case. Because of all this, she was validly terminated on the
ground that she willfully breached the trust and confidence reposed in
her by her employed. The SC denied her original petition.

On April 29, 2008: Petitioner moved for reconsideration praying that


her dismissal be declared illegal and that she be awarded separation
pay and retirement benefits out of humanitarian considerations. But the
SC denied MR.

On August 21, 2008: Petitioner filed a Motion for Leave to File a


Second Motion for Reconsideration for the reconsideration of the SC
resolution raising essentially the same arguments and prayers.

ISSUE: Whether or not Tirazona’s dismissal was justified to not warrant the
award of separation pay out of humanitarian consideration.

RULING: Yes, the dismissal was legally justified.

First of all, the second Motion for Reconsideration is prohibited,


except for extraordinarily persuasive reasons [Sec. 2, Rule 52, ROC]. Here,
no extraordinary persuasive reasons are present to allow the second MR.
Next, as re petitioner’s dismissal, the general rule is that an employee who
has been dismissed for any of the just causes enumerated under Art. 282
of the Labor Code is not entitled to separation pay. Only unjustly
dismissed employees are entitled to retirement benefits and other
privileges including reinstatement and backwages.

An exception, however, is that separation pay or other financial


assistance may be allowed to an employee dismissed for just causes on
the basis of equity. This shall be allowed as a measure of social justice only
in those instances where the employee is validly dismissed for causes
other than serious misconduct or those reflecting on his moral character.

Given the above, petitioner is not entitled to the award of


separation pay for violating the trust and confidence reposed in her by
her employer when she arrogantly demanded from respondent the
exorbitant amount of Php 2M in damages with a threat of a lawsuit if the
money was not paid within five days. She also continually refused to
cooperate with PET’s investigation of her case. Lastly, petitioner tried to
persuade the Court to consider in her favor the length of her service to
PET, but in the end, failed. She claimed that she was employed by PET for
26 years. However, it was later on found out that she had only been there
for 2 years and 9 months. The cases she cited to support her case were
misleading as the circumstances were totally different from hers.

Dispositive Portion:

WHEREFORE, premises considered, the instant petition is hereby DENIED for


lack of merit and the Decision of the Court of Appeals dated 24 May 2005
is hereby AFFIRMED. Costs against the petitioner.

CAPITOL WIRELESS INC VS. CARLOS ANTONIO BALAGOT


513 SCRA 672 [2007]

Jurisprudence recognizes as a valid ground for dismissal of an employee


unauthorized use of company time. It would be unfair to compensate
private respondent who does not devote his time and effort to his
employer. The primary duty of the employee is to carry out his employer’s
policies.

FACTS: Capitol Wireless, Inc. (Capwire) hired Carlos Antonio Balagot


(Balagot) as collector on September 16, 1987. Carlos is required to work
outside the office and Capwire assigned to him a motorcycle as a service
vehicle, for which it shouldered expenses for gasoline and maintenance.

Balagot was discovered to have been rendering services to China


Bank and that since 1992, Carlos had been concurrently employed with
Contractual Concepts, Inc. (CCI), a local manpower company, which
assigned him to render messengerial services to China Bank in the same
year.

Capwire terminated his services on the ground of grave


misconduct and willful breach of trust and confidence. Capwire
contends that the time of work of Balagot to other companies overlaps
with his work at Capwire. Balagot admitted the charge but he filed a
complaint for illegal dismissal against Capwire and its President Epifanio
Marquez.

ISSUE: Whether or not Balagot’s actuation warrants his dismissal.

HELD: Yes. The Supreme Court ruled that jurisprudence recognizes as a


valid ground for dismissal of an employee’s unauthorized use of company
time. And from the evidence presented, Balagot used the company
vehicle in pursuing his own interests, on company time and deviating from
his authorized route without permission. Capwire has all the right and
reason to cry foul as this is a clear case of moonlighting and using the
company’s time, money, and equipment to render service to another
company.

The court said that there is no denying that taking on double job
per se is not illegal according to the Labor Code, as extra income would
go a long way for an ordinary worker like Balagot. The only limitation is
where one job overlaps with the other in terms of time and/or poses a
clear case of conflict of interest as to the nature of business of
complainant’s two employers.
The contention of Balagot that he is working for China Bank after
5:00 pm is untenable because he was sighted by the HR director within
the premises of the bank at 3:35 pm and as general knowledge, the
banking industry follows the ordinary working hours from 8:00 am to 5:00
pm and a bank has no use for an employee who can only be of service
to it after 5:00 pm.

Dispositive Portion:

WHEREFORE, the petition is GRANTED. The appealed decision of the Court


of Appeals is REVERSED and SET ASIDE, and the September 23, 2002
decision of the National Labor Relations Commission is REINSTATED.

PLANTATION BAY AND SPA VS DUBRICO


G.R. NO. 182216 December 4, 2009

Principle: It is the employer’s responsibility to ensure that drug tests are


properly administered, the results thereof being the bases in terminating
the employees’ services; How can the presence of shabu be confirmed
when the results of the initial screening were not yet out?

Facts: Respondents are former employees of Plantation Bay located in


Cebu, of which Belarmino is the Manager. On several dates in September
2004, after Plantation Bay issued a series of memoranda and conducted
seminars relative to its drug-free workplace policy, Plantation Bay, in
compliance with Republic Act No. 9165 (Comprehensive Dangerous
Drugs Act of 2002), conducted surprise random drug tests on its
employees. The drug tests, said to have been carried out with the
assistance of the Philippine National Police-Scene of Crime Operations
(SOCO), were administered on about 122 employees by the Martell
Medical Trade and Lab Services (Martell), a drug testing laboratory. And
confirmatory tests were conducted by the Philippine Drug Screening
Laboratory, Inc. (Phil. Drug), a Department of Health-accredited
laboratory.

Respondent RomelDubrico (Dubrico) failed to take the drug test


conducted on September 14, 2004, hence, he was issued a
memorandum requiring him to appear in a mandatory conference on
September 20, 2004. Before the scheduled conference or on September
19, 2004, Dubrico explained in writing his failure to undergo the drug test,
he averring that, inter alia, the procedure for the random drug testing
was not followed such that he was not informed about his selection; and
that he was at the appointed time and place for the pre-test meeting but
that the duty manager was not around, hence, he left and failed to be
tested.Dubrico was later tested and found positive for use of
methamphetamine hydrochloride (shabu).

Twenty other employees were found positive for use of shabu including
herein respondents Godfrey Ngujo (Ngujo) and Julius Villaflor (Villaflor).

Their explanations having been found unsatisfactory, Plantation Bay


dismissed them including herein respondents.

Respondents Dubrico, Ngujo and Villaflor and three others thereupon


filed on November 18, 2004 their respective complaints for illegal dismissal,
questioning the conduct of the drug tests without the presence of the
DOLE Regional Director or his representative.

The Labor Arbiter Jose G. Gutierrez dismissed the employees' complaints,


holding that in testing positive for the use of shabu, they were guilty of
serious misconduct, hence, Plantation Bay validly terminated their
employment The Labor Arbiter discredited the drug test results presented
by the employees as the tests were taken more than 72 hours after the
conduct of the random drug tests.

On appeal, the NLRC, by Decision of October 26, 2005, affirmed the


Decision of the Labor Arbiter. On respondents' motion for reconsideration,
it, however reversed its October 26, 2005 Decision and declared that
respondents were illegally dismissed.

In finding for respondents, the NLRC held that the results of


the confirmatory drug tests cannot be given credence since they were
conducted prior to the conduct by the employer of the drug tests.

Its motion for reconsideration having been denied by Resolution of June


23, 2006, Plantation Bay appealed to the Court of Appeals.

The appellate court affirmed the NLRC March 24, 2006 Resolution with
modification by deleting the award of damages.

Hence, the present petition.

Issue: Whether or not Plantation Bay is justified in dismissing the


respondents for serious misconduct.

Ruling:
No, the petition is bereft of merit.

On the merits, it fails. The importance of the confirmatory test is


underscored in Plantation Bay's own "Policy and Procedures," in
compliance with Republic Act No. 9165, requiring that a confirmatory test
must be conducted if an employee is found positive for drugs in the
Employee's Prior Screening Test, and that both tests must arrive at the
same positive results.

Records show the following timeline, based on the reports on


respondents' respective drug tests administered by Martell and
confirmatory tests19 undertaken by the Phil. Drug:

Name Drug Test Confirmatory Test


RomelDubrico Urine sample received Issued on 09/29/04
on 09/29/04 at 5:14 p.m. at 3:57 p.m.
Godfrey Urine sample received Issued on 09/29/04
Ngujo on 09/29/04 at 5:24 p.m. at 3:57 p.m.
Julius Villaflor Urine sample received Issued on 09/29/04
on 09/29/04 at 5:32 p.m. at 4:15 p.m.

As reflected in the above matrix, the confirmatory test results were


released earlier than those of the drug test, thereby casting doubts on the
veracity of the confirmatory results.

Indeed, how can the presence of shabu be confirmed when the results of
the initial screening were not yet out? Plantation Bay's arguments that it
should not be made liable thereof and that the doubt arising from the
time of the conduct of the drug and confirmatory tests was the result of
the big volume of printouts being handled by Martell do not thus lie. It
was Plantation Bay's responsibility to ensure that the tests would be
properly administered, the results thereof being the bases in terminating
the employees' services.

Time and again, we have ruled that where there is no showing of a clear,
valid and legal cause for termination of employment, the law considers
the case a matter of illegal dismissal. The burden is on the employer to
prove that the termination of employment was for a valid and legal
cause. For an employee's dismissal to be valid,

(a) the dismissal must be for a valid cause and


(b) the employee must be afforded due process.

In fine, as petitioners failed to indubitably prove that respondents were


guilty of drug use in contravention of its drug-free workplace policy
amounting to serious misconduct, respondents are deemed to have
been illegally dismissed.

As to the appellate court's deletion of the award of damages, the same is


in order, there being no clear showing that the termination of
respondents' services was actuated by bad faith.

Dispositive Portion:

WHEREFORE, the petition is DENIED.

SAMAHAN NG MANGGAGAWA SA HYATT-NUWHRAIN VS HON. MAGSALIN


GR No. 164939 June 06, 2011

Principle: Series of irregularities when put together may constitute serious


misconduct, which under Article 282 of the Labor Code, as amended, is a
just cause for dismissal. Caragdag’s dismissal being due to serious
misconduct, it follows that he should not be entitled to financial
assistance. To rule otherwise would be to reward him for the grave
misconduct he committed. We must emphasize that social justice is
extended only to those who deserve its compassion.

FACTS: Petitioner Samahan ng mga Manggagawa sa Hyatt-NUWHRAIN-


APL is a duly registered union and the certified bargaining representative
of the rank-and-file employees of Hyatt Regency Manila, a five-star hotel
owned and operated by respondent Hotel Enterprises of the Philippines,
Inc. Mr.Angelito Caragdag (waiter) was dimissed by the Hotel for several
offenses. The Voluntary Arbitrator ruled that the dismissal Mr. Caragdag
for commiting offenses penalized with three suspensions within the 12
month period was valid.

However, due to humanitarian considerations, it ordered financial assista


nce.Petitioner assailed the decision of the Voluntary Arbitrator before the
CA in a petition for certiorari which was dismissed outright for being the wr
ong remedy.
The CA explained that Rule 43, Section 5 of the 1997 Rules of Civil Proced
ure explicitly provides that the proper mode of appeal from judgments, fi
nal orders or resolution of voluntary arbitrators is through a Petition for Revi
ew which should be filed within fifteen (15) days from the receipt of notic
e of judgment, order or resolution of the voluntary arbitrator. Considering t
hat petitioner intended the petition to be a Petition for Certiorari, the Cou
rt hereby resolves to dismiss the petition outright for being an improper mo
de of appeal.

ISSUE: WHETHER OR NOT MR. CARAGDAG IS ENTITLED TO THE AWARD OF


FINANCIAL ASSISTANCE FOR COMMITTING OFFENSES PENALIZED WITH
THREE SUSPENSIONS WITHIN A PERIOD OF 12 MONTHS.

RULING: NO, he is not entitled as ruled by the Supreme Court. On the


second issue, petitioner argues that Caragdag is entitled to financial
assistance in the amount of ₱100,000 on humanitarian considerations.
Petitioner stresses that Caragdag’s infractions were due to his being a
union officer and his acts did not show moral depravity. Petitioner also
adds that, while it is true that the award of financial assistance is given
only for dismissals due to causes specified under Articles 283 and 284 of
the Labor Code, as amended, this Court has, by way of exception,
allowed the grant of financial assistance to an employee dismissed for
just causes based on equity.

Respondent on the other hand, asserts that the CA correctly


deleted the award of financial assistance erroneously granted to
Caragdag considering that he was found guilty of serious misconduct
and other acts adversely reflecting on his moral character. Respondent
stresses that Caragdag’swillful defiance of the hotel’s security policy,
disrespect and intimidation of a superior, and unjustifiable desertion of his
work assignment during working hours without permission, patently show
his serious and gross misconduct as well as amoral character.

Here, Caragdag’s dismissal was due to several instances of willful


disobedience to the reasonable rules and regulations prescribed by his
employer. The Voluntary Arbitrator pointed out that according to the
hotel’s Code of Discipline, an employee who commits three different acts
of misconduct within a twelve (12)-month period commits serious
misconduct. He stressed that Caragdag’s infractions were not even
spread in a period of twelve (12) months, but rather in a period of a little
over a month. Records show the various violations of the hotel’s rules and
regulations were committed by Caragdag. He was suspended for
violating the hotel policy on bag inspection and body frisking. He was
likewise suspended for threatening and intimidating a superior while the
latter was counseling his staff. He was again suspended for leaving his
work assignment without permission. Evidently, Caragdag’s acts
constitute serious misconduct

Dispositive Portion:

WHEREFORE, the petitions for review on certiorari are DENIED. The October
3, 2003 and August 13, 2004 Court of Appeals Resolutions in CA-G.R. SP
No. 78364, as well as the Court of Appeals December 16, 2005 Decision
and April 12, 2006 Resolution in CA-G.R. SP No. 77478, are AFFIRMED and
UPHELD

Mario Danilo B. Villafloresvs National Labor Relations Commission and


Radio Communications Philippines Inc.
G.R. No.114777 (July 5, 1996)

FACTS:
On October 29, 1990, Villaflores, the Assistant Vice- President of
RCPI and German Bernardo Mattus, Manager of the Information System
Department had an unpleasant confrontation with each other due to the
removal of the posted copy of invitation to a computer seminar on the
bulletin board. The said post was not approved by the former. Upon
learning of the removal, Mattusbarged into the working place of the
petitioner and confronted him and without the timely intervention of the
other employees present, the two would have assaulted each other as
Villaflores has already attempted to throw a stapler at him. As Mattus was
leaving the room, Villaflores shouted invectives such as “bullshit ka”,
“baby ka” and “gago ka” at him.
On the same day, Mattus lodged a complaint against the
petitioner for conduct unbecoming of a company official, physical injury
threat and shouting invectives. The next day, the executive president
asked Villaflores for an explanation for no administrative action shall be
taken against him. In his explanation, he claimed that Mattus attempted
to attack him and those invectives were mere expressions of disgust.
After conducting formal investigation, Villaflores was placed under
preventive suspension. Ultimately, he was terminated on the grounds of
gross misconduct and gross violations of their Company Rules and
Regulations losing the company’s trust and confidence in him.
Villaflores filed a complaint before the NLRC Arbitration Branch
against RCPI for illegal dismissal, illegal suspension, illegal deduction of
allowances and nonpayment of 13th month pay with claim for moral and
exemplary damages and attorney’s fees. The Labor Arbiter rendered a
decision that Villaflores was illegally dismissed and was not guilty of serious
misconduct. He further held that the attempt of throwing staler at Mattus
and uttering foul language at him although constituting misconduct
cannot fall under the category of serious misconduct.Moreover, the
Labor Arbiter found Mattus to be guilty of disrespect to a superior officer.
But due to the unduly soured and strained relationship between the
parties and the loss of trust and confidence, reinstatement was not
considered.
Both parties appealed to the NLRC. RCPI maintained that
Villafloreswas guilty of grave misconduct. Villaflores, on the other hand,
contended that there was no just cause of his dismissal so the Labor
Arbiter should have ordered reinstatement. However, the NLRC affirmed
the assailed decision. Consequently, both parties elevated the case to
the Supreme Court on separate petitions for certiorari.

ISSUE:
Whether the actuations of Villaflores constitutes serious misconduct

HELD:
No.
As found by the Labor Arbiter and affirmed by the NLRC and the
Supreme Court, to wit:
The petitioner reacted to the posting by Mattus of a poster at the
bulletin board without his consent and the latter's angrily barging into the
room where he was seated but his reaction -- his attempt to throw a
stapler at Mattus and, thereafter, his uttering foul language at him
although constituting misconduct cannotfall under the category of a
serious misconduct. The petitioner was provoked by Mattus who
unjustifiably barged into his room. He did not actually throw a stapler at
Mattus. He could have just tried to scare him with the stapler. He allowed
himself to be pacified by cooler heads. These attending circumstances
removed petitioner's reaction from the classification of a serious
misconduct. However, the petitioner is guilty of minor misconduct as he
should have met anger with sobriety and authority for he degraded his
position by engaging a subordinate in a shouting match of foul
language.
Consequently, the Court agree with the public respondents that
the termination of employment of petitioner Villaflores on account of a
minor misconduct was illegal because Art. 282 of the Labor Code
mentions "serious misconduct" as a cause for cessation of employment.
VH Manufacturing, Inc vs National Labor Relations Commission and
Herminio C. Gamido
G.R. No. 130957 (January 19, 2000)

FACTS:
Since November 5, 1985, private respondent was employed in
petitioner’s business of manufacturing liquefied petroleum gas (LPG)
cylinders as a quality control inspector with the principal duty of
inspecting LPG cylinders for any possible defects. His service with the
company was abruptly interrupted on February 14, 1995, when he was
served a notice of termination of his employment.
His dismissal stemmed from an incident on February 10, 1995
wherein petitioner’s company President, Alejandro Dy Juanco, allegedly
caught private respondent sleeping on the job, violating Company Rule
15-b. On that same day, private respondent was asked through a written
notice from the petitioner’s Personnel Department to explain within
twenty-four (24) hours why no disciplinary action should be taken against
him. Without delay, private respondent replied in a letter.
Notwithstanding, his foregoing reply, he was terminated.
Feeling aggrieved, private respondent fileda complaint for illegal
dismissal, praying for reinstatement. Labor Arbiter Ricardo C. Nora
rendered his decision upholding petitioner’s position and declared that
private respondent’s dismissal is anchored on a valid and just cause and
the latter’s contention of denial of due process as devoid of merit.
Private respondent then appealed the decisionto the NLRC. The
NLRC reversed the decision of the Labor Arbiter and orderedfor his
reinstatement with full backwages. It also denied petitioner’s motion for
reconsideration. Petitioner now challenges the correctness of the NLRC’s
decision and order via the instant petition.

ISSUE:
Whether or not the termination was anchored on a just and valid
cause

HELD:
No.
In termination disputes, the burden of proof is always on the
employer to prove that the dismissal was for a just and valid cause.
However, petitioner’s claim that private respondent slept on the job was
not substantiated by any evidence. Furthermore, the doctrine that
sleeping on the job is a valid ground for dismissal relied by the petitioner is
misplaced as such would be appliable to security guards due to the
nature of their works.
The Court further held that while an employer enjoys a wide
latitude of discretion in the promulgation of policies, rules and regulations
on work-related activities of the employees, those directives, however,
must always be fair and reasonable, and the corresponding penalties,
when prescribed, must be commensurate to the offense involved and to
the degree of the infraction. In the case at bar, the dismissal meted out
on private respondent for allegedly sleeping on the job, under the
attendant circumstances, appears to be too harsh a penalty,considering
that he was being held liable for first time, after nine (9) long years of
unblemished service, for an alleged offense which caused no prejudice
to the employer, aside from absence of substantiation of the alleged
offense.

Roberto Norberto F. Samson vs National Labor Relations Commission,


Schering-Plough Corporation, LeoRiconalla and Jose L. Estingor
G.R. No. 121035 (April 12, 2000)

FACTS:
Samson, the petitioner received a letter calling the attention of his
conduct during the Sales And Marketing Christmas gathering where he
allegedly uttered obscene and offensive words directed to the SPC’s
Management Committee in the presence of several employees. Such
utterances and malicious and lewd gestures were also made on the
same occasion directed to the President and General Manager of SPC.
He was given two days to explain why no disciplinary action,
including termination should be taken against him and he was placed on
preventive suspension until further notice. The petitioner made a reply to
the memo categorically denying the allegations and thereby,
complaining the preventive suspension placed on him. However, the
petitioner received a letter terminating his employment.
The Labor Arbiter ruled that the petitioner’s conduct was not so
serious as to warrant his dismissal ordering the company for his
reinstatement. Both parties appealed to the NLRC. In its decision, the
NLRC reversed the labor arbiter’ decision finding that there was just
cause, that is, gross misconduct, for petitioner’s dismissal. Hence, this
petition.

ISSUE:
Whether or not the petitioner was validly dismissed
HELD:
No.
Misconduct is improper or wrong conduct. It is the transgression of
some established and definite rule of action, a forbidden act, a
dereliction of duty, willful in character, and implies wrongful intent and
not mere error in judgment. The misconduct to be serious must be of such
grave and aggravated character and not merely trivial and unimportant.
Such misconduct, however serious, must, nevertheless, be in connection
with the employee’s work to constitute just cause for his separation.
In this case, the alleged misconduct of petitioner is not of such
serious and grave character as to warrant his dismissal. First, petitioner
made the alleged offensive utterances and obscene gesture during an
informal Christmas gathering of respondent company’s district sales
managers and marketing staff. The gathering was just a casual get-
together of employees. It is to be expected during this kind of gatherings,
where tongues are more often than not loosened by liquor or other
alcoholic beverages, that employees freely express their grievances and
gripes against their employers. Employees should be allowed wider
latitude to freely express their sentiments during these kinds of occasions
which are beyond the disciplinary authority of the employer. Significantly,
it does not appear in the records that petitioner possessed any
ascendancy over the employees who heard his utterances as to cause
demoralization in the ranks.
Second, using the words "bullshit" and "putangina" and making
lewd gesture to express his dissatisfaction over a particular management
decision were clearly in bad taste but these acts were not intended to
malign or cast aspersion on the person of respondent company’s
president and general manager. Furthermore, when petitioner was heard
to have uttered the alleged offensive words against respondent
company’s president and general manager, the latter was not around.
Third, respondent company itself did not seem to consider the
offense of petitioner serious and grave enough to warrant an immediate
investigation on the matter. In fact, respondent company allowed several
weeks to pass before it deemed it necessary to require petitioner to
explain why no disciplinary action should be taken against him for his
behavior. This seeming lack of urgency on the part of respondent
company in taking any disciplinary action against petitioner negates its
charge that the latter’s misbehavior constituted serious misconduct.
Furthermore, it is also provided in the company rules and regulations on
the use of violent language, a first offense can only be imposed with a
“verbal reminder and not dismissal.
Indeed, the penalty of dismissal is unduly harsh considering that
petitioner had been in the employ of respondent company for eleven
(11) years and it does not appear that he had a previous derogatory
record. It is settled that notwithstanding the existence of a valid cause for
dismissal, such as breach of trust by an employee, nevertheless, dismissal
should not be imposed, as it is too severe a penalty if the latter had been
employed for a considerable length of time in the service of his employer,
and such employment is untainted by any kind of dishonesty and
irregularity.

Lorna DisingPunzal vs ESTI Technologies, Inc., Werner Geiserrt and


Carmelo D. Remudaro
G.R. Nos. 170384-85 (March 9, 2007)

FACTS:
Petitioner Punzal had been working for ESTI Technologies as
Department Secretary. Sometime in 2001, she sent and email to her
officemates announcing the holding of a Halloween party to be held in
the office. Her immediate supervisor advised her to first secure the
approval of Wesner Geisert, the Senior Vice-President. Upon learning that
the latter did not approve the plan, she sent another e-mail to her
officemates stating that Geisert was so unfair, among others.
The Human Resources of the Company informed the petitioner that
Geisert secured a copy of her e-mail and that he required her to explain
in writing within 48 hours why she should not be given disciplinary actions
due to her improper conducts or acts of discourtesy or disrespectand
making malicious statements concerning company officer. The Petitioner
replied that she had no malicious intent in her second email and she did
not expect that such words can be called as acts of discourtesy or
disrespect. Finding her explanation not acceptable, she was terminated.
The petitioner filed a complaint for illegal dismissal against ESTI, et
al. The Labor Arbiter dismissed the complaint finding that she was legally
dismissed for serious misconduct and that she was afforded due process.
On petitioner’s appeal, the NLRC found that while she was guilty of
misconduct, the penalty of dismissal was disproportionate to her
infraction. But due to the strained relations, her reinstatement was no
longer feasible.
Both parties filed their respective petitions for certiorari with the
Court of Appeals. The appellate court on its decision reinstated the Labor
Arbiter’s Order as her e-mail message to her officemates tended to cast
scorn and disrespect toward a senior company officer. The message itself
resounds of subversion and undermine the authority and credibility of
management. Hence, this petition for review on certiorari.

ISSUE:
Whether or not the petitioner’s statement was discourteous and
disrespectful as to constitute gross disrespect and serious misconduct

HELD:
Yes.
In Philippines Today, Inc. v. NLRC,this Court, passing on the attitude
or respect that an employee is expected to observe towards an
employer, held:
x xxA cordial or, at the very least, civil attitude, according due
deference to one's superiors, is still observed, especially among high-
ranking management officers. The Court takes judicial notice of the
Filipino values of pakikisama and paggalang which are not only
prevalent among members of a family and community but within
organizations as well, including work sites. An employee is expected to
extend due respect to management, the employer being the "proverbial
hen that lays the golden egg," so to speak. An aggrieved employee who
wants to unburden himself of his disappointments and frustrations in his
job or relations with his immediate superior would normally approach said
superior directly or otherwise ask some other officer possibly to mediate
and discuss the problem with the end in view of settling their differences
without causing ferocious conflicts. No matter how [much] the employee
dislikes the employer professionally, and even if he is in a confrontational
disposition, he cannot afford to be disrespectful and dare to talk with an
unguarded tongue and/or with a bileful pen.
Additionally, petitioner sent the e-mail message in reaction to
Geisert's decision which he had all the right to make. That it has been a
tradition in ETSI to celebrate occasions such as Christmas, birthdays,
Halloween, and othersdoes not remove Geisert's prerogative to approve
or disapprove plans to hold such celebrations in office premises and
during company time. It is settled that
x xx it is the prerogative of management to regulate, according to
its discretion and judgment, all aspects of employment. This flows from the
established rule that labor law does not authorize the substitution of the
judgment of the employer in the conduct of its business. Such
management prerogative may be availed of without fear of any liability
so long as it is exercised in good faith for the advancement of the
employers' interest and not for the purpose of defeating or circumventing
the rights of employees under special laws or valid agreement and are
not exercised in a malicious, harsh, oppressive, vindictive or wanton
manner or out of malice or spite.
Furthermore, petitioner's reliance on Samson vs NLRCis misplaced.
First, in that case, this Court found that the misconduct committed was
not related with the employee's work as the offensive remarks were
verbally made during an informal Christmas gathering of the employees,
an occasion "where tongues are more often than not loosened by liquor
or other alcoholic beverages” and "it is to be expected x xx that
employees freely express their grievances and gripes against their
employers."
In petitioner's case, her assailed conduct was related to her work. It
reflects an unwillingness to comply with reasonable management
directives.While in Samson, Samson was held to be merely expressing his
dissatisfaction over a management decision,in this case, petitioner's
offensive remarks were directed against Geisert.Finally, in Samson, this
Court found that the "lack of urgency on the part of the respondent
company in taking any disciplinary action againstthe employee negates
its charge that the latter's misbehavior constituted serious misconduct." In
the case at bar, the management acted 14 days after petitioner
circulated the quoted e-mail message.

G.R. NO. 167345: November 23, 2007

e PACIFIC GLOBAL CONTACT CENTER, INC. and/or JOSE VICTOR SISON


v. MA. LOURDES CABANSAY

Principle:

While compassion and human consideration should guide the disposition


of cases involving termination of employment, since it affects one's
means of livelihood, it should not be overlooked that the benefits
accorded to labor do not include compelling an employer to retain the
services of an employee who has been shown to be a gross liability to the
employer.

Facts:

Ma. Lourdes Cabansay (Cabansay) was hired as Senior Traning Manager


of ePacific Global Contact Center, Inc. on April 18, 2001and became a
regular employee on August 1, 2001. In March 2002, respondent was
tasked to prepare a new training process for the company's Telesales
Trainees.
After Mr.Rosendo S. Ballesteros (Ballesteros) reviewed the training module,
the company's Senior Vice President-Business Development Group, found
that the module did not contain any changes and that they were not
ready for presentation. He instructed Cabansaythrough an electronic
mail (e-mail) to postpone the presentation and the implementation of the
new training process and emphasized that the Department needed more
time to teach the trainees on how to get leads, focus on developing their
telemarketing skills and acquire proper motivation.

Cabansayreplied the email on April 5, 2002 with the message as follows:

“Ro, the presentation is going to be discussed in detail. As we discussed


yesterday i (sic) SPECIFICALLY told you that I WILL DISCUSS the new
training process and explain it to them in detail. Didn't you see the last
past (sic) of the 5-day classroom training, (sic) the last day includes
PROSPECTING, and that’s where the CCA trainees will be taught how to
get leads both local and abroad.

The criteria for the evaluation? It's already done by Richie, we’re going to
distribute the hard copies and discuss it in DETAIL in this afternoon's
briefing.

This is a very simple presentation and I WILL NOT POSTPONE it today, it's
very easy to comprehend and as per YOUR INSTRUCTION we will be
implementing it next week, so when should we present this to the
TLs?cralawlibrary

Let's not make SIMPLE THINGS COMPLICATED.

I will go on with the presentation this afternoon.”

On the same day, no presentation of the training module was made as


the Senior Manager for Telesales, Ms. Lorna Garcia informed all the
participants that the same was postponed because Management was
not yet ready to present the module.

On April 6, 2002, Ballesteros sent a Memo to Cabansay, informing her that


the he found the message to be a clear act of insubordination, causing
him to lose his trust and confidence in her as Manager of the Training
Department. He then asked her to explain in writing why she should not
be terminated as a consequence of her acts.
Cabansay submitted two memoranda dated April 8 and 11, 2002
explaining that she had no intention to disregard the order to postpone
the implementation of the new training process.

On April 11, 2002, she submitted her second explanation and also
received a memorandum from the HR Department/Office of the
President notifying her that she had been terminated from the service
effective immediately for having committed an act of insubordination
resulting in the management's loss of trust and confidence in her.

Issue:

Whether or not respondent Cabansay was illegally dismissed?

Ruling:

The court ruled that her email reply constitutes willful disobedience,
making her dismissal valid.

ARTICLE 282. TERMINATION BY EMPLOYER

An employer may terminate an employment for any of the following


causes:

(a) Serious misconduct or willful disobedience by the employee of the


lawful orders of his employer or representative in connection with his work;

x xx

(c) Fraud or willful breach by the employee of the trust reposed in him by
his employer or duly authorized representative;

Willful disobedience or insubordination necessitates the concurrence of


at least two requisites: (1) the employee's assailed conduct must have
been willful, that is, characterized by a wrongful and perverse attitude;
and (2) the order violated must have been reasonable, lawful, made
known to the employee and must pertain to the duties which he had
been engaged to discharge.On the other hand, loss of trust and
confidence, to be a valid ground for dismissal, must be based on a willful
breach of trust and founded on clearly established facts. A breach is
willful if it is done intentionally, knowingly and purposely, without justifiable
excuse, as distinguished from an act done carelessly, thoughtlessly,
heedlessly or inadvertently. It must rest on substantial grounds and not on
the employer's arbitrariness, whims, caprices or suspicion; otherwise, the
employee would eternally remain at the mercy of the employer. Loss of
confidence must not also be indiscriminately used as a shield by the
employer against a claim that the dismissal of an employee was arbitrary.
And, in order to constitute a just cause for dismissal, the act complained
of must be work-related and show that the employee concerned is unfit
to continue working for the employer.44

The fact that she well-understood the directive for her to postpone the
presentation of the module, as she herself acknowledged in her e-mail
reply to SVP Ballesteros that she would "discuss the new training process
and explain it to them in detail" in the afternoon on that day, thus, she
would not postpone the scheduled presentation. There is no doubt,
therefore, that the order of Ballesteros was clearly made known to
respondent.

As to the willfulness of her conduct, the same is manifest in her e-mail


reply, which, as it is written, is characterized by abject aggressiveness and
antagonism: the e-mail has a begrudging tone and is replete with
capitalized words eliciting her resolve to indeed contravene the SVP's
directive. Thus, she categorically said, "This is a very simple presentation
and I WILL NOT POSTPONE it today, it's very easy to comprehend and as
per YOUR INSTRUCTION we will be implementing it next week, so when
should we present this to the TLs? Let's not make SIMPLE THINGS
COMPLICATED. I will go on with the presentation this afternoon."

Cabansay was a managerial employee, a Senior Training Manager


entrusted with the delicate matter of molding the minds and characters
of call center agents and team leaders, and clothed with discretion to
determine what was in the best interest of the company, her managerial
discretion was not without limits. Its parameters were contained the
moment her discretion was exercised and then opposed by the
immediate superior officer/employer for being against the policies and
welfare of the company. Hence, any action in pursuit of the discretion
thus opposed ceased to be discretionary and could be considered as
willful disobedience.

Let it be stressed that insofar as the application of the doctrine of trust


and confidence is concerned, jurisprudence has distinguished the
treatment of managerial employees or employees occupying positions of
trust and confidence from that of rank-and-file personnel. With respect to
the latter, loss of trust and confidence as a ground for dismissal requires
proof of involvement in the alleged events in question, but as regards
managerial employees, the mere existence of a basis for believing that
such employee has breached the trust of his employer would suffice for
his or her dismissal.For this purpose, there is no need to present proof
beyond reasonable doubt. It is sufficient that there is some basis for the
loss of trust or that the employer has reasonable ground to believe that
the employee is responsible for the misconduct which renders him
unworthy of the trust and confidence demanded by his
position.Cabansay’s conduct, is sufficient basis for the company to lose its
trust and confidence in her. Under the circumstances, the company
cannot be expected to retain its trust and confidence in and continue to
employ a manager whose attitude is perceived to be inimical to its
interests. Unlike other just causes for dismissal, trust in an employee, once
lost, is difficult, if not impossible to regain.
G.R. No. 164860: February 02, 2010

HILTON HEAVY EQUIPMENT CORPORATION AND PETER LIM VS. ANANIAS P.


DY

Principle:

The violation of the statutory due process, even if the dismissal was for a
just cause, warrants the payment of indemnity in the form of nominal
damages. This indemnity is intended not to penalize the employer but to
vindicate or recognize the employee's right to statutory due process
which was violated by the employer.

Facts:

Ananias Dy (hereafter, "DY") was employed at Hilton Heavy Equipment


Corporation. He was assigned as the personal bodyguard of Peter Lim
(hereafter, "LIM"), the President of the said Corporation. On 19 April 2000,
in the presence of the Corporation's employees and Lim, Dy mauled
Duke Echiverri, a co-employee, within the premises of the principal office
of the Corporation. Dy defied orders of Lim to stop mauling Duke
Echiverri. Dy also threatened to kill the latter, and uttered that if he will be
given monetary consideration, he will cease working in the company.
Geraldine Chan, Secretary of the Corporation, executed an affidavit
attesting to the fact of Dy's utterance of his intention to resign from his job.
Thereafter, Dy stopped reporting to work. Subsequently, Duke Echiverri
filed criminal complaints against Dy for grave threats and less serious
physical injuries and the corresponding Informations were filed before the
Municipal Trial Court in Cities, Mandaue City. These cases were later
dismissed upon motion filed by Duke Echiverri. A month after the mauling
incident, on 19 May 2000, Lim requested Dy to come to the office where
he was confronted by Lim and Wellington Lim, Lim's brother. Thereat, Dy
was paid by Wellington Lim the amount of P120, 000.00 as may be shown
by SolidbankMandaue Branch Check No. CD 0590750 dated 19 May
2000 payable to cash, as separation pay

Issue

Whether or not Dy is illegally dismissed?


Ruling:

It was upheld that Dy is validly dismissed but failure of the company to


follow the statutory due process of terminating an employee makes them
liable for nominal damages.

Art. 282. Termination by Employer. -- An employer may terminate an


employment for any of the following causes:

(a) Serious misconduct or willful disobedience by the employee of the


lawful orders of his employer or representative in connection with his work;

(b) Gross and habitual neglect by the employee of his duties;

(c) Fraud or willful breach by the employee of the trust reposed in him by
his employer or duly authorized representative;

(d) Commission of a crime or offense by the employee against the person


of his employer or any immediate member of his family or his duly
authorized representative; and

(e) Other causes analogous to the foregoing.

Art. 285. Termination by Employee. -- (a) An employee may terminate


without just cause the employee-employer relationship by serving a
written notice on the employer at least one (1) month in advance. The
employer upon whom no such notice was served may hold the
employee liable for damages.

(b) An employee may put an end to the relationship without serving any
notice on the employer for any of the following just causes:

1. Serious insult by the employer or his representative on the honor and


person of the employee;

2. Inhuman and unbearable treatment accorded the employee by the


employer or his representative;

3. Commission of a crime or offense by the employer or his representative


against the person of the employee or any of the immediate members of
his family; and
4. Other causes analogous to any of the foregoing.

The court ruled thatDy was the perpetrator in a mauling incident and his
acts is a just cause for termination. However, the company failed to
accord Dy due process as Section 2, Rule XXIII, and Book V of the
Omnibus Rules Implementing the Labor Code.

Section 2.Standards of due process; requirements of due notice. - In all


cases of termination of employment, the following standards of due
process shall be substantially observed:

I. For termination of employment based on just causes as defined in


Article 282 of the Code

(a) A written notice served on the employee specifying the ground or


grounds for termination, and giving to said employee reasonable
opportunity within which to explain his side;

(b) A hearing or conference during which the employee concerned, with


the assistance of counsel if the employee so desires, is given opportunity
to respond to the charge, present his evidence or rebut the evidence
presented against him; and

(c) A written notice of termination served on the employee indicating


that upon due consideration of all the circumstances, grounds have
been established to justify his termination.

In case of termination, the foregoing notices shall be served on the


employee's last known address.

The law requires that the employer must furnish the worker sought to be
dismissed with two written notices before termination of employment can
be legally effected: (1) notice which apprises the employee of the
particular acts or omissions for which his dismissal is sought; and (2) the
subsequent notice which informs the employee of the employer's
decision to dismiss him. Failure to comply with the requirements taints the
dismissal with illegality.
LORES REALTY ENTERPRISES, INC., LORENZO Y. SUMULONG III, PETITIONERS,
VS. VIRGINIA E. PACIA, RESPONDENT.

Principle:

If doubt exists between the evidence presented by the employer and the
employee, the scales of justice must be tilted in favor of the employee.
The rule in controversies between a laborer and his master distinctly states
that doubts reasonably arising from the evidence, or in the interpretation
of agreements and writing, should be resolved in favour of the labourer.

Facts:

In 1982,Virginia E. Pacia (Pacia) was hired by LREI. At the time of her


dismissal, she was the assistant manager and officer-in-charge of LREI's
Accounting Department under the Finance Administrative Division.

On October 28, 1998, LREI's acting general manager, petitioner


Sumulong, through Ms. Julie Ontal, directed Pacia to prepare Check
Voucher No. 16477 worth P150,000.00 as partial payment for LREI's
outstanding obligation to the Bank of the Philippine Islands-Family
Bank (BPI-FB). Pacia did not immediately comply with the instruction. After
two repeated directives, Pacia eventually prepared Check No.
0000737526 in the amount of P150,000.00. Later, Sumulong again directed
Pacia to prepare Check Voucher No. 16478 in the amount of P175,000.00
to settle the balance of LREI's outstanding indebtedness with BPI-FB. Pacia
once again was slow in obeying the order. Due to the insistence of
Sumulong, however, Pacia eventually prepared Check No. 0000737527 in
the amount of P175,000.00.

To explain her refusal to immediately follow the directive, Pacia reasoned


out that the funds in LREI's account were not sufficient to cover the
amounts to be indicated in the checks.

On October 29, 1998, Sumulong issued a memorandumordering Pacia to


explain in writing why she refused to follow a clear and lawful directive.

On the same day, Pacia replied in writing and explained that her initial
refusal to prepare the checks was due to the unavailability of funds to
cover the amounts and that she only wanted to protect LREI from liability
under the Bouncing Checks Law.

On November 6, 1998, Pacia received a notice of termination stating,


among others, that she was being dismissed because of her willful
disobedience and their loss of trust and confidence in her.

Issue:

Whether or not Pacia is illegally dismissed?

Ruling:

The court ruled that Pacia is illegally dismissed

Article 282 of the Labor Code enumerates the just causes for which an
employer may terminate the services of an employee, to wit:

ARTICLE 282.Termination by employer. - An employer may terminate an


employment for any of the following causes:

(a) Serious misconduct or willful disobedience by the employee of the


lawful orders of his employer or representative in connection with his
work;

(b) Gross and habitual neglect by the employee of his duties;

(c) Fraud or willful breach by the employee of the trust reposed in him by
his employer or duly authorized representative;

(d) Commission of a crime or offense by the employee against the person


of his employer or any immediate member of his family or his duly
authorized representative; and

(e) Other causes analogous to the foregoing. [Emphasis supplied]

The offense of willful disobedience requires the concurrence of two (2)


requisites: (1) the employee's assailed conduct must have been willful,
that is characterized by a wrongful and perverse attitude; and (2) the
order violated must have been reasonable, lawful, made known to the
employee and must pertain to the duties which he had been engaged
to discharge.

The reason of her initial reluctance is to protect the company from liability
under the Bouncing Checks Law. It was not wrongful or willful. Neither can
it be considered an obstinate defiance of company authority. The Court
takes into consideration that Pacia, despite her initial reluctance,
eventually did prepare the checks on the same day she was tasked to do
it.Thus, her actuation should not be construed as improper conduct.

ARSENIO S. QUIAMBAO, Petitioner, vs. MANILA ELECTRIC COMPANY,


Respondent
G.R. No. 171023 December 18, 2009

Facts:
On July 16, 1986, petitioner was employed as branch teller by respondent
Manila Electric Company. He was assigned at respondent’s
Mandaluyong office and was responsible for the handling and processing
of payments made by respondent’s customers.
It appears from his employment records, however, that petitioner has
repeatedly violated the Company Code of Employee Discipline and has
exhibited poor performance in the latter part of his employment.
On March 10, 2000, a Notice of Investigation was served upon petitioner
for his unauthorized and unexcused absences on November 10, 25, 26,
29, 1999; December 1, 2, 14, 15, 16, 17, 20, 21, 22, 2000; and from February
17, 2000 up to the date of such notification letter.
Petitioner was likewise required to appear at the investigation and to
present his evidence in support of his defense.
However, despite receipt of such notice, petitioner did not participate in
the investigation. Consequently, in a Memorandum dated March 21,
2000, the legal department recommended petitioner’s dismissal from
employment due to excessive, unauthorized, and unexcused absences,
which constitute (i) abandonment of work under the provisions of the
Company Code of Employee Discipline (ii) and gross and habitual
neglect of duty under Article 282 of the Labor Code of the Philippines.
Through a Notice of Dismissal dated March 28, 2000, petitioner’s
employment was terminated effective March 29, 2000.
On July 3, 2001, petitioner filed a complaint before the Arbitration Branch
of the NLRC against respondent assailing the legality of his dismissal. While
petitioner did not dispute his absences, he nonetheless averred that the
same were incurred with the corresponding approved application for
leave of absence. He also claimed that he was denied due process.
The Labor Arbiter rendered a Decision dismissing petitioner’s complaint for
lack of merit.
Petitioner appealed to the NLRC which affirmed the legality of his
dismissal due to habitual absenteeism. Nonetheless, the NLRC awarded
separation pay in favor of petitioner.
Aggrieved, respondent filed with the CA a petition for certiorari. On
October 28, 2005, the CA nullified the NLRC’s Decision and reinstated the
Labor Arbiter’s Decision dismissing the complaint. It ruled that the award
of separation pay is neither justified nor warranted under the
circumstances.

Issue:
Whether or not the petitioner is guilty of serious misconduct.

Held:
Yes. The Supreme Court has examined the records which indeed show
that petitioner’s unauthorized absences as well as tardiness are habitual
despite having been penalized for past infractions.
In Gustilo v. Wyeth Philippines, Inc., the Supreme Court held that a series
of irregularities when put together may constitute serious misconduct.
The Supreme Court also held that gross neglect of duty becomes serious
in character due to frequency of instances. Serious misconduct is said to
be a transgression of some established and definite rule of action, a
forbidden act, a dereliction of duty, willful in character, and indicative of
wrongful intent and not mere error of judgment. Oddly, petitioner never
advanced any valid reason to justify his absences. Petitioner’s intentional
and willful violation of company rules shows his utter disregard of his work
and his employer’s interest. Indeed, there can be no good faith in
intentionally and habitually incurring unexcusable absences. Thus, the
CA did not commit grave abuse of discretion amounting to lack or excess
of jurisdiction in equating petitioner’s gross neglect of duty to serious
misconduct.
Moreover, petitioner is not entitled to separation pay. Labor adjudicatory
officials and the CA must demur the award of separation pay based on
social justice when an employee’s dismissal is based on serious
misconduct or willful disobedience; gross and habitual neglect of duty;
fraud or willful breach of trust; or commission of a crime against the
person of the employer or his immediate family - grounds under Art. 282 of
the Labor Code that sanction dismissals of employees.
Petition is dismissed. The Resolution of the Court of Appeals is affirmed.
ST. LUKE'S MEDICAL CENTER, INC. and ROBERT KUAN, Chairman, Petitioners,
vs. ESTRELITO NOTARIO, Respondent.
G.R. No. 152166 October 20, 2010

Facts:
St. Luke’s Medical Center, Inc. (petitioner hospital), located at Quezon
City, employed respondent as In-House Security Guard. In August 1996,
Nimaya Electro Corporation installed a closed-circuit television (CCTV)
system in the premises of petitioner hospital to enhance its security
measures and conducted an orientation seminar for the in-house security
personnel on the proper way of monitoring video cameras, subject to
certain guidelines.
Respondent was on duty from 6:00 p.m. to 6:00 a.m. of the following day,
December 31, 1996. His work consisted mainly of monitoring the video
cameras. In the evening of December 30, 1996, Justin Tibon, a foreigner
from Majuro, Marshall Island, then attending to his 3-year-old daughter,
Andanie De Brum, who was admitted since December 20, 1996 at room
257, cardiovascular unit of petitioner hospital, reported to the
management of petitioner hospital about the loss of his mint green
traveling bag. When the tapes of video camera recorder (VCR) no. 3
covering the subject period were reviewed, it was shown that the VCR
was focused on camera no. 2 and camera no. 1. The cameras failed to
record any incident of theft at room 257.
On January 6, 1997, petitioner hospital, through Abdul A. Karim, issued a
Memorandum to respondent, the CCTV monitoring staff on duty,
directing him to explain in writing, within 24 hours upon receipt thereof,
why no disciplinary action should be taken against him for violating the
normal rotation/sequencing process of the VCR and, consequently, failed
to capture the theft of Tibon's traveling bag at room 257.
In his letter, respondent explained that on the subject dates, he was the
only personnel on duty as nobody wanted to assist him. Finding the
written explanation of respondent to be unsatisfactory, petitioner hospital,
through Calixton, served on respondent a copy of the Notice of
Termination dated January 24, 1997, dismissing him on the ground of gross
negligence/inefficiency under Section 1, Rule VII of its Code of Discipline.
Thus, respondent filed a Complaint for illegal dismissal against petitioner
hospital and its Chairman, Robert Kuan, seeking reinstatement with
payment of full backwages from the time of his dismissal up to actual
reinstatement, without of loss of seniority rights and other benefits.
The Labor Arbiter dismissed respondent’s complaint for illegal dismissal
against petitioners. On appeal by the respondent, the NLRC issued a
Resolution reversing the Decision of the Labor Arbiter. The CA dismissed
petitioners' petition for certiorari, affirming the NLRC’s finding.
Issue:
Whether or not a single isolated act of negligence insufficient ground for
termination.

Held:
No. Respondent was illegally dismissed without just cause and
compliance with the notice requirement.
Under Article 282 (b) of the Labor Code, an employer may terminate an
employee for gross and habitual neglect of duties. Neglect of duty, to be
a ground for dismissal, must be both gross and habitual. Gross negligence
connotes want of care in the performance of one’s duties. Habitual
neglect implies repeated failure to perform one’s duties for a period of
time, depending upon the circumstances. A single or isolated act of
negligence does not constitute a just cause for the dismissal of the
employee. Under the prevailing circumstances, respondent exercised his
best judgment in monitoring the CCTV cameras so as to ensure the
security within the hospital premises. Verily, assuming arguendo that
respondent was negligent, although this Court finds otherwise, the lapse
or inaction could only be regarded as a single or isolated act of
negligence that cannot be categorized as habitual and, hence, not a
just cause for his dismissal.
Petitioners anchor on the postulate that even a single or isolated act of
negligence by respondent constitutes a just cause for his dismissal as it
engendered the possibility of a legal action that may be taken against
them by the owner of the lost items. This is purely speculative. The
Certification issued by the Station Commander of Galas Police Station
(Station II), located at Unang Hakbang Street, corner Luzon Avenue,
Galas, Quezon City, stated that no incident of theft was reported by the
management of petitioner hospital or any of its authorized
representatives involving the loss of the plane tickets and other personal
belongings of Justin Tibon and Andanie De Brum. Even the supposed
complainant, Tibon, did not institute any complaint against petitioner
hospital. Therefore, it cannot be said that petitioners incurred actual loss
or pecuniary damage.

Petition is denied. The decision of the NLRC is affirmed.


LAKPUE DRUG, INC., LA CROESUS PHARMA, INC., TROPICAL BIOLOGICAL
PHILS., INC. (all known as LAKPUE GROUP OF COMPANIES) and/or ENRIQUE
CASTILLO, JR., Petitioners, vs. MA. LOURDES BELGA, Respondent.
G.R. No. 166379 October 20, 2005

Facts:
Petitioner Tropical Biological Phils., Inc. (Tropical), a subsidiary of Lakpue
Group of Companies, hired on March 1, 1995 respondent Ma. Lourdes
Belga (Belga) as bookkeeper and subsequently promoted as assistant
cashier. On March 19, 2001, Belga brought her daughter to the Philippine
General Hospital (PGH) for treatment of broncho-pneumonia. On her way
to the hospital, Belga dropped by the house of Marylinda O. Vegafria,
Technical Manager of Tropical, to hand over the documents she worked
on over the weekend and to give notice of her emergency leave.
While at the PGH, Belga who was pregnant experienced labor pains and
gave birth on the same day. Two days after giving birth, Tropical
summoned Belga to report for work but the latter replied that she could
not comply because of her situation. Then, Tropical sent Belga another
memorandum ordering her to report for work and also informing her of
the clarificatory conference. Belga requested that the conference be
moved as her newborn was scheduled for check-up. When Belga
attended the clarificatory conference, she was informed of her dismissal
effective that day.
Tropical terminated Belga on the following grounds: (1) Absence without
official leave for 16 days; (2) Dishonesty, for deliberately concealing her
pregnancy; (3) Insubordination, for her deliberate refusal to heed and
comply with the memoranda sent by the Personnel Department on
March 21 and 30, 2001 respectively.
The Labor Arbiter ruled in favor of Belga and found that she was illegally
dismissed. Tropical appealed to the NLRC, which reversed the findings of
the labor arbiter.
Belga filed a petition for certiorari with the Court of Appeals which found
in favor of Belga.

Issue:
Whether or not the respondent was illegally dismissed due to alleged
concealment of pregnancy.

Held:
Yes. Tropical’s ground for terminating Belga is her alleged concealment
of pregnancy. It argues that such non-disclosure is tantamount to
dishonesty and impresses upon this Court the importance of Belga’s
position and the gravity of the disruption her unexpected absence
brought to the company. Tropical also charges Belga with
insubordination for refusing to comply with its directives to report for work
and to explain her absence.
The Supreme Court has defined misconduct as a transgression of some
established and definite rule of action, a forbidden act, a dereliction of
duty, willful in character, and implies wrongful intent and not mere error in
judgment. The misconduct to be serious must be of such grave and
aggravated character and not merely trivial and unimportant. Such
misconduct, however serious, must, nevertheless, be in connection with
the employee’s work to constitute just cause for his separation.
In the instant case, the alleged misconduct of Belga barely falls within the
situation contemplated by the law. Her absence for 16 days was justified
considering that she had just delivered a child, which can hardly be
considered a forbidden act, a dereliction of duty; much less does it imply
wrongful intent on the part of Belga. Tropical harps on the alleged
concealment by Belga of her pregnancy. This argument, however, begs
the question as to how one can conceal a full-term pregnancy. The
Supreme Court agree with respondent’s position that it can hardly
escape notice how she grows bigger each day. While there may be
instances where the pregnancy may be inconspicuous, it has not been
sufficiently proven by Tropical that Belga’s case is such.
The Petition is denied. The decision of the Court of Appeals is affirmed.

ABELARDO P. ABEL, Petitioner, vs. PHILEX MINING CORPORATION,


represented by FERNANDO AGUSTIN, Respondent.
G.R. No. 178976 July 31, 2009

Facts:
Petitioner was first hired by respondent in January, 1988. He was
eventually assigned to respondent’s Legal Department as a Contract
Claims Assistant, a position he occupied for five years prior to his transfer
to the Mine Engineering and Draw Control Department wherein he was
appointed Unit Head in early 2002.
Sometime in September, 2002, petitioner was implicated in an irregularity
occurring in the subsidence area of respondent’s mine site at Pacdal,
Tuba, Benguet. Petitioner’s co-worker Danilo R. Lupega (Lupega), a
Subsidence Checker at the mine site who was himself under
administrative investigation for what came to be known as the
"subsidence area anomaly," executed an affidavit.
The incidents alleged in Lupega’s affidavit supposedly took place when
petitioner was still a Contract Claims Assistant at respondent’s Legal
Department.
In compliance with respondent’s directive to respond to Lupega’s
charges, petitioner wrote a letter to Fernando Agustin (Agustin),
respondent’s Vice President for Operations, denying Lupega’s allegations
of extortion from Anseca Development Corporation (ANSECA) and failure
to report the incidents of underloading of ANSECA’s trucks during
backfilling operations.
An investigation was promptly launched by respondent’s officers by
conducting several fact-finding meetings for the purpose. Petitioner
attended the meetings but claimed that he was neither asked if he
needed the assistance of counsel nor allowed to properly present his side.
By Memorandum dated December 7, 2002, respondent’s Administrative
Division, Litigation and Investigation Section found petitioner guilty of (1)
fraud resulting in loss of trust and confidence and (2) gross neglect of
duty, and was meted out the penalty of dismissal from employment
effective December 8, 2002.
Petitioner thus filed a complaint for illegal dismissal. The Labor Arbiter ruled
that petitioner was dismissed illegally. On respondent’s appeal, the NLRC
reversed the decision of the Labor Arbiter. Petitioner’s Motion for
Reconsideration having been denied by Resolution of July 7, 2005, he
appealed to the Court of Appeals via certiorari. The appellate court
denied due course to, and dismissed, petitioner’s appeal.

Issue:
Whether or not the petitioner was illegally dismissed for lack of trust and
confidence.

Held:
Yes. The petitioner was illegally dismissed.
Respondent dismissed petitioner on the following grounds: (1) fraud
resulting in loss of trust and confidence and (2) gross neglect of duty.
Respecting the first ground, Article 282(c) of the Labor Code allows an
employer to terminate the services of an employee for loss of trust and
confidence:
ART. 282. Termination by employer. - An employer may terminate an
employment for any of the following causes:
xxxx
c) Fraud or willful breach by the employee of the trust reposed in him by
his employer or his duly authorized representative.
The first requisite for dismissal on the ground of loss of trust and confidence
is that the employee concerned must be holding a position of trust and
confidence. Verily, the Court must first determine if petitioner holds such a
position.
There are two classes of positions of trust. The first class consists of
managerial employees. They are defined as those vested with the powers
or prerogatives to lay down management policies and to hire, transfer,
suspend, lay-off, recall, discharge, assign or discipline employees or
effectively recommend such managerial actions. The second class
consists of cashiers, auditors, property custodians, etc.. They are defined
as those who, in the normal and routine exercise of their functions,
regularly handle significant amounts of money or property.
In this case, petitioner was a Contract Claims Assistant at respondent’s
Legal Department at the time he allegedly committed the acts which led
to its loss of trust and confidence. It is not the job title but the actual work
that the employee performs. It was part of petitioner’s responsibilities to
monitor the performance of respondent’s contractors in relation to the
scope of work contracted out to them.
The second requisite is that there must be an act that would justify the loss
of trust and confidence. Loss of trust and confidence, to be a valid cause
for dismissal, must be based on a willful breach of trust and founded on
clearly established facts. The basis for the dismissal must be clearly and
convincingly established but proof beyond reasonable doubt is not
necessary. Respondent’s evidence against petitioner fails to meet this
standard. Its lone witness, Lupega, did not support his affidavit and
testimony during the company investigation with any piece of evidence
at all. No other employee working at respondent’s mine site attested to
the truth of any of his statements. Standing alone, Lupega’s account of
the subsidence area anomaly could hardly be considered substantial
evidence. And while there is no concrete showing of any ill motive on the
part of Lupega to falsely accuse petitioner, that Lupega himself was
under investigation when he implicated petitioner in the subsidence area
anomaly makes his uncorroborated version suspect.
The Labor Arbiter correctly found that the alleged telephone
conversations between petitioner and Didith Caballero of ANSECA would
not suffice to lay the basis for respondent’s loss of trust and confidence in
petitioner.
The assailed Decision of the Court of Appeals is REVERSED and SET ASIDE.
Respondent is ordered to reinstate petitioner to his former position or its
equivalent without loss of seniority rights and privileges, and to pay him full
backwages inclusive of allowances and other benefits or their monetary
equivalent, from the time of his dismissal until his actual reinstatement; or,
if reinstatement is no longer feasible, to give him separation pay
equivalent to at least one month salary for every year of service,
computed from the time of engagement up to the finality of this decision.
VALENZUELA VS CALTEX PHILIPPINES, INC. G.R. Nos. 169965-66
December 15, 2010

FACTS:This petition for review on certiorari assails the Decision 1 dated July
20, 2005 of the Court of Appeals (CA) in CA-G.R. SP Nos. 80494 and 80638.
The appellate court had reversed and set aside the Decision 2 of the
National Labor Relations Commission (NLRC) and reinstated the Decision 3
of the Labor Arbiter which dismissed petitioner’s complaint for illegal
dismissal for lack of merit.

Carlos Valenzuela was hired by respondent Caltex Philippines, Inc.


sometime in March 1965 as Laborer and assigned in the Lube Oil Section
of its Pandacan Terminal in Manila. Sometime in 1970, petitioner
requested that he be transferred to respondent’s main office. Since the
position available then was that of a messenger, he accepted the same.
One year later, petitioner was given a new assignment as Aviation
Attendant of respondent’s Manila Aviation Service. 5

After twenty-two (22) years at the Manila Aviation Service, petitioner was
moved to respondent’s Lapu-Lapu Terminal in Lapu-Lapu City. The
transfer was part of the penalty for the charge of not servicing an
aircraft’s fuel needs, which petitioner denied. Reluctantly, petitioner
acceded to the transfer.Petitioner was initially designated as Gauger but
he also handled Bulk Receiving, Tank Truck Loading and Bunkering. In
1996, the Warehouseman retired and the functions of the warehouseman
were given to petitioner.7 As warehouseman, petitioner’s duties included,
among others, the maintenance of stock cards for storehouse materials
and supplies, the conduct of physical inventory of the company’s
merchandise stocks and monitoring the movement of said stocks.8

On November 23, 1999, a spot operational audit was conducted on the


Lapu-Lapu City District Office, and several irregularities in the handling of
respondent’s merchandise were discovered. A net inventory shortage
amounting to P823,100.49 was discovered.9Petitioner was required to
explain within forty-eight (48) hours such shortage and the other
irregularities discovered during the spot audit. He was further informed 10
that an administrative investigation will be conducted on the matter and
because of the nature of his offense and his position in the Company, he
was preventively suspended to prevent further losses and/or possible
tampering of the documents and other evidence.11

The administrative investigation was conducted with two hearings held


on December 15, 1999 and January 18, 2000. On both dates, petitioner
was present, together with his counsel and/or union officer. Thereafter,
based on the findings from the administrative investigation, respondent
found cause to terminate petitioner’s employment.12 Specifically,
respondent found petitioner liable for (1) Gross and Habitual neglect of
duties and responsibilities as warehouse clerk, (2) Not performing month-
end inventory duties, (3) Not investigating the shortages of stocks under
his custody and (4) Commission of Fraud.13

Aggrieved by the respondent’s decision to terminate his employment,


petitioner filed a complaint14 for illegal dismissal with the NLRC Regional
Arbitration Branch No. VII in Cebu City. On May 19, 2000, Labor Arbiter
Ernesto F. Carreon rendered a Decision 17 declaring the claim for illegal
dismissal unmeritorious.

On appeal to the NLRC, the NLRC set aside the decision of the Labor
Arbiter and declared that petitioner was illegally dimissed. On September
20, 2005, the CA denied the motion for reconsideration. Hence, this
petition.

ISSUE: Whether or not the CA correctly ruled that petitioner was validly
dismissed.

RULING: Yes.

There is no compelling reason in this case for us to reverse the ruling of the
CA sustaining the finding of the Labor Arbiter that petitioner’s dismissal
was effected with just cause. The findings of the Labor Arbiter are
supported by more than substantial evidence and even petitioner’s
admissions during the administrative hearings.27 As the CA correctly held,

Evidence overwhelmingly shows that petitioner Valenzuela was indeed


guilty of habitual and gross neglect of his duties. It was not the first time
that there occurred a shortage of the merchandise stocks but apparently
petitioner Valenzuela did nothing about it and, instead, manipulated
documents and records, i.e., stock cards, to create the illusion that all
merchandise stocks were accounted for, when in fact a lot of these
merchandise were already missing from petitioner Company’s Lapu-Lapu
terminal.

Furthermore, petitioner Valenzuela likewise committed fraud and willful


breach of the trust reposed in him by petitioner Caltex. He was in-charge
of the custody and monitoring of the merchandise stocks, and, as found
by the Labor Arbiter, was entrusted with confidence on delicate matters,
i.e., the handling and care and protection of the employer's property.
Considering that the merchandise stocks are the lifeblood of petitioner
Caltex, petitioner Valenzuela's act of allowing the loss of merchandise
stocks and concealing these from the employer is reason enough for his
termination from his employment.29

Under Article 282 of the Labor Code, as amended, gross and habitual
neglect by the employee of his duties is a sufficient and legal ground to
terminate employment. Jurisprudence provides that serious misconduct
and habitual neglect of duties are among the just causes for terminating
an employee. Gross negligence connotes want of care in the
performance of one’s duties. Habitual neglect implies repeated failure to
perform one’s duties for a period of time, depending upon the
circumstances.30

Further, Article 282 of the Labor Code, as amended, also provides fraud
or willful breach by employee of the trust reposed in him by his employer
as a just cause for termination. It is always a serious issue for the employer
when an employee performs acts which diminish or break the trust and
confidence reposed in him. The Labor Code, as amended, although
sympathetic to the working class, is aware of this scenario and in pursuit
of fairness, included fraud or willful breach of trust as a just cause for
termination of employment.

One last point on the preventive suspension imposed by the


respondents.Sections 8 and 9 of Rule XXIII, Implementing Book V of the
Omnibus Rules Implementing the Labor Code provides:SEC. 8. Preventive
suspension. – The employer may place the worker concerned under
preventive suspension if his continued employment poses a serious and
imminent threat to the life or property of the employer or of his co-
workers.

SEC. 9. Period of suspension. – No preventive suspension shall last longer


than thirty (30) days. The employer shall thereafter reinstate the worker in
his former or in a substantially equivalent position or the employer may
extend the period of suspension provided that during the period of
extension, he pays the wages and other benefits due to the worker. In
such case, the worker shall not be bound to reimburse the amount paid
to him during the extension if the employer decides, after completion of
the hearing, to dismiss the worker.

In this case, petitioner was preventively suspended from November 26,


1999 to December 25, 1999. Respondents extended his preventive
suspension for thirty days, from December 26, 1999 to January 24, 2000. 31
After the conclusion of the administrative investigation, he was finally
terminated on January 21, 2000.32 There is no showing that petitioner was
paid his wages and benefits during the additional period of suspension.
Clearly, petitioner is entitled to his salary and other benefits prior to his
dismissal, from December 26, 1999 to January 21, 2000.

WHEREFORE, the petition is DENIED. The assailed Decision dated July 20,
2005 of the Court of Appeals in the consolidated cases of CA-G.R. SP Nos.
80494 and 80638 is hereby AFFIRMED.

PHIL. TRANSMARINE CARRIERS VS CARILLA, 535 SCRA 893 (2007)

FACTS: Apetition for Review on Certiorari filed by Philippine Transmarine


Carriers, Inc. (petitioner) seeking to annul and set aside the Decision 1 of
the Court of Appeals (CA) dated November 26, 2002. On November 18,
1993, Felicisimo Carilla (respondent) was hired by petitioner, a manning
agent, in behalf of its principal, Anglo-Eastern Shipmanagement Ltd., to
work as Master on board MV Handy-Cam Azobe for twelve months. Their
approved POEA contract provided that respondent would get a basic
monthly pay of US$1700.00, fixed monthly overtime of US$765.00, master's
allowance of US$170.00 and leave with pay of six days per month or
US$340.00 or a total of US$2,975.00 a month.

On November 29, 1993, respondent boarded the vessel in Abidjan, Ivory


Coast, Africa. On June 6, 1994, while the vessel was in Bombay, India,
respondent was dismissed and repatriated to the Philippines.

On August 25, 1994, respondent filed with the Philippine Overseas and
Employment Agency (POEA) a complaint3 for illegal dismissal with claims
for salaries and other benefits for the unexpired portion of his contract as
well as unremitted allotments and damages. He alleged that: he was
dismissed without notice and hearing and without any valid reason;
petitioner's unlawful act deprived him of his expected monthly benefits
for the unexpired portion of his contract. Petitioner filed its Answer4
contending that: respondent's termination was for cause; he failed to
take the necessary steps to ensure the safety of the vessel and its cargo
while plying the waters of South Korea and Keelung port causing
petitioner to incur a huge amount of damages on cargo claims and
vessel repairs; respondent's incompetence is therefore penalized with
dismissal; despite the fact that respondent was warned of his lapses, he
had not shown any improvement which forced petitioner to dismiss and
replace him with a competent one; thus, cost had to be incurred.
Petitioner asked for moral and exemplary damages and attorneys fees as
its counterclaim.
On December 1, 1999, the Labor Arbiter (LA) rendered a decision 6 in
favor of respondent. The LA found that respondent's long experience as a
seaman and his various recommendations from his previous employers
contradicted any finding of incompetence; that the unauthenticated
logbook extract submitted by petitioner lacked even an iota of
admissibility as the entries appearing therein had been merely copied
from the original logbook. The LA gave credence to respondent's
allegation that he was unceremoniously removed from his job and found
that petitioner had not submitted any proof of payment of respondent's
claims.

Aggrieved, petitioner filed its appeal with the NLRC. In its Decision 8 dated
June 14, 2001, the NLRC dismissed the appeal and affirmed the LA's
decision.The NLRC found that petitioner's evidence which consisted of a
document dated June 1, 1994, entitled "Logs of Events During
Respondent's Command" and the Senior Officer Evaluation Reports, did
not prove anything as these documents, besides being unauthenticated,
were self-serving and unreliable.Petitioner's motion for reconsideration
was denied in a Resolution dated April 10, 2003.

Hence, the instant Petition

ISSUE: Whether or not respondent was illegally dismissed

RULING: Yes

In termination cases, the burden of proof rests upon the employer to show
that the dismissal of the employee is for just cause 16 and failure to do so
would mean that the dismissal is not justified. A dismissed employee is not
required to prove his innocence of the charges leveled against him by his
employer.17 The determination of the existence and sufficiency of a just
cause must be exercised with fairness and in good faith and after
observing due process.18

Respondent was dismissed because of his alleged incompetence. To


prove respondent's incompetence while on board the vessel, petitioner
presented a piece of paper dated June 1, 1994 entitled "Logs of Events
During Capt Carilla (sic) Command,"19 enumerating therein the alleged
incidents where damages to timber products and on the vessel occurred;
and the Senior Officer Evaluation Reports20 showing respondent's
unsatisfactory performance, prepared by Chief Officer R. Miu and Chief
Engineer N.K. Jaggi, who allegedly had served with respondent and had
seen his work on board the vessel.
We agree with the LA, NLRC and the CA in their finding that petitioner's
documents were not authenticated and, hence, were self-serving and
unreliable. It appears from the "Logs of Events During Capt. Carilla
Command" that it is merely a typewritten enumeration of several alleged
incidents of damages to the cargoes and to the vessel, but it does not
state the source and who prepared the same. While petitioner claims
that it was prepared by the vessel's technical superintendent, he was not
identified at all. The log of events did not also provide a detailed account
of respondent's act of incompetence which caused those alleged
incidents. There is no way of verifying the truth of these entries, and if they
were actually recorded in the vessel logbook on the dates the alleged
incidents took place. In its Memorandum of Appeal filed with the NLRC,
petitioner claims that the original copies were not available, as they were
on file with the vessel at that time. Be that as it may, it was still petitioner's
duty to secure the same to prove the validity of respondent's dismissal.

In Wallem Maritime Services, Inc. v. National Labor Relations


Commission,21 we rejected a typewritten collation of excerpts from what
could be the logbook and found that what should have been submitted
as evidence was the logbook itself or even authenticated copies of
pertinent pages thereof, which could have been easily xeroxed or
photocopied, considering the present technology on reproduction of
documents. On the other hand, in Abacast Shipping and Management
Agency, Inc. v. National Labor Relations Commission,22 we held:

The log book is a respectable record that can be relied upon to


authenticate the charges filed and the procedure taken against the
employees prior to their dismissal. Curiously, however, no entry from such
log book was presented at all in this case. What was offered instead was
the shipmasters report, which was later claimed to be a collation of
excerpts from such book.

It would have been a simple matter, considering the ease of reproducing


the same, to make photocopies of the pertinent pages of the log book to
substantiate the petitioner's contention. Why this was not done is
something that reasonably arouses the curiosity of this Court and suggests
that there probably were no entries in the log book at all that could have
proved the alleged offenses of the private respondents."

Petitioner's arguments are that respondent, being the person responsible


for accomplishing the vessel logbook by writing entries on the day-to-day
events on board, could not be expected to reflect any derogatory
reports about his own performance; and that the officers next in rank,
who are the technical superintendent and the chief engineer are the
only ones who could check on respondent's performance. These
arguments are not sufficient to disturb the findings of the CA.

Assuming the vessel logbook kept by respondent did not reflect the
different untoward incidents that occurred in the vessel, petitioner should
have presented other evidence to substantiate these incidents.
Petitioner's log of events purports to show that the timber products on the
vessel were damaged, and that the vessel was towed to a port for repair.
It was also alleged in petitioner's pleadings that it had incurred huge
amounts for damages on cargo claims. However, petitioner failed to
present these cargo claims from the shipper/consignees, and petitioner's
payment thereof as well as its expenses for the cost of the repair of the
vessel.

Moreover, the two sets of Senior Officer Evaluation Reports allegedly


prepared by the officers next in rank to respondent did not help to justify
respondent's dismissal for incompetency. While the reports showed that
respondent was given an unsatisfactory performance rating and a
recommendation for his replacement, they failed to show the exact
designations of the persons who prepared the same, and neither do their
signatures appear over the typewritten names. In fact, these alleged
officers did not even execute an affidavit to attest to the truth of those
reports. Thus, we agree with the LA and the NLRC that these documents,
being unauthenticated, have no probative value.

Respondent was terminated without having been given the opportunity


to defend himself. He was summarily dismissed and repatriated to the
Philippines without being informed of the charges against him; nor was he
given the chance to refute the charges.

Petitioner's claim that it has a wider latitude of discretion in terminating


respondent, since the latter was a managerial employee, is not plausible.
It is well settled in this jurisdiction that confidential and managerial
employees cannot be arbitrarily dismissed at any time, and without cause
as reasonably established in an appropriate investigation. 23 Such
employees, too, are entitled to security of tenure, fair standards of
employment and the protection of labor laws.24 Managerial employees,
no less than rank-and-file laborers, are entitled to due process.25 The
captain of a vessel is a confidential and managerial employee within the
meaning of this doctrine.26 Thus, respondent was illegally dismissed as he
was not accorded a fair investigation as required by law and the ground
invoked for his dismissal was not proven.
BAGO VS NRLC, 520 SCRA 644 (2007)

FACTS:In a complaint filed on November 20, 2002 1 with the Human


Resource Development Department (HRDD) of respondent Standard
Insurance Company Incorporated (SICI), Celia P. Abordo (Celia), the
Head of the Tuguegarao Branch of SICI, charged five employees
including herein petitioner Arlyn Bago (Arlyn), an encoder, and Elsie
Pagarigan (Elsie), an assistant underwriter, with "manipulating money out
of the agents/zone managers and [Celia’s] commissions." 2 She further
charged Arlyn and two other employees with "spreading rumors to
clients/agents/zone managers that [Celia] is having an ‘affair’ with the
claims assistant."3Arlyn et al. were later terminated effective March 31,
2003.15

Arlyn and Elsie subsequently filed separate complaints for illegal dismissal
against respondent SICI and its President-co-respondent Ernesto T.
Echaus. The complaints were consolidated. 16

By Decision of October 27, 2003, Executive Labor Arbiter Salvador V.


Reyes found that Arlyn and Elsie were illegally dismissed and accordingly
ordered their reinstatement to their former positions, without loss of
seniority rights,17 and the award to them of full backwages and other
benefits they normally enjoyed under existing company policy, moral
damages, exemplary damages, and attorney’s fees.18

SICI later manifested that it opted to adopt payroll reinstatement for Arlyn
and Elsie pending appeal which the Labor Arbiter approved on
December 10, 2003.19

On appeal20 by respondents, the National Labor Relations Commission


(NLRC), by Decision21 dated September 27, 2004, reversed the Labor
Arbiter’s decision and declared valid the termination of Arlyn and Elsie’s
services on the grounds of loss of trust and confidence and dishonesty. 22
Arlyn and Elsie’s Joint Motion for Reconsideration 23 having been denied24
by the NLRC, Arlyn filed a Petition for Certiorari and Prohibition with the
Court of Appeals which, by Decision 25 dated August 25, 2005, it denied.

Hence, Arlyn’s present Petition for Review on Certiorari

ISSUE: Whether or not petitioner as an ordinary rank-and-file employee


that she could be dismissed for loss of trust and confidence

RULING:Arlyn’s claim that she is an ordinary rank-and-file employee,


hence, she cannot be dismissed for loss of trust and confidence does not
lie. The observation of the Court of Appeals that "[h]er work is of such
nature as to require a substantial amount of trust and confidence on the
part of x x x her employer"32 is well-taken in light of her following functions,
as enumerated by the NLRC:1. Batches, collates and encode[s] policies,
endorsements and official receipts;2. Generates printed production,
collection, statistical and receivable reports for submission to the Head
Office;3. Reconciles and finalizes production and collection reports; 4.
Maintains the computer hardware and software; and5. Performs other
related functions as may be assigned to her by her superior from time to
time, 33which functions "required the use of judgment and discretion."34

Arlyn of course incorrectly assumes that mere rank-and-file employees


cannot be dismissed on the ground of loss of confidence. Jurisprudence
holds otherwise albeit it requires "a higher proof of involvement" in the
questioned acts.

As a general rule, employers are allowed a wide latitude of discretion in


terminating the employment of managerial personnel or those who, while
not of similar rank, perform functions which by their nature require the
employer’s full trust and confidence. Proof beyond reasonable doubt is
not required. It is sufficient that there is some basis for loss of confidence,
such as when the employer has reasonable ground to believe that the
employee concerned is responsible for the purported misconduct, and
the nature of his participation therein renders him unworthy of the trust
and confidence demanded by his position. This must be distinguished
from the case of ordinary rank-and-file employees, whose termination on
the basis of these same grounds requires a higher proof of involvement in
the events in question; mere uncorroborated assertions and accusations
by the employer will not suffice.35 Even assuming that Arlyn may be
considered a rank and file employee, sufficient evidence of her
involvement in the dishonest scheme of SICI’s accountant and cashier
who were also charged and found guilty exists. Not only was her
participation established by the internal audit conducted; the cashier
identified her as part of the scheme,36 and she herself admitted her
involvement. Her claim that she merely received money from the cashier
and the accountant without knowledge of its illegal source 37 is
contradicted by her subsequent statement of January 7, 2003 submitted
to the HRDD owning up to having participated in the scheme.38

But even assuming further that Arlyn may not be dismissed for loss of
confidence, she can, on the ground of fraud or betrayal of trust, following
Article 282 of the Labor Code which provides that:An employer may
terminate an employee for any of the following causes:(c) Fraudor willful
breach by the employee of the trust reposed in him by his employer or
duly authorized representative;(e) Other causes analogous to the
foregoing.39

Arlyn’s argument that "Even granting that there was withdrawal from the
[Branch Head’s] commissions, [SICI] was not even prejudiced financially
[and] its income was not diminished [as the withdrawn amounts were not]
diverted from its coffers"40 fails. Etcuban, Jr. v. Sulpicio Lines, Inc.41 instructs
that: "x x x Whether or not the respondent was financially prejudiced is
immaterial. Also, what matters is not the amount involved, be it paltry or
gargantuan; rather the fraudulent scheme in which the petitioner was
involved, which constitutes a clear betrayal of trust and confidence." x x
x42 (Underscoring supplied)

As for the propriety of dismissal as a penalty in light of Arlyn’s eight years


of service during which, so she claims, she committed no infraction, the
doctrines established in Salvador v. Philippine Mining Service Corp., 45 to
wit: To be sure, length of service is taken into consideration in imposing
the penalty to be meted an erring employee. However, the case at bar
involves dishonesty and pilferage by petitioner which resulted in
respondent’s loss of confidence in him. Unlike other just causes for
dismissal, trust in an employee, once lost is difficult, if not impossible, to
regain.46 (Underscoring supplied)and in Flores v. NLRC, 47 to wit:The fact
that petitioner worked for private respondent for twenty-one (21) years, if
it is to be considered at all, should be taken against him. The infraction
that he committed, vis-à-vis his long years of service with the company,
reflects a regrettable lack of loyalty. Loyalty that he should have
strengthened instead of betrayed. If an employee’s length of service is to
be regarded as a justifying circumstance in moderating the penalty of
dismissal, it will actually become a prize for disloyalty, perverting the
meaning of social justice and undermining the efforts of labor to cleanse
its ranks of all undesirables.48

TIRAZONA VS CA, 548 SCRA 560 (2008)

FACTS: Private respondent Philippine EDS-Techno Services Inc. (PET) is a


corporation duly registered under Philippine laws and is engaged in the
business of designing automotive wiring harnesses for automobile
manufacturers. Private respondents Ken Kubota, Mamoru Ono and
Junichi Hirose are all Japanese nationals, the first being the President and
the latter two being the directors of PET.
On 21 July 1999, PET employed Ma. Wenelita S. Tirazona (Tirazona) as
Administrative Manager. Being the top-ranking Filipino Manager, she
acted as the liaison between the Japanese management and the
Filipino staff.

On 15 January 2002, Fe Balonzo, a rank-and-file employee, wrote a letter5


that was addressed to nobody in particular, but was later acquired by PET
management. In her letter, Balonzo complained that Tirazona humiliated
her while she was reporting back to work after recuperating from a bout
of tuberculosis. Balonzo explained that Tirazona insinuated, in a manner
loud enough to be heard from the outside, that Balonzo still had the
disease. This allegedly occurred despite Balonzo’s possession of a
medical clearance that proved her fitness to return to work. Balonzo thus
requested that the necessary action be undertaken to address the said
incident.

Upon receiving the letter, the PET management directed Tirazona to file
her comment. Tirazona replied accordingly in a letter 6 wherein she
denied the accusations against her. Tirazona stated that her only
intention was to orient Balonzo about the latter’s rights as a sick
employee, i.e., that under the law, if the latter planned to resign, the
company can give her separation pay. Tirazona likewise asked for an
independent investigation and threatened to file a libel case against
Balonzo for allegedly trying to destroy her reputation and credibility.PET
director Ono sent a memorandum to Tirazona reminding her to be more
circumspect in handling the incident or situation like this. However,
Tirazona treated as an affront to ther honor and dignity. This, instead of
seeking a dialogue with respondent on her felt grievance, petitioner
through her lawyer send questioned demand letter to respondent stating
that the act of petitioner bared animosity in the company and even
demanding claims of 2 million. Because of Tirazona’s obstinate demand
for compensation, PET sent her a Notice of Charge,9 which informed her
that they were considering her termination from employment by reason
of serious misconduct and breach of trust. According to the
management, they found her letter libelous, since it falsely accused the
company of finding her guilty of the charges of Balonzo and depriving
her of due process. The only issue for consideration was Tirazona’s "ill-
advised response to the Management’s disposition to the Fe Balonzo
incident," for which an administrative hearing was scheduled on 4 April
2002.

On 10 April 2002, Tirazona and her counsel did not appear at the
administrative hearing. The PET management informed them through a
memorandum14 dated 12 April 2002 that the hearing was carried out
despite their absence. Nevertheless, Tirazona was granted a final chance
to submit a supplemental written explanation or additional documents to
substantiate her claims.

After finding the explanations unsatisfactory, PET sent Tirazona a Notice of


Termination,16 which found her guilty of serious misconduct and breach of
trust because of her demand against the company and her invasion of
PET’s right to privileged communication.Tirazona then instituted with the
NLRC a complaint for illegal dismissal, non-payment of salaries, and
damages against PET, docketed as NLRC-CA No. 034872-03.In the
Decision17 dated 22 January 2003, Labor Arbiter Veneranda C. Guerrero
ruled in favor of Tirazona, holding that the latter’s termination from
employment was illegal.The Arbiter declared that there was no breach of
trust when Tirazona sent the demand letter, as the same was against Ono
in his personal capacity, not against the company. The decision also
ruled that PET failed to discharge the burden of proving that the alleged
breach of trust was fraudulent and willful, and that the company was
careless in handling its communications. The Arbiter further stated that
Tirazona was deprived of her right to due process when she was denied a
fair hearing.

On appeal by PET, the NLRC reversed the rulings of the Labor Arbiter: It
ruled that Tirazona’s demand letter addressed to Ono constituted a just
cause for dismissal, as the same was "an openly hostile act" by a high-
ranking managerial employee against the company.19 The NLRC likewise
found that PET complied with the notice and hearing requirements of due
process, inasmuch as Tirazona’s demand for a special panel was without
any legal basis. Furthermore, petitioner breached the company’s trust
when she read the confidential legal opinion of PET’s counsel without
permission.

ISSUE: Whether or not there Tirazona was legally dismissed from


employment

RULING: Yes

On the matter of Tirazona’s demand letter, this Court is bound by the


following findings of the Court of Appeals:

Clearly, petitioner Tirazona’s letter to respondent Ono dated 27 February


2002, as DIRECTOR of PET was addressed to an officer and representative
of the corporation. The accusations in the aforesaid demand letter were
directed against respondent Ono’s official act as a representative of
respondent PET. Suffice it to stress, an attack on the integrity of his (Ono)
corporate act is necessarily aimed at respondent PET because a
corporation can only act through its officers, agents and representatives.

In fine, the confluence of events and circumstances surrounding the


petitioner Tirazona’s actions or omissions affecting her employer’s rights
and interest, would undoubtedly show that she is no longer worthy of
being a recipient of the trust and confidence of her employer. x x x.44

Likewise conclusive upon this Court is the Court of Appeals’


pronouncement that Tirazona is in fact a managerial employee, to
wit:The records would indubitably show that it is only now that petitioner
Tirazona is asserting that she is not a managerial employee of respondent
PET. From the very start, her dismissal was premised on the fact that she is
a managerial and confidential employee, and she never denied that
fact. It was never an issue at all before the Labor Arbiter and the public
respondent NLRC. Therefore, she is estopped to claim now that she is [just
a] rank and file employee of respondent PET, especially that she herself
admitted in her pleading that she is a managerial employee:If the
respondent Company has to protect Respondent Mamoru Ono, the
Complainant [petitioner] has also the right to be protected from the
baseless accusations of a Rank and File Employee for she [petitioner] is a
part of the management like Mr. Mamoru Ono"

As correctly held by the NLRC and the Court of Appeals, Tirazona’s


stance is without any legal basis. On the contrary, this Court’s ruling in
Foster Parents Plan International/Bicol v. Demetriou48 is controlling:

The right to dismiss or otherwise impose disciplinary sanctions upon an


employee for just and valid cause, pertains in the first place to the
employer, as well as the authority to determine the existence of said
cause in accordance with the norms of due process. In the very nature of
things, any investigation by the employer of any alleged cause for
disciplinary punishment of an employee will have to be conducted by
the employer himself or his duly designated representative; and the
investigation cannot be thwarted or nullified by arguing that it is the
employer who is accuser, prosecutor and judge at the same time. x x x Of
course, the decision of the employer meting out sanctions against an
employee and the evidentiary and procedural bases thereof may
subsequently be passed upon by the corresponding labor arbiter (and
the NLRC on appeal) upon the filing by the aggrieved employee of the
appropriate complaint.
As a final plea for her case, Tirazona asserts that her dismissal from
employment was too harsh and arbitrary a penalty to mete out for
whatever violation that she has committed, if indeed there was
one.Tirazona ought to bear in mind this Court’s pronouncement in Metro
Drug Corporation v. NLRC50 that:

When an employee accepts a promotion to a managerial position or to


an office requiring full trust and confidence, she gives up some of the rigid
guaranties available to ordinary workers. Infractions which if committed
by others would be overlooked or condoned or penalties mitigated may
be visited with more severe disciplinary action. A company’s resort to acts
of self-defense would be more easily justified. x x x.

Tirazona, in this case, has given PET more than enough reasons to distrust
her. The arrogance and hostility she has shown towards the company
and her stubborn, uncompromising stance in almost all instances justify
the company’s termination of her employment. Moreover, Tirazona’s
reading of what was supposed to be a confidential letter between the
counsel and directors of the PET, even if it concerns her, only further
supports her employer’s view that she cannot be trusted. In fine, the Court
cannot fault the actions of PET in dismissing petitioner.

WHEREFORE, premises considered, the instant petition is hereby DENIED for


lack of merit and the Decision of the Court of Appeals dated 24 May 2005
is hereby AFFIRMED. Costs against the petitioner.
Eric Dela Cruz V. Coca-Cola Bottlers, G.R. 180465, 31 July 2009

FACTS:
Raymund Sales (Sales), a salesman of Coca-Cola Bottlers Phils., Inc. (respondent),
figured in a motor vehicle accident while driving respondent’s motor vehicle
which he was then not authorized to use.
He was hospitalized and was found to have been under the influence of liquor at
the time of the accident. The police blotter of the incident indeed indicates that
Sales was under the influence of liquor.
Respondent soon discovered that Sales’ co-employees (petitioners) secured
apolice report and a medical certificate which omitted the statement that Sales
was under the influence of liquor, to which they all denied. However, further
investigation by the respondent showed that the petitioners conspired to have an
"altered report" prepared to make it appear that Sales was not under the
influence of liquor. Petitioners were thereupon dismissed from employment.
Labor Arbiter: Petitioners were illegally dismissed.
NLRC: Affirmed
CA: Petitioners were validly dismissed

ISSUE: Whether or notpetitionerswere validly terminated

RULING:Yes
An award of back wages and separation pay is justified only if there is a finding of
illegal dismissal. Since petitioners were supervisory employees and were thus
covered by the trust and confidence rule, the Court of Appeals correctly
overturned the ruling of the NLRC and the Labor Arbiter.
Petitioners contend, however, that for loss of trust and confidence to be a ground
for termination of employment, it must be willful and must be connected with the
employee’s work.
Indeed, by obtaining an altered police report and medical certificate, petitioners
deliberately attempted to cover up the fact that Sales was under the influence of
liquor at the time the accident took place. In so doing, they committed acts
inimical to respondent’s interests. They thus committed a work-related willful
breach of the trust and confidence reposed in them.
Bebina G. Salvaloza vs. National Labor Relations Commission, Pacific Security
Agency, Inc., and Angel Quizon, G.R. No. 182086, 24 November 2010

FACTS:
Petitioner Gregorio G. Salvaloza(Gregorio) filed a complaint for illegal dismissal
against respondent Gulf Pacific Security Agency, Inc. (Gulf Pacific). He alleged
that he was employed by Gulf Pacific as a security guard. He reported daily to
Gulf Pacific, waiting for his new assignment, but he was not given any because
there was no position available for him.
Gulf Pacific and private respondent Angel Quizon (Quizon), the owner and
manager of the agency, denied Gregorio’s allegations. He had been relieved
several times from his assignments for various reasons or had been on Absence
Without Leave (AWOL). Also, when Gregorio wanted to be posted, he was told by
Gulf Pacific to first renew and update his license as a security guard. Instead of
reporting back to work, Gregorio filed his complaint.

1STISSUE: Whether or not the petitioner was inefficient at work whenhe failed to
renew his security guard license despite constant reminders to do so.

RULING: Yes
It is settled that, in labor cases, the employer has the burden of proving that the
employee was not dismissed, or, if dismissed, that the dismissal was not illegal.
Failure to discharge this burden would be tantamount to an unjustified and illegal
dismissal. Gregorio contends that Gulf Pacific failed to discharge this burden
when they claimed that Gregorio’s employment was severed for his failure to
renew his security guard license, for his alleged inefficiency at work, and for his
submission of a spurious security guard license.
As per law, a security guard has the personal responsibility to obtain his license.
Notwithstanding the practice of some security agencies to procure the licenses of
their security guards for a fee, it remains the personal obligation of a security
guard to ensure that he has a valid and subsisting license to be qualified and
available for an assignment.
Thus, when Gregorio was directed him to complete his 201 file requirements, it
meant that he had to submit each and every document to show his qualifications
to work as a security guard, most important of which is his security guard license.
Thus, his excuse that he was not informed that he already had an expired license
and had to renew the same cannot be sustained.

2nd ISSUE:Whether or not Gregorio was constructively dismissed by Gulf Pacific

RULING: Yes
It should be pointed out that, per his service record, Gregorio was thrice put on
"floating status" by Gulf Pacific. Of the three instances when Gregorio was
temporarily "off-detailed," we find that the last two already ripened into
constructive dismissal.
While we acknowledge that Gregorio’s service record shows that his performance
as a security guard was below par, we join the LA in his finding that Gulf Pacific
neverissued any memo citing him for the alleged repeated errors, inefficiency,
and poor performance while on duty, and instead continued to assign him to
various posts. This amounts to condonation by Gulf Pacific of whatever infractions
Gregorio may have committed.
Torreda vs. Toshiba Info Equip., 515 SCRA 133 [2007]

FACTS:
Jeffrey O. Torreda was employed by Toshiba Information Equipment (Phils.), Inc.
He was mainly responsible for payroll processing and management, and for the
bookkeeping of T&P Properties, Inc. Thereafter, he was employed on a regular
basis as finance accountantunder the Finance and Accounting Department,
headed by Kazuo Kobayashi, Vice-President, and Teresita Sepulveda, Finance
Manager.
Sepulveda ordered Torreda and his other co-employees to prepare petty cash
vouchers in their names. The sums covered by the vouchers were received by
Sepulveda for her own personal use. This was reported to the HR by Tanaka, and
restrictions were imposed on Sepulveda’s authority to approve petty cash
vouchers.
Thereafter, Sepulveda opened Torreda’s personal computer and read his Lotus
Notes mail and other personal files, specifically the report he had sent to Tanaka
about her. She reprimanded Torreda, who in turn reported the incident to Tanaka.
Sepulveda received complaints regarding payroll kept in Torreda’s drawer. As
Torreda failed to process the claims before taking a leave of absence,
Sepulvedahad the drawer forcibly opened. P200.00 was allegedly lost by Torreda,
and he accused Sepulveda of stealing it by calling her a robber (through public
email).
Torreda was therefore terminated for grave slander, which under the Employee
Handbook is punishable by dismissal.
LA: The dismissal from employment was unjustified. Torreda was harassed by
Sepulveda because of his exposé of irregularities she had committed. The
opening of his drawer formed part of her harassment tactics. Thus, Torreda had all
the right to demand an explanation for the forcible opening of his computer files
and drawer which resulted in the loss of some amount of money.
NLRC: Reversed. He committed a serious misconduct when he accused his
immediate superior of stealing ₱200.00 and calling her a robber (through an e-
mail message), without any evidence at all, and forwarding copies to the other
officers of the company.
CA: Affirmed NLRC. He committed grave slander when he concocted the charge
of theft against Sepulveda, the penalty for which, under the Employee’s
Handbook, is dismissal.

ISSUE: Whether or not Torreda’s dismissal from employment was legal

RULING:
Yes.
The CA correctly affirmed the NLRC Resolution ordering the Labor Arbiter to
dismiss petitioner’s complaint. However, the appellate court erred in ruling that
petitioner committed grave slander against Sepulveda and in applying the
Employee’s Handbook as basis for his dismissal.
The false attribution by the petitioner of robbery (theft) against Sepulveda was
made in writing; patently then, petitioner committed libel, not grave slander
against Sepulveda.
There is abundant evidence on record showing that petitioner committed libel
against his immediate superior, Sepulveda, an act constituting serious misconduct
which warrants the dismissal from employment.
Indeed, an employee may be dismissed from employment for acts punishable by
dismissal under Article 282(a) of the Labor Code, which reads:
Article 282. Termination by employer. – An employer may terminate an
employment for any of the following causes: (a) Serious misconduct or willful
disobedience by the employee of the lawful orders of his employer or
representative in connection with his work; x x x
Petitioner maliciously and publicly imputed on Sepulveda the crime of robbery of
P200.00. He knew that it was Delos Santos who opened his drawer and not
Sepulveda.Thus, by his own admission, petitioner was well aware that the robbery
charge againstSepulveda was a concoction, a mere fabrication with the sole
purpose of retaliating againstSepulveda’s previous acts.

IMASEN PHILIPPINE MANUFACTURING CORPORATION, Petitioner,vs.RAMONCHITO T.


ALCON and JOANN S. PAPA, Respondents.

G.R. No. 194884 October 22, 2014

FACTS:

Petitioner Imasen Philippine Manufacturing Corporation is a domestic


corporation engaged in the manufacture of auto seat-recliners and slide-
adjusters. It hired the respondents as manual welders in 2001.

On October 5, 2002, the respondents reported for work on the second


shift – from 8:00 pm to 5:00 am of the following day. At around 12:40 am, Cyrus A.
Altiche, Imasen’s security guard on duty, went to patrol and inspect the
production plant’s premises. When Altiche reached Imasen’s Press Area, he heard
the sound of a running industrial fan. Intending to turn the fan off, he followed the
sound that led him to the plant’s “Tool and Die” section.

At the “Tool and Die” section, Altiche saw the respondents having sexual
intercourse on the floor, using a piece of carton as mattress. Altiche immediately
went back to the guard house and relayed what he saw to Danilo S. Ogana,
another security guard on duty.

Respondent’s defense: they claimed that they were merely sleeping in


the “Tool and Die” section at the time of the incident. They also claimed that
other employees were near the area, making the commission of the act charged
impossible.
Both LA and NLRC held that the dismissal was valid. CA however nullified
NLRC’s decision and held that sexual intercourse inside company premises is not
serious misconduct.

ISSUE:

Whether or not the respondents’ infraction – engaging in sexual


intercourse inside company premises during work hours – amounts to serious
misconduct justifying their dismissal.

RULING:

YES. Sexual acts and intimacies between two consenting adults belong,
as a principled ideal, to the realm of purely private relations. Whether aroused by
lust or inflamed by sincere affection, sexual acts should be carried out at such
place, time and circumstance that, by the generally accepted norms of conduct,
will not offend public decency nor disturb the generally held or accepted social
morals. Under these parameters, sexual acts between two consenting adults do
not have a place in the work environment.

Indisputably, the respondents engaged in sexual intercourse inside


company premises and during work hours. These circumstances, by themselves,
are already punishable misconduct. Added to these considerations, however, is
the implication that the respondents did not only disregard company rules but
flaunted their disregard in a manner that could reflect adversely on the status of
ethics and morality in the company.

Additionally, the respondents engaged in sexual intercourse in an area


where co-employees or other company personnel have ready and available
access. The respondents likewise committed their act at a time when the
employees were expected to be and had, in fact, been at their respective posts,
and when they themselves were supposed to be, as all other employees had in
fact been, working.

The Court also considered the respondents’ misconduct to be of grave


and aggravated character so that the company was justified in imposing the
highest penalty available ― dismissal.

Their infraction transgressed the bounds of socially and morally


accepted human public behavior, and at the same time showed brazen
disregard for the respect that their employer expected of them as employees. By
their misconduct, the respondents, in effect, issued an open invitation for others to
commit the same infraction, with like disregard for their employer’s rules, for the
respect owed to their employer, and for their co-employees’ sensitivities.
BILLY M. REALDA,Petitioner,vs.NEW AGE GRAPHICS, INC. and JULIAN I. MIRASOL,
JR.,Respondents.
G.R. No. 192190, April 25, 2012,

FACTS:

Respondent Julian Mirasol’s business is a printing press whose production


schedule is sometimes flexible and varying. It is only reasonable that workers are
sometimes asked to render overtime work in order to meet production deadlines.

Petitioner Billy Realda, who was the former machine operator of respondent
New Age Graphics Inc.(Graphics, Inc.), was asked to render overtime work but he
refused to do so despite the "rush" orders of customers and petitioner’s need to
meet its deadlines set by the former. In fact, he reneged on his promise to do the
same, after being issued an Overtime Slip Form. He knew that he was going to be
unavailable for work on the following day, but instead of trying to finish his work
before that date by rendering overtime, due to the "rush" in meeting the
deadlines, he opted to forego with the same, and thereby rejecting the order of
petitioner.

Petitioner is also accused of insubordination for the reason that he


stubbornly refused to follow the orders of his General Manager to show the latter
and check on the computer using the CMYK guide, whether the colors he is
running in his printing machine are correct. After initially following the said order,
and confirming that the first color, cyan, running in the machine was correct, he
failed to observe the same procedure on the second color magenta and did not
even bother to remedy it after it was pointed out by the Computer Graphic Artist
supervising him.

Respondent further alleges habitual tardiness on the part of petitioner for


which he received a warning notice in April and May 2004. For the month of
January and February 2004 alone, he reported late for work 23 times, and just prior
to his suspension, he was yet again late for 6 times. The Daily Time Records of
petitioner contained the entries which were personally written by him.

The LA found that the petitioner was illegally dismissed. The NLRC affirmed
the LA.

The CA reversed the NLRC. It ruled that the petitioner’s unjustified refusal to render
overtime work, unexplained failure to observe prescribed work standards, habitual
tardiness and chronic absenteeism despite warning and non-compliance with the
directive for him to explain his numerous unauthorized absences constitute
sufficient grounds for his termination. It found that private respondent should be
dismissed on the ground of willful disobedience of the warning and memoranda
issued by petitioner. Nonetheless, while the CA recognized the existence of just
causes for petitioner’s dismissal, it found the petitioner entitled to nominal
damages in the amount of
₱5,000.00 due to Graphics, Inc.’s failure to observe the procedural requirements of
due process.

ISSUE:

Whether or not the petitioner was validly dismissed on the grounds of willful
disobedience.

RULING:

YES. The petitioner’s arbitrary defiance to Graphics, Inc.’s order for him to
render overtime work constitutes wilful disobedience. Taking this in conjunction
with his inclination to absent himself and to report late for work despite being
previously penalized, the petitioner is indeed defiant of thelawful orders and the
reasonable work standards prescribed by his employer.

For wilful disobedience to be a valid cause for dismissal, these two elements
must concur: (1) the employee’s assailed conduct must have been wilful, that is,
characterized by a wrongful and perverse attitude; and (2) the order violated
must have been reasonable, lawful, made known to the employee, and must
pertain to the duties which he had been engaged to discharge.

In the present case, there is no question that petitioners’ order for


respondent to render overtime service to meet a production deadline complies
with the second requisite. Art. 89 of the Labor Code empowers the employer to
legally compel his employees to perform overtime work against their will to
prevent serious loss or damage. The petitioner’s business is a printing press whose
production schedule is sometimes flexible and varying. It is only reasonable that
workers are sometimes asked to render overtime work in order to meet production
deadlines. In this case, the petitionerexhibited willful disobedience to a
reasonable order from his employer.

The petitioner’s failure to observe Graphics, Inc.’s work standards constitutes


inefficiency that is a valid cause for dismissal. Failure to observe prescribed
standards of work, or to fulfilreasonable work assignments due to inefficiency may
constitute just cause for dismissal. Such inefficiency is understood to mean failure
to attain work goals or work quotas, either by failing to complete the same within
the alloted reasonable period, or by producing unsatisfactory results. As the
operator of Graphics, Inc.’s printer, he is mandated to check whether the colors
that would be printed are in accordance with the client’s specifications and for
him to do so, he must consult the General Manager and the color guide used by
Graphics, Inc. before making a full run. Unfortunately, he failed to observe this
simple procedure and proceeded to print without making sure that the colors
were at par with the client’s demands. This resulted to delays in the delivery of
output, client dissatisfaction, and additional costs on Graphics, Inc.’s part.

Undoubtedly, Graphics, Inc. complied with the substantive requirements of due


process in effecting employee dismissal. However, the same cannot be said
insofar as the procedural requirements are concerned. Graphics, Inc. failed to
afford the petitioner with a reasonable opportunity to be heard and defend itself.
An administrative hearing set on the same day that the petitionerreceived the
memorandum and the 24-hour period for him to submit a written explanation are
far from being reasonable. Furthermore, there is no indication that Graphics, Inc.
issued a second notice, informing the petitioner of his dismissal. Graphics, Inc.
decided to terminate the petitioner’ semployment after he ceased reporting for
work from the time he received the memorandum requiring him to explain, and
subsequent to his failure to submit a written explanation. However, there is nothing
on record showing that Graphics, Inc. placed its decision to dismiss the petitioner
in writing and that a copy thereof was sent to the latter. Thus, Graphics, Inc was
ordered to paypetitioner nominal damages in the amount of P30,000.00 pursuant
to Agabon vs. NLRC.

INTERNATIONAL SCHOOL MANILA AND/OR BRIAN McCAULEY, Petitioners,


vs. INTERNATIONAL SCHOOL ALLIANCE OF EDUCATORS (ISAE) AND MEMBERS
REPRESENTED BY RAQUEL DAVID CHING, PRESIDENT, EVANGELINE SANTOS, JOSELYN
RUCIO AND METHELYN FILLER, Respondents.

G.R. No. 167286 February 5, 2014

FACTS:

Santos was first hired by the School in 1978 as a full-time Spanish language
teacher. In April 1992, Santos filed for and was granted a leave of absence for the
school year 1992-1993. She came back from her leave of absence sometime in
August 1993. Upon Santos’s return to the School, only one class of Spanish was
available for her to teach. Thus, for the school year 1993-1994, Santos agreed to
teach one class of Spanish and four other classes of Filipino that were left behind
by a retired teacher.

On October 26, 1993, Dale Hill, then Assistant Principal, observed Santos’s
Filipino II class. In the Classroom Standards Evaluation Form,7 Hill remarked that the
lesson plan that Santos provided "was written with little detail given." Santos was
also noted as needing improvement in the following criteria: (1) uses effective
questioning techniques; (2) is punctual and time efficient; (3) states and enforces
academic and classroom behavior expectations in a positive manner; and (4)
reinforces appropriate behavior. Hill also stated that Santos’s management of the
class left much to be desired. Hill added that "[t]he beginning and the end of the
class were poorly structured with students both coming late and leaving early with
no apparent expectations to the contrary."

In the meantime, for the school year 1994-1995, Santos agreed to teach five
classes of Filipino. On November 7, 1994, Santos also informed the School of her
assignment preference for the incoming school year 1995-1996. In a
memorandum/formsubmitted to the Personnel Department of the School, Santos
indicated that she did not prefer a change of teaching assignment. In the school
year 1995-1996, Santos again taught five classes of Filipino.

On February 1, 1996, then Assistant Principal Peter Loy observed a Filipino


IBS1 class of Santos. In the Classroom Standards Evaluation Formhe completed
thereafter, Loy noted that Santos needed improvement on the following aspects:
(1) has daily lesson plans written out; (2) incorporates a variety of activities,
resources and teaching strategies into the lesson; (3) plans for the entire
instructional period; (4) provides an instructional sequence which is clear and
logical, leading to stated objectives; (5) uses effective questioning techniques; (6)
develops rapport with and between students by creating a supportive
environment; (7) is punctual and time efficient; and (8) reinforces appropriate
behavior. Loy also observed that Santos did not meet the minimum standards in
these areas of concern: (1) has clearly defined lesson objectives that tie into unit
objectives as well as into the school curriculum; and (2) states and enforces
academic and classroom behavior expectations in a positive manner.

Subsequently, on April 10, 1997, McCauley sent a letterto Santos directing


her to explain in writing why her employment from the School should not be
terminated because of her failure to meet the criteria for improvement set out in
her Professional Growth Plan and her substandard performance as a teacher.

In her reply letterdated April 14, 1997, Santos blamed the School for her
predicament. She said that, in the last few years, she had been forced to teach
Filipino, a subject which she had no preparation for. The School allegedly made
this happen against her objections and despite the fact that she had no training
in FilIn a letter dated May 29, 1997, McCauley informed Santos that he was
adopting the recommendation of the investigation committee that Santos’s
employment from the School cannot be continued. According to McCauley, the
committee found that the numerous consultations of Santos with her supervisors
for the last three school years did not result in any appreciable improvement on
her part. McCauley pointed out that Santos categorically indicated that she
preferred to continue teaching Filipino for the school years 1994-1995 and 1995-
1996. Given that Santos was duly licensed to teach Filipino, McCauley stated that
the committee could not accept her claim that she was ill-equipped to teach the
language. McCauley then told Santos that her employment with the School
would cease effective June 7, 1997.ipino linguistics and literature. Santos also
asked for clarification on why she was being asked to explain and the reasons
therefor

ISSUE:

Whether or not Santos dismissal was valid on the grounds of gross inefficiency?

RULING:

Yes.
Article 282 of the Labor Code provides:

ART. 282. Termination by employer. – An employer may terminate an employment


for any of the following causes:

(a) Serious misconduct or willful disobedience by the employee of the


lawful orders of his employer or representative in connection with his work;

(b) Gross and habitual neglect by the employee of his duties;

(c) Fraud or willful breach by the employee of the trust reposed in him by his
employer or duly authorized representative;

(d) Commission of a crime or offense by the employee against the person


of his employer or any immediate member of his family or his duly
authorized representative; and

(e) Other causes analogous to the foregoing.

The Supreme Court held that Gross inefficiency falls within the purview of
"other causes analogous to the foregoing," and constitutes, therefore, just cause
to terminate an employee under Article 282 of the Labor Code. One is analogous
to another if it is susceptible of comparison with the latter either in general or in
some specific detail; or has a close relationship with the latter. "Gross inefficiency"
is closely related to "gross neglect," for both involve specific acts of omission on
the part of the employee resulting in damage to the employer or to his business. In
Buiser vs. Leogardo, this Court ruled that failure to observe prescribed standards of
work, or to fulfill reasonable work assignments due to inefficiency may constitute
just cause for dismissal. (Emphases ours; citations omitted.)

Viewed in light of the above doctrines, the Court is not convinced that the
actuations of Santos complained of by the petitioners constituted gross and
habitual neglect of her duties.

From the very beginning of her tenure as a teacher of the Filipino language,
the recurring problem observed of Santos was that her lesson plans lacked details
and coherent correlation to each other, to the course, and to the curriculum,
which in turn affected how lessons and instructions were conveyed to the
students. After Santos was placed in a Professional Growth Plan on March 29,
1996, petitioners observed a noticeable improvement on her part. In his memo
dated May 24, 1996, then Assistant Principal Loy even stated that Santos’s
improvement was a result of her positive attitude in approaching her growth plan.
Unfortunately, though, Santos could not sustain this progress. Not long after, the
School administrators were again admonishing Santos for her vague lesson plans
that lacked specifics.

What can be gathered from a thorough review of the records of this case is
that the inadequacies of Santos as a teacher did not stem from a reckless
disregard of the welfare of her students or of the issues raised by the School
regarding her teaching. Far from being tainted with bad faith, Santos’s failings
appeared to have resulted from her lack of necessary skills, in-depth knowledge,
and expertise to teach the Filipino language at the standards required of her by
the School.

Be that as it may, the Court finds that the petitioners had sufficiently proved
the charge of gross inefficiency, which warranted the dismissal of Santos from the
School.

SCHOOL OF THE HOLY SPIRIT OF QUEZON CITY and/or SR. CRISPINA A. TOLENTINO,
S.Sp.S., Petitioners,
vs.
CORAZON P. TAGUIAM, Respondent.

G.R. No. 165565 July 14, 2008

FACTS:

Respondent Taguiam was the class adviser of a Grade 5 class of petitioner


school. After obtaining permission from the principal, they were allowed to use the
school swimming pool for their year-endactivity. With this, respondent Taguiam
distributed the parent’s/guardian’s permit forms to the students.

The permit form of student Chiara Mae was unsigned. But because the
mother personally brought her to the school with her packed lunch and swimsuit,
Taguiam concluded that the mother allowed her to join. Before the activity
started, respondent warned the pupils who did not know how to swim to avoid
the deeper area. However, while the pupils were swimming, two of them sneaked
out.Respondent went after them to verify where they were going. Unfortunately,
while respondent was away, Chiara Mae drowned. When respondent returned,
the maintenance man was already administering cardiopulmonary resuscitation
on Chiara Mae. She was still alive when respondent rushed her to the General
Malvar Hospital where she was pronounced dead on arrival.

The petitioner school conducted a clarificatory hearing to which


respondent attended and submittedher Affidavit of Explanation. A month later,
petitioner school dismissed respondent on the ground of gross negligence
resulting to loss of trust and confidence.

ISSUE:

Whether or not respondent’s dismissal on the ground of gross negligence resulting


to loss of trust and confidence was valid.

RULING:
Yes. Under Article 282 of the Labor Code, gross and habitual neglect of
duties is a valid ground for an employer to terminate an employee. Gross
negligence implies a want or absence of or a failure to exercise slight care or
diligence, or the entire absence of care. It evinces a thoughtless disregard of
consequences without exerting any effort to avoid them. Habitual neglect implies
repeated failure to perform one’s duties for a period of time, depending upon the
circumstances.

The SC concluded that respondent had been grossly negligent. First, it is


undisputed that Chiara Mae’s permit form was unsigned. Yet, respondent allowed
her to join the activity because she assumed that Chiara Mae’s mother has
allowed her to join it by personally bringing her to the school with her packed
lunch and swimsuit. Second, it was respondent’s responsibility as Class Adviser to
supervise her class in all activities sanctioned by the school. Thus, she should have
coordinated with the school to ensure that proper safeguards, such as adequate
first aid and sufficient adult personnel, were present during their activity. She
should have been mindful of the fact that with the number of pupils involved, it
would be impossible for her by herself alone to keep an eye on each one of
them.

Fernanandez vs.. Newfield Staff, G.R. No. 165565, July 14, 2008

Facts:
Newfield hired Fernandez as Recruitment manager and Beltran as probationary
Recruitment Specialist. Newfield’s General Manager terminates Petitioners on the
ground that they failed to perform satisfactorily. A week later, Petitioner’s received
return-to-work letters from the General Manager. The letters stated that they did
not report to work without resigning in violation of their employment agreements.
They were directed to report and explain their failure to file resignation letters.
Petitioners filed a complaint for illegal dismissal against respondents.
Fernandez contended that she was able to hire a team leader and 12 agents in 3
weeks but Newfield still found her performance unsatisfactory and told her to file
resignation letter.
Respondents contended that petitioners signed fixed-term employment
agreements where they agreed to perform their tasks for 6 months. They also
agreed to give written notice 45 days in advance if they want to terminate their
employment agreements. But they never complied with their undertakings. Three
weeks after working for Newfield, Fernandez did not report for work. She never
even bothered to communicate with respondents despite the return-to-work
letter. Hence, Newfield declared her absent without official leave and terminated
her employment on the ground of breach of contract. Beltran stopped reporting
2 weeks after she was hired and never bothered to communicate with
respondents despite the return-to-work letter. Respondents claimed that no
evidence shows or even hints that petitioners were forced not to report for work.
Petitioners simply no longer showed up for work.
The Labor Arbiter ruled that Petitioners’ dismissal was illegal. The NLRC affirmed the
Labor Arbiter’s decision and said that it is supported by substantial evidence.

Issue:
1. WON Petitioners were invalidly dismissed
2. WON Petitioners abandoned their jobs

Ruling
1. Yes.

Petitioners argue that for dismissal to be valid there must be a just or


authorized cause and due process must be observed. And that they were
dismissed without any written notice informing them of the cause for their
termination.

Respondents do not deny what happened as stated by the Petitioners.


Respondents merely said that no evidence shows that petitioners were
forced not to report to work. Even respondent’s appeal memorandum
before the NLRC is silent on petitioner’s claim. Respondent’s silence
constitutes an admission that fortifies the truth of Petitioner’s narration.

2. No.

Abandonment is a form of neglect of duty, one of the just causes for an


employer to terminate an employee. For abandonment to exist, two factors
must be present: (1) the failure to report for work or absence without valid or
justifiable reason, and (2) a clear intention to sever the employer-employee
relationship, with the second element as the more determinative factor
being manifested by some overt acts.

Since both factors are not present, Petitioners are not guilty of
abandonment. One, Petitioners were absent because Newfield’s General
Manager fired them. Thus, we cannot fault them for refusing to comply with
the return-to-work letters and responding instead with their demand letters.
Neither can they be accused of being AWOL or of breaching their
employment agreements. Two, Petitioner’s protest of their dismissal by
sending demand letters and filing a complaint for illegal dismissal with
prayer for reinstatement convinces us that petitioners have no intention to
sever the employment relationship.
Sanden Aircon vs. Rosales, G.R. No. 169260, March 23, 2011

Facts:
Sanden employed Rosales as Management Information System (MIS) Department
Secretary. She was promoted as Data Custodian and Coordinator. As such,
Rosales had access to all computer programs and marketing computer data,
including the Delivery Receipt Transaction files of Sanden.

Sanden discovered that the marketing delivery receipt transactions computer files
were missing. Hence, a technical investigation was conducted. On the basis of
the investigation, a letter to Rosales charging her with data sabotage and
absences without leave (AWOL). She was given 24 hours to explain her side.
Rosales denied the allegations.

Rosales received a Notice of Disciplinary Action from Sanden notifying Rosales


that management is terminating Rosaless employment effective upon receipt of
the said communication. The reason cited by Sanden was the loss of trust on her
capability to continue as its Coordinator and Data Custodian. For this reason,
Rosales filed a complaint for illegal dismissal.

The LA rendered a decision finding that there was illegal dismissal. On appeal, the
NLRC dismissed the complaint for lack of merit. Aggrieved, Rosales filed a petition
for certiorari before the CA where the NLRC decision was reversed and set aside.

Issue: WON Rosales was illegally dismissed

Ruling: Yes.
"A breach is willful if it is done intentionally and knowingly without any justifiable
excuse, as distinguished from an act done carelessly, thoughtlessly or
inadvertently."

As firmly entrenched in our jurisprudence, loss of trust and confidence as a just


cause for termination of employment is premised on the fact that an employee
concerned holds a position where greater trust is placed by management and
from whom greater fidelity to duty is correspondingly expected.

The first requisite for dismissal on the ground of loss of trust and confidence is that
the employee concerned must be holding a position of trust and confidence.

In this case, we agree that Rosales, who had immediate access to Sandens
confidential files, papers and documents, held a position of trust and confidence
as Coordinator and Data Custodian of the MIS Department.
The second requisite is that there must be an act that would justify the loss of trust
and confidence. Loss of trust and confidence, to be a valid cause for dismissal,
must be based on a willful breach of trust and founded on clearly established
facts. The basis for the dismissal must be clearly and convincingly established but
proof beyond reasonable doubt is not necessary.

As correctly found by the Labor Arbiter, nowhere in the records can be found
evidence that directly point to complainant as having committed acts of
sabotage. Also, during the administrative investigation, the guilt of complainant-
appellee was based on mere allegations not supported by documentary
evidence nor any factual basis. Even appellants cannot directly pinpoint
appellee as the culprit. They were only thinking of her as the one probably
responsible thereto, considering that when she used the computer, she told the
other users to log out and thereafter, used the computer for 16 minutes, with only
1 minute as usage time. But these allegations would not suffice termination of
employment of appellee. Note that security of tenure is protected by
constitutional mandate.

On the other hand, Rosales was able to provide documentary evidence to show
that Sandens computer system was experiencing some problems even before
May 16, 1997. The entries as reported by the System Administrator clearly show
that the problem of missing data already existed as early as 1995, when Rosales
was still an MIS Secretary and was not yet tasked to back up the Marketing
Delivery Receipt Transaction files.
. Lhuiller vs. Velayo, G.R. No. 198620, November 12, 2014

Facts:
Lhuillier hired respondent as Accounting Clerk. Respondent was served with a
Show Cause Memo by Lhuillier, ordering her to explain why no disciplinary action
should be taken against her for dishonesty, misappropriation, theft or
embezzlement of company. She was placed under preventive suspension while
her case was under investigation.
The charges against respondent were based on the Audit Findings conducted
where the overage amount of P540.00 was not reported immediately to the
supervisor and not recorded at the end of that day.
Respondent admitted that she was not able to report the overage to the
supervisor since the latter was on leave on that day and that the omission or
failure to report immediately the overage was just a simple mistake without intent
to defraud her employer.
After the conduct of a formal investigation and after finding respondent's
explanations without merit, Lhuillier terminated her employment on grounds of
serious misconduct and breach of trust.
The respondent filed a complaint for illegal dismissal, separation pay and other
damages against Lhuillier.
Labor Arbiter ordered the dismissal of the instant complaint for lack of merit.
The LA found that the respondent's termination was valid and based not on a
mere act of simple negligence in the performance of her duties as cashier.
On appeal, the NLRC countermanded the LA, holding that the respondent was
illegally dismissed since the petitioners failed to prove a just cause of serious
misconduct and willful breach of trust

Issue: WON there was illegal dismissal

Ruling: No.
Article 282 of the Labor Code allows an employer to dismiss an employee for
willful breach of trust or loss of confidence. It has been held that a special and
unique employment relationship exists between a corporation and its cashier. Truly,
more than most key positions, that of a cashier calls for utmost trust and
confidence, and it is the breach of this trust that results in an employer’s loss of
confidence in the employee.
Aliling vs. Feliciano, G.R. No. 174893, July 11, 2012

Facts:
Respondent Wide Wide World Express Corporation (WWWEC) offered to employ
petitioner Aliling on as Account Executive (Seafreight Sales). The offer came with
a 6 month probation period condition with this express caveat: “Performance
during probationary period shall be made as basis for confirmation to Regular or
Permanent Status.”
Aliling and WWWEC inked an Employment Contract under the terms of
conversion to regular status shall be determined on the basis of work
performance; and employment services may, at any time, be terminated for just
cause or in accordance with the standards defined at the time of engagement.
A month after assigning Aliling a new company product launch instead of a
seafreight sale assignment, WWWEC emailed AIiling to express dissatisfaction with
his performance and asked Aliling to report to Human Resources to explain his
absence taken without leave.
Aliling responded two days later denying his absence, attaching a copy of his
timesheet. Aliling questioned the withholding of his salary.
Aliling tendered his resignation. While WWWEC took no action on his tender, Aliling
demanded reinstatement and a written apology, claiming he was forced to
resign.
Aliling was informed that his case was still in the process of being evaluated. And
then was informed that the termination of his services was due to his non-
satisfactory performance during his probation period. He was then paid his
outstanding salary.
Aliling filed a complaint for illegal dismissal with the NLRC stating that he was not
informed of the standards to qualify as a regular employee.
Refuting Aliling’s basic posture, WWWEC stated that in the letter offer and
employment contract adverted to, WWWEC and Aliling have signed a letter of ap
pointment containing the terms of engagement.
WWWEC also attached to its Position Paper a memo in which San Mateo asked Ali
ling to explain why he should not be terminated for failure to meet the expected j
ob performance, considering that the load factor was only 0.18% as opposed to t
he allegedlyagreed upon load of 80%. According to WWWEC, Aliling,
instead of explaining himself, simply submitted a resignation letter. The LA
issued a decision declaring that the grounds upon which complainant’s
dismissal was based did not conform not only the standardbut also the complianc
e required under Article 281 of the Labor Code, Necessarily, complainant’s
termination is not justified for failure to comply with the mandate the law
requires.
The Labor Arbiter explained that Aliling cannot be validly terminated for non-
compliance with the quota threshold absent a prior advisory of the reasonable
standards upon which his performance would be evaluated.
Both parties appealed the decision to the NLRC, which affirmed the decision of
the Labor Arbiter. The separate motions for reconsideration were also denied by
the NLRC.
The CA anchored its assailed action on the strength of the following premises:
(a) respondents failed to prove that Aliling’s dismal performance constituted gross
and habitual neglect necessary to justify his dismissal;
(b) not having been informed at thetime of his engagement of the reasonable
standards under which he will qualify as a regular employee; and (c) the strained
relationship existing between the parties argues against the propriety or
reinstatement.

Issue: WON there was illegal dismissal

Ruling: Yes.
Settled is the rule that the findings of the Labor Arbiter, when affirmed by the
NLRC and the Court of Appeals, are binding on the Supreme Court, unless
patentlyerroneous. It is not the function of the Supreme Court to analyze or
weigh all over againthe evidence already considered in the proceedings below. T
he jurisdiction of this Court in a petition for review on certiorari is limited to
reviewing only errors of law, not of fact, unless the factual findings assailed are
not supported by evidence on record or the impugned judgment is based on a m
isapprehension of facts.
Long-established is thedoctrine that findings of fact of quasi-judicial
bodies are accorded respect, even finality,if supported by substantial evidence.
When passed upon and upheld by the CA, theyare binding and conclusive upon
this Court and will not normally be disturbed. Thoughthis doctrine is not
without exceptions, the Court finds that none are applicable to thepresent case

G.R. No. 174893. July 11, 2012.*


FLORDELIZA MARIA REYES-RAYEL, petitioner, vs. PHILIPPINE LUEN THAI HOLDINGS,
CORPORATION/L&T INTERNATIONAL GROUP PHILIPPINES, INC., respondents.

FACTS:

PLTHC hired petitioner as Corporate Human Resources (CHR) Director for


Manufacturing for L&T. In the employment contract, petitioner was tasked
to perform functions in relation to administration, recruitment, benefits,
audit/compliance, policy development/ structure, project plan, and such
other works as may be assigned by her immediate superior, Frank
Sauceda, PLTHC’s Corporate Director for Human Resources.

A Prerequisite Notice from Sauceda was sent to Reyes-Rayel which cited


specific incidents from various instances that have collectively contributed
to the company’s loss of trust and confidence in her. To wit: 1.) Her
pronouncements against the Human Resource Information System (HRIS) or
HR2 Program, crucial to the enhancement of personnel management; 2.)
Her negative attitude towards the Company, its officers and people; 3.) Her
unsatisfactory performance rating, and in the departure of promising
employees who could not work with her.

She was afforded the opportunity to submit her written reply to the
memorandum within 48 hours from its receipt.

Petitioner’s written response explained that her alleged failure to perform


management directives could be attributed to the lack of effective
communication with her superiors due to malfunctioning email system. She
further denied uttering negative comments about the HR2 Program and
denied causing disharmony in her division.

A Termination Notice was served which dismissed petitioner from the


service for loss of confidence on her ability to promote the interests of the
company. Petitioner filed a complaint for illegal dismissal, not afforded due
process, payment of separation pay, among others.

Labor Arbiter: declared petitioner to have been illegally dismissed.


Respondents was directed to reinstate complainant to her former position
and pay her full backwages.

NLRC: found respondents to have sufficiently established the validity of


petitioner’s dismissal on the ground of loss of trust and confidence. Modified
the decision of the LA and respondents to pay complainant 3 months’ worth
of salary

NLRC MR: granted and found that petitioner was not afforded due process
as she was not given the opportunity to refute the charges against her
through an investigation and an appeal at the company level. The dismissal
was illegal and thus reinstated the Labor Arbiter’s Decision with modification
that respondents be ordered to pay petitioner separation pay in lieu of
reinstatement.

CA: found sufficient evidence to support the dismissal of petitioner on the


ground of loss of trust and confidence. It also did not subscribe to petitioner’s
allegation that she was denied due process.

ISSUES:
1. WON there exists a valid ground for petitioner’s termination from
employment
2. WON petitioner was accorded due process

RULING:

1. YES. A managerial personnel who performs functions which by their


nature require the employer’s full trust and confidence. The burden of
proving that the termination was for a valid cause lies on the employer.
Here, respondents were able to overcome this burden as the evidence
presented clearly support the validity of petitioner’s dismissal; various
emails showing that she failed to update Sauceda on the progress of her
important assignments on several occasions, narrations of several
instances which demonstrated petitioner’s notoriously bad temper.
which notably remain uncontroverted and unrefuted, and on two
occasions, gave wrong information regarding issues on leave and
holiday pay which generated confusion among employees in the
computation of salaries and wages.
2. YES. The Prerequisite Notice was free from any ambiguity. The said notice
properly advised petitioner to explain through a written response her
failure to perform in accordance with management directives and it
cited specific incidents from various instances that have collectively
contributed to the company’s loss of trust and confidence in her.

G.R. No. 121314. February 12, 1998.*


EDGE APPAREL, INC., petitioner, vs. NATIONAL LABOR RELATIONS COMMISSION

FACTS:

Pursuing its retrenchment program, Edge Apparel, Inc., dismissed private


respondents Josephine Antipuesto, et al. The subsequent receipt of their
separation pay benefits did not deter Antipuesto, et al., from going through
with their complaint for illegal dismissal which averred that the retrenchment
program was a mere subterfuge used by Edge Apparel to give a semblance
of regularity and validity to the dismissal of the complainants. Edge Apparel
countered that its financial obligations had begun to eat up most of its
capital outlay and resulted in unabated losses.

NLRC: There is basis in the retrenchment of these workers. 27 workers were


assigned to row #8 of the sewing line for simple garments which was phased
out due in fact to the dropping of this particular line of business. The
termination of the 27 retrenched employees is considered a redundancy.
Hence, the complainants, who were already paid the separation pay
equivalent to 1/2 month pay per year of service, are entitled to be paid the
additional separation pay equivalent to 1/2 month pay for every year of
service.

ISSUE:

WON NLRC’s gravely abused its discretion

RULING:

YES. Court agrees with the Solicitor General that here the NLRC has gravely
abused its discretion. For just causes, the employee is not entitled to
payment of separation benefits. For authorized causes or employee found
to be suffering from a disease if his/her continued employment is prohibited
by law or prejudicial to his or his fellow employees health, is entitled to
separation pay.
In exercising its right to retrench employees, the firm may choose to close
all, or a part of, its business to avoid further losses or mitigate expenses. The
fact alone that a mere portion of the business of an employer, not the
whole of it, is shut down does not necessarily remove that measure from the
ambit of the term "retrenchment.” The respondents should only be entitled
to severance compensation equivalent to one-half (1/2) month pay for
every year of service.

G.R. No. 114952. January 29, 1996.*


MAGNOLIA DAIRY PRODUCTS CORPORATION, petitioner, vs. NATIONAL LABOR
RELATIONS COMMISSION

FACTS:

Private respondent applied with Lippercon Services, Inc., a corporation


engaged in providing manpower services, assigned her to petitioner's Tetra
Paster Division as a cleaning aide. she was later on terminated from service
due to petitioner's installation of automated machines.
Respondent instituted a complaint for illegal dismissal and petitioner
averred that it has no employer-employee relationship and dismissal
was prompted by an authorized cause.

LA: ruled that petitioner is the private respondent's employer because


Lippercon Services, Inc., were mere "labor-only" contractors. For its failure to
observe the due process, petitioner was ordered "to pay her backwages
and in lieu of reinstatement,... to pay her separation pay”

NLRC: modified decision and directed respondent's reinstatement and


payment of backwages not exceeding three (3) years

ISSUES:

1. WON an employer-employee relationship exists


2. WON private respondent was not illegally dismissed since the termination
of her employment was due to a cause expressly authorized by the Labor
Code and the absence of notice therefor did not make it so.

RULING:

1. YES. These workers, in performing their works, utilized the premises, tools,
equipments and machineries of respondent Magnolia and not those of
the former. The work being performed by complainant, are directly
related to the day to day operations of respondent Magnolia. Lipercon
was merely an agent of the respondent Magnolia and that the latter
was the real employer. Where "labor only" contracting exists, the status
itself implies or establishes an employer-employee relationship between
the employer and the employees of the "labor- only" contractor.
Petitioner also exercises the power to discipline and suspend private
respondent.
2. NO. The law authorizes an employer to terminate the employment of any
employee due to the installation of labor saving devices but this did not
excuse petitioner from complying with the required written notice to the
employee and to DOLE at least one month before the intended date of
termination. This procedure enables an employee to contest the reality or
good faith character of the asserted ground for the termination of his
services before the DOLE. However, failure to serve written notice does not
make it an illegal dismissal—it merely makes it defective because such
was not tainted with bad faith or arbitrariness and was due
to a valid cause. It is a well-settled rule that employer shall be sanctioned
for non-compliance with the requirements of, or for failure to observe due
process in terminating from service its employee. Under the attendant
facts, P5,000.00 is just and reasonable for indemnification.
NLRC's grant of backwages and order of reinstatement are untenable.
These awards are proper for illegally dismissed employees which
obviously is not the situation in this case and Article 283 of the Labor
Code, an employee removed from service due to the installation of
labor saving devices is entitled to separation pay.

G.R. No. 181719 April 21, 2014


EUGENE S. ARABIT vs. JARDINE PACIFIC FINANCE, INC. (FORMERLY MB FINANCE)

FACTS:

The former regular employees of respondent were also officers and members
of MB Finance
Employees Association-FFW Chapter (the Union), a legitimate labor union
and the sole exclusive bargaining agent. On the claim of financial losses,
Jardine decided to reorganize and implement a redundancy program
among its employees. Petitioners were dismissed and Jardine thereafter
hired contractual employees to undertake the functions these employees
used to perform.

The Union filed a notice of strike with the NCMB, questioning the termination
of employment of the petitioners who were also union officers. The Union
alleged unfair labor practice as well as discrimination in the dismissal of its
officers and members.

Both parties reached an amicable settlement. The petitioners accepted


their redundancy pay without prejudice to their right to question the
legality of their dismissal. Jardine paid a separation package (severance
pay, plus grossed up transportation allowance.)

The petitioners and the Union filed a complaint against Jardine with
the NLRC for illegal dismissal and unfair labor practice.

LA: ruled in the petitioners’ favor. The hiring of contractual employees


directly contradicts redundancy. The company’s action was a
circumvention of the right of the petitioners to security of tenure. Jardine did
not even explain why their respective positions became superfluous.

NLRC: dismissed the appeals and affirmed the LA’s decision.

CA: reversed the LA’s and the NLRC’s rulings. The hiring contractual
employees does not run counter that their positions are already
superfluous— it is a management prerogative and absence of any showing
of malice or arbitrariness, courts must not interfere with the exercise of a
management decision.

ISSUE:

WON the CA correctly rule that the NLRC committed grave abuse of
discretion when it found that Jardine validly terminated the petitioners’
employment because of redundancy

RULING:

YES. It is illogical for Jardine to terminate the petitioners’ employment and


replace them with
contractual employees. The replacement effectively belies Jardine’s claim
that the petitioners’
positions were abolished due to superfluity. The petitioners’ services have
not really become in excess of what Jardine’s business requires.

GUIDELINES IN IMPLEMENTING REDUNDANCY:

Golden Thread Knitting Industries, Inc. v. NLRC


1. less preferred status (e.g. temporary employee);
2. efficiency; and
3. seniority.

The records are bereft of indications that Jardine employed clear criteria
when it decided who among its employees should be removed from their
posts because of redundancy. Jardine’s acts became more suspect
given that the petitioners were all union officers and some of them were
panel members in the scheduled CBA negotiations.

GUIDELINES FOR REDUNDANCY TO BE CHARACTERIZED AS VALIDLY UNDERTAKEN:

Asian Alcohol Corp. v. NLRC

1. written notice served on both the employees and the Department of


Labor and Employment at least one month prior to the intended date of
retrenchment;
2. payment of separation pay equivalent to at least one month pay or at
least one month pay for every year of service, whichever is higher;
3. good faith in abolishing the redundant positions; and
4. fair and reasonable criteria in ascertaining what positions are to be
declared redundant and accordingly abolished.

The last two guidelines are interrelated to ensure good faith in abolishing
redundant positions. Jardine failed to set the required fair and reasonable
criteria in the termination of the petitioners’ employment, leading to the
conclusion that the termination from the service was arbitrary and in bad
faith.

The Court reversed the decision of the Court of Appeals.


G.R. No. 172846 July 24, 2013

MANILA POLO CLUB EMPLOYEES' UNION (MPCEU) FUR-TUCP, Petitioner,


vs.
MANILA POLO CLUB, INC., Respondent.

FACTS:

Petitioner Manila Polo Club Employees Union (MPCEU), which is affiliated with the
Federation of Unions of Rizal (FUR)-TUCP, is a legitimate labor organization duly
registered with the Department of Labor and Employment (DOLE), while
respondent Manila Polo Club, Inc. is a non-profit and proprietary membership
organization which provides recreation and sports facilities to its proprietary
members, their dependents, and guests.

On December 13, 2001, the Board of Directors of respondent unanimously


resolved to completely terminate the entire operations of its Food and Beverage
(F & B) outlets, except the Last Chukker, and award its operations to a qualified
restaurant operator or caterer.

Cited as reasons were as follows:

1. The Food and Beverage (F & B) operations has resulted in yearly losses to
the Club in six (6) out of the last eight (8) years and that these losses are due
mainly to the exceedingly high manpower cost and other management
inefficiencies;

2. Due to the substantial losses incurred by the Club in both F&B operations
and in its recurring operations, the Board and management had instituted
cost and loss-cutting measures;

3. The non-viability of the operations of the Food and Beverage Department


and that its continued operations by the Club will result in substantial losses
that will seriously impair the Club’s financial health and membership
satisfaction;

Subsequently, on March 22, 2002, respondent’s Board approved the


implementation of the retrenchment program of employees who are directly and
indirectly involved with the operations of the F & B outlets and authorized then
General Manager Philippe D. Bartholomi to pay the employees’ separation pay.

On even date, respondent sent notices to the petitioner and the affected
employees (via registered mail) as well as submitted an Establishment Termination
Report to the DOLE. Respondent informed, among others, of the retrenchment of
123 employees in the F & B Division and those whose functions are related to its
operations; the discontinuance of the F & B operations effective March 25, 2002;
the termination of the employment relationship on April 30, 2002; and, the
continued payment of the employees’ salaries despite the directive not to report
to work effective immediately.

Unaware yet of the termination notice sent to them by respondent, the affected
employees of petitioner were surprised when they were prevented from entering
the Club premises as they reported for work on March 25, 2002. They later learned
that the F & B operations of respondent had been awarded to Makati Skyline, Inc.
effective that day. Treating the incident as respondent’s way of terminating union
members under the pretense of retrenchment to prevent losses, petitioner filed a
Step II grievance and requested for an immediate meeting with the
Management. When the Management refused, petitioner filed a Notice of Strike
before the National Conciliation and Mediation Board (NCMB) for illegal dismissal,
violation/non-implementation of the Collective Bargaining Agreement (CBA),
union busting, and other unfair labor practices (ULP). In view of the position of
respondent not to refer the issues to a voluntary arbitrator or to the Secretary of
DOLE, petitioner withdrew the notice on April 9, 2002 and resolved to exhaust all
remedies at the enterprise level.

Later, on May 10, 2002, petitioner again filed a Notice of Strike, based on the
same grounds, when it sensed the brewing tension brought about by the CBA
negotiation that was in the meantime taking place. A month after, however, the
parties agreed, among others, to maintain the existing provisions of the CBA
(except those pertaining to wage increases and signing bonus) and to refer to the
Voluntary Arbitrator the issue of retrenchment of 117 union members, with the
qualification that "the retrenched employees subject of the VA will receive
separation package without executing quitclaim and release, and without
prejudice to the decision of the voluntary arbitrator."

On August 28, 2002, VA Diamonon dismissed petitioner’s complaint for lack of


merit, but without prejudice to the payment of separation pay to the affected
employees.

ISSUE: Whether or not the employees were illegally dismissed.

RULING:

No.

It is apparent from the records that this case involves a closure of business
undertaking, not retrenchment. The legal requirements and consequences of
these two authorized causes in the termination of employment are discernible.

We distinguished, in Alabang Country Club Inc. v. NLRC:

 While retrenchment and closure of a business establishment or undertaking


are often used interchangeably and are interrelated, they are actually two
separate and independent authorized causes for termination of
employment.
 Retrenchment is the reduction of personnel for the purpose of cutting down
on costs of operations in terms of salaries and wages resorted to by an
employer because of losses in operation of a business occasioned by lack
of work and considerable reduction in the volume of business.
 Closure of a business or undertaking due to business losses is the reversal of
fortune of the employer whereby there is a complete cessation of business
operations to prevent further financial drain upon an employer who cannot
pay anymore his employees since business has already stopped.
 One of the prerogatives of management is the decision to close the entire
establishment or to close or abolish a department or section thereof for
economic reasons, such as to minimize expenses and reduce capitalization.
 While the Labor Code provides for the payment of separation package in
case of retrenchment to prevent losses, it does not obligate the employer
for the payment thereof if there is closure of business due to serious losses.

Likewise, the case of Eastridge Golf Club, Inc. v. Eastridge Golf Club, Inc., Labor-
Union, Super stressed the differences:

Retrenchment or lay-off is the termination of employment initiated by the


employer, through no fault of the employees and without prejudice to the latter,
during periods of business recession, industrial depression, or seasonal fluctuations,
or during lulls occasioned by lack of orders, shortage of materials, conversion of
the plant for a new production program or the introduction of new methods or
more efficient machinery, or of automation. It is an exercise of management
prerogative which the Court upholds if compliant with certain substantive and
procedural requirements, namely:

1. That retrenchment is necessary to prevent losses and it is proven, by


sufficient and convincing evidence such as the employer's financial
statements audited by an independent and credible external auditor, that
such losses are substantial and not merely flimsy and actual or reasonably
imminent; and that retrenchment is the only effective measure to prevent
such imminent losses;

2. That written notice is served on to the employees and the DOLE at least
one (1) month prior to the intended date of retrenchment; and

3. That the retrenched employees receive separation pay equivalent to one


(1) month pay or at least one-half (1/2) month pay for every year of service,
whichever is higher.

The employer must prove compliance with all the foregoing requirements. Failure
to prove the first requirement will render the retrenchment illegal and make the
employer liable for the reinstatement of its employees and payment of full
backwages. However, were the retrenchment undertaken by the employer is
bona fide, the same will not be invalidated by the latter's failure to serve prior
notice on the employees and the DOLE; the employer will only be liable in
nominal damages, the reasonable rate of which the Court En Banc has set at
₱50,000.00 for each employee.

Closure or cessation of business is the complete or partial cessation of the


operations and/or shut-down of the establishment of the employer. It is carried out
to either stave off the financial ruin or promote the business interest of the
employer.

Unlike retrenchment, closure or cessation of business, as an authorized cause of


termination of employment, need not depend for validity on evidence of actual
or imminent reversal of the employer's fortune. Article 283 authorizes termination of
employment due to business closure, regardless of the underlying reasons and
motivations therefor, be it financial losses or not.

To be precise, closure or cessation of an employer’s business operations, whether


in whole or in part, is governed by Article 283 of the Labor Code, as amended. It
states:

Article 283.Closure of establishment and reduction of personnel. - The employer


may also terminate the employment of any employee due to the installation of
labor-saving devices, redundancy, retrenchment to prevent losses or the closing
or cessation of operation of the establishment or undertaking unless the closing is
for the purpose of circumventing the provisions of this Title, by serving a written
notice on the workers and the Ministry of Labor and Employment at least one (1)
month before the intended date thereof. In case of termination due to the
installation of labor-saving devices or redundancy, the worker affected thereby
shall be entitled to a separation pay equivalent to at least his one (1) month pay
or to at least one (1) month pay for every year of service, whichever is higher. In
case of retrenchment to prevent losses and in cases of closures or cessation of
operations of establishment or undertaking not due to serious business losses or
financial reverses, the separation pay shall be equivalent to one (1) month pay or
at least one-half (1/2) month pay for every year of service, whichever is higher. A
fraction of at least six (6) months shall be considered one (1) whole year.26

In Industrial Timber Corporation v. Ababon, the Court explained the above-


quoted provision in this wise:

A reading of the foregoing law shows that a partial or total closure or cessation of
operations of establishment or undertaking may either be due to serious business
losses or financial reverses or otherwise. Under the first kind, the employer must
sufficiently and convincingly prove its allegation of substantial losses, while under
the second kind, the employer can lawfully close shop anytime as long as
cessation of or withdrawal from business operations was bona fide in character
and not impelled by a motive to defeat or circumvent the tenurial rights of
employees, and as long as he pays his employees their termination pay in the
amount corresponding to their length of service. Just as no law forces anyone to
go into business, no law can compel anybody to continue the same. It would be
stretching the intent and spirit of the law if a court interferes with management's
prerogative to close or cease its business operations just because the business is
not suffering from any loss or because of the desire to provide the workers
continued employment.

Under Article 283 of the Labor Code, three requirements are necessary for a valid
cessation of business operations:

(a) service of a written notice to the employees and to the DOLE at least one
month before the intended date thereof;

(b) the cessation of business must be bona fide in character; and

(c) payment to the employees of termination pay amounting to one month pay or
at least one-half month pay for every year of service, whichever is higher.

Based on the cases of similar import, The SC summarized:

1. Closure or cessation of operations of establishment or undertaking may


either be partial or total.

2. Closure or cessation of operations of establishment or undertaking may or


may not be due to serious business losses or financial reverses. However, in
both instances, proof must be shown that: (1) it was done in good faith to
advance the employer's interest and not for the purpose of defeating or
circumventing the rights of employees under the law or a valid agreement;
and (2) a written notice on the affected employees and the DOLE is served
at least one month before the intended date of termination of
employment.

3. The employer can lawfully close shop even if not due to serious business
losses or financial reverses but separation pay, which is equivalent to at least
one month pay as provided for by Article 283 of the Labor Code, as
amended, must be given to all the affected employees.

4. If the closure or cessation of operations of establishment or undertaking is


due to serious business losses or financial reverses, the employer must prove
such allegation in order to avoid the payment of separation pay. Otherwise,
the affected employees are entitled to separation pay.

5. The burden of proving compliance with all the above-stated falls upon
the employer.

Guided by the foregoing, the Court shall refuse to dwell on the issue of whether
respondent was in sound financial condition when it resolved to stop the
operations of its F & B Department. As stated, an employer can lawfully close shop
anytime even if not due to serious business losses or financial reverses.
We have already resolved that the characterization of the employee's service as
no longer necessary or sustainable, and therefore, properly terminable, is an
exercise of business judgment on the part of the employer; the determination of
the continuing necessity of a particular officer or position in a business corporation
is a management prerogative, and the courts will not interfere with the exercise of
such so long as no abuse of discretion or arbitrary or malicious action on the part
of the employer is shown.As recognized by both the VA and the CA, evident
proofs of respondent’s good faith to arrest the losses which the F & B Department
had been incurring since 1994 are: engagement of an independent consulting
firm to conduct manpower audit/organizational development; institution of cost-
saving programs, termination of the services of probationary employees,
substantial reduction of a number of agency staff and personnel, and the
retrenchment of eight (8) managers. After the effective date of the termination of
employment relation, respondent even went on to aid the displaced employees
in finding gainful employment by soliciting the assistance of respondent’s
members, Makati Skyline, Human Resource Managers of some companies, and
the Association of Human Resource Managers.33 These were not refuted by
petitioner. Only that, it perceives them as inadequate and insists that the
operational losses are very well covered by the other income of respondent and
that less drastic measures could have been resorted to, like increasing the
membership dues and the prices of food and beverage. Yet the wisdom or
soundness of the Management decision is not subject to discretionary review of
the Court for, even the VA admitted, it enjoys a pre-eminent role and is presumed
to possess all relevant and necessary information to guide its business decisions
and actions.

Further, there is nothing on record to indicate that the closure of respondent’s F &
B Department was made in bad faith. It was not motivated by any specific and
clearly determinable union activity of the employees; rather, it was truly dictated
by economic necessity. Despite petitioner’s allegations, no convincing and
credible proofs were presented to establish the claim that such closure qualifies as
an act of union-busting and ULP. No evidence was shown that the closure is stirred
not by a desire to avoid further losses but to discourage the workers from
organizing themselves into a union for more effective negotiations with the
management. Allegations are not proofs and it is incumbent upon petitioner to
substantiate the same. On the contrary, respondent continued to negotiate with
petitioner even after April 30, 2002. In fact, a Memorandum of Agreement was
executed before the NCMB between petitioner and respondent on June 10, 2002
whereby the parties agreed, among others, to maintain the existing provisions of
the CBA, except those pertaining to wage increases and signing bonus.

Finally, even if the members of petitioner are not considered as illegally dismissed,
they are entitled to separation pay pursuant to Article 283 of the Labor Code, as
amended. Per respondent's information, however, the separation packages of all
117 union members were already paid during the pendency of the
case. Petitioner did not oppose this representation;
Hence, We shall treat the fact of receipt of separation pay as having been
voluntarily entered into, with a full understanding of its import, and the amount
received as credible and reasonable settlement that should be respected by the
Court as the law between the parties are valid and binding between them.

DISPOSITIVE PORTION:

WHEREFORE, the foregoing considered, the instant Petition is DENIED. The February
2, 2006 Decision and May 29, 2006 Resolution of the Court of Appeals in CA-G.R.
SP No. 73127 sustaining in toto the August 28, 2002 Decision and September 13,
2002 Resolution of VoluntaryArbitrator Jesus B. Diamonon, which dismissed
petitioner’s complaint for illegal retrenchment, are AFFIRMED.

G.R. No. 202996 June 18, 2014

MARLO A. DEOFERIO, Petitioner,


vs.
INTEL TECHNOLOGY PHILIPPINES, INC. and/or MIKE WENTLING, Respondents.

FACTS:

On February 1, 1996, respondent Intel Technology Philippines, Inc. (Intel)employed


Deoferio as a product quality and reliability engineer with a monthly salary of
₱9,000.00. In July 2001, Intel assigned him to the United States as a validation
engineer for an agreed period of two years and with a monthly salary of
US$3,000.00. On January 27, 2002, Deoferio was repatriated to the Philippines after
being confined at Providence St. Vincent Medical Center for major depression
with psychosis. In the Philippines, he worked as a product engineer with a monthly
salary of ₱23,000.00.

Deoferio underwent a series of medical and psychiatric treatment at Intel’s


expense after his confinement in the United States. In 2002, Dr. Elizabeth Rondain
of Makati Medical Center diagnosed him to be suffering from mood disorder,
major depression, and auditory hallucination.He was also referred to Dr. Norieta
Balderrama, Intel’s forensic psychologist, and to a certain Dr. Cynthia Leynes who
both confirmed his mental condition.On August 8, 2005, Dr. Paul Lee, a consultant
psychiatrist of the Philippine General Hospital, concluded that Deoferio was
suffering from schizophrenia. After several consultations, Dr. Lee issued a
psychiatric report dated January 17,2006 concluding and stating that Deoferio’s
psychotic symptoms are not curable within a period of six months and "will
negatively affect his work and social relation with his co-worker[s]."Pursuant to
these findings, Intel issued Deoferio a notice of termination on March 10, 2006.
Deoferio responded to his termination of employment by filing a complaint for
illegal dismissal with prayer for money claims against respondents Intel and Mike
Wentling (respondents). He denied that he ever had mental illness and insisted
that he satisfactorily performed his duties as a product engineer. He argued that
Intel violated his statutory right to procedural due process when it summarily issued
a notice of termination. He further claimed that he was entitled to a salary
differential equivalent to the pre-terminated period of his assignment in the United
States minus the base pay that he had already received. Deoferio also prayed for
backwages, separation pay, moral and exemplary damages, as well as attorney’s
fees.

In defense, the respondents argued that Deoferio’s dismissal was based on Dr.
Lee’s certification that: (1) his schizophrenia was not curable within a period of six
months even with proper medical treatment; and (2) his continued employment
would be prejudicial to his and to the other employees’ health. The respondents
also insisted that Deoferio’s presence at Intel’s premises would pose an actual
harm to his co-employees as shown by his previous acts. On May 8, 2003, Deoferio
emailed an Intel employee with this message: "All soul’s day back to work Monday
WW45.1." On January 18, 2005, he cut the mouse cables, stepped on the
keyboards, and disarranged the desks of his co-employees. The respondents also
highlighted that Deoferio incurred numerous absences from work due to his
mental condition, specifically, from January 31, 2002 until February 28, 2002, from
August 2002 until September 2002, and from May 2003 until July 2003. Deoferio
also took an administrative leave with pay from January 2005 until December
2005.

The respondents further asserted that the twin-notice requirement in dismissals


does not apply to terminations under Article 284 of the Labor Code. They
emphasized that the Labor Code’s implementing rules (IRR) only requires a
competent public health authority’s certification to effectively terminate the
services of an employee.

LA Ruling

In a decision dated March 6, 2008,the Labor Arbiter (LA) ruled that Deoferio had
been validly dismissed. The LA gave weight to Dr. Lee’s certification that Deoferio
had been suffering from schizophrenia and was not fit for employment. The
evidence on record shows that Deoferio’s continued employment at Intel would
pose a threat to the health of his co-employees. The LA further held that the Labor
Code and its IRR do not require the employer to comply with the twin-notice
requirement in dismissals due to disease. The LA also found unmeritorious
Deoferio’s money claims against Intel.

NLRC Ruling

On appeal by Deoferio, the National Labor Relations Commission (NLRC) wholly


affirmed the LA’s ruling.The NLRC also denied Deoferio’s motion for
reconsideration.
CA Ruling

On February 24, 2012, the CA affirmed the NLRC decision. It agreed with the lower
tribunals’ findings that Deoferio was suffering from schizophrenia and that his
continued employment at Intel would be prejudicial to his health and to those of
his co-employees. It ruled that the only procedural requirement under the IRR is
the certification by a competent public health authority on the non-curability of
the disease within a period of six months even with proper medical treatment. I

Petitioner’s Argument

Deoferio argues that the uniform finding that he was suffering from schizophrenia
is belied by his subsequent employment at Maxim Philippines Operating Corp. and
Philips Semiconductors Corp., which both offered him higher compensations. He
also asserts that the Labor Code does not exempt the employer from complying
with the twin-notice requirement in terminations due to disease.

Respondents’ Argument

Respondents posit that the petition raises purely questions of fact which a petition
for review on certiorari does not allow. They submit that Deoferio’s arguments
have been fully passed upon and found unmeritorious by the lower tribunals and
by the CA. They additionally argue that Deoferio’s subsequent employment in
other corporations is irrelevant in determining the validity of his dismissal; the law
merely requires the non-curability of the disease within a period of six months even
with proper medical treatment.

The respondents also maintain that Deoferio’s claim for salary differential is
already barred by prescription under Article 291 of the Labor Code. Even
assuming that the claim for salary differential has been timely filed, the
respondents assert that the parties expressly agreed in the International
Assignment Relocation Agreement that "the assignment length is only an estimate
and not a guarantee of employment for any particular length of time."31Moreover,
his assignment in the United States was merely temporary and did not change his
salary base, an amount which he already received.

ISSUES:

1. Whether or not Intel has a valid authorized cause to dismiss Deoferio


2. Whether the twin-notice requirement in dismissals applies to terminations
due to disease and thus resulting to Deoferio being entitled to nominal
damages for violation of his right to statutory procedural due process; and

RULING:

1. Yes

Intel had an authorized cause to dismiss Deoferio from employment.


Concomitant to the employer’s right to freely select and engage an employee is
the employer’s right to discharge the employee for just and/or authorized causes.
To validly effect terminations of employment, the discharge must be for a valid
cause in the manner required by law. The purpose of these two-pronged
qualifications is to protect the working class from the employer’s arbitrary and
unreasonable exercise of its right to dismiss. Thus, in termination cases, the law
places the burden of proof upon the employer to show by substantial evidence
that the termination was for a lawful cause and in the manner required by law.

In concrete terms, these qualifications embody the due process requirement in


labor cases - substantive and procedural due process. Substantive due process
means that the termination must be based on just and/or authorized causes of
dismissal. On the other hand, procedural due process requires the employer to
effect the dismissal in a manner specified in the Labor Code and its IRR.

The present case involves termination due to disease – an authorized cause for
dismissal under Article 284 of the Labor Code. As substantive requirements, the
Labor Code and its IRR require the presence of the following elements:

(1) An employee has been found to be suffering from any disease.

(2) His continued employment is prohibited by law or prejudicial to his


health, as well as to the health of his co-employees.

(3) A competent public health authority certifies that the disease is of such
nature or at such a stage that it cannot be cured within a period of six
months even with proper medical treatment. With respect to the first and
second elements, the Court liberally construed the phrase "prejudicial to his
health as well as to the health of his co-employees" to mean "prejudicial to
his health or to the health of his co-employees." We did not limit the scope
of this phrase to contagious diseases for the reason that this phrase is
preceded by the phrase "any disease" under Article 284 of the Labor Code,
to wit:

Art. 284. Disease as ground for termination. – An employer may terminate the
services of an employee who has been found to be suffering from any disease
and whose continued employment is prohibited by law or is prejudicial to his
health as well as to the health of his co-employees: Provided, That he is paid
separation pay equivalent to at least one (1) month salary or to one-half (1/2)
month salary for every year of service, whichever is greater, a fraction of at least
six (6) months being considered as one (1) whole year.

Consistent with this construction, we applied this provision in resolving illegal


dismissal cases due to non-contagious diseases such as stroke, heart attack,
osteoarthritis, and eye cataract, among others. In Baby Bus, Inc. v. Minister of
Labor,34 we upheld the labor arbitration’s finding that Jacinto Mangalino’s
continued employment – after he suffered several strokes – would be prejudicial
to his health. In Duterte v. Kingswood Trading Co., Inc., 35 we recognized the
applicability of Article 284 of the Labor Code to heart attacks. In that case, we
held that the employer- company’s failure to present a certification from a public
health authority rendered Roque Duterte’s termination due to a heart attack
illegal. We also applied this provision in Sy v. Court of Appeals 36 to determine
whether Jaime Sahot was illegally dismissed dueto various ailments such as
presleyopia, hypertensive retinopathy, osteoarthritis, and heart enlargement,
among others. In Manly Express, Inc. v. Payong, Jr.,37 we ruled that the employer-
company’s non-presentment of a certification from a public health authority with
respect to RomualdoPayong Jr.’s eye cataract was fatal to its defense.

The third element substantiates the contention that the employee has indeed
been suffering from a disease that: (1) is prejudicial to his health as well as to the
health of his co-employees; and (2) cannot be cured within a period of six months
even with proper medical treatment. Without the medical certificate, there can
be no authorized cause for the employee’s dismissal. The absence of this element
thus renders the dismissal void and illegal.

The third requirement is not merely a procedural requirement, but a substantive


one.The certification from a competent public health authority is precisely the
substantial evidence required by law to prove the existence of the disease itself,
its non-curability within a period of six months even with proper medical
treatment, and the prejudice that it would cause to the health of the sick
employee and to those of his co-employees.

In the current case, we agree with the CA that Dr. Lee’s psychiatric report
substantially proves that Deoferio was suffering from schizophrenia, that his disease
was not curable within a period of six months even with proper medical
treatment, and that his continued employment would be prejudicial to his mental
health. This conclusion is further substantiated by the unusual and bizarre acts that
Deoferio committed while at Intel’s employ.

The twin-notice requirement applies to terminations under Article 284 of the Labor
Code

The Labor Code and its IRR are silent on the procedural due process required in
terminations due to disease. Despite the seeming gap in the law, Section 2, Rule 1,
Book VI of the IRR expressly states that the employee should be afforded
procedural due process in all cases of dismissals.38

In Sy v. Court of Appeals39 and Manly Express, Inc. v. Payong, Jr.,40 promulgated in


2003 and 2005, respectively, the Court finally pronounced the rule that the
employer must furnish the employee two written notices in terminations due to
disease, namely: (1) the notice to apprise the employee of the ground for which
his dismissal is sought; and (2) the notice informing the employee of his dismissal, to
be issued after the employee has been given reasonable opportunity to answer
and to be heard on his defense. These rulings reinforce the State policy of
protecting the workers from being terminated without cause and without
affording them the opportunity to explain their side of the controversy.
From these perspectives, the CA erred in not finding that the NLRC gravely
abused its discretion when it ruled that the twin-notice requirement does not
apply to Article 284 of the Labor Code. This conclusion is totally devoid of any
legal basis; its ruling is wholly unsupported by law and jurisprudence. In other
words, the NLRC’s unprecedented, whimsical and arbitrary ruling, which the CA
erroneously affirmed, amounted to a jurisdictional error.

Deoferio is entitled to nominal damages for violation of his right to statutory


procedural due process

Intel’s violation of Deoferio’s right to statutory procedural due process warrants the
payment of indemnity in the form of nominal damages. In Jaka Food Processing
Corp. v. Pacot,41 we distinguished between terminations based on Article 282 of
the Labor Code42 and dismissals under Article 283 of the Labor Code.43 We then
pegged the nominal damages at ₱30,000.00 if the dismissal is based on a just
cause but the employer failed to comply with the twin-notice requirement. On the
other hand, we fixed the nominal damages at ₱50,000.00 if the dismissal is due to
an authorized cause under Article 283 of the Labor Code but the employer failed
to comply with the notice requirement. The reason is that dismissals for just cause
imply that the employee has committed a violation against the employer, while
terminations under Article 283 of the Labor Code are initiated by the employer in
the exercise of his management prerogative.

With respect to Article 284 of the Labor Code, terminations due to disease do not
entail any wrongdoing on the part of the employee. It also does not purely involve
the employer’s willful and voluntary exercise of management prerogative – a
function associated with the employer's inherent right to control and effectively
manage its enterprise.44 Rather, terminations due to disease are occasioned by
matters generally beyond the worker and the employer's control.

In fixing the amount of nominal damages whose determination is addressed to our


sound discretion, the Court should take into account several factors surrounding
the case, such as: (1) the employer’s financial, medical, and/or moral assistance
to the sick employee; (2) the flexibility and leeway that the employer allowed the
sick employee in performing his duties while attending to his medical needs; (3)
the employer’s grant of other termination benefits in favor of the employee; and
(4) whether there was a bona fide attempt on the part of the employer to comply
with the twin-notice requirement as opposed to giving no notice at all.

We award Deoferio the sum of ₱30,000.00 as nominal damages for violation of his
statutory right to procedural due process. In so ruling, we take into account Intel’s
faithful compliance with Article 284 of the Labor Code and Section 8, Rule 1, Book
6 of the IRR. We also note that Deoferio’s separation pay equivalent to one-half
month salary for every year of service 45 was validly offset by his matured car loan.
Under Article 1278 of the Civil Code, in relation to Article 1706 of the Civil
Code46 and Article 113(c) of the Labor Code,47 compensation shall take place
when two persons are creditors and debtors of each other in their own right. We
likewise consider the fact that Intel exhibited real concern to Deoferio when it
financed his medical expenses for more than four years. Furthermore, prior to his
termination, Intel liberally allowed Deoferio to take lengthy leave of absences to
allow him to attend to his medical needs.

DISPOSITIVE PORTION:

WHEREFORE, premises considered, we partially grant the petition; the assailed


February 24, 2012 decision and the August 2, 2012 resolution of the Court of
Appeals stand but respondent Intel Technology Philippines, Inc. is ordered to pay
petitioner Marlo A. Deoferio nominal damages in the amount of ₱30,000.00. We
totally deny the petition with respect to respondent Mike Wending.

G.R. No. 115394 September 27, 1995

FE S. SEBUGUERO, CARLOS ONG, NENE MANAOG, JUANITO CUSTODIO, CRISANTA


LACSAM, SATURNINO GURAL, WILMA BALDERA, LEONILA VALDEZ, FATIMA
POTESTAD, EVANGELINE AGNADO, RESTITUTO GLORIOSO, JANESE DE LOS REYES,
RODOLFO SANCHEZ, WILMA ORBELLO, DAISY PASCUA, and ALEX
MASAYA, petitioners,
vs.
NATIONAL LABOR RELATIONS COMMISSION, G.T.I. SPORTSWEAR CORPORATION
and/or BENEDICTO YUJUICO, respondents.

FACTS:

The petitioners were among the thirty-eight (38) regular employees of private
respondent GTI Sportswear Corporation (hereinafter GTI), a corporation engaged
in the manufacture and export of ready-to-wear garments, who were given
"temporary lay-off" notices by the latter on 22 January 1991 due to alleged lack of
work and heavy losses caused by the cancellation of orders from abroad and by
the garments embargo of 1990.

Believing that their "temporary lay-off" was a ploy to dismiss them, resorted to
because of their union activities and was in violation of their right to security of
tenure since there was no valid ground therefor, the 38 laid-off employees filed
with the Labor Arbiter's office in the National Capital Region complaints for illegal
dismissal, unfair labor practice, underpayment of wages under Wage Orders Nos.
01 and 02, and non-payment of overtime pay and 13th month pay.

Private respondent GTI denied the claim of illegal dismissal and asserted that it
was its prerogative to lay-off its employees temporarily for a period not exceeding
six months to prevent losses due to lack of work or job orders from abroad, and
that the lay-off affected both union and non-union members. It justified its failure
to recall the 38 laid-off employees after the lapse of six months because of the
subsequent cancellations of job orders made by its foreign principals, a fact which
was communicated to the petitioners and the other complainants who were all
offered severance pay. Twenty-two (22) of the 38 complainants accepted the
separation pay. The petitioners herein did not.

The cases then involving those who accepted the separation pay were pro
tanto dismissed with prejudice.

In his decision of 26 February 1993 with respect to the claims of the petitioners,
Labor Arbiter Pablo C. Espiritu, Jr. found for them and disposed as follows:

WHEREFORE, above premises considered, judgment is hereby


rendered finding Respondent, G.T.I. Sportswear Corporation, liable for
constructive dismissal, underpayment of wages under NCR 01 and
02, and 13th-month pay differentials and concomitantly, Respondent
corporation is hereby ordered:

a. To pay the following complainants backwages from


the time of their constructive dismissal (July 22, 1991) till
promulgation considering that reinstatement is no longer
decreed: . . .

b. To pay complainants separation pay of 1/2 month for


every year of service in lieu of reinstatement in the
following amounts: . . .

c. xxx:…

In support of the disposition, the Labor Arbiter made the following ratiocinations:

On the validity of the temporary lay-off, this Arbitration Branch finds


that there was ample justification on the part of Respondent
company to lay-off temporarily some of its employees to prevent
losses as a result of the reduction of the garment quota allocated to
Respondent company due to the garment embargo of 1990. In fact,
in the months of March, April, and May of 1991 respondent company
received several messages/correspondence from its foreign principals
informing them (Respondent) that they are canceling/transferring
some of their quotas/orders to other countries. The evidence
presented by Respondent company proves this fact

This is sustainable, as in this case, where the Respondent found it


unnecessary to continue employing some of its workers because of
business recession, lack of materials to work on due to government
controls (garments embargo) and due to the lack of the demand for
export quota from its principal foreign buyers.
Although, as a general rule, Respondent company has the
prerogative and right to resort to temporary lay-off, such right is
likewise limited to a period of six (6) months applying Art. 286 of the
Labor Code on suspension of employer-employee relationship not
exceeding six (6) months.

In this case, respondent company was justified in the temporary lay-


off of some of its employees. However, Respondent company should
have recalled them after the end of the six month period or at the
least reasonably informed them (complainants) that the Respondent
company is still not in a position to recall them due to the continuous
drop of demand in the export market (locally or internationally),
thereby extending the temporary lay-off with a definite period of
recall and if the same cannot be met, then the company should
implement retrenchment and pay its employees separation pay.
Failing in this regard, respondent company chose not to recall nor
send notice to the complainants after the lapse of the six (6) month
period. Hence, there is in this complaint a clear case of constructive
dismissal. While there is a valid reason for the temporary lay-off, the
same is also limited to a duration of six months. Thereafter the
employees, complainants herein, are entitled under the law (Art. 286)
to be recalled back to work. As result thereof, the temporary lay-off of
the complainants from January 22, 1991 (date of lay-off) to July 22,
1991 is valid, however, thereafter complainants are already entitled
to backwages, in view of constructive dismissal, due to the fact that
they were no longer recalled back to work. Complainants cannot be
placed on temporary lay-off forever. The limited period of six (6)
months is based provisionally too prevent circumvention on the right
to security of tenure and to prevent grave abuse of discretion on the
part of the employer. However, since during the trial it was proven, as
testified by the Vice-President for marketing and personnel manager,
that the lack of work and selection of personnel continued to persist
and considering the antagonism and hostility displayed by both
litigants, as observed by this Arbiter, during the trial of this case and in
view of the strained relations between the parties, reinstatement of
the complainants would not be prudent. Hence, separation pay of
1/2 month for every year of service in lieu of reinstatement is in order. .
..

GTI appealed to the NLRC. The NLRC concurred with the findings of the Labor
Arbiter that there was a valid lay-off of the petitioners due to lack of work, but
disagreed with the latter's ruling granting back wages after 22 July 1991.

GTI appealedto the SC and argued that there was a valid and legal reduction of
business and in sustaining the theory of redundancy in justifying the dismissal of the
petitioners
ISSUE: Whether or not there was a valid dismissal

RULING:

No.

Redundancy exists where the services of an employee are in excess of what is


reasonably demanded by the actual requirements of the enterprise. A position is
redundant where it is superfluous, and superfluity of a position or positions may be
the outcome of a number of factors, such as overhiring of workers, decreased
volume of business, or dropping of a particular product line or service activity
previously manufactured or undertaken by the enterprise.

Retrenchment, on the other hand, is used interchangeably with the term "lay-off."
It is the termination of employment initiated by the employer through no fault of
the employee's and without prejudice to the latter, resorted to by management
during periods of business recession, industrial depression, or seasonal fluctuations,
or during lulls occasioned by lack of orders, shortage of materials, conversion of
the plant for a new production program or the introduction of new methods or
more efficient machinery, or of automation. Simply put, it is an act of the
employer of dismissing employees because of losses in the operation of a business,
lack of work, and considerable reduction on the volume of his business, a right
consistently recognized and affirmed by this Court.

Article 283 of the Labor code which covers retrenchment, reads as follows:

Art. 283. Closure of establishment and reduction of personnel. — The


employer may also terminate the employment of any employee due
to the installation of labor saving devices, redundancy, retrenchment
to prevent losses or the closing or cessation of operation of the
establishment or undertaking unless the closing is for the purpose of
circumventing the provisions of this Title, by servicing a written notice
on the workers and the Ministry of Labor and Employment at least one
(1) month before the intended date thereof. In case of termination
due to the installation of labor saving devices or redundancy, the
worker affected thereby shall be entitled to a separation pay
equivalent to at least his one (1) month pay or to at least one (1)
month pay for every year of service, whichever is higher. In case of
retrenchment to prevent losses and in cases of closure or cessation of
operations of establishment or undertaking not due to serious business
losses or financial reverses, the separation pay shall be equivalent to
one (1) month pay or at least one-half (1/2) month pay for every year
of service, whichever is higher. A fraction of at least six (6) months
shall be considered one (1) whole year.
This provision, however, speaks of a permanent retrenchment as opposed to
a temporary lay-off as is the case here. There is no specific provision of law
which treats of a temporary retrenchment or lay-off and provides for the
requisites in effecting it or a period or duration therefor. These employees
cannot forever be temporarily laid-off. To remedy this situation or fill the
hiatus, Article 286 may be applied but only by analogy to set a specific
period that employees may remain temporarily laid-off or in floating
status.Six months is the period set by law that the operation of a business or
undertaking may be suspended thereby suspending the employment of the
employees concerned. The temporary lay-off wherein the employees
likewise cease to work should also not last longer than six months. After six
months, the employees should either be recalled to work or permanently
retrenched following the requirements of the law, and that failing to comply
with this would be tantamount to dismissing the employees and the
employer would thus be liable for such dismissal.

To determine, therefore, whether the petitioners were validly retrenched or were


illegally dismissed, we must determine whether there was compliance with the law
regarding a valid retrenchment at anytime within the six month-period that they
were temporarily laid-off.

Under the aforequoted Article 283 of the Labor Code, there are three basic
requisites for a valid retrenchment:

(1) the retrenchment is necessary to prevent losses and such losses


are proven;

(2) written notice to the employees and to the Department of Labor


and Employment at least one month prior to the intended date of
retrenchment; and

(3) payment of separation pay equivalent to one month pay or at


least 1/2 month pay for every year of service, whichever is higher.

As for the first requisite, whether or not an employer would imminently suffer serious
or substantial losses for economic reasons is essentially a question of fact for the
Labor Arbiter and the NLRC to determine.14 Here, both the Labor Arbiter and the
NLRC found that the private respondent was suffering and would continue to
suffer serious losses, thereby justifying the retrenchment of some of its employees,
including the petitioners. We are not prepared to disregard this finding of fact. It is
settled that findings of quasi-judicial agencies which have acquired expertise in
the matters entrusted to their jurisdiction are accorded by this Court not only with
respect but with finality if they are supported by substantial evidence.15 The latter
means that amount of relevant evidence which a reasonable mind might accept
as adequate to justify a conclusion.16 In the instant case, no claim was made by
any of the parties that such a finding was not supported by substantial evidence.
Furthermore, the petitioners did not appeal the finding of the Labor Arbiter that
their temporary lay-off to prevent losses was amply justified. They cannot now
question this finding that there is a valid ground to lay-off or retrench them.

The requirement of notice to both the employees concerned and the


Department of Labor and Employment (DOLE) is mandatory and must be written
and given at least one month before the intended date of retrenchment. In this
case, it is undisputed that the petitioners were given notice of the temporary lay-
off. There is, however, no evidence that any written notice to permanently
retrench them was given at least one month prior to the date of the intended
retrenchment. The NLRC found that GTI conveyed to the petitioners the
impossibility of recalling them due to the continued unavailability of work.17 But
what the law requires is a written notice to the employees concerned and that
requirement is mandatory.18 The notice must also be given at least one month in
advance of the intended date of retrenchment to enable the employees to look
for other means of employment and therefore to ease the impact of the loss of
their jobs and the corresponding income.19 That they were already on temporary
lay-off at the time notice should have been given to them is not an excuse to
forego the one-month written notice because by this time, their lay-off is to
become permanent and they were definitely losing their employment.

There is also nothing in the records to prove that a written notice was ever given to
the DOLE as required by law. GTI's position paper,20 offer of exhibits,21 Comment to
the Petition,22 and Memorandum23 in this case do not mention of any such written
notice. The law requires two notices — one to the employee/s concerned and
another to the DOLE — not just one. The notice to the DOLE is essential because
the right to retrench is not an absolute prerogative of an employer but is subject
to the requirement of law that retrenchment be done to prevent losses. The DOLE
is the agency that will determine whether the planned retrenchment is justified
and adequately supported by facts.

The lack of written notice to the petitioners and to the DOLE does not, however,
make the petitioners' retrenchment illegal such that they are entitled to the
payment of back wages and separation pay in lieu of reinstatement as they
contend. Their retrenchment, for not having been effected with the required
notices, is merely defective. In those cases where we found the retrenchment to
be illegal and ordered the employees' reinstatement and the payment of back
wages, the validity of the cause for retrenchment, that is the existence of
imminent or actual serious or substantial losses, was not proven.26 But here, such a
cause is present as found by both the Labor Arbiter and the NLRC. There is only a
violation by GTI of the procedure prescribed in Article 283 of the Labor Code in
effecting the retrenchment of the petitioners.

It is now settled that where the dismissal of an employee is in fact for a just and
valid cause and is so proven to be but he is not accorded his right to due
process, i.e., he was not furnished the twin requirements of notice and the
opportunity to be heard, the dismissal shall be upheld but the employer must be
sanctioned for non-compliance with the requirements of or for failure to observe
due process. The sanction, in the nature of indemnification or penalty, depends
on the facts of each case and the gravity of the omission committed by the
employer.

DISPOSITIVE PORTION:

WHEREFORE, the instant petition is partially GRANTED and the challenged decision
of public respondent National Labor Relations Commission in NLRC NCR CA Case
No. 004673-93 is modified by reversing and setting aside its deletion of the awards
in the Labor Arbiter's decision of proportionate 13th month pay for 1991 and
attorney's fees, the latter being reduced to P25,000.00. Separation pay equivalent
to one-half (1/2) month pay for every year of service shall be computed from the
dates of the commencement of the petitioners' respective employment until the
end of their six-month temporary lay-off which is 22 July 1991. In addition, private
respondent G.T.I. Sportswear Corporation is ordered to pay each of the petitioners
the sum of P2,000.00 as indemnification for its failure to observe due process in
effecting the retrenchment.

G.R. No. 181738 January 30, 2013

GENERAL MILLING CORPORATION, Petitioner,


vs.
VIOLETA L. VIAJAR, Respondent.

FACTS:

GMC is a domestic corporation with principal office in Makati City and a


manufacturing plant in Lapu-Lapu City.

In October 2003, GMC terminated the services of thirteen (13) employees for
redundancy, including herein respondent, Violeta Viajar (Viajar). GMC alleged
that it has been gradually downsizing its Vismin (Visayas-Mindanao) Operations in
Cebu where a sizeable number of positions became redundant over a period of
time.

On December 2, 2003, Viajar filed a Complaint 7 for Illegal Dismissal with damages
against GMC, its Human Resource Department (HRD) Manager, Johnny T.
Almocera (Almocera), and Purchasing Manager, Joel Paulino before the Regional
Arbitration Branch (RAB) No. VII, NLRC, Cebu City.

In her Position Paper,8 Viajar alleged that she was employed by GMC on August 6,
1979 as Invoicing Clerk. Through the years, the respondent held various positions in
the company until she became Purchasing Staff.

On October 30, 2003, Viajar received a Letter-Memorandum dated October 27,


2003 from GMC, through Almocera, informing her that her services were no longer
needed, effective November 30, 2003 because her position as Purchasing Staff at
the Purchasing Group, Cebu Operations was deemed redundant. Immediately
thereafter, the respondent consulted her immediate superior at that time,
Thaddeus Oyas, who told her that he too was shocked upon learning about it.

When Viajar reported for work on October 31, 2003, almost a month before the
effectivity of her severance from the company, the guard on duty barred her from
entering GMC’s premises. She was also denied access to her office computer and
was restricted from punching her daily time record in the bundy clock.

On November 7, 2003, Viajar was invited to the HRD Cebu Office where she was
asked to sign certain documents, which turned out to be an "Application for
Retirement and Benefits." The respondent refused to sign and sought clarification
because she did not apply for retirement and instead asserted that her services
were terminated for alleged redundancy. Almocera told her that her signature on
the Application for Retirement and Benefits was needed to process her separation
pay. The respondent also claimed that between the period of July 4, 2003 and
October 13, 2003, GMC hired fifteen (15) new employees which aroused her
suspicion that her dismissal was not necessary. At the time of her termination, the
respondent was receiving the salary rate of ₱19,651.41 per month.

For its part, the petitioner insisted that Viajar’s dismissal was due to the
redundancy of her position. GMC reasoned out that it was forced to terminate
the services of the respondent because of the economic setbacks the company
was suffering which affected the company’s profitability, and the continuing rise
of its operating and interest expenditures. Redundancy was part of the petitioner’s
concrete and actual cost reduction measures. GMC also presented the required
"Establishment Termination Report" which it filed before the Department of Labor
and Employment (DOLE) on October 28, 2003, involving thirteen (13) of its
employees, including Viajar. Subsequently, GMC issued to the respondent two (2)
checks respectively amounting to ₱440,253.02 and ₱21,211.35 as her separation
pay.

On April 18, 2005, the Labor Arbiter dismissed the case for lack of
meritrationatingthe respondent was properly notified on October 30, 2003 through
a Letter-Memorandum dated October 27, 2003, signed by GMC’s HRD Manager
Almocera, that her position as Purchasing Staff had been declared redundant. It
also found that the petitioner submitted to the DOLE on October 28, 2003 the
"Establishment Termination Report." The LA even faulted the respondent for not
questioning the company’s action before the DOLE Regional Office, Region VII,
Cebu City so as to compel the petitioner to prove that Viajar’s position was
indeed redundant. It ruled that the petitioner complied with the requirements
under Article 283 of the Labor Code, considering that the nation was then
experiencing an economic downturn and that GMC must adopt measures for its
survival.15

Viajar appealed the aforesaid decision to the NLRC. On October 28, 2005, the
NLRC affirmed the ruling of the LA.
The NLRC stated that the characterization of positions as redundant is an exercise
of the employer’s business judgment and prerogative. It also ruled that the
petitioner did not exercise this prerogative in bad faith and that the payment of
separation pay in the amount of ₱461,464.37 was in compliance with Article 283 of
the Labor Code.20

Respondent Viajar filed a Motion for Reconsideration which was denied by the
NLRC in its Resolution dated January 31, 2006.

Undaunted, Viajar filed a petition for certiorari before the CA. In the now assailed
Decision dated September 21, 2007, the CA granted the petition, reversing the
decision of the NLRC.

The NLRC declared the dismissal ILLEGAL and ordered respondent to reinstate
petitioner without loss of seniority rights and other privileges with full backwages
inclusive of allowances and other benefits computed from the time she was
dismissed on 30 November 2003 up to the date of actual reinstatement.

Aggrieved by the reversal of the NLRC decision, GMC filed a motion for
reconsideration. However, in its Resolution dated January 30, 2008, the CA denied
the same; hence, this petition.

ISSUE: Whether or not there was a valid dismissal

RULING:

No.Viajar was not validly dismissed on the ground of redundancy.

Petitioner’s argument:

GMC claims that Viajar was validly dismissed on the ground of redundancy which
is one of the authorized causes for termination of employment. The petitioner
asserts that it has observed the procedure provided by law and that the same
was done in good faith. To justify the respondent’s dismissal, the petitioner
presented: (i) the notification Letter-Memorandum dated October 27, 2003
addressed to the respondent which was received on October 30, 2003; 29 (ii) the
"Establishment Termination Report" as prescribed by the DOLE; 30 (iii) the two (2)
checks issued in the respondent’s name amounting to ₱440,253.02 and ₱21,211.35
as separation pay;31 and (iv) the list of dismissed employees as of June 6, 2006 to
show that GMC was in a "reduction mode." Both the LA and the NLRC found these
sufficient to prove that the dismissal on the ground of redundancy was done in
good faith.

Article 283 of the Labor Code provides that redundancy is one of the authorized
causes for dismissal. It reads:

Article 283. Closure of establishment and reduction of personnel. – The employer


may also terminate the employment of any employee due to the installment of
labor-saving devices, redundancy, retrenchment to prevent losses or the closing
or cessation of operation of the establishment or undertaking unless the closing is
for the purpose of circumventing the provisions of this Title, by serving a written
notice on the worker and the Ministry of Labor and Employment at least one (1)
month before the intended date thereof. In case of termination due to the
installation of labor-saving devices or redundancy, the worker affected thereby
shall be entitled to a separation pay equivalent to at least his one (1) month pay
or to at least one (1) month pay for every year of service, whichever is higher. In
case of retrenchment to prevent losses and in cases of closures or cessation of
operations of establishment or undertaking not due to serious business losses or
reverses, the separation pay shall be equivalent to one (1) month pay or at least
one-half (1/2) month pay for every year of service, whichever is higher. A fraction
of at least six (6) months shall be considered one (1) whole year. (Emphasis
supplied)

From the above provision, it is imperative that the employer must comply with the
requirements for a valid implementation of the company’s redundancy program,
to wit: (a) the employer must serve a written notice to the affected employees and
the DOLE at least one (1) month before the intended date of retrenchment; (b) the
employer must pay the employees a separation pay equivalent to at least one
month pay or at least one month pay for every year of service, whichever is
higher; (c) the employer must abolish the redundant positions in good faith; and
(d) the employer must set fair and reasonable criteria in ascertaining which
positions are redundant and may be abolished.

In Smart Communications, Inc., v. Astorga,34 the Court held that:

The nature of redundancy as an authorized cause for dismissal is explained in the


leading case of Wiltshire File Co., Inc. v. National Labor Relations Commission, viz:

"x x x redundancy in an employer’s personnel force necessarily or even ordinarily


refers to duplication of work. That no other person was holding the same position
that private respondent held prior to termination of his services does not show that
his position had not become redundant. Indeed, in any well organized business
enterprise, it would be surprising to find duplication of work and two (2) or more
people doing the work of one person. We believe that redundancy, for purposes
of the Labor Code, exists where the services of an employee are in excess of what
is reasonably demanded by the actual requirements of the enterprise. Succinctly
put, a position is redundant where it is superfluous, and superfluity of a position or
positions may be the outcome of a number of factors, such as overhiring of
workers, decreased volume of business, or dropping of a particular product line or
service activity previously manufactured or undertaken by the enterprise."

The characterization of an employee’s services as superfluous or no longer


necessary and, therefore, properly terminable, is an exercise of business judgment
on the part of the employer. The wisdom and soundness of such characterization
or decision is not subject to discretionary review provided, of course, that a
violation of law or arbitrary or malicious action is not shown.
The Court has always stressed that a company cannot simply declare
redundancy without basis. To exhibit its good faith and that there was a fair and
reasonable criteria in ascertaining redundant positions, a company claiming to be
over manned must produce adequate proof of the same.

We reiterate what was held in Caltex (Phils.), Inc. v. NLRC:

In Asufrin, Jr. v. San Miguel Corporation, we ruled that it is not enough for a
company to merely declare that it has become overmanned (sic). It must
produce adequate proof of such redundancy to justify the dismissal of the
affected employees.

In Panlilio v. National Labor Relations Commission, we held that evidence must be


presented to substantiate redundancy such as but not limited to the new staffing
pattern, feasibility studies/proposal, on the viability of the newly created positions,
job description and the approval by the management of the restructuring.

In the instant case, the petitioner failed to present substantial proof to support
GMC’s general allegations of redundancy. As shown from the records, the
petitioner simply presented as its evidence of good faith and compliance with the
law the notification letter to respondent Viajar; the "Establishment Termination
Report" it submitted to the DOLE Office; the two (2) checks issued in the
respondent’s name amounting to ₱440,253.02 and ₱21,211.35; and the list of
terminated employees as of June 6, 2006. These are not enough proof for the valid
termination of Viajar’s employment on the ground of redundancy.

The letter-memorandum which contains general allegations is not enough to


convince this Court that Viajar’s termination of employment due to redundancy
was warranted under the circumstances. There is no showing that GMC made an
evaluation of the existing positions and their effect to the company. Neither did
GMC exert efforts to present tangible proof that it was experiencing business slow
down or over hiring. The "Establishment Termination Report" it submitted to the
DOLE Office did not account for anything to justify declaring the positions
redundant. The Court notes that the list of terminated employees presented by
GMC was a list taken as of June 6, 2006 or almost three years after the respondent
was illegally dismissed and almost a year after the LA promulgated its decision.
While the petitioner had been harping that it was on a "reduction mode" of its
employees, it has not presented any evidence (such as new staffing pattern,
feasibility studies or proposal, viability of newly created positions, job description
and the approval of the management of the restructuring,audited financial
documents like balance sheets, annual income tax returns and others) which
could readily show that the company’s declaration of redundant positions was
justified. Such proofs, if presented, would suffice to show the good faith on the part
of the employer or that this business prerogative was not whimsically exercised in
terminating respondent’s employment on the ground of redundancy.

On the other hand, the respondent presented proof that the petitioner had been
hiring new employees while it was firing the old ones,negating the claim of
redundancy. It must, however, be pointed out that in termination cases, like the
one before us, the burden of proving that the dismissal of the employees was for a
valid and authorized cause rests on the employer. It was incumbent upon the
petitioner to show by substantial evidence that the termination of the employment
of the respondent was validly made and failure to discharge that duty would
mean that the dismissal is not justified and therefore illegal.

DISPOSITIVE PORTION:

WHEREFORE, the petition is DENIED. The Decision dated September 21, 2007 of the
Court of Appeals, as well as its Resolution dated January 30, 2008 in CA-G.R. SP
No. 01734, are hereby AFFIRMED.

NORTH DAVAO MINING CORPORATION VS. NATIONAL LABOR RELATIONS


COMMISSION
G.R. No. 112546 March 13, 1996

FACTS:
Petitioner North Davao Mining Corporation (North Davao) was incorporated in
1974 as a 100% privately-owned company.On May 31, 1992, petitioner North
Davao completely ceased operations due to serious business reverses. From 1988
until its closure in 1992, North Davao suffered net losses averaging three billion
pesos (P3,000,000,000.00) per year, for each of the five years prior to its closure. All
told, as of December 31, 1991, or five months prior to its closure, its total liabilities
had exceeded its assets by 20,392 billion pesos, as shown by its financial
statements audited by the Commission on Audit.

When North Davao ceased operations, its remaining employees were separated
and given the equivalent of 12.5 days' pay for every year of service, computed on
their basic monthly pay, in addition to the commutation to cash of their unused
vacation and sick leaves. However, it appears that, during the life of the petitioner
corporation, from the beginning of its operations in 1981 until its closure in 1992, it
had been giving separation pay equivalent to thirty (30) days' pay for every year
of service. Moreover, inasmuch as the region where North Davao operated was
plagued by insurgency and other peace and order problems, the employees had
to collect their salaries at a bank in Tagum, Davao del Norte, some 58 kilometers
from their workplace and about 2 1/2 hours' travel time by public transportation;
this arrangement lasted from 1981 up to 1990.

Subsequently, a complaint was filed with respondent Labor Arbiter by respondent


Wilfredo Guillema and 271 other separated employees for: (1) additional
separation pay of 17.5 days for every year of service; (2) back wages equivalent
to two days a month; (3) transportation allowance; (4) hazard pay; (5) housing
allowance; (6) food allowance; (7) post-employment medical clearance; and (8)
future medical allowance, all of which amounted to P58,022,878.31 as computed
by private respondent.

ISSUE:
a. Whether or not North Davao is liable for separation pay to its separated
employees
b. Whether or not the separated employees are entitled to back wages and
transportation allowance

RULING:
a. No, North Davao is not liable to pay separation benefits to its separated
employees.

Art. 298 of the Labor Code provides that “The employer may also terminate the
employment of any employee due to the installation of labor saving devices,
redundancy, retrenchment to prevent losses or the closing or cessation of
operation of the establishment or undertaking unless the closing is for the purpose
of circumventing the provisions of this Title, by serving a written notice on the
workers and the Ministry of Labor and Employment at least one (1) month before
the intended date thereof. In case of termination due to the installation of labor
saving devices or redundancy, the worker affected thereby shall be entitled to a
separation pay equivalent to at least his one (1) month pay or to at least one (1)
month pay for every year of service, whichever is higher. In case of retrenchment
to prevent losses and in cases of closures or cessation of operations of
establishment or undertaking not due to serious business losses or financial
reverses, the separation pay shall be equivalent to one (1) month pay or at least
one-half (1/2) month pay for every year of service, whichever is higher. A fraction
of at least six (6) months shall be considered one (1) whole year.”

The Code does not obligate an employer to pay separation benefits when the
closure is due to losses. In the case before us, the basis for the claim of the
additional separation benefit of 17.5 days is alleged discrimination, i.e., unequal
treatment of employees, which is proscribed as an unfair labor practice by Art.
248 (e) of said Code. Under the facts and circumstances of the present case, the
grant of a lesser amount of separation pay to private respondent was done, not
by reason of discrimination, but rather, out of sheer financial bankruptcy — a fact
that is not controlled by management prerogatives.The fact that North Davao at
the point of its forced closure voluntarily paid any separation benefits at all —
although not required by law — and 12.5-days worth at that, should have elicited
admiration instead of condemnation. But to require it to continue being generous
when it is no longer in a position to do so would certainly be unduly oppressive,
unfair and most revolting to the conscience.

b. Yes, the separated employees are entitled to back wages and transportation
allowance.
Section 4, Rule VIII, Book III of the Omnibus Rules Implementing the Labor Code
provides that:

Sec. 4. Place of payment. — (a) As a general rule, the place of payment shall be
at or near the place of undertaking. Payment in a place other than the workplace
shall be permissible only under the following circumstances:

(1) When payment cannot be effected at or near the place of work by reason of
the deterioration of peace and order conditions, or by reason of actual or
impending emergencies caused by fire, flood, epidemic or other calamity
rendering payment thereat impossible;

(2) When the employer provides free transportation to the employees back and
forth; and

(3) Under any analogous circumstances; provided that the time spent by the
employees in collecting their wages shall be considered as compensable hours
worked.

From the evidence on record, the hours spent by complainants in collecting


salaries at a bank in Tagum, Davao del Norte shall be considered compensable
hours worked. Considering further the distance between Amacan, Maco to
Tagum which is 2 1/2 hours by travel and the risks in commuting all the time in
collecting the employees' salaries, would justify the granting of backwages
equivalent to two (2) days in a month as prayed for.

Corollary to the above findings, and for equitable reasons, North Davao shall be
liable for the transportation expenses incurred by the separated employees at
P40.00 round trip fare during pay days.

VIRGILIO G. ANABE VS. ASIAN CONSTRUCTION (ASIAKONSTRUKT)


G.R. No. 183233, December 23, 2009

FACTS:
Virgilio G. Anabe (petitioner) was hired by respondent Asian Construction
(Asiakonstrukt) as radio technician/operator on April 15, 1993. By notice dated
September 8, 1999, he was advised that his services would be, as he was in fact,
terminated effective October 8, 1999 on the ground of retrenchment. Petitioner
thus filed on February 10, 2000 a complaint for illegal dismissal and illegal
deduction and payment of overtime pay, premium pay, holiday pay, service
incentive leave pay, and 13th month pay.

Asiakonstrukt, attributing petitioner’s retrenchment to sudden business reversal in


the construction industry, averred, however, that petitioner’s money claims have
been offset against his outstanding accountabilities.
The Labor Arbiter, finding that Asiakonstrukt failed to submit financial statements to
prove losses, ruled that petitioner was not validly dismissed.

On appeal, the National Labor Relations Commission (NLRC), taking into


consideration the certified true copies of the Audited Financial Statements from
1998 to 2000 submitted by Asiakonstrukt, partly granted the appeal modifying the
Labor Arbiter’s Decision by holding that petitioner was not illegally dismissed. While
it affirmed the award of the 13th month pay, overtime pay and attorney’s fees, it
ordered the payment to petitioner of ₱19,170 as separation pay.

Moreover, the NLRC reduced the reimbursable amount of illegal deductions from
₱164,960.24 to ₱88,000.00, ratiocinating that petitioner is only entitled to money
claims from 1997-1999, the claims prior thereto having already prescribed.

On appeal, the Court of Appeals held that there was no grave abuse of discretion
on the part of the NLRC when it considered the financial statements as they
"already form part of the records on appeal."

ISSUE:
Whether or not petitioner was illegally dismissed

RULING:
Yes, the petitioner was illegally dismissed.

Retrenchment is the termination of employment initiated by the employer through


no fault of and without prejudice to the employees, it is resorted to during periods
of business recession, industrial depression, or seasonal fluctuations or during lulls
occasioned by lack of orders, shortage of materials, conversion of the plant for a
new production program or the introduction of new methods or more efficient
machinery or of automation. It is a management prerogative resorted to, to avoid
or minimize business losses, and is recognized by Article 298 of the Labor Code, as
amended, viz:

Art. 298. Closure of establishment and reduction of personnel.––The employer may


also terminate the employment of any employee due to x xx retrenchment to
prevent losses or the closing or cessation of operations of the establishment x xx by
serving a written notice on the worker and the [DOLE] at least one month before
the intended date thereof. x xxIn case of retrenchment to prevent losses, the
separation pay shall be equivalent to one (1) month pay or at least one-half
month pay for every year of service whichever is higher. x xx

To effect a valid retrenchment, the following elements must be present: (1) the
retrenchment is reasonably necessary and likely to prevent business losses which, if
already incurred, are not merely de minimis, but substantial, serious, and real, or
only if expected, are reasonably imminent as perceived objectively and in good
faith by the employer; (2) the employer serves written notice both to the
employee/s concerned and the Department of Labor and Employment at least a
month before the intended date of retrenchment; (3) the employer pays the
retrenched employee separation pay in an amount prescribed by the Code; (4)
the employer exercises its prerogative to retrench in good faith; and (5) the
employer uses fair and reasonable criteria in ascertaining who would be
retrenched or retained.

The losses must be supported by sufficient and convincing evidence,the normal


method of discharging which is the submission of financial statements duly
audited by independent external auditors.

In the present case, Asiakonstrukt failed to submit its audited financial statements
within the two years that the case was pending before the Labor Arbiter. It
submitted them only after it received the adverse judgment of the Labor Arbiter.

Indubitably, the NLRC is not precluded from receiving evidence on appeal as


technical rules of evidence are not binding in labor cases. There is, however, a
caveat to this policy. The delay in the submission of evidence should be clearly
explained and should adequately prove the employer’s allegation of the cause
for termination. In the present case, Asiakonstrukt proffered no explanation behind
the belated submission. And the financial statements it submitted covered the
period 1998-2000. Further, note that the audited financial statementcovering the
period 1998-2000 was prepared in April 2001, which begs the question of how the
management knew at such date of the company’s huge losses to justify
petitioner’s retrenchment in 1999.

Furthermore, from the certificationissued by the Securities and Exchange


Commission (SEC), it would appear that Asiakonstrukt failed to submit its financial
statements to the SEC, as required under the law, for the period 1998-2000 and
2003-2005, thereby lending credence to petitioner’s theory that the financial
statements submitted on appeal may have been fabricated. Indeed,
Asiakonstrukt could have easily submitted its audited financial statements during
the pendency of the proceedings at the labor arbiter’s level, especially
considering that it was in late 2001 that the case was decided.

For failure then of Asiakonstrukt to clearly and satisfactorily substantiate its


financial losses,the dismissal of petitioner on account of retrenchment is unjustified.
Petitioner is thus entitled to the twin reliefs of payment of backwages and other
benefits from the time of his dismissal up to the finality of this Court’s Decision, and
reinstatement without loss of seniority rights or, in lieu thereof, payment of
separation pay.
CAPITOL MEDICAL CENTER, INC. VS. DR. CESAR E. MERIS
G.R. No. 155098, September 16, 2005

FACTS:
On January 16, 1974, petitioner Capitol Medical Center, Inc. (Capitol) hired Dr.
Cesar Meris (Dr. Meris),one of its stockholders, as in charge of its Industrial Service
Unit (ISU) at a monthly salary of ₱10,270.00.Until the closure of the ISU on April 30,
1992,Dr. Meris performed dual functions of providing medical services to Capitol’s
more than 500 employees and health workers as well as to employees and
workers of companies having retainer contracts with it.

On March 31, 1992, Dr. Meris received from Capitol’s president and chairman of
the board, Dr. Thelma Navarette-Clemente (Dr. Clemente), a notice advising him
of the management’s decision to close or abolish the ISU and the consequent
termination of his services as Chief thereof, effective April 30, 1992.

Dr. Meris, doubting the reason behind the management’s decision to close the ISU
and believing that the ISU was not in fact abolished as it continued to operate
and offer services to the client companies with Dr. Clemente as its head and the
notice of closure was a mere ploy for his ouster in view of his refusal to retire
despite Dr. Clemente’s previous prodding for him to do so,sought his
reinstatement but it was unheeded.

Dr. Meris thus filed on September 7, 1992 a complaint against Capitol and Dr.
Clemente for illegal dismissal and reinstatement with claims for backwages, moral
and exemplary damages, plus attorney’s fees.

Finding for Capitol and Dr. Clemente, the Labor Arbiter held that the abolition of
the ISU was a valid and lawful exercise of management prerogatives and there
was convincing evidence to show that ISU was being operated at a loss.

On appeal by Dr. Meris, the National Labor Relations Commission (NLRC) modified
the Labor Arbiter’s decision. It held that in the exercise of Capitol’s management
prerogatives, it had the right to close the ISU even if it was not suffering business
losses in light of Article 298 of the Labor Code and jurisprudence. The NLRC further
set aside the Labor Arbiter’s directive for the payment of retirement benefits to Dr.
Meris because he did not retire. Instead, it ordered the payment of separation
pay as provided under Article 298 as he was discharged due to closure of ISU, to
be charged against the retirement fund.

The Court of Appeals held that Capitol failed to strictly comply with both
procedural and substantive due process, a condition sine qua non for the validity
of a case of termination,hence, Dr. Meris was illegally dismissed.
ISSUE:
Whether or not respondent was illegally dismissed

RULING:
The right to close the operation of an establishment or undertaking is explicitly
recognized under the Labor Code as one of the authorized causes in terminating
employment of workers, the only limitation being that the closure must not be for
the purpose of circumventing the provisions on termination of employment
embodied in the Labor Code.

The phrase "closures or cessation of operations of establishment or undertaking"


includes a partial or total closure or cessation. The phrase "closures or cessation x
xx not due to serious business losses or financial reverses" recognizes the right of
the employer to close or cease his business operations or undertaking even if he is
not suffering from serious business losses or financial reverses, as long as he pays his
employees their termination pay in the amount corresponding to their length of
service.As long as the company’s exercise of closing or ceasing its business
operations is in good faith to advance its interest and not for the purpose of
defeating or circumventing the rights of employees under the law or a valid
agreement, such exercise will be upheld.The ultimate test of the validity of closure
or cessation of establishment or undertaking is that it must be bona fide in
character. And the burden of proving such falls upon the employer.

In the case at bar, Capitol failed to sufficiently prove its good faith in closing the
ISU.The records of the case failed to show that there was indeed extinct demand
for the medical services rendered by the ISU, as what was indicated in the notice
given to Dr. Meris.

The termination of the services of Dr. Meris not having been premised on a just or
authorized cause, he is entitled to either reinstatement or separation pay if
reinstatement is no longer viable, and to backwages.

Reinstatement, however, is not feasible in case of a strained employer-employee


relationship or when the work or position formerly held by the dismissed employee
no longer exists, as in the instant case. Dr. Meris is thus entitled to payment of
separation pay at the rate of one (1) month salary for every year of his
employment, with a fraction of at least six (6) months being considered as one(1)
year, and full backwages from the time of his dismissal from April 30, 1992 until the
expiration of his term as Chief of ISU or his mandatory retirement, whichever
comes first.
PEÑAFRANCIA TOURS AND TRAVEL TRANSPORT, INC., VS. JOSELITO P. SARMIENTO
and RICARDO S. CATIMBANG
G.R. No. 178397, October 20, 2010

FACTS:
Herein respondents Joselito Sarmiento (Sarmiento) and Ricardo
Catimbangworked as bus inspectors of petitioner Peñafrancia Tours and Travel
Transport, Inc.

Both Sarmiento and Catimbang (respondents) averred that they were required to
work seven (7) days a week, and that they had no rest day and worked even
during the holidays, except Good Friday, Christmas Eve, and New Year’s Eve.
Sometime in the first week of October 2002, they received notices of termination
on the ground of petitioner’s alleged irreversible business losses.

In the middle of October 2002, a meeting was called by petitioner’s President and
General Manager, Bonifacio Cu, wherein respondents were introduced to Alfredo
Perez, the owner of ALPS Transportation, as the new owner of petitioner, having
allegedly bought the same. On October 30, 2002, respondents received their last
pay with a letter informing them that their application with the company had
been held in abeyance. Respondents filed a case for illegal dismissal. They,
however, learned that, several days after their termination, Bonifacio Cu
continued to operate petitioner bus company.

While respondents’ case for illegal dismissal was pending before the Labor Arbiter
(LA), a notice was issued by Perez to all employees of petitioner that the
management of the company shall revert to its former President, Bonifacio Cu
decided to rescind the sale for Perez’ failure to comply with their
agreement.Thereafter, Cu entered into a transaction, denominated as a "Deed of
Sale with Assignment of Franchise (By Way of Dation in Payment)," with Southern
Comfort Bus Co., Inc. (SCBC), represented by its President and General Manager,
Willy Deterala
.

Petitioneraverred that due to severe business losses, it decided to stop its


operation and sell the business enterprise to the Perez family of ALPS
Transportation. It alleged that due notice was given to the Department of Labor
and Employment,and that all its employees were duly notified and were paid their
corresponding separation pay, as well as their 13th month pay. Petitioner argued
that the matter of rehiring respondents rested on the sound discretion of its new
owners, and the latter could not be compelled to absorb petitioner’s former
employees since the same was not part of the deal. Petitioner alleged that
respondents submitted their application for reemployment but, after evaluation,
the new owners opted not to hire respondents.
The Labor Arbiter dismissed respondents’ complaint of illegal dismissal for lack of
merit. On recourse, the National Labor Relations Commission (NLRC) favored the
respondents, finding that no sale of the business actually took place. On appeal,
the Court of Appeals ruled in favor of respondents.

ISSUE:
Whether respondents were legally terminated from employment by reason of the
sale of the business enterprise and the consequent change or transfer of
ownership/management

RULING:
No. Closure of business is the reversal of fortune of the employer whereby there is
a complete cessation of business operations and/or an actual locking-up of the
doors of the establishment, usually due to financial losses. Closure of business, as
an authorized cause for termination of employment, aims to prevent further
financial drain upon an employer who can no longer pay his employees since
business has already stopped.Closure or cessation of operation of the
establishment is an authorized cause for terminating an employee.

In this case, petitioner terminated the employment of respondents on this ground.


However, what petitioner apparently made was a transfer of ownership. Where
the charge of ownership is done in bad faith, or is used to defeat the rights of
labor, the successor-employer is deemed to have absorbed the employees and is
held liable for the transgressions of his or her predecessor. In this case, there is no
successor-employer because there was no actual change of ownership. The
records explicitly show that it (PTTTI) failed to establish its allegation that it was
suffering from business reverses. Neither was there proof that indeed a sale was
made and executed involving the company’s assets in favor of ALPS
Transportation owned by the Perez family.Furthermore, the alleged second sale
made by petitioner PTTTI, this time in favor of Southern Comfort Bus Co., Inc.
represented by one Willy D. Deter[a]la is also simulated, and PTTTI did not refute
respondents’ claim that Southern Comfort Bus Co., is a dummy corporation since it
has not operated any single bus under its name, even prior to the sale and up to
the present and that its principal business office at No. 4 Cathedral St., Ateneo
Avenue 4400 Naga City is not even known.
Hence, no actual sale transpired and, as such, there is no closure or cessation of
business that can serve as an authorized cause for the dismissal of respondents.
Eleazar S. Padillo vs. Rural Bank of Nabunturan, Inc., et al. G.R. No. 199338, 21
January 2013.

Facts:

Petitioner, the late Eleazar Padillo, was employed by respondent Rural Bank of
Nabunturan, Inc. as its SA Bookkeeper. Due to liquidity problems, the Bank took
out retirement/insurance plans with Philippine American Life and General
Insurance Company for all its employees in anticipation of its possible closure and
the concomitant severance of its personnel.

Respondent Mark S. Oropeza, the President of the Bank, bought majority shares of
stock in the Bank and took over its management which brought about its gradual
rehabilitation. The Bank’s finances improved and eventually, its liquidity was
regained.

Padillo suffered a mild stroke due to hypertension which consequently impaired his
ability to effectively pursue his work. He wrote a letter addressed to respondent
Oropeza expressing his intention to avail of an early retirement package. Despite
several follow-ups, his request remained unheeded.

Padillo was separated from employment due to his poor and failing health as
reflected in a Certification issued by the Bank. Not having received his claimed
retirement benefits, Padillo filedwith the NLRC Regional Arbitration Branch No. XI of
Davao City a complaint for the recovery of unpaid retirement benefits. He
asserted, among others, that the Bank had adopted a policy of granting its aging
employees early retirement packages, pointing out that one of his co-employees,
Nenita Lusan (Lusan), was accorded retirement benefits in the amount of
₱348,672.72 when she retired at the age of only fifty-three (53). The Bank and
Oropeza (respondents) countered that the claim of Padillo for retirement benefits
was not favorably acted upon for lack of any basis to grant the same.

The LA issued a Decision dismissing Padillo’s complaint but directed the Bank to
pay him the amount of ₱100,000.00 as financial assistance, treated as an
advance from the amounts receivable under the Philam Life Plan.

The NLRC’s Fifth Division reversed and set aside the LA’s ruling and ordered
respondents to pay Padillo the amount of ₱164,903.70 as separation pay, on top
of the ₱100,000.00 Philam Life Plan benefit.

The CA granted respondents’ petition for certiorari and rendered a decision


setting aside the NLRC’sResolutions, thereby reinstating the LA’s Decision but with
modification. It directed the respondents to pay Padillo the amount of ₱50,000.00
as financial assistance exclusive of the ₱100,000.00 Philam Life Plan benefit which
already matured.

The CA held that Padillo could not, absent any agreement with the Bank, receive
any retirement benefits pursuant to Article 300 of the Labor Code considering that
he was only fifty-five (55) years old when he retired.It likewise found the evidence
insufficient to prove that the Bank has an existing company policy of granting
retirement benefits to its aging employees.

Issue:

Whether or not separation pay on the ground of the disease under Article 297
should be given to Padillo

Ruling:

No. Separation pay should not be given.

At the outset, it must be maintained that the Labor Code provision on termination
on the ground of disease under Article 297 does not apply in this case, considering
that it was the petitioner and not the Bank who severed the employment relations.
As borne from the records, the clear import of Padillo’s September 10, 2007
letter and the fact that he stopped working before the foregoing date and never
reported for work even thereafter show that it was Padillo who voluntarily retired
and that he was not terminated by the Bank.

Thus, given the inapplicability of Article 297 of the Labor Code to the case at bar,
it necessarily follows that petitioners’ claim for separation pay anchored on such
provision must be denied.

What remains applicable, however, is the Labor Code provision on retirement. In


particular, Article 300 of the Labor Code as amended by Republic Act Nos.
7641 and 8558 partly provides:

Art. 300. Retirement. — Any employee may be retired upon reaching the
retirement age established in the collective bargaining agreement or other
applicable employment contract.

In case of retirement, the employee shall be entitled to receive such retirement


benefits as he may have earned under existing laws and any collective
bargaining agreement and other agreements: Provided, however, That an
employee's retirement benefits under any collective bargaining and other
agreements shall not be less than those provided herein.

In the absence of a retirement plan or agreement providing for retirement


benefits of employees in the establishment, an employee upon reaching the age
of sixty (60) years or more, but not beyond sixty-five (65) years which is hereby
declared the compulsory retirement age, who has served at least five (5) years in
the said establishment, may retire and shall be entitled to retirement pay
equivalent to at least one-half (1/2) month salary for every year of service, a
fraction of at least six (6) months being considered as one whole year.
Manlimos vs. NLRC, 242 SCRA 145 [1995]

Facts:

Petitioners were among the regular employees of the Super Mahogany Plywood
Corporation, a domestic corporation based in Butuan City. They were hired as
patchers, taper-graders and receivers-dryers.

However, a new owner/ management group acquired complete ownership of


the corporation headed by Alfredo Roxas.

Upon the change of ownership, petitioners continued to work for the new owner
until their termination when they received their separation pay and other benefits
due them. Each of them executed a release a waiver acknowledged by Atty.
Discipulo and Hearing Officer of Butuan City District Office of DOLE.

The new owner caused a publication for the hiring of workers. Petitioners filed their
applications and were hired on probationary basis except for Rosario Cuarto.

Two (2) of the employees hired were terminated for their alleged absence without
leave and were considered to have abandoned their work. The rest were
dismissed. Thus, the filing of a complaint for illegal dismissal.

The Labor Arbiter declared the dismissal invalid. Saying that the transfer of
ownership partook of a cessation of business operation not due to business
reverses and must comply with the following requisites: (1) service of written notice
to the employees and to the MOLE at least one (1) month before the intended
date thereof, (2) the cessation of or withdrawal from business operations must be
bona fide in character and (3) payment to the employees of termination pay
amounting to at least one-half month pay for each year of service or one month
pay whichever is higher. The first and third requisites were present in this case.
However, there was no cessation of operations which would lead to the dismissal
of employees. And that upon resumption of work, the complainants were regular
employees for they were engaged in work which was necessary and desirable to
the company’s operations. Thus, they could not be dismissed without cause and
due process.

NLRC however, reversed the judgment of the Labor Arbiter finding that the
change of ownership was made in good faith since there was no evidence that
the former owners conspired with the new owners to insulate the former
management of any liability to its workers. And sale or disposition of a business
enterprise which has been motivated by good faith is an element of exemption
from liability. Thus, an innocent transferee of a business has no liability to the
employees of the transferor to continue employing them. Nor is the transferee
liable for past unfair labor practices of the previous owner, except, when the
liability is assumed by the new employer under the contract of sale, or when
liability arises because the new owners participated in thwarting or defeating the
right of the employees.
Hence, this special civil action for certiorari.

Issue:

Whether or not the transfer of ownership was done in good faith making private
respondent not guilty of illegal dismissal

Ruling:

Yes. The transfer was done in good faith.

The rule is that sale or disposition of a business enterprise which has been
motivated by good faith is an element of exemption from liability. Thus, an
innocent transferee of a business has no liability to the employees of the transferor
to continue employing them. Nor is the transferee liable for past unfair labor
practices of the previous owner, except, when the liability is assumed by the new
employer under the contract of sale, or when liability arises because the new
owners participated in thwarting or defeating the right of the employees.

Where such transfer of ownership is in good faith, the transferee is under no


obligation to absorb the transferor’s employees as there is no law compelling such
absorption.

In this case, the transfer of ownership was made in good faith given that there was
no evidence that there was conspiracy to insulate the former management of
any liability to its workers. Thus, petitioners were validly dismissed.
. Nippon Housing Phil. Inc.,et al. vs. Maia Angela Leynes, G.R. 177816, 03 August
2011.

Facts:

Nippon Housing Philippines, Inc. (NPHI) hired respondent Maiah Angela Leynes for
the position of Property Manager.

Leynes had a misunderstanding with Engr. Honesto Cantuba, the Building


Engineer assigned at Bay Gardens Condominium Project (the Project), regarding
the extension of the latter working hours. Aside from instructing the security guards
to bar Engr. Cantuba from entry into the Project and to tell him to report to the
NHPI main office in Makati, Leynes also sent a letter by telefax to Joel Reyes, NHPI
HR Head, apprising the latter of Cantuba supposed insubordination and
disrespectful conduct. With Engr. Cantuba submission of a reply in turn accusing
Leynes of pride, conceit and poor managerial skills, Hiroshi Takada, NHPI VP, went
on to issue a memorandum, attributing the incident to "simple personal
differences" and directing Leynes to allow Engr. Cantuba to report back for work.

Disappointed with the foregoing management decision, Leynes submitted to a


letter asking for an emergency leave of absence for the supposed purpose of
coordinating with her lawyer regarding her resignation letter. While NHPI offered
the Property Manager position to Engr. Carlos Jose on as a consequence Leynes’
signification of her intention to resign, it also appears that Leynes sent another
letter to Reyes by telefax on the same day, expressing her intention to return to
work and to call off her planned resignation upon the advice of her lawyer.Leynes
was further served with a letter and memorandum relieving her from her position
and directing her to report to NHPI main office while she was on floating status.

Aggrieved, Leynes lost no time in filing against NHPI and its above-named officers
a complaint for illegal dismissal, unpaid salaries, benefits, damages and attorney
fees before the NLRC. NHPI and its officers asserted that the management
exercise of the prerogative to put an employee on floating status for a period not
exceeding six months was justified in view of her threatened resignation from her
position and BGCC request for her replacement.During the pendency of the case,
however, Reyes eventually served the DOLE and Leynes with a notice terminating
her services effective on the ground of redundancy or lack of a posting
commensurate to her position at the Project.Leynes was offered by NHPI the sum
ofP28,188.16 representing her unpaid wages, proportionate 13th month pay, tax
refund and service incentive leave pay (SILP).

The LA found that NHPI act of putting Leynes on floating status was equivalent to
termination from employment without just cause and compliance with the twin
requirements of notice and hearing.

On appeal, the NLRC reversed the LA decision. Leynes elevated the case to the
CA on a Rule 65 petition for certiorari and the CA reversed the NLRC decision.
Issue:

Whether or not the CA erred in finding that Leynes was constructively dismissed
when she was placed on floating status prior to her termination from employment
on the ground of redundancy

Ruling:

Yes. CA erred in finding that Leynes was constructively dismissed.

Although the CA correctly found that the record is bereft of any showing that
Leynes was unacceptable to BGCC, the evidence the parties adduced a quo
clearly indicates that petitioners were not in bad faith when they placed the
former under floating status. Disgruntled by NHPI countermanding of her decision
to bar Engr. Cantuba from the Project, Leynes twice signified her intention to
resign from her position. In her application letter for an immediate emergency
leave, Leynes also distinctly expressed her dissatisfaction over NHPI resolution of
her dispute with Engr. Cantuba and announced her plan of coordinating with her
lawyer regarding her resignation letter.

In view of the sensitive nature of Leynes position and the critical stage of the
Project business development, NHPI was constrained to relay the situation to
BGCC which, in turn, requested the immediate adoption of remedial measures
from Takada, including the appointment of a new Property Manager for the
Project. Upon BGCC recommendation, NHPI consequently hired Engr. Jose on 13
February 2002 as Leynes replacement. Far from being the indication of bad faith
the CA construed the same to be, these factual antecedents suggest that NHPI
immediate hiring of Engr. Jose as the new Property Manager for the Project was
brought about by Leynesown rash announcement of her intention to resign from
her position. Although she subsequently changed her mind and sent Reyes a letter
by telefax on 13 February 2002 announcing the reconsideration of her planned
resignation and her intention to return to work on 15 February 2002, Leynes
evidently had only herself to blame for precipitately setting in motion the events
which led to NHPI hiring of her own replacement.

The record, moreover, shows that NHPI simply placed her on floating status "until
such time that another project could be secured" for her. Traditionally invoked by
security agencies when guards are temporarily sidelined from duty while waiting
to be transferred or assigned to a new post or client, Article 286 of the Labor Code
has been applied to other industries when, as a consequence of the bona fide
suspension of the operation of a business or undertaking, an employer is
constrained to put employees on floating status for a period not exceeding six
months.

Considering that even labor laws discourage intrusion in the employer's judgment
concerning the conduct of their business, courts often decline to interfere in their
legitimate business decisions,absent showing of illegality, bad faith or arbitrariness.
Indeed, the right of employees to security of tenure does not give them vested
rights to their positions to the extent of depriving management of its prerogative to
change their assignments or to transfer them.The record shows that Leynes filed
the complaint for actual illegal dismissal from which the case originated on 22
February 2002 or immediately upon being placed on floating status as a
consequence of NHPI hiring of a new Property Manager for the Project. The rule is
settled, however, that "off-detailing" is not equivalent to dismissal, so long as such
status does not continue beyond a reasonable time and that it is only when such
a "floating status" lasts for more than six months that the employee may be
considered to have been constructively dismissed. A complaint for illegal dismissal
filed prior to the lapse of said six-month and/or the actual dismissal of the
employee is generally considered as prematurely filed.

Viewed in the light of the foregoing factual antecedents, the Court finds that the
CA reversibly erred in holding petitioners liable for constructively dismissing Leynes
from her employment. There is said to be constructive dismissal when an act of
clear discrimination, insensitivity or disdain on the part of the employer has
become so unbearable as to leave an employee with no choice but to forego
continued employment. Constructive dismissal exists where there is cessation of
work because continued employment is rendered impossible, unreasonable or
unlikely, as an offer involving a demotion in rank and a diminution in pay. Stated
otherwise, it is a dismissal in disguise or an act amounting to dismissal but made to
appear as if it were not.

With no other client aside from BGCC for the building management side of its
business, the Court finds that NHPI was acting well within its prerogatives when it
eventually terminated Leynesservices on the ground of redundancy. One of the
recognized authorized causes for the termination of employment, redundancy
exists when the service capability of the workforce is in excess of what is
reasonably needed to meet the demands of the business enterprise. A redundant
position is one rendered superfluous by any number of factors, such as overhiring
of workers, decreased volume of business, dropping of a particular product line
previously manufactured by the company or phasing out of service activity priorly
undertaken by the business.It has been held that the exercise of business
judgment to characterize an employee service as no longer necessary or
sustainable is not subject to discretionary review where, as here, it is exercised
there is no showing of violation of the law or arbitrariness or malice on the part of
the employer.

Having been validly terminated on the ground of redundancy, Leynes is entitled


to separation pay equivalent to one-month salary for every year of service but not
to the backwages adjudicated in her favor by the Labor Arbiter.
. Maya Farms Employees Org. vs. NLRC, 239 SCRA 508

Facts:

Private respondents Maya Farms, Inc. and Maya Realty and Livestock Corporation
belong to the Liberty Mills group of companies whose undertakings include the
operation of a meat processing plant which produces ham, bacon, cold cuts,
sausages and other meat and poultry products.

Petitioners, on the other hand, are the exclusive bargaining agents of the
employees of Maya Farms, Inc. and the Maya Realty and Livestock Corporation.

Private respondents announced the adoption of an early retirement program as a


cost-cutting measure considering that their business operations suffered major
setbacks over the years. The program was voluntary and could be availed of only
by employees with at least eight (8) years of service.Dialogues were thereafter
conducted to give the parties an opportunity to discuss the details of the
program. Accordingly, the program was amended to reduce the minimum
requirement of eight (8) years of service to only five (5) years.

However, the response to the program was nil. There were only a few takers. To
avert further losses, private respondents were constrained to look into the
companies' organizational set-up in order to streamline operations. Consequently,
the early retirement program was converted into a special redundancy program
intended to reduce the work force to an optimum number so as to make
operations more viable.

A total of sixty-nine (69) employees from the two companies availed of the special
redundancy program.

The two companies sent letters to sixty-six (66) employees informing them that their
respective positions had been declared redundant. The notices likewise stated
that their services would be terminated effective thirty (30) days from receipt
thereof. Separation benefits, including the conversion of all earned leave credits
and other benefits due under existing CBAs were thereafter paid to those
affected.

A notice of strike was filed by the petitioners which accused private respondents,
among others, of unfair labor practice, violation of CBA and discrimination.
Conciliation proceedings were held by the National Conciliation and Mediation
Board (NCMB) but the parties failed to arrive at a settlement.

The two companies filed a petition with the Secretary of Labor and Employment
asking the latter to assume jurisdiction over the case and/or certify the same for
compulsory arbitration. Thus, the then Acting Labor Secretary (now Secretary)
Nieves Confesor certified the case to herein public respondent for compulsory
arbitration.
The parties were called to a hearing to identify the issues involved in the case.
Thereafter, they were ordered to submit their respective position papers.

In their position paper, petitioners averred that in the dismissal ofsixty-six (66) union
officers and members on the ground of redundancy, private respondents
circumvented the provisions in their CBA, more particularly, Section 2, Article III
thereof. Said provision reads:

Sec. 2. LIFO RULE. — In all cases of lay-off or retrenchment resulting in termination


of employment in the line of work, theLast-In-First-Out (LIFO) Rule must always be
strictly observed.

Petitioners also alleged that the companies' claim that they were in economic
crisis was fabricated because in 1990, a net income of over 83 million pesos was
realized by Liberty Flour Mills Group of Companies. Furthermore, with the
termination of the sixty-six (66) employees pursuant to the special redundancy
program, the remaining work force, especially the drivers, became overworked
and overburdened so much so that they found themselves doing overtime work
and reporting for duty even during rest days.

Invoking the workers' constitutional right to security of tenure, petitioners prayed


for the reinstatement of the sixty-six (66) employees and the payment of attorney's
fees as they were constrained to hire the services of counsel in order to protect
the workers' rights.

On their part, private respondents contend that their decision to implement a


special redundancy program was an exercise of management prerogative which
could not be interfered with unless it is shown to be tainted with bad faith and ill
motive. Private respondents explained that they had no choice but to reduce
their work force, otherwise, they would suffer more losses. Furthermore, they
denied that the program violated CBA provisions.

On June 29, 1992, public respondent rendered a decision,confirming the legality


of the separation of the 66 employees of management thereby dismissing the
charges of violation of CBA and unfair labor practice on the part of
management.

Not satisfied with the above-quoted decision, petitioners interposed the instant
petition.

Issue:

Whether or not the NLRC grossly erred and gravely abused its discretion when it
ruled that:

(a) the termination of the sixty-six (66) employees was in accordance with the LIFO
rule in the CBA;
(b) the termination of the sixty-six (66) employees was in accordance with Article
283 of the Labor Code

Ruling:

No. The NLRC did not grossly err and gravely abuse its discretion in its rulings.

The termination of the sixty-six employees was done in accordance with Article
283 of the Labor Code. The basis for this was the companies' study to streamline
operations so as to make them more viable. Positions which overlapped each
other, or which are in excess of the requirements of the service, were declared
redundant.

Article 283 provides:

Art. 283. Closure of establishment and reduction of personnel. — The


employer may also terminate the employment of any employee due
to the installation of labor-saving devises, redundancy, retrenchment
to prevent losses or the closing or cessation of operation of the
establishment or undertaking unless the closing is for the purpose of
circumventing in the provisions of this title, by serving a written notice
on the workers and the Department of Labor and Employment at
least one (1) month before the intended date thereof. In case of
retrenchment to prevent losses of operations of establishment or
undertaking not due to serious business losses or financial reverses, the
one (1) month pay or at least
one-half (1/2) pay for every year of service, whichever is higher. A
fraction of at least six (6) months shall be considered one (1) whole
year.

The rule is well-settled that labor laws discourage interference with an employer's
judgment in the conduct of his business. Even as the law is solicitous of the welfare
of employees, it must also protect the right of an employer to exercise what are
clearly management prerogatives. As long as the company's exercise of the same
is in good faith to advance its interest and not for the purpose of defeating or
circumventing the rights of employees under the laws or valid agreements, such
exercise will be upheld. 11

The NLRC correctly held that private respondents did not violate the LIFO rule
under Section 2, Article III of the CBA which provides:

Sec. 2. LIFO RULE. In all cases of lay-off or retrenchment resulting in


termination of employment in the line of work, the
Last-in-First-Out (LIFO) Rule must always be strictly observed.

It is not disputed that the LIFO rule applies to termination of employment in the line
of work. 12 Verily, what is contemplated in the LIFO rule is that when there are two
or more employees occupying the same position in the company affected by the
retrenchment program, the last one employed will necessarily be the first to go.

Finally, contrary to petitioners' contention, there is nothing on record to show that


the 30-day notice of termination to the workers was disregarded and that the
same substituted with separation pay by private respondents. As found by public
respondent, written notices of separation were sent to the employees on January
17, 1992. The notices expressly stated that the termination of employment was to
take effect one month from receipt thereof. Therefore, the allegation that
separation pay was given in lieu of the 30-day notice required by law is baseless.

[G.R. No. 111222. January 18, 1995.]

CITIBANK, N.A., Petitioner, v. DR. JOSE C. GATCHALIAN, in his capacity as


Voluntary Arbitrator, CITIBANK PHILIPPINES EMPLOYEES UNION (CPEU) and EMY
LLONILLO, Respondents.

FACTS:
Petitioner Citibank N.A. prays for the reversal of the decision of voluntary arbitrator
Dr. Jose C. Gatchalian reinstating respondent Emerita "Emy" Llonillo to her former
position as clerk-typist/maker without backwages.

Respondent Llonillo, together with Teresita Supnad, her co-employee and


Florence Verendia, an employee of Asian-Pacific Broadcasting Company, Inc.
(APBCI), were implicated in a scheme to defraud petitioner bank in the amount of
P740,000.00.
Petitioner bank received thirty-one (31) applications from alleged APBCI
employees 1 for the issuance of Citibank credit cards, popularly known as
Mastercard.

A Citibank employee verified by phone the data which appeared on the


application forms. It was Florence Verendia, as secretary of the APBCI General
Manager, who answered the check calls. The applications were then approved
and the corresponding new and unsigned credit cards were issued.
Petitioner bank’s policy is for new and unsigned credit cards to be released only
to the cardholders concerned or their duly authorized representatives. However, a
Citibank employee may himself take delivery of new and unsigned credit cards
after accomplishing a Card Pull-Out Request Form wherein the employee assumes
the responsibility of delivering the same to the cardholder concerned.

Teresita Supnad, an employee of petitioner bank and Florence Verendia, took


delivery of nineteen (19) credit cards issued in the name of the said alleged APBCI
credit applicants.
On the other hand, on five (5) separate occasions, respondent Llonillo personally
picked up the newly approved and unsigned credit cards issued to the other
seven (7) alleged APBCI employee and delivered them to Verendia.

In July 1992, petitioner bank discovered that the credit card applications of the
alleged APBCI employees were fictitious. Per report of the PNP-Crime Laboratory,
Supnad and Verendia falsified the signature of the alleged applicants.
Petitioner bank required respondent Llonillo to explain. In her reply, Llonillo
admitted she personally picked up seven (7) credit cards issued to Anjenette
Caballa, Miriam Ramiro, Alen Malic, Caroline Ramiro, Cecilia Ibañez, Lalaine Perez
and Marife Bacuetes. She allegedly wanted to help the bank deliver "fast,
competent and problem-free service to clients." She disclaimed knowledge that
the APBCI applicants were fictitious. She also denied participation in the
fraudulent use of said credit cards.
Respondent revealed that on five (5) occasions, she was asked by Verendia to
take delivery of newly approved and unsigned credit cards issued to some of the
latter’s alleged officemates, namely: Anjenette Caballa, Miriam Ramiro, Allen
Malic, Caroline Ramiro, Cecilia Ibañez, Lalaine Perez and Marife Bacuetes. 4 On
said occasions, Verendia informed her by telephone she was on the way to the
bank to pick up some of the newly approved credit cards issued to her alleged
co-employees at APBCI. Each time, she acceded to Verendia’s request and
delivered the newly approved and unsigned credit cards to the latter without
knowing that the cardholders were fictitious. In every case, respondent signed the
Card Pull-Out Request Form, acknowledging receipt of the credit cards and
taking responsibility for their delivery to the cardholder concerned. Respondent
further disclosed that Verendia was introduced to her by a mutual friend.

On November 19, 1992, the committee recommended the termination of


respondent’s employment with the bank for loss of trust and confidence and gross
negligence. Petitioner bank adopted the committee’s recommendation and
notified respondent of her immediate dismissal.
On the basis of the record and the evidence presented by petitioner bank,
voluntary arbitrator Dr. Gatchalian rendered a decision ordering the
reinstatement of respondent Llonillo without payment of backwages.
In ordering the reinstatement of respondent Llonillo, the voluntary arbitrator ruled
that the law requires that an employee’s negligence, to be a valid ground for
dismissal, must be both gross and habitual. He did not find the negligence of
respondent Llonillo as within this category.

ISSUE:
Whether or not there was illegal termination of employment
HELD:
NO.
Gross negligence implies a want or absence of or failure to exercise slight care or
diligence, or the entire absence of care. It evinces a thoughtless disregard of
consequences without exerting any effort to avoid them.
The evidence on record succinctly established the gross negligence of
respondent Llonillo. All of her acts and omissions were in patent violation of
petitioner bank’s policy that an employee may take delivery of newly approved
and unused credit cards issued in another’s name, but in doing so, he/she
assumes the responsibility of delivering the credit card to the cardholder
concerned or to the latter’s duly authorized representative. We also rule that
respondent Llonillo’s negligence is both gross and habitual. It was proved that she
picked up the newly approved credit cards on five (5) separate occasions and
delivered the same to Verendia and the latter’s messenger. Certainly, these
repetitive acts and omissions bespeak of habituality.

G.R. No. 80609 August 23, 1988

PHILIPPINE LONG DISTANCE TELEPHONE COMPANY, petitioner,


vs.
THE NATIONAL LABOR RELATIONS COMMISSION and MARILYN
ABUCAY, respondents.

FACTS:
Marilyn Abucay, a traffic operator of the Philippine Long Distance Telephone
Company, was accused by two complainants of having demanded and
received from them the total amount of P3,800.00 in consideration of her promise
to facilitate approval of their applications for telephone installation. 1 Investigated
and heard, she was found guilty as charged and accordingly separated from the
service.2 She went to the Ministry of Labor and Employment claiming she had
been illegally removed. After consideration of the evidence and arguments of the
parties, the company was sustained and the complaint was dismissed for lack of
merit. Nevertheless, the dispositive portion of labor arbiter's decision declared:

WHEREFORE, the instant complaint is dismissed for lack of merit.

Considering that Dr. Helen Bangayan and Mrs. Consolacion Martinez


are not totally blameless in the light of the fact that the deal
happened outhide the premises of respondent company and that
their act of giving P3,800.00 without any receipt is tantamount to
corruption of public officers, complainant must be given one month
pay for every year of service as financial assistance. 3
Both the petitioner and the private respondent appealed to the National Labor
Relations Board, which upheld the said decision in toto and dismissed the
appeals. 4 The private respondent took no further action, thereby impliedly
accepting the validity of her dismissal. The petitioner, however, is now before us to
question the affirmance of the above- quoted award as having been made with
grave abuse of discretion.

The position of the petitioner is simply stated: It is conceded that an employee


illegally dismissed is entitled to reinstatement and backwages as required by the
labor laws. However, an employee dismissed for cause is entitled to neither
reinstatement nor backwages and is not allowed any relief at all because his
dismissal is in accordance with law. In the case of the private respondent, she has
been awarded financial assistance equivalent to ten months pay corresponding
to her 10 year service in the company despite her removal for cause. She is,
therefore, in effect rewarded rather than punished for her dishonesty, and without
any legal authorization or justification. The award is made on the ground of equity
and compassion, which cannot be a substitute for law. Moreover, such award
puts a premium on dishonesty and encourages instead of deterring corruption.

For its part, the public respondent claims that the employee is sufficiently punished
with her dismissal. The grant of financial assistance is not intended as a reward for
her offense but merely to help her for the loss of her employment after working
faithfully with the company for ten years.

ISSUE:

Whether or not the length of service of an employee is to be considered in


granting a separation pay for a terminated employee

HELD:

NO.

The court ruled that the grant of separation pay in the case at bar is unjustified.
The private respondent has been dismissed for dishonesty, as found by the labor
arbiter and affirmed by the NLRC and as she herself has impliedly admitted. The
fact that she has worked with the PLDT for more than a decade, if it is to be
considered at all, should be taken against her as it reflects a regrettable lack of
loyalty that she should have strengthened instead of betraying during all of her 10
years of service with the company. If regarded as a justification for moderating
the penalty of dismissal, it will actually become a prize for disloyalty, perverting the
meaning of social justice and undermining the efforts of labor to cleanse its ranks
of all undesirables.

if the employee who steals from the company is granted separation pay even as
he is validly dismissed, it is not unlikely that he will commit a similar offense in his
next employment because he thinks he can expect a like leniency if he is again
found out. This kind of misplaced compassion is not going to do labor in general
any good as it will encourage the infiltration of its ranks by those who do not
deserve the protection and concern of the Constitution.

The policy of social justice is not intended to countenance wrongdoing simply


because it is committed by the underprivileged. At best it may mitigate the
penalty but it certainly will not condone the offense. Compassion for the poor is
an imperative of every humane society but only when the recipient is not a rascal
claiming an undeserved privilege. Social justice cannot be permitted to be refuge
of scoundrels any more than can equity be an impediment to the punishment of
the guilty. Those who invoke social justice may do so only if their hands are clean
and their motives blameless and not simply because they happen to be poor. This
great policy of our Constitution is not meant for the protection of those who have
proved they are not worthy of it, like the workers who have tainted the cause of
labor with the blemishes of their own character.
G.R. No. 125303 June 16, 2000

DANILO LEONARDO, petitioner,


vs.
NATIONAL LABOR RELATIONS COMMISSION and REYNALDO'S MARKETING
CORPORATION, ET. AL., respondents.

x - - - - - - - - - - - - - - - - - - - - - - -x

G.R. No. 126937

AURELIO FUERTE and DANILO LEONARDO, petitioners,


vs.
RAUL T. AQUINO, VICTORIANO R. CALAYCAY and ROGELIO I. RALAYA, as Chairman
and Members of the NATIONAL LABOR RELATIONS COMMISSION, SECOND
DIVISION and REYNALDO'S MARKETING and/or REYNALDO PADUA, respondents.

FACTS:

Before us is a consolidation of G.R. Nos. 125303 and 126937, both petitions


for certiorari under Rule 65 of the 1997 Rules of Civil Procedure, seeking the
annulment of a Decision1 and Resolution2 dated March 28, 1996 and May 29, 1996,
respectively, of the public respondent in NLRC NCR 00-02-01024-92.

Petitioner AURELIO FUERTE was originally employed by private respondent


REYNALDO'S MARKETING CORPORATION on August 11, 1981 as a muffler specialist,
receiving P45.00 per day. When he was appointed supervisor in 1988, his
compensation was increased to P122.00 a day, augmented by a weekly
supervisor's allowance of P600.00. On the other hand, DANILO LEONARDO was
hired by private respondent on March 4, 1988 as an auto-aircon mechanic at a
salary rate of P35.00 per day. His pay was increased to P90.00 a day when he
attained regular status six months later. From such time until he was allegedly
terminated, he claims to have also received a monthly allowance equal to
P2,500.00 as his share in the profits of the auto-aircon division.

FUERTE alleges that on January 3, 1992, he was instructed to report at private


respondent's main office where he was informed by the company's personnel
manager that he would be transferred to its Sucat plant due to his failure to meet
his sales quota, and for that reason, his supervisor's allowance would be
withdrawn. For a short time, FUERTE reported for work at the Sucat plant; however,
he protested his transfer, subsequently filing a complaint for illegal termination.
On his part, LEONARDO alleges that on April 22, 1991, private respondent was
approached by the same personnel manager who informed him that his services
were no longer needed. He, too, filed a complaint for illegal termination.

The case was heard by Labor Arbiter Jesus N. Rodriguez, Jr. On December 15,
1994, Labor Arbiter Emerson C. Tumanon, to whom the case was subsequently
assigned, rendered judgment in favor of petitioners. The dispositive portion of the
arbiter's decision3 states:

WHEREFORE, premises considered, respondents are hereby ordered:

1. To reinstate complainant Aurelio Fuerte, to the position he was holding


before the demotion, and to reinstate likewise complainant Danilo
Leogardo to his former position or in lieu thereof, they be reinstated through
payroll reinstatement without any of them losing their seniority rights and
other privileges, inclusive of allowance and to their other benefits;

2. To pay AURELIO FUERTE, the sum of TWO HUNDRED EIGHTY THOUSAND


EIGHT HUNDRED NINETY-SIX PESOS and 72/100 (280,896.72);

3. To pay DANILO LEOGARDO, the sum of TWO HUNDRED FORTY ONE


THOUSAND NINE HUNDRED EIGHT PESOS and 67/100 (P241,908.67).

On appeal, the respondent Commission modified the aforesaid decision as


follows:

WHEREFORE, premises considered, the Decision of December 15, 1994 is


hereby modified as follows:

1. Ordering the reinstatement of complainant Aurelio Fuerte to his former


position without loss of his seniority rights but without backwages;

2. Dismissing the complaint of Danilo leonardo [sic] for lack of merit; and

3. Deleting the rests [sic] of the monetary award as well as the award of
moral damages and attorney's fees in favor of the complainants also for
lack of merit.

On July 1, 1996, LEONARDO, represented by the Public Attorney's Office, filed G.R.
No. 125303, a special civil action for certiorari assailing the Commission's decision
and resolution. However, on November 15, 1996, FUERTE, again joined by
LEONARDO, filed G.R. No. 126937, a similar action praying for the annulment of the
same decision and resolution.

On October 7, 1997, private respondent filed its Comment 5 to the petition in G.R.
No. 125303. On April 2, 1997, it filed its Comments 6 to the petition in G.R. No.
126937 with a motion to drop petitioner LEONARDO and consolidate G.R. No.
126937 with G.R. No. 125303. We granted private respondent's motion in our
Resolution dated June 16, 1997.7

Private respondent contends that it never terminated petitioners' services. In


FUERTE's case, private respondent claims that the latter was demoted pursuant to
a company policy intended to foster competition among its employees. Under
this scheme, private respondent's employees are required to comply with a
monthly sales quota. Should a supervisor such as FUERTE fail to meet his quota for a
certain number of consecutive months, he will be demoted, whereupon his
supervisor's allowance will be withdrawn and be given to the individual who takes
his place. When the employee concerned succeeds in meeting the quota again,
he is re-appointed supervisor and his allowance is restored.9

With regard to LEONARDO, private respondent likewise insists that it never severed
the former's employment. On the contrary, the company claims that it was
LEONARDO who abandoned his post following an investigation wherein he was
asked to explain an incident of alleged "sideline" work which occurred on April 22,
1991. It would appear that late in the evening of the day in question, the driver of
a red Corolla arrived at the shop looking for LEONARDO. The driver said that, as
prearranged, he was to pick up LEONARDO who would perform a private service
on the vehicle. When reports of the "sideline" work reached management, it
confronted LEONARDO and asked for an explanation. According to private
respondent, LEONARDO gave contradictory excuses, eventually claiming that the
unauthorized service was for an aunt. When pressed to present his aunt, it was
then that LEONARDO stopped reporting for work, filing his complaint for illegal
dismissal some ten months after his alleged termination.

ISSUE:

Whether or not demotion of employment is valid

HELD:

YES.

This arrangement appears to us to be an allowable exercise of company rights.


An employer is entitled to impose productivity standards for its workers, and in
fact, non-compliance may be visited with a penalty even more severe than
demotion. Thus,

[t]he practice of a company in laying off workers because they failed to


make the work quota has been recognized in this jurisdiction. (Philippine
American Embroideries vs. Embroidery and Garment Workers, 26 SCRA 634,
639). In the case at bar, the petitioners' failure to meet the sales quota
assigned to each of them constitute a just cause of their dismissal,
regardless of the permanent or probationary status of their employment.
Failure to observe prescribed standards of work, or to fulfill reasonable work
assignments due to inefficiency may constitute just cause for dismissal. Such
inefficiency is understood to mean failure to attain work goals or work
quotas, either by failing to complete the same within the allotted
reasonable period, or by producing unsatisfactory results. This management
prerogative of requiring standards may be availed of so long as they are
exercised in good faith for the advancement of the employer's interest. 14

Neither can we say that FUERTE's actions are indicative of abandonment. To


constitute such a ground for dismissal, there must be (1) failure to report for work or
absence without valid or justifiable reason; and (2) a clear intention, as
manifested by some overt acts, to sever the employer-employee
relationship. 15 We have accordingly held that the filing of a complaint for illegal
dismissal, as in this case, is inconsistent with a charge of abandonment.

Neither do we discern any grave abuse of discretion in the Commission's ruling


dismissing LEONARDO's complaint. On this score, the public respondent found
that:

Coming now to the case of complainant Danilo Leonardo, the evidence on


record indubitably shows that he abandoned his work with the respondents.
As sufficiently established by respondents, complainant Leonardo, after
being pressed by the respondent company to present the customer
regarding his unauthorized solicitation of sideline work from the latter and
whom he claims to be his aunt, he never reported back to work anymore.
This finding is bolstered by the fact that after he left the respondent
company, he got employed with Dennis Motors Corporation as Air-Con
Mechanic from October 12, 1992 to April 3, 1995 (Certification attached to
respondents' Manifestation filed June 5, 1996)

It must be stressed that while Leonardo alleges that he was illegally


dismissed from his employment by the respondents, surprisingly, he never
stated any reason why the respondents would want to ease him out from his
job. Moreover, why did it take him ten (10) long months to file his case if
indeed he was aggrieved by respondents. All the above facts clearly point
that the filing of his case is a mere afterthought on the part of complainant
Leonardo.

LEONARDO protests that he was never accorded due process.1awphi1 This begs
the question, for he was never terminated; 20 he only became the subject of an
investigation in which he was apparently loath to participate.

As testified to by Merlin P. Orallo, the personnel manager, he was given a


memorandum 21 asking him to explain the incident in question, but he refused to
receive it. 22 In an analogous instance, we held that an employee's refusal to sign
the minutes of an investigation cannot negate the fact that he was accorded
due process. 23 So should it be here. We find no reason to disturb the Commission's
ruling that LEONARDO had abandoned his position, the instant case being a
petition for certiorari where questions of fact are not entertained. 24 Whether a
worker has abandoned his employment is essentially a question of fact. 25 We
reiterate that it is not for us "to neither re-examine conflicting evidence, re-
evaluate the credibility of witnesses, nor substitute the findings of fact of an
administrative tribunal which has gained expertise in its special field”.

G.R. No. 188002 February 1, 2010

GOODRICH MANUFACTURING CORPORATION & MR. NILO CHUA GOY, Petitioners,


vs.
EMERLINA ATIVO, LOVITO SEBUANO, MICHAEL FERNANDEZ, JUNIFER* CASAS,
ROLANDO ISLA, ELISEO DEL ROSARIO, MARK JON MARTIN, EDISON GAMIDO, WARRY
BALINTON, ROBERT RAGO and ROBERTO MENDOZA, Respondents.

FACTS:

Respondents are former employees of petitioner Goodrich Manufacturing


Corporation (Goodrich) assigned as machine or maintenance operators for the
different sections of the company. Sometime in the latter part of 2004, on account
of lingering financial constraints, Goodrich gave all its employees the option to
voluntarily resign from the company. Several employees, including respondents,
decided to avail of the voluntary resignation option. On December 29, 2004,
respondents were paid their separation pay.6 On January 3, 2005, respondents
executed their respective waivers and quitclaims.7

The following day, January 4, 2005, some of Goodrich’s former employees,


including herein respondents, filed complaints against Goodrich for illegal
dismissal with prayer for payment of their full monetary benefits before the NLRC.
Despite several conferences, no amicable settlement was reached by the parties.

On November 22, 2005, Labor Arbiter Florentino R. Darlucio rendered a Decision


declaring that there was no illegal dismissal but held that petitioners were still
liable to the respondents for their unpaid emergency cost of living allowance
(ECOLA), 13th month pay, and service incentive leave (SIL) pay. The Labor Arbiter
likewise found the separation pay paid by Goodrich to be insufficient.

ISSUE:

Whether or not the release, waiver and quitclaim signed by respondents are valid
and binding
HELD:

It is true that the law looks with disfavor on quitclaims and releases by employees
who have been inveigled or pressured into signing them by unscrupulous
employers seeking to evade their legal responsibilities and frustrate just claims of
employees.14 In certain cases, however, the Court has given effect to quitclaims
executed by employees if the employer is able to prove the following requisites, to
wit: (1) the employee executes a deed of quitclaim voluntarily; (2) there is no
fraud or deceit on the part of any of the parties; (3) the consideration of the
quitclaim is credible and reasonable; and (4) the contract is not contrary to law,
public order, public policy, morals or good customs, or prejudicial to a third person
with a right recognized by law.15

The pronouncement in Periquet v. National Labor Relations Commission 16 on this


matter cannot be more explicit:

Not all waivers and quitclaims are invalid as against public policy. If the
agreement was voluntarily entered into and represents a reasonable settlement, it
is binding on the parties and may not later be disowned simply because of a
change of mind. It is only where there is clear proof that the waiver was wangled
from an unsuspecting or gullible person, or the terms of settlement are
unconscionable on its face, that the law will step in to annul the questionable
transaction. But where it is shown that the person making the waiver did so
voluntarily, with full understanding of what he was doing, and the consideration
for the quitclaim is credible and reasonable, the transaction must be recognized
as a valid and binding undertaking.17

In the case at bar, both the Labor Arbiter and the NLRC ruled that respondents
executed the quitclaims absent any coercion from the petitioners following their
voluntary resignation from the company.

The contents of the quitclaim documents that have been signed by the
respondents are simple, clear and unequivocal.23 The records of the case are
bereft of any substantial evidence to show that respondents did not know that
they were relinquishing their right short of what they had expected to receive and
contrary to what they have so declared. Put differently, at the time they were
signing their quitclaims, respondents honestly believed that the amounts received
by them were fair and reasonable settlements of the amounts which they would
have received had they refused to voluntarily resign from the said company.
Skippers United Pacific, Inc. and Skippers Maritime Services, Inc. Ltd. vs. Nathaniel
Doza, et al., G.R. No. 175558. 08 February 2012

Facts:
Skippers United Pacific, Inc. deployed, in behalf of Skippers, De Gracia, Lata, and
Aprosta to work on board the vessel MV Wisdom Star.

De Gracia, et al., claimed that Skippers failed to remit their respective allotments
for almost five months, compelling them to air their grievances with the Romanian
Seafarers Free Union.||| On 28 January 1999, De Gracia, et al. were
unceremoniously discharged from MV Wisdom Stars and immediately
repatriated.Upon arrival in the Philippines, De Gracia, et al. filed a complaint for
illegal dismissal with the Labor Arbiter on 4 April 1999 and prayed for payment of
their home allotment for the month of December 1998, salaries for the unexpired
portion of their contracts, moral damages, exemplary damages, and attorney's
fees.|||

Skippers, on the other hand, claims that at around 2:00 a.m. on 3 December 1998,
Skippers alleges that De Garcia smelling strongly of alcohol went to the cabinof
Gabriel Oleszek, MV Wisdom Stars’ Master and was rude, shouting noisily to the
master. De Gracia left the master's cabin after a few . This incident was
evidenced by the Captain's Report sent via telex to Skippers on said date.
Furthermore, Skippers also claim that on January 22, 1999, Aprosta, De Gracia,
Lata and Daza arrived in the master’s cabin and demanded immediate
repatriation because they were not satisfied with theship. De Gracia, et al.
threatened that they may become crazy any moment and demanded for all
outstanding payments due to them. The incident is evidenced by a telex of
Cosmoship MV Wisdom toskippers but had conflicting dates.
The LA dismissed the seafarers’ complaint as the seafarers’ demand for immediate
repatriation due to the dissatisfaction with the ship is considered a voluntary pre-
termination of employment. Such act was deemed akin to resignation recognized
under Article 285 of the LC. The LA gave credence to the telex of the master’s
report that the seafarers indeed demanded immediate repatriation.

The NLRC agreed with the LA’s decision.

TheCA however reversed the LA’s and the NLRC’s decision.The Court
deemedthe telex message as aself-serving document that does not satisfy the
requirement of substantial evidence, or that amount ofrelevant evidence which a
reasonable mind might accept as adequate to justify the conclusion
thatpetitioners indeed voluntarily demanded their immediate repatriation.
Aggrieved, Skippers appeals the case with the Supreme Court.

Issue:
Whether or not the Court of Appeals seriously erred in not giving due credence to
the master's telex message showing that the respondents voluntarily requested to
be repatriated
Ruling of the Supreme Court:
No.For a worker's dismissal to be considered valid, it must comply with both
procedural and substantive due process. The legality of the manner of dismissal
constitutes procedural due process, while the legality of the act of dismissal
constitutes substantive due process.In this case, there was no written notice
furnished to De Gracia, et al., regarding the cause of their dismissal. Cosmoship
furnished a written notice (telex) to Skippers, the local manning agency, claiming
that De Gracia, et al., were repatriated because the latter voluntarily pre-
terminated their contracts. This telex was given credibility and weight by the Labor
Arbiter and NLRC in deciding that there was pre-termination of the employment
contract "akin to resignation" and no illegal dismissal. However, as correctly ruled
by the CA, the telex message is "a biased and self-serving document that does
not satisfy the requirement of substantial evidence." If, indeed, De Gracia, et al.,
voluntarily pre-terminated their contracts, then De Gracia, et al., should have
submitted their written resignations.

Article 285 of the Labor Code recognizes termination by the employee of


the employment contract by "serving written notice on the employer at least one
(1) month in advance." Given that provision, the law contemplates the
requirement of a written notice of resignation. In the absence of a written
resignation, it is safe to presume that the employer terminated the seafarers. In
addition, the telex message relied upon by the Labor Arbiter and NLRC bore
conflicting dates of 22 January 1998 and 22 January 1999, giving doubt to the
veracity and authenticity of the document. In 22 January 1998, De Gracia, et al.,
were not even employed yet by the foreign principal. For these reasons, the
dismissal of De Gracia, et al., was illegal.

CheryllLeus Vs. St. Scholastica’sWestgrove G.R. No. 187226, 28 Jan 2015

Facts:
Petitioner was hired by St. Scholastica's College Westgrove (SSCW), a Catholic
educational institution, as a non-teaching personnel.
Sometime in 2003, the petitioner and her boyfriend conceived a child out of
wedlock. When SSCW learned of the petitioner's pregnancy, SSCW's Directress,
advised her to file a resignation letter. In response, the petitioner informed Sr.
Quiambao that she would not resign from her employment just because she got
pregnant without the benefit of marriage
On May 28, 2003, Sr. Quiambao formally directed the petitioner to explain in
writing why she should not be dismissed for engaging in pre-marital sexual relations
and getting pregnant as a result thereof, which amounts to serious misconduct
and conduct unbecoming of an employee of a Catholic school.
Petitioner replied that her pregnancy outside of wedlock does not amount to
serious misconduct. She there after request for a copy of SSCW`s policy so that she
can better respond to the charge against her.
On June 2, 2003, Sr. Quiambao informed the petitioner thattpending the
promulgation of a "Support Staff Handbook," SSCW follows the 1992 MRPS on the
causes for termination of employments; that Section 94 (e) of the 1992 MRPS cites
"disgraceful or immoral conduct" as a ground for dismissal in addition to the just
causes for termination of employment provided under Article 282 of theLabor
Code.
In a letter dated June 6, 2003, SSCW, through counsel, maintained that pre-marital
sexual relations, even if between two consenting adults without legal impediment
to marry, is considered a disgraceful and immoral conduct or a serious
misconduct, which are grounds for the termination of employment.
Thereupon, the petitioner filed a complaint for illegal dismissal with the Regional
Arbitration Branch of the NLRC in Quezon City against SSCW and Sr. Quiambao
(respondents). In her position paper, 14 the petitioner claimed that SSCW gravely
abused its management prerogative as there was no just cause for her dismissal.
She maintained that her pregnancy out of wedlock cannot be considered as
serious misconduct since the same is a purely private affair and not connected in
any way with her duties as an employee of SSCW. Further, the petitioner averred
that she and her boyfriend eventually got married even prior to her dismissal.|||
On February 28, 2006, the Labor Arbiter (LA) rendered a Decision, which dismissed
the complaint filed by the petitioner. The LA found that there was a valid ground
for the petitioner's dismissal; that her pregnancy out of wedlock is considered as a
"disgraceful and immoral conduct." The LA pointed out that, as an employee of a
Catholic educational institution, the petitioner is expected to live up to the
Catholic values taught by SSCW to its students.
The NLRC and CA affirmed the validity of the petitioner`s dismissal pursuant to Sec.
94(e) of the 1992 MRPS.
Issues:
Whether or not the petitioner `s pregnancy out of wedlock amounts to disgraceful
and immoral conduct.

Ruling:

No. The Supreme Court stressed a regular employee may not be dismissed unless
for cause provided under theLabor Code and other relevant laws, in this case,
the 1992 MRPS. When the law refers to morality, it necessarily pertains to public
and secular morality and not religious morality. Thus, the proscription against
"disgraceful or immoral conduct" under Section 94 (e) of the 1992 MRPS, which is
made as a cause for dismissal, must necessarily refer to public and secular
morality. Accordingly, in order for a conduct to be considered as disgraceful or
immoral, it must be "'detrimental (or dangerous) to those conditions upon which
depend the existence and progress of human society' and not because the
conduct is proscribed by the beliefs of one religion or the other.”
As the Court held in Radam, there is no law which penalizes an unmarried
mother by reason of her sexual conduct or proscribes the consensual sexual
activity between two unmarried persons; that neither does such situation
contravenes any fundamental state policy enshrined in the Constitution. DTAESI
Admittedly, the petitioner is employed in an educational institution where
the teachings and doctrines of the Catholic Church, including that on pre-marital
sexual relations, is strictly upheld and taught to the students. That her indiscretion,
which resulted in her pregnancy out of wedlock, is anathema to the doctrines of
the Catholic Church. However, viewed against the prevailing norms of conduct,
the petitioner's conduct cannot be considered as disgraceful or immoral; such
conduct is not denounced by public and secular morality. It may be an unusual
arrangement, but it certainly is not disgraceful or immoral within the
contemplation of the law.

. EVELYN CHUA-QUA, petitioner, vs. HON. JACOBO C. CLAVE, in his capacity as


Presidential Executive Assistant, and TAY TUNG HIGH SCHOOL, INC., respondent

FACTS:

Petitioner worked as a teacher in Tay Tung High School in Bacolod City since
1963. In 1976, petitioner was a Grade VI class adviser where one Bobby Qua, 16
years old, was enrolled. Petitioner was giving remedial lessons to Bobby Qua as
per policy of the school when petitioner and Bobby became very close. On
December 24, 1975, they were married in a civil ceremony in Iloilo City, petitioner
was then 30 years old. Bobby, only 16 years old, received the consent and advice
of the latter’s mother, Mrs. Concepcion Ong. Evelyn and Bobby were married in a
church wedding on January 10, 1976.

On February 4, 1976, Tay Tung High School filed with the Department of Labor in
Bacolod City an application for clearance to terminate petitioner’s employment
on the ground of “abusive and unethical conduct unbecoming of a dignified
school teacher….” Petitioner was suspended without pay on March 12, 1976.
Labor Arbiter Jose Aguirre, without conducting any formal hearing, awarded in
favor of Tay Tung High School. Petitioner appealed to the NLRC claiming denial of
due process for not receiving copies of affidavits relied by labor arbiter. On
December 27, 1976, NLRC reversed the labor arbiter’s decision. This was in turn
reversed by the Minister of Labor, but awarding 6 months salary to petitioner as
financial assistance. Petitioner appealed to the Office of the President of the
Philippines, and through Executive Assistant Jacobo C. Clave, reversed the
decision of the Minister of Labor and ordered petitioner to be reinstated. Public
respondent reversed his earlier decision however and supported petitioner’s
dismissal from work.
ISSUE:

Whether or not thepetitioner commits an immoral act as a teacher warranting


dismissal from work.

RULING:

No. The Supreme Court declared the dismissal illegal. Private respondent utterly
failed to show that petitioner took advantage of her position to court her student.
If the two eventually fell in love,despite the disparity in their ages and academic
levels, this only lends substance to the truism thatthe heart has reasons of its own
which reason does not know. But, definitely, yielding to thisgentle and universal
emotion is not to be so casually equated with immorality. The deviation ofthe
circumstances of their marriage from the usual societal pattern cannot be
considered as adefiance of contemporary social mores.

RE: REGIDOR R. TOLEDO, RONALDO TOLEDO, AND JOEFFREY TOLEDO * vs. ATTY.
JERRY RADAM TOLEDO

Facts:
This is a Complaint for violation of the lawyer's oath, violation of the Code of
Professional Responsibility, oppression, dishonesty, harassment, and immorality
against Atty. Jerry Radam Toledo, Branch Clerk of Court, Regional Trial Court,
Branch 259, Parañaque City.
Complainants, all relatives of respondent, allege that the latter is utilizing his
profession as a lawyer and his position in the judiciary to harass them and make
them agree to an unequal distribution of the estate of the late Florencia
R. Toledo. Complainants claim that respondent, after Florencia's death, never
informed them that he was in possession of the Owner's Duplicate Copy of TCT
No. 125017. As a result of such concealment, complainants executed an
Affidavit of Loss of the document on the basis of which they filed a Verified
Petition for the issuance of the Owner's Duplicate Copy before the RTC of Tarlac
City. Respondent opposed the petition on the ground that he had the subject
document in his possession allegedly because he bought part of the land from
Florencia. Thus, complainants withdrew the petition before the Tarlac court.
Meanwhile, on November 28, 2003, respondent filed another case against
complainants Regidor and Zenaida, and yet another relative,
CresenciaAgduma, this time for violation of Presidential Decree (PD) No. 651. The
case arose when Florencia died and was to be buried in San Clemente, Tarlac.
Complainants had to secure her death certificate, which they failed to obtain in
Parañaque City. Complainants sought advice from respondent, he being the
lawyer in the family, who advised them to get a permit from the Local Civil
Registrar in San Clemente. They followed his advice. Because of this, a case for
violation of PD No. 651 was filed against the three.
Complainants accuse respondent of immorality. They allege that they have
personal knowledge of the fact that respondent is living with his common-law
wife, Normita, whom he allegedly treats as a "maid servant." They further allege
that during the hearings of their cases, respondent was seen with a woman, not
Normita, who was always at his side, and they were very sweet to each other.
They also attribute his unruly and bullying behavior to his being a drunkard with a
fondness for the "night life."
The complainants filed the present petition praying that this Court conduct a
formal investigation of respondent's actions and impose on him the proper
penalty which, they submit, should be the dismissal of respondent from the
service as Branch Clerk of Court.
Issue:
Whether or not a lawyer's sexual congress with a woman not his wife or without
the benefit of marriage should be characterized as "grossly immoral conduct.

Ruling:
This Court has further ruled that intimacy between a man and a woman who are
not married, where both suffer from no impediment to marry, voluntarily carried
on and devoid of any deceit on the part of respondent, is neither so corrupt as
to constitute a criminal act nor so unprincipled as to warrant disbarment or
disciplinary action against a member of the Bar.
Based on the allegations in the Complaint and in respondent's Comment, we
cannot conclude that his act of cohabiting with a woman and begetting
children by her without the benefit of marriage falls within the category of
"grossly immoral conduct."
However, we take this occasion to remind the respondent of the high standards
of conduct imposed upon lawyers in the judiciary. Lawyers in the government
service are under an even greater obligation to observe the basic tenets of the
legal profession because public office is a public trust. They should be more
circumspect in their adherence to their professional obligations under the Code
of Professional Responsibility, for their disreputable conduct is more likely to be
magnified in the public eye.
Domingo v. Rayala
G.R. No. 155831, 18 February 2008

Facts:Ma. Lourdes T. Domingo, then Stenographic Reporter III at the NLRC, filed a
Complaint for sexual harassment against Domingo I. Rayala, the Chairman of
NLRC.She alleged that Rayala called her in his office and touched her shoulder,
part of her neck then tickled her ears. Rayala argued that his acts does not
constitute sexual harassment because for it to exist, there must be a demand,
request or requirement of sexual favor.

Issue: Whether or not Rayala committed sexual harassment.

Held: Yes.The law penalizing sexual harassment in our jurisdiction is RA


7877.Section 3 thereof defines work-related sexual harassment in this wise:

Sec. 3. Work, education or training-related sexual harassment defined. –


Work, education or training-related sexual harassment is committed by an
employer, manager, supervisor, agent of the employer, teacher, instructor,
professor, coach, trainor, or any other person who, having authority,
influence or moral ascendancy over another in a work or training or
education environment, demands, requests or otherwise requires any sexual
favor from the other, regardless of whether the demand, request or
requirement for submission is accepted by the object of said Act.
(a) In a work-related or employment environment, sexual harassment is
committed when:
1. The sexual favor is made as a condition in the hiring or in the
employment, re-employment or continued employment of said
individual, or in granting said individual favorable compensation,
terms, conditions, promotions, or privileges; or the refusal to grant
the sexual favor results in limiting, segregating or classifying the
employee which in a way would discriminate, deprive or diminish
employment opportunities or otherwise adversely affect said
employee;
2. The above acts would impair the employee’s rights or privileges
under existing labor laws; or
3. The above acts would result in an intimidating, hostile, or offensive
environment for the employee.

It is true that this provision calls for a “demand, request or requirement of a sexual
favor.” But it is not necessary that the demand, request or requirement of a sexual
favor be articulated in a categorical oral or written statement. It may be
discerned, with equal certitude, from the acts of the offender. Holding and
squeezing Domingo’s shoulders, running his fingers across her neck and tickling her
ear, having inappropriate conversations with her, giving her money allegedly for
school expenses with a promise of future privileges, and making statements with
unmistakable sexual overtones – all these acts of Rayala resound with deafening
clarity the unspoken request for a sexual favor.

Aquino v. Acosta
A.M. No. CTA-01-1, 02 April 2002

Facts: In 2000, Atty. Susan M. Aquino, Chief of the Legal and Technical Staff of the
Court of Tax Appeals (CTA), reported for work after her vacation in the U.S.,
bringing gifts for the
three judges of the CTA, including respondent, Judge Ernesto Acosta, Presiding
Judge of the same court. In the afternoon of the same day, he entered herroom
and greeted her by shaking her hand. Suddenly, he pulled her towards him and
kissed heron her cheek. In another occasion, while respondent was on official
leave, he called complainant byphone, saying he will get something in her office.
Shortly thereafter, he entered her room, shookher hand and greeted her, "Merry
Christmas." Thereupon, he embraced her and kissed her. Shewas able to free
herself by slightly pushing him away.On the first working day in 2001, respondent
phoned complainant, asking if she could seehim in his chambers inorder to discuss
some matters. When complainantarrived there,respondent tried to kiss her but
she was able to evade his sexual attempt.Weeks later, after the Senate approved
the proposed bill expanding the jurisdiction of theCTA, while complainant and her
companions were congratulating and kissing each other,respondent suddenly
placed his arms around her shoulders and kissed her.The last incident happened
the next day when respondent called complainant and asked her to see him in
his office todiscuss the Senate bill on the CTA. Complainant sat in front of
respondent's table and asked himwhat he wanted to know about the Senate bill.
Respondent then approached complainant saying,“May gusto
akonggawinsaiyokahapon pa”. Thereupon, he tried to grab her. Complainant
instinctively raised her hands to protect herself but respondent held her arms
tightly, pulled her towards him and kissed her. She pushed him away, then
slumped on a chair trembling. Meantime, respondent sat on his chair and
covered his face with his hands. Thereafter, complainant left crying and locked
herself inside a comfort room. After that incident, respondent went to her office
and tossed a note stating, “Sorry, it won’t happen again.”

Issue: Whether or not respondent judge could be held guilty for sexual
harassment.

Held: No, respondent judge could not be held guilty for sexual harassment. The
complainant failedto show by convincing evidence that the acts of Judge
Acosta in greeting her with a kiss on thecheek, in a 'beso-beso' fashion, were
carried out with lustful and lascivious desires or weremotivated by malice or ill-
motive. It is clear under the circumstances that most of the kissingincidents were
done on festive and special occasions. Notably, complainant declared in
heraffidavit-complaint that she brought some 'pasalubongs' for the respondent
judge from her tripabroad. Therefore, Atty. Aquino could not have been 'taken
aback' by the respondent's act ofgreeting her in a friendly manner and thanking
her by way of a kiss on the cheek. Atty. Aquino
failed to state categorically in her affidavit-complaint that respondent demanded
sexual advancesor favors from her, or that the former had committed physical
conduct of sexual nature againsther.

MalayangSamahansa M. Greenfield v. Ramos


326 SCRA 428 [2000]

Facts: Petitioner MSMS, (local union), is an affiliate of ULGWP (federation). A local


union election was held under the action of the federation. The defeated
candidates filed a petition for impeachment. The local union held a general
membership meeting. Several union members failed to attend the meeting. The
local union requested the company to deduct the union fines from the wage of
those union members who failed to attend the general membership meeting. The
Secretary General of the federation disapproved the resolution imposing the
Php50 fine. The company then sent a reply to petitioner’s request stating it cannot
deduct fines without going against certain laws. The imposition of the fine
became the subject of a bitter disagreement between the Federation and the
local union culminating to the latter’s declaration of general autonomy from the
former. The federation asked the company to stop the remittance of the local
union’s share in the education funds.

The company led a complaint of interpleader with the DOLE. The federation
called a meeting placing the local union under trusteeship and appointing an
administrator. Petitioner union officers received letters from the administrator
requiring them to explain why they should not be removed from the office and
expelled from union membership. The officers were expelled from the federation.
The federation advised the company of the expulsion of the 30 union officers and
demanded their separation pursuant to the Union Security Clause in the CBA. The
Federation filed a notice of strike with the NCMB to compel the company to
effect the immediate termination of the expelled union officers. Under the
pressure of a strike, the company terminated the 30 union officers from
employment. The petitioners filed a notice of strike on the grounds of
discrimination; interference; mass dismissal of union officers and shop stewards;
threats, coercion and intimidation; and union busting. The petitioners prayed for
the suspension of the effects of their termination. Secretary Drilon dismissed the
petition stating it was an intra-union matter. Later, 78 union shop stewards were
placed under preventive suspension. The union members staged a walk-out and
officially declared a strike that afternoon. The strike was attended by violence.
Issues:
1. Whether or not the company committed illegal dismissal.
2. Whether or not the strike was illegal.
3. Whether or not petitioners can be deemed to have abandoned their
work.

Held:

1. The chargesagainst respondent company proceeds from one main issue – the
termination of several employees upon the demand of the federation pursuant
to the union security clause. Although the union security clause may be validly
enforced, such must comply with due process. In this case, petitioner union
officers were expelled for allegedly committing acts of disloyalty to the
federation. The company did not inquire into the cause of the expulsion and
merely relied upon the federation’s allegations. The issue is not a purely intra-
union matter as it was later on converted into a termination dispute when the
company dismissed the petitioners from work without the benefit of a separate
notice and hearing. Although it started as an intra-union dispute within the
exclusive jurisdiction of the BLR, to remand the same to the BLR would
intolerably delay the case and the Labor Arbiter could rule upon it. As to the
act of disaffiliation by the local union; it is settled that a local union has the right
to disaffiliate from its mother union in the absence of specific provisions in the
federation’s constitution prohibiting such. There was no such provision in
federation ULGWP’s constitution

2. No. As to the legally of the strike; it was based on the termination dispute and
petitioners believed in good faith in dismissing them, the company was guilty of
ULP. A no-strike, no lockout provision in the CBA can only be invoked when the
strike is economic. As to the violence, the parties agreed that the violence was
not attributed to the striking employees alone as the company itself hired men
to pacify the strikers. Such violence cannot be a ground for declaring the strike
illegal.

3. As to the dismissal of the petitioners; respondents failed to prove that there was
abandonment absent any proof of petitioner’s intention to sever the
employee-employer relationship.
Alabang Country v. NRLC
545 SCRA 351 [2008]

Facts: Petitioner, Alabang Country Club, Inc. (ACCI), requested its Internal Auditor,
Irene Campos-Ugalde, to conduct a study on the profitability of its Food and
Beverage (F&B) Department. Ugalde found out that the business had been
incurring substantial losses. Consequently, the management decided to transfer
the operation of the department to La Tasca Restaurant Inc. (La Tasca). ACCI
then sent its F&B Department employees individual letters informing them that their
services were being terminated and that they would receive separation pay.The
private respondent, Alabang Country Club Independent Employees Union
(Union), filed before the National Labor Relations Commission (NLRC) a complaint
for illegal dismissal, unfair labor practice, regularization and damages with prayer
for the issuance of a writ of preliminary injunction against ACCI. The Labor Arbiter
(LA) dismissed the complaint for illegal dismissal which was upheld by the NLRC.
The Court of Appeals (CA) reversed the decisions of the LA and NLRC.

Issue: Whether or not the ACCI can terminate its business operation.

Held: One of the prerogatives of management is the decision to close the entire
establishment or to close or abolish a department or section thereof for economic
reasons, such as to minimize expenses and reduce capitalization. While the Labor
Code provides for the payment of separation package in case of retrenchment
to prevent losses, it does not obligate the employer for the payment thereof if
there is closure of business due to serious losses. As in the case of retrenchment,
however, for the closure of a business or a department due to serious business
losses to be regarded as an authorized cause for terminating employees, it must
be proven that the losses incurred are substantial and actual or reasonably
imminent; that the same increased through a period of time; and that the
condition of the company is not likely to improve in the near future.

The closure of operation of an establishment or undertaking not due to serious


business losses or financial reverses includes both the complete cessation of
operations and the cessation of only part of a company’s activities. For any bona
fide reason, an employer can lawfully close shop anytime. Just as no law forces
anyone to go into business, no law can compel anybody to continue the same. It
would be stretching the intent and spirit of the law if a court interferes with
management’s prerogative to close or cease its business operations just because
the business is not suffering from any loss or because of the desire to provide the
workers continued employment.
Herminigildo Inguillo and Zenaida Bergante vs. First Philippine Scales, Inc. (FPSI)
and/or Amparo Policarpio, manager
G.R. No. 165407 (June 5, 2009)

FACTS:
In 1991, FPSI and First Philippine Scales Industries Labor Union (FPSILU) entered into
a Collective Bargaining Agreement (CBA) for a period of five (5) years in a
document entitled RATIPIKASYON NG KASUNDUAN. Bergante and Inguillo, who
were members of FPSILU, signed the said document.

Bergante, Inguillo and several FPSI employees joined another union,


the Nagkakaisang Lakas ng Manggagawa (NLM). [The latter] filed with the
Department of Labor and Employment (DOLE) an intra-union dispute against
FPSILU and FPSI. Meanwhile, on March 29, 1996, the executive board and
members of the FPSILU addressed a document dated March 18, 1996
denominated as “Petisyon” to FPSI's general manager, Amparo Policarpio
(Policarpio), seeking the termination of the services of [several employees,
including herein petitioners. This was granted upon by FPSI, which terminated,
among others, herein petitioners.]

In their Petition, Bergante and Inguillo assail the legality of their termination based
on the Union Security Clause in the CBA between FPSI and FPSILU.

ISSUE:
Whether or not the termination was valid instigated by Union on account of Union
Security Clause?

HELD:
Yes. The Labor Code of the Philippines has several provisions under which an
employee may be validly terminated, namely: (1) just causes under Article 282; (2)
authorized causes under Article 283; (3) termination due to disease under Article
284; and (4) termination by the employee or resignation under Article 285. While
the said provisions did not mention as ground the enforcement of the Union
Security Clause in the CBA, the dismissal from employment based on the same is
recognized and accepted in our jurisdiction.

“Union security” is a generic term, which is applied to and comprehends “closed


shop,” “union shop,” “maintenance of membership” or any other form of
agreement which imposes upon employees the obligation to acquire or retain
union membership as a condition affecting employment. There is union shop
when all new regular employees are required to join the union within a certain
period as a condition for their continued employment. There is maintenance of
membership shop when employees, who are union members as of the effective
date of the agreement, or who thereafter become members, must maintain union
membership as a condition for continued employment until they are promoted or
transferred out of the bargaining unit or the agreement is terminated. A closed-
shop, on the other hand, may be defined as an enterprise in which, by agreement
between the employer and his employees or their representatives, no person may
be employed in any or certain agreed departments of the enterprise unless he or
she is, becomes, and, for the duration of the agreement, remains a member in
good standing of a union entirely comprised of or of which the employees in
interest are a part.

Bergante and Inguillo assail the legality of their termination based on the Union
Security Clause in the CBA between FPSI and FPSILU. Article II of the CBA pertains
to Union Security and Representatives, which provides:

“The Company hereby agrees to a UNION SECURITY [CLAUSE] with the following
terms:
1. All bonafide union members x x x x shall, as a condition to their continued
employment, maintain their membership with the UNION;
xxx
5. Any employee/union member who fails to retain union membership in
good standing may be recommended for suspension or dismissal by the Union
Directorate and/or FPSILU Executive Council x x x”
Verily, the aforesaid provision requires all members to maintain their membership
with FPSILU during the lifetime of the CBA. Failing so, and for any of the causes
enumerated therein, the Union Directorate and/or FPSILU Executive Council may
recommend to FPSI an employee/union member's suspension or
dismissal. Records show that Bergante and Inguillo were former members of FPSILU
based on their signatures in the document which ratified the CBA. It can also be
inferred that they disaffiliated from FPSILU when the CBA was still in force and
subsisting, as can be gleaned from the documents relative to the intra-union
dispute between FPSILU and NLM-KATIPUNAN. In view of their disaffiliation, as well
as other acts allegedly detrimental to the interest of both FPSILU and FPSI, a
“Petisyon” was submitted to Policarpio, asking for the termination of the services of
employees who failed to maintain their Union membership.

In terminating the employment of an employee by enforcing the Union Security


Clause, the employer needs only to determine and prove that: (1) the union
security clause is applicable; (2) the union is requesting for the enforcement of the
union security provision in the CBA; and (3) there is sufficient evidence to support
the union's decision to expel the employee from the union or company. All the
requisites have been sufficiently met and FPSI was justified in enforcing the Union
Security Clause.

KING OF KINGS TRANSPORT INC., CLAIRE DELA FUENTE and MELISSA


LIM, petitioners,
vs.
SANTIAGO O. MAMAC, respondent.
G.R. No. 166208 June 29, 2007

FACTS:
Petitioner Kings of Kings Transport Inc. (KKTI) is a corporation engaged in public
transportation and managed by Claire Dela Fuente and Melissa Lim. Respondent
was a conductor for Don Mariano Transit Corporation (DMTC). He was one of the
few people who established Damayan ng mga Manggagawa, Tsuper at
Conductor-Transport Workers Union. Pending the union’s certification election,
respondent was transferred to KKTI. The KKTI employees later organized the
Kaisahan ng mga Kawani sa King of Kings (KKKK) which was registered with DOLE.
Respondent was elected KKKK president.
Upon audit of the October 28, 2001 Conductor’s Report of respondent, KKTI noted
an irregularity. It discovered that respondent declared several sold tickets as
returned tickets causing KKTI to lose an income of eight hundred and ninety pesos.
While no irregularity report was prepared on the October 28, 2001 incident, KKTI
nevertheless asked respondent to explain the discrepancy. In his
letter, respondent said that the erroneous declaration in his October 28, 2001 Trip
Report was unintentional. He explained that during that day’s trip, the windshield
of the bus assigned to them was smashed; and they had to cut short the trip in
order to immediately report the matter to the police. As a result of the incident, he
got confused in making the trip report.
On November 26, 2001, respondent received a letter terminating his employment
effective November 29, 2001. The dismissal letter alleged that the October 28,
2001 irregularity was an act of fraud against the company. KKTI also cited as basis
for respondent’s dismissal the other offenses he allegedly committed since 1999.
After that, he filed an action for illegal dismissal, among other claims. He denied
committing any infraction and alleged that his dismissal was intended to bust
union activities. Moreover, he claimed that his dismissal was effected without due
process.
KKTI averred that it had observed due process in dismissing respondent and
maintained that respondent was not entitled to his money claims such as service
incentive leave and 13th-month pay because he was paid on commission or
percentage basis.
LABOR ARBITER: he was validly dismissed
NLRC: Affirmed. CA held that there was just cause for respondent’s dismissal. It
ruled that respondent’s act in “declaring sold tickets as returned tickets x x x
constituted fraud or acts of dishonesty justifying his dismissal.”

ISSUE:
Whether or not the respondent was accorded procedural due process?

HELD:
NO. There was failure to observe the requirements of due process
Due process under the Labor Code involves two aspects: first, substantive––the
valid and authorized causes of termination of employment under the Labor Code;
and second, procedural––the manner of dismissal.
Section 2(d) of Rule I of Book VI of the Omnibus Rules Implementing the Labor
Code provides:
SEC. 2. Standards of due process; requirements of notice.––In all cases of
termination of employment, the following standards of due process shall be
substantially observed:
For termination of employment based on just causes as defined in Article 282 of
the Code:
(a) A written notice served on the employee specifying the ground or grounds for
termination, and giving said employee reasonable opportunity within which to
explain his side.
(b) A hearing or conference during which the employee concerned, with the
assistance of counsel if he so desires is given opportunity to respond to the
charge, present his evidence, or rebut the evidence presented against him.
(c) A written notice of termination served on the employee, indicating that upon
due consideration of all the circumstances, grounds have been established to
justify his termination.

The first written notice to be served on the employees should contain the specific
causes or grounds for termination against them, and a directive that the
employees are given the opportunity to submit their written explanation within a
reasonable period. “Reasonable opportunity” under the Omnibus Rules means
every kind of assistance that management must accord to the employees to
enable them to prepare adequately for their defense. This should be construed as
a period of at least five (5) calendar days from receipt of the notice to give the
employees an opportunity to study the accusation against them, consult a union
official or lawyer, gather data and evidence, and decide on the defenses they
will raise against the complaint. Moreover, in order to enable the employees to
intelligently prepare their explanation and defenses, the notice should contain a
detailed narration of the facts and circumstances that will serve as basis for the
charge against the employees. A general description of the charge will not
suffice. Lastly, the notice should specifically mention which company rules, if any,
are violated and/or which among the grounds under Art. 282 is being charged
against the employees.
After serving the first notice, the employers should schedule and conduct a
hearing or conference wherein the employees will be given the opportunity to: (1)
explain and clarify their defenses to the charge against them; (2) present
evidence in support of their defenses; and (3) rebut the evidence presented
against them by the management. During the hearing or conference, the
employees are given the chance to defend themselves personally, with the
assistance of a representative or counsel of their choice. Moreover, this
conference or hearing could be used by the parties as an opportunity to come to
an amicable settlement.
After determining that termination of employment is justified, the employers shall
serve the employees a written notice of termination indicating that: (1) all
circumstances involving the charge against the employees have been
considered; and (2) grounds have been established to justify the severance of
their employment.
Respondent was not issued a written notice charging him of committing an
infraction. A verbal appraisal of the charges against an employee does not
comply with the first notice requirement.
The court observed from the irregularity reports against respondent for his other
offenses that such contained merely a general description of the charges against
him. The reports did not even state a company rule or policy that the employee
had allegedly violated.
No hearing was conducted. Regardless of respondent’s written explanation, a
hearing was still necessary in order for him to clarify and present evidence in
support of his defense. Moreover, respondent made the letter merely to explain
the circumstances relating to the irregularity in his October 28, 2001 Conductor’s
Trip Report. He was unaware that a dismissal proceeding was already being
effected. Thus, he was surprised to receive the November 26, 2001 termination
letter indicating as grounds, not only his October 28, 2001 infraction, but also his
previous infractions.

MAGRO PLACEMENT AND GENERAL SERVICES, registered in the name of Marina G.


Sobremesana, Petitioner, v. CRESENCIANO E. HERNANDEZ, Respondent.
526 SCRA 408 [2007]
FACTS:
Magro Placement and General Services (petitioner) is a duly licensed recruitment
agency. It is the local agency of Orbit Recruitment Office of Jeddah, Kingdom of
Saudi Arabia (K.S.A.).
On November 6, 1999, Cresenciano E. Hernandez (respondent), was hired as
Auto Electrician of Al Yamama Est. (Al Yamama) in Jeddah, K.S.A. for a two-year
contract with a basic monthly salary of US$450.00 for 10 hours a day, 6 days a
week regular working hours, 15 days vacation leave and 15 days sick leave with
full pay per year of service, and free food allowance of US$50.00 a month with
free suitable housing.
On January 16, 2000, respondent left for Jeddah, K.S.A. Respondent worked at the
Al Yamama as an electrician. Because of lack of equipment or tools, the work
became harder. After 10 days, his employer took his passport and brought him to
Orbit. His employer told the agency that respondent did not know his job as
electrician. Respondent explained that since he used to repair Japanese cars
only, he needed time to adjust to American cars. Respondent further stated that
he was willing to continue his job. When respondent was subjected to a trade test
using an American car, he failed. Respondent requested that he be sent back to
the Philippines as early as possible. On March 3, 2000, respondent was repatriated
to the Philippines.
On March 16, 2000, respondent filed a Complaint for illegal dismissal against
petitioner before the NLRC.
The Labor Arbiter held that there was no illegal dismissal
ISSUE:
Whether respondent was accorded procedural due process before his separation
from work?
HELD:
No. In dismissing an employee, the employer has the burden of proving that the
dismissed worker has been served two notices: (1) the first to inform the employee
of the particular acts or omissions for which the employer seeks his dismissal; and
(2) the second to inform the employee of his employer's decision to terminate him.
The first notice must state that the employer seeks dismissal for the act or omission
charged against the employee, otherwise, the notice does not comply with the
rules.
In Maquiling v. Philippine Tuberculosis Society, Inc., the Court held that the first
notice must inform the employee outright that an investigation will be conducted
on the charges specified in such notice which, if proven, will result in the
employee's dismissal.

In the case at bar, Al Yamama failed to satisfy the two-notice requirement.


Without prior notice or explanation, Al Yamama took respondent's passport and
simply brought him to petitioner's foreign principal, Orbit, and told the latter that
respondent did not know his job as electrician. Respondent heard his employer's
complaint against him at that instance only.
From these facts, it is clear that respondent's dismissal was effected without the
notice required by law.

Marilou S. Genuino vs. NLRC


539 SCRA 342 [2007]
FACTS:
Genuino was employed by Citibank in January 1992 as Treasury Sales Division
Head with the rank of Assistant Vice- President. On August 23, 1993, Citibank sent
Genuino a letter charging her with “knowledge and involvement” in transactions
“which were irregular or event fraudulent.” In the same letter, Genuino was
informed she was under preventive suspension. On September 27, 1993, Citibank
informed Genuino of the result of their investigation. It found that Genuino,
together with Santos personally and actively participated through the use of
“facilities of Genuino’s family corporation, Global Pacific” in the diversion of bank
client’s funds to products of other companies that yielded higher interests than
Citibank offers. And that Genuino and Santos realized substantial financial gains,
all in violation of existing company policy and Corporation Code under which
carries penal sanction. In view of the foregoing circumstances, Genuino’s
employment was terminated by Citibank on grounds of (1) serious misconduct, (2)
wilful breach of the trust reposed upon her by the bank, and (3) commission of a
crime against the bank.

Genuino filed before the Labor Arbiter a Complaint against Citibank for illegal
suspension and illegal dismissal with damages and prayer for temporary
restraining order and/or writ of preliminary injunction. The Labor Arbiter rendered a
Decision finding the dismissal of Genuino to be without just cause. The NLRC
reversed the decision of the Labor Arbiter. The Court of Appeals then
promulgated its decision denying due course to and dismissing the petitions.

ISSUE:
Whether or not the dismissal of Genuino is for a just cause and in accordance with
due process?

HELD:
The dismissal was for a just cause but lacked due process.
The requirements of twin notices must be met. The two-notice requirement of the
Labor Code is an essential part of the due process. The first notice informing the
employee of the charges should neither be pro-forma nor vague. It should set out
clearly what the employee is being held liable for. The employee should be
afforded ample opportunity to be heard and not mere opportunity. Ample
opportunity to be heard is especially accorded the employees sought to be
dismissed after they are specifically informed of the charges in order to give them
an opportunity to refute such accusations leveled against them. Since the notice
of charges given to Genuino is inadequate, the dismissal could not be in
accordance with due process. While the Court held that Citibank failed to
observe procedural due process, it never the less found Genuino’s dismissal
justified.

While the bank gave genuine an opportunity to deny the truth of the allegations
in writing and participate in the administrative investigation, the fact remains that
the charges were too general to enable Genuino to intelligently and adequately
prepare her defense.

G.R. No. 173012 June 13, 2012


DOLORES T. ESGUERRAvs. VALLE VERDE COUNTRY CLUB, INC. and ERNESTO
VILLALUNA

FACTS:
On 1978, Valle Verde hired Esguerra as Head Food Checker and eventually was
promoted to Cost Control Supervisor in 1999.

On January 15, 2000, the Couples for Christ held a seminar at the country club.
Esguerrawas tasked to oversee the seminar held in the two function rooms the
Ballroom and the Tanay Room. The arrangement was that the food shall be
served in the form of pre-paid buffet, while the drinks shall be paid in a "pay as
you order" basis.

The Valle Verde Management found out the following day that only the proceeds
from the Tanay Room had been remitted to the accounting department. To
resolve the issue, Valle Verde conducted an investigation; the employees who
were assigned in the two function rooms were summoned and made to explain, in
writing, what had transpired.

Valle Verde sent a memorandum to Esguerra requiring her to show cause as to


why no disciplinary action should be taken against her for the non-remittance of
the Ballroom sales. Esguerra was placed under preventive suspension with pay,
pending investigation.

In her letter-response, Esguerra denied having committed any misappropriation.


She explained that it had been her daughter (who was assigned as a food
checker) who lost the money. To settle the matter, Esguerra paid the
unaccounted amount as soon as her daughter informed her about it.

Valle Verde found Esguerra explanation unsatisfactory and issued a second


memorandum terminating Esguerra employment.
Esguerra filed a complaint for illegal dismissal. The Labor Arbiter dismissed the
complaint for lack of merit. Esguerra appealed the case to the NLRC. The NLRC
affirmed with modification the ruling of the Labor Arbiter.

Esguerra filed a partial motion for reconsideration, while Valle Verde filed its own
motion for reconsideration.The NLRC denied Esguerra motion, but granted Valle
Verde motion. Thus, it set aside itsdecision and affirmed the decision of the labor
arbiter.

Aggrieved, Esguerra elevated her case to the CA but it was denied. Her Motion
for Reconsideration was also denied.

ISSUE:
Whether or not intention to terminate should be included in the notice of
informing of charges against an employee.

RULING:
No. There was valid notice and hearing. The Court failed to find any irregularities in
the service of notice to Esguerra. The memorandum dated March 6, 2000
informed her of the charges, and clearly directed her to show cause, in writing,
why no disciplinary action should be imposed against her. Esguerra allegation that
the notice was insufficient since it failed to contain any intention to terminate her
is incorrect.

In Perez v. Philippine Telegraph and Telephone Company, the Court underscored


the significance of the two-notice rule in dismissing an employee:

To meet the requirements of due process in the dismissal of an employee, an


employer must furnish the worker with two written notices: (1) a written notice
specifying the grounds for termination and giving to said employee a reasonable
opportunity to explain his side and (2) another written notice indicating that, upon
due consideration of all circumstances, grounds have been established to justify
the employer decision to dismiss the employee.

Contrary to Esguerra allegation, the law does not require that an intention to
terminate one employment should be included in the first notice. It is enough that
employees are properly apprised of the charges brought against them so they
can properly prepare their defenses; it is only during the second notice that the
intention to terminate one employment should be explicitly stated.

There is also no basis to question the absence of a proper hearing. The existence
of an actual, formal "trial-type" hearing, although preferred, is not absolutely
necessary to satisfy the employee's right to be heard. Esguerra was able to
present her defenses; and only upon proper consideration of it did Valle Verde
send the second memorandum terminating her employment. Since Valle Verde
complied with the two-notice requirement, no procedural defect exists in Esguerra
termination
G.R. No. 80587 February 8, 1989

WENPHIL CORPORATIONvs.NATIONAL LABOR RELATIONS COMMISSION AND


ROBERTO MALLARE

FACTS:
Private respondent Mallare was hired by petitioneras a crew member at its Cubao
Branch. He thereafter became the assistant head of the Backroom department of
the same branch. At about 2:30 P.M. on May 20, 1985 private respondent had an
altercation with a co-employee, Job Barrameda, as a result of which he and
Barrameda were suspended on the following morning and in the afternoon of the
same day a memorandum was issued by the Operations Manager advising
Mallare of his dismissal from the service in accordance with their Personnel
Manual. The notice of dismissal was served on Mallare on May 25, 1985.

Thus Mallare filed a complaint against petitioner for unfair labor practice, illegal
suspension and illegal dismissal. After submitting their respective position papers to
the Labor Arbiter and as the hearing could not be conducted due to repeated
absence of counsel for respondent, the case was submitted for resolution.
Thereafter a decision was rendered by the Labor Arbiter dismissing the complaint
for lack of merit.

Mallare appealed to NLRC which sets aside the appealed decision and ordering
the reinstatement of Mallare to his former position without loss of seniority and
other related benefits and one (1) year backwages without qualification and
deduction.

Hence this petition alleging that the NLRC committed a grave abuse of discretion
in rendering its decision contrary to the evidence on record.

ISSUES:
1. Whether or not Mallare waived his right to investigation.
2. Whether or not an employee dismissed for just cause but without due process
be reinstated to work.

RULING:
1. No.The incident happened on May 20, 1985 and right then and there as afore
repeated on the following day private respondent was suspended in the
morning and was dismissed from the service in the afternoon. He received an
official notice of his termination four (4) days later.

The defiant attitude of Mallare immediately after the incident amounted to


insubordination. Nevertheless his refusal to explain his side under the
circumstances cannot be considered as a waiver of his right to an
investigation.
Although in the Personnel Manual of the petitioner, it states that an erring
employee must request for an investigation it does not thereby mean that
petitioner is thereby relieved of the duty to conduct an investigation before
dismissing private respondent. Indeed said provision of the Personnel Manual of
petitioner which may effectively deprive its employees of the right to due
process is clearly against the law and hence null and void. The security of
tenure of a laborer or employee is enshrined in the Constitution, the Labor
Code and other related laws.

Under Section 1, Rule XIV of the Implementing Regulations of the Labor Code, it
is provided that "No worker shall be dismissed except for just or authorized
cause provided by law and after due process." Sections 2, 5, 6, and 7 of the
same rules require that before an employer may dismiss an employee the latter
must be given a written notice stating the particular act or omission constituting
the grounds thereof; that the employee may answer the allegations within a
reasonable period; that the employer shall afford him ample opportunity to be
heard and to defend himself with the assistance of his representative, if he so
desires; and that it is only then that the employer may dismiss the employee by
notifying him of the decision in writing stating clearly the reasons therefor. Such
dismissal is without prejudice to the right of the employee to contest its validity
in the Regional Branch of the NLRC.

2. No.The failure of petitioner to give private respondent the benefit of a hearing


before he was dismissed constitutes an infringement of his constitutional right to
due process of law and equal protection of the laws. The standards of due
process in judicial as well as administrative proceedings have long been
established. In its bare minimum due process of law simply means giving notice
and opportunity to be heard before judgment is rendered.

However, it is a matter of fact that when Mallare filed a complaint against


petitioner he was afforded the right to an investigation by the labor arbiter. He
presented his position paper as did the petitioner. If no hearing was had, it was
the fault Mallare as his counsel failed to appear at the scheduled hearings. The
labor arbiter concluded that the dismissal of private respondent was for just
cause. He was found guilty of grave misconduct and insubordination. This is
borne by the sworn statements of witnesses. The Court is bound by this finding
of the labor arbiter.

By the same token, the conclusion of theNLRC on appeal that Mallare was not
afforded due process before he was dismissed is binding on the Court. Indeed,
it is well taken and supported by the records. However, it can not justify a ruling
that Mallare should be reinstated with back wages as the NLRC so decreed.
Although belatedly, Mallare was afforded due process before the labor arbiter
wherein the just cause of his dismissal bad been established. With such finding,
it would be arbitrary and unfair to order his reinstatement with back wages.

The Court holds that the policy of ordering the reinstatement to the service of
an employee without loss of seniority and the payment of his wages during the
period of his separation until his actual reinstatement but not exceeding three
(3) years without qualification or deduction, when it appears he was not
afforded due process, although his dismissal was found to be for just and
authorized cause in an appropriate proceeding in the Ministry of Labor and
Employment, should be re-examined. It will be highly prejudicial to the interests
of the employer to impose on him the services of an employee who has been
shown to be guilty of the charges that warranted his dismissal from
employment. Indeed, it will demoralize the rank and file if the undeserving, if
not undesirable, remains in the service.

Thus in the present case, where Mallare, who appears to be of violent temper,
caused trouble during office hours and even defied his superiors as they tried
to pacify him, should not be rewarded with re-employment and back wages. It
may encourage him to do even worse and will render a mockery of the rules of
discipline that employees are required to observe. Under the circumstances
the dismissal Mallare for just cause should be maintained. He has no right to
return to his former employer.

However, the petitioner must nevertheless be held to account for failure to


extend to Mallare his right to an investigation before causing his dismissal. The
rule is explicit as above discussed. The dismissal of an employee must be for just
or authorized cause and after due process. Petitioner committed an infraction
of the second requirement. Thus, it must be imposed a sanction for its failure to
give a formal notice and conduct an investigation as required by law before
dismissing petitioner from employment. Considering the circumstances of this
case petitioner must indemnify Mallare

G.R. No. 117040 January 27, 2000

RUBEN SERRANOvs.NATIONAL LABOR RELATIONS COMMISSION and ISETANN


DEPARTMENT STORE

FACTS:
Petitioner was hired by private respondent Isetann Department Store as a security
checker to apprehend shoplifters and prevent pilferage of merchandise. Initially
hired on contractual basis, petitioner eventually became a regular employee on.
In 1988, he became head of the Security Checkers Section of private respondent.

Sometime in 1991, as a cost-cutting measure, private respondent decided to


phase out its entire security section and engage the services of an independent
security agency. For this reason, it wrote petitioner a memorandum.

The loss of his employment prompted petitioner to file a complaintfor illegal


dismissal, illegal layoff, unfair labor practice, underpayment of wages, and
nonpayment of salary and overtime pay.
The Labor Arbiter rendered a decision finding petitioner to have been illegally
dismissed. He ruled that private respondent failed to establish that it had
retrenched its security section to prevent or minimize losses to its business; that
private respondent failed to accord due process to petitioner; that private
respondent failed to use reasonable standards in selecting employees whose
employment would be terminated; that private respondent had not shown that
petitioner and other employees in the security section were so inefficient so as to
justify their replacement by a security agency, or that "cost-saving devices [such
as] secret video cameras (to monitor and prevent shoplifting) and secret code
tags on the merchandise" could not have been employed; instead, the day after
petitioner's dismissal, private respondent employed a safety and security
supervisor with duties and functions similar to those of petitioner.

Private respondent appealed to the NLRC which reversed the decision of the
Labor Arbiter and ordered petitioner to be given separation pay equivalent to
one month pay for every year of service, unpaid salary, and proportionate 13th
month pay. Petitioner filed a motion for reconsideration, but his motion was
denied.

The NLRC held that the phase-out of private respondent's security section and the
hiring of an independent security agency constituted an exercise by private
respondent of "[a] legitimate business decision whose wisdom we do not intend to
inquire into and for which we cannot substitute our judgment"; that the distinction
made by the Labor Arbiter between "retrenchment" and the employment of cost-
saving devices" under Art. 283 of the Labor Code was insignificant because the
company official who wrote the dismissal letter apparently used the term
"retrenchment" in its "plain and ordinary sense: to layoff or remove from one's job,
regardless of the reason therefor"; that the rule of "reasonable criteria" in the
selection of the employees to be retrenched did not apply because all positions in
the security section had been abolished; and that the appointment of a safety
and security supervisor referred to by petitioner to prove bad faith on private
respondent's part was of no moment because the position had long been in
existence and was separate from petitioner's position as head of the Security
Checkers Section. Hence this petition.

ISSUE:
Whether or not the hiring of an independent Security Agency by Isetann to
replace its current Security Section a valid grounds for the dismissal of the
employees.

RULING:
Yes. Art. 283 provides:

Closure of establishment and reduction of personnel. — The employer may


also terminate the employment of any employee due to the installation of
labor-saving devices, redundancy, retrenchment to prevent losses or the
closing or cessation of operations of the establishment or undertaking unless
the closing is for the purpose of circumventing the provisions of this Title, by
serving a written notice on the, workers and the Department of Labor and
Employment at least one (1) month before the intended date thereof. In
case of termination due to the installation of labor-saving devices or
redundancy, the worker affected thereby shall be entitled to a separation
pay equivalent to at least one (1) month pay or to at least one (1) month
pay for every year of service, whichever is higher. In case of retrenchment
to prevent losses and in cases of closure or cessation of operations of
establishment or undertaking not due to serious business losses or financial
reverses, the separation pay shall be equivalent to at least one (1) month
pay or at least one-half (1/2) month pay for every year of service,
whichever is higher. A fraction of at least six (6) months shall be considered
as one (1) whole year.

In the case at bar, the Court have only the bare assertion of petitioner that, in
abolishing the security section, private respondent's real purpose was to avoid
payment to the security checkers of the wage increases provided in the collective
bargaining agreement approved in 1990. Such an assertion is not sufficient basis
for concluding that the termination of petitioner's employment was not a bona
fide decision of management to obtain reasonable return from its investment,
which is a right guaranteed to employers under the Constitution. Indeed, that the
phase-out of the security section constituted a "legitimate business decision" is a
factual finding of an administrative agency which must be accorded respect and
even finality by the Court since nothing can be found in the record which fairly
detracts from such finding.

Accordingly, the Court hold that the termination of petitioner's services was for an
authorized cause, i.e., redundancy. Hence, pursuant to Art. 283 of the Labor
Code, petitioner should be given separation pay at the rate of one month pay for
every year of service.

Art. 283 also provides that to terminate the employment of an employee for any
of the authorized causes the employer must serve "a written notice on the workers
and the Department of Labor and Employment at least one (1) month before the
intended date thereof." In the case at bar, petitioner was given a notice of
termination on October 11, 1991. On the same day, his services were terminated.
He was thus denied his right to be given written notice before the termination of
his employment, and the question is the appropriate sanction for the violation of
petitioner's right.

With respect to Art. 283 of the Labor Code, the employer's failure to comply with
the notice requirement does not constitute a denial of due process but a mere
failure to observe a procedure for the termination of employment which makes
the termination of employment merely ineffectual.Under the Labor Code, only the
absence of a just cause for the termination of employment can make the
dismissal of an employee illegal. This is clear from Art. 279 which provides the
Security of Tenure.
Given the nature of the violation, therefore, the appropriate sanction for the
failure to give notice is the payment of backwages for the period when the
employee is considered not to have been effectively dismissed or his employment
terminated. The sanction is not the payment alone of nominal damages

In sum, the Court hold that if in proceedings for reinstatement under Art. 283, it is
shown that the termination of employment was due to an authorized cause, then
the employee concerned should not be ordered reinstated even though there is
failure to comply with the 30-day notice requirement. Instead, he must be granted
separation pay in accordance with Art. 283.

In case of termination due to the installation of labor-saving devices or


redundancy, the worker affected thereby shall be entitled to a separation pay
equivalent to at least his one (1) month pay or to at least one month for every
year of service, whichever is higher. In case of retrenchment to prevent losses and
in cases of closures or cessation of operations of establishment or undertaking not
due to serious business losses or financial reverses, the separation pay shall be
equivalent to one (1) month pay or at least one-half (1/2) month pay for every
year of service, whichever is higher. A fraction of at least six months shall be
considered one (1) whole year.

If the employee's separation is without cause, instead of being given separation


pay, he should be reinstated. In either case, whether he is reinstated or only
granted separation pay, he should be paid full backwages if he has been laid off
without written notice at least 30 days in advance.

On the other hand, with respect to dismissals for cause under Art. 282, if it is shown
that the employee was dismissed for any of the just causes mentioned in said Art.
282, then, in accordance with that article, he should not be reinstated. However,
he must be paid backwages from the time his employment was terminated until it
is determined that the termination of employment is for a just cause because the
failure to hear him before he is dismissed renders the termination of his
employment without legal effect.
G.R. No. 158693 November 17, 2004

JENNY M. AGABON and VIRGILIO C. AGABON vs. NATIONAL LABOR RELATIONS


COMMISSION (NLRC), RIVIERA HOME IMPROVEMENTS, INC. and VICENTE ANGELES,

FACTS:
Private respondent Riviera Home Improvements, Inc. is engaged in the business of
selling and installing ornamental and construction materials. It employed
petitioners Virgilio Agabon and Jenny Agabon as gypsum board and cornice
installers on January1992 until February 1999 when they were dismissed for
abandonment of work.

Petitioners then filed a complaint for illegal dismissal and payment of money
claims and then the Labor Arbiter rendered a decision declaring the dismissals
illegal and ordered private respondent to pay the monetary claims.

On appeal, the NLRC reversed the Labor Arbiter because it found that the
petitioners had abandoned their work, and were not entitled to backwages and
separation pay. The other money claims awarded by the Labor Arbiter were also
denied for lack of evidence.

Upon denial of their motion for reconsideration, petitioners filed a petition for
certiorari with the Court of Appeals.

The Court of Appeals in turn ruled that the dismissal of the petitioners was not
illegal because they had abandoned their employment but ordered the payment
of money claims. Hence, petitioners elevated the case.

Petitioners assert that they were dismissed because the private respondent
refused to give them assignments unless they agreed to work on a "pakyaw" basis
when they reported for duty on February 23, 1999. They did not agree on this
arrangement because it would mean losing benefits as Social Security System
(SSS) members. Petitioners also claim that private respondent did not comply with
the twin requirements of notice and hearing.

On the other hand, private respondent maintained that petitioners were not
dismissed but had abandoned their work. In fact, private respondent sent two
letters to the last known addresses of the petitioners advising them to report for
work. Private respondent's manager even talked to petitioner Virgilio Agabon by
telephone sometime in June 1999 to tell him about the new assignment at Pacific
Plaza Towers involving 40,000 square meters of cornice installation work. However,
petitioners did not report for work because they had subcontracted to perform
installation work for another company. Petitioners also demanded for an increase
in their wage to P280.00 per day. When this was not granted, petitioners stopped
reporting for work and filed the illegal dismissal case.

ISSUES:
1. Whether or not the petitioners abandoned their work.
2. Whether or not the failure of the employer to observe the procedural due
process of twin notice in the termination of the employee due to just or
authorized cause nullifies the termination.

RULING:
1. Yes. Article 282 of the Labor Code enumerates the just causes for termination
by the employer: (a) serious misconduct or willful disobedience by the
employee of the lawful orders of his employer or the latter's representative in
connection with the employee's work; (b) gross and habitual neglect by the
employee of his duties; (c) fraud or willful breach by the employee of the trust
reposed in him by his employer or his duly authorized representative; (d)
commission of a crime or offense by the employee against the person of his
employer or any immediate member of his family or his duly authorized
representative; and (e) other causes analogous to the foregoing.

Abandonment is the deliberate and unjustified refusal of an employee to


resume his employment.It is a form of neglect of duty, hence, a just cause for
termination of employment by the employer.For a valid finding of
abandonment, these two factors should be present: (1) the failure to report for
work or absence without valid or justifiable reason; and (2) a clear intention to
sever employer-employee relationship, with the second as the more
determinative factor which is manifested by overt acts from which it may be
deduced that the employees has no more intention to work. The intent to
discontinue the employment must be shown by clear proof that it was
deliberate and unjustified.

In February 1999, petitioners were frequently absent having subcontracted for


an installation work for another company. Subcontracting for another
company clearly showed the intention to sever the employer-employee
relationship with private respondent. This was not the first time they did this. In
January 1996, they did not report for work because they were working for
another company. Private respondent at that time warned petitioners that
they would be dismissed if this happened again. Petitioners disregarded the
warning and exhibited a clear intention to sever their employer-employee
relationship.

2. No. Dismissals based on just causes contemplate acts or omissions attributable


to the employee while dismissals based on authorized causes involve grounds
under the Labor Code which allow the employer to terminate employees. A
termination for an authorized cause requires payment of separation pay. When
the termination of employment is declared illegal, reinstatement and full
backwages are mandated under Article 279. If reinstatement is no longer
possible where the dismissal was unjust, separation pay may be granted.

Procedurally, (1) if the dismissal is based on a just cause under Article 282, the
employer must give the employee two written notices and a hearing or
opportunity to be heard if requested by the employee before terminating the
employment: a notice specifying the grounds for which dismissal is sought a
hearing or an opportunity to be heard and after hearing or opportunity to be
heard, a notice of the decision to dismiss; and (2) if the dismissal is based on
authorized causes under Articles 283 and 284, the employer must give the
employee and the Department of Labor and Employment written notices 30
days prior to the effectivity of his separation.

From the foregoing rules four possible situations may be derived: (1) the
dismissal is for a just cause under Article 282 of the Labor Code, for an
authorized cause under Article 283, or for health reasons under Article 284, and
due process was observed; (2) the dismissal is without just or authorized cause
but due process was observed; (3) the dismissal is without just or authorized
cause and there was no due process; and (4) the dismissal is for just or
authorized cause but due process was not observed.

In the first situation, the dismissal is undoubtedly valid and the employer will not
suffer any liability.

In the second and third situations where the dismissals are illegal, Article 279
mandates that the employee is entitled to reinstatement without loss of
seniority rights and other privileges and full backwages, inclusive of allowances,
and other benefits or their monetary equivalent computed from the time the
compensation was not paid up to the time of actual reinstatement.

In the fourth situation, the dismissal should be upheld. While the procedural
infirmity cannot be cured, it should not invalidate the dismissal. However, the
employer should be held liable for non-compliance with the procedural
requirements of due process.

The present case squarely falls under the fourth situation. The dismissal should
be upheld because it was established that the petitioners abandoned their
jobs to work for another company. Private respondent, however, did not follow
the notice requirements and instead argued that sending notices to the last
known addresses would have been useless because they did not reside there
anymore. Unfortunately for the private respondent, this is not a valid excuse
because the law mandates the twin notice requirements to the employee's last
known address. Thus, it should be held liable for non-compliance with the
procedural requirements of due process.

Where the employer had a valid reason to dismiss an employee but did not
follow the due process requirement, the dismissal may be upheld but the
employer will be penalized to pay an indemnity to the employee. This became
known as the Wenphil or Belated Due Process Rule.

On January 27, 2000, in Serrano, the rule on the extent of the sanction was
changed. The Court held that the violation by the employer of the notice
requirement in termination for just or authorized causes was not a denial of due
process that will nullify the termination. However, the dismissal is ineffectual and
the employer must pay full backwages from the time of termination until it is
judicially declared that the dismissal was for a just or authorized cause.

Serrano was confronting the practice of employers to "dismiss now and pay
later" by imposing full backwages.

The Court believe, however, that the ruling in Serrano did not consider the full
meaning of Article 279 of the Labor Code which states:

ART. 279. Security of Tenure. – In cases of regular employment, the employer


shall not terminate the services of an employee except for a just cause or
when authorized by this Title. An employee who is unjustly dismissed from
work shall be entitled to reinstatement without loss of seniority rights and
other privileges and to his full backwages, inclusive of allowances, and to his
other benefits or their monetary equivalent computed from the time his
compensation was withheld from him up to the time of his actual
reinstatement.

This means that the termination is illegal only if it is not for any of the justified or
authorized causes provided by law. Payment of backwages and other
benefits, including reinstatement, is justified only if the employee was unjustly
dismissed.

The fact that the Serrano ruling can cause unfairness and injustice which
elicited strong dissent has prompted the Court to revisit the doctrine.

After carefully analyzing the consequences of the divergent doctrines in the


law on employment termination, we believe that in cases involving dismissals
for cause but without observance of the twin requirements of notice and
hearing, the better rule is to abandon the Serrano doctrine and to follow
Wenphil by holding that the dismissal was for just cause but imposing sanctions
on the employer. Such sanctions, however, must be stiffer than that imposed in
Wenphil. By doing so, the Court would be able to achieve a fair result by
dispensing justice not just to employees, but to employers as well.

The unfairness of declaring illegal or ineffectual dismissals for valid or authorized


causes but not complying with statutory due process may have far-reaching
consequences.This would encourage frivolous suits, where even the most
notorious violators of company policy are rewarded by invoking due process.
This also creates absurd situations where there is a just or authorized cause for
dismissal but a procedural infirmity invalidates the termination.

The constitutional policy to provide full protection to labor is not meant to be a


sword to oppress employers. The commitment of this Court to the cause of
labor does not prevent us from sustaining the employer when it is in the right, as
in this case. Certainly, an employer should not be compelled to pay
employees for work not actually performed and in fact abandoned.
The employer should not be compelled to continue employing a person who is
admittedly guilty of misfeasance or malfeasance and whose continued
employment is patently inimical to the employer. The law protecting the rights
of the laborer authorizes neither oppression nor self-destruction of the
employer.

It must be stressed that in the present case, the petitioners committed a grave
offense, i.e., abandonment, which, if the requirements of due process were
complied with, would undoubtedly result in a valid dismissal.An employee who
is clearly guilty of conduct violative of Article 282 should not be protected by
the Social Justice Clause of the Constitution. Social justice, as the term
suggests, should be used only to correct an injustice.This is not to say that the
Court was wrong when it ruled the way it did in Wenphil, Serrano and related
cases. Social justice is not based on rigid formulas set in stone. It has to allow for
changing times and circumstances.

The violation of the petitioners' right to statutory due process by the private
respondent warrants the payment of indemnity in the form of nominal
damages. The amount of such damages is addressed to the sound discretion
of the court, taking into account the relevant circumstances.

FELIX B. PEREZ and AMANTE G. DORIA, Petitioners, vs PHILIPPINE TELEGRAPH AND


TELEPHONE COMPANY and JOSE LUIS SANTIAGO, Respondents.

G.R. No. 152048; April 7, 2009

FACTS:

Petitioners Felix B. Perez and Amante G. Doria were employed by respondent


Philippine Telegraph and Telephone Company (PT&T) as shipping clerk and
supervisor, respectively, in PT&T’s Shipping Section, Materials Management Group.
Acting on an alleged unsigned letter regarding anomalous transactions at the
Shipping Section, respondents formed a special audit team to investigate the
matter. It was discovered that the Shipping Section jacked up the value of the
freight costs for goods shipped and that the duplicates of the shipping documents
allegedly showed traces of tampering, alteration and superimposition.

Petitioners were placed on preventive suspension for 30 days for their alleged
involvement in the anomaly. Their suspension was extended for 15 days twice.
Then in a Memorandum, petitioners were dismissed from the service for having
falsified company documents. Petitioners filed a complaint for illegal suspension
and illegal dismissal alleging that they were dismissed on November 8, 1993, the
date they received the above-mentioned memorandum.
LA favored petitioners. NLRC reversed the decision of LA. Petitioners appealed to
CA. CA affirmed the NLRC decision insofar as petitioners’ illegal suspension for 15
days and dismissal for just cause were concerned. However, it found that
petitioners were dismissed without due process. Petitioners now seek a reversal of
the CA decision before the SC. They contend that there was no just cause for their
dismissal, that they were not accorded due process and that they were illegally
suspended for 30 days.

ISSUE:

Whether respondents were dismissed for just cause and with the observance of
due process.

RULING:

Respondents’ evidence is insufficient to clearly and convincingly establish the


facts from which the loss of confidence resulted. Other than their bare allegations
and the fact that such documents came into petitioners’ hands at some point,
respondents should have provided evidence of petitioners’ functions, the extent
of their duties, the procedure in the handling and approval of shipping requests
and the fact that no personnel other than petitioners were involved. The
alterations on the shipping documents could not reasonably be attributed to
petitioners because it was never proven that petitioners alone had control of or
access to these documents.

Willful breach by the employee of the trust reposed in him by his employer or duly
authorized representative is a just cause for termination. However, loss of
confidence should not be simulated. It should not be used as a subterfuge for
causes which are improper, illegal or unjustified. Loss of confidence may not be
arbitrarily asserted in the face of overwhelming evidence to the contrary. It must
be genuine, not a mere afterthought to justify an earlier action taken in bad faith.

The burden of proof rests on the employer to establish that the dismissal is for
cause in view of the security of tenure that employees enjoy under the
Constitution and the Labor Code. The employer’s evidence must clearly and
convincingly show the facts on which the loss of confidence in the employee may
be fairly made to rest. It must be adequately proven by substantial evidence.
Respondents failed to discharge this burden.

Respondents’ illegal act of dismissing petitioners was aggravated by their failure to


observe due process. To meet the requirements of due process in the dismissal of
an employee, an employer must furnish the worker with 2 written notices: (1) a
written notice specifying the grounds for termination and giving to said employee
a reasonable opportunity to explain his side and (2) another written notice
indicating that, upon due consideration of all circumstances, grounds have been
established to justify the employer’s decision to dismiss the employee.
Petitioners were neither apprised of the charges against them nor given a
chance to defend themselves. They were simply and arbitrarily separated from
work and served notices of termination in total disregard of their rights to due
process and security of tenure. Respondents failed to comply with the two-notice
requirement for terminating employees.

We note a marked difference in the standards of due process to be followed as


prescribed in the Labor Code and its implementing rules. The Labor Code provides
that an employer must provide the employee ample opportunity to be heard and
to defend himself with the assistance of his representative if he so desires.

The omnibus rules implementing the Labor Code, on the other hand, require a
hearing and conference during which the employee concerned is given the
opportunity to respond to the charge, present his evidence or rebut the evidence
presented against him.

In case of conflict, the law prevails over the administrative regulations


implementing it. The authority to promulgate implementing rules proceeds from
the law itself. To be valid, a rule or regulation must conform to and be consistent
with the provisions of the enabling statute. As such, it cannot amend the law
either by abridging or expanding its scope.

Article 277(b) of the Labor Code provides that, in cases of termination for a just
cause, an employee must be given “ample opportunity to be heard and to
defend himself.” Thus, the opportunity to be heard afforded by law to the
employee is qualified by the word “ample” which ordinarily means “considerably
more than adequate or sufficient.” In this regard, the phrase “ample opportunity
to be heard” can be reasonably interpreted as extensive enough to cover actual
hearing or conference. To this extent, Section 2(d), Rule I of the Implementing
Rules of Book VI of the Labor Code is in conformity with Article 277(b).

Nonetheless, Section 2(d), Rule I of the Implementing Rules of Book VI of the Labor
Code should not be taken to mean that holding an actual hearing or conference
is a condition sine qua non for compliance with the due process requirement in
termination of employment. The test for the fair procedure guaranteed under
Article 277(b) cannot be whether there has been a formal pretermination
confrontation between the employer and the employee. The “ample opportunity
to be heard” standard is neither synonymous nor similar to a formal hearing.

The standard for the hearing requirement, ample opportunity, is couched in


general language revealing the legislative intent to give some degree of flexibility
or adaptability to meet the peculiarities of a given situation. To confine it to a
single rigid proceeding such as a formal hearing will defeat its spirit.

Section 2(d), Rule I of the Implementing Rules of Book VI of the Labor Code itself
provides that the so-called standards of due process outlined therein shall be
observed “substantially,” not strictly. This is a recognition that while a formal
hearing or conference is ideal, it is not an absolute, mandatory or exclusive
avenue of due process.

A hearing means that a party should be given a chance to adduce his evidence
to support his side of the case and that the evidence should be taken into
account in the adjudication of the controversy. “To be heard” does not mean
verbal argumentation alone inasmuch as one may be heard just as effectively
through written explanations, submissions or pleadings. Therefore, while the phrase
“ample opportunity to be heard” may in fact include an actual hearing, it is not
limited to a formal hearing only. The existence of an actual, formal “trial-type”
hearing, although preferred, is not absolutely necessary to satisfy the employee’s
right to be heard.

Due process of law simply means giving opportunity to be heard before


judgment is rendered. In fact, there is no violation of due process even if no
hearing was conducted, where the party was given a chance to explain his side
of the controversy. What is frowned upon is the denial of the opportunity to be
heard. Twin requirements of notice and hearing constitute the essential elements
of due process in the dismissal of employees. It is deemed sufficient for the
employer to follow the natural sequence of notice, hearing and judgment.

In sum, the following are the guiding principles in connection with the hearing
requirement in dismissal cases:

(a) “ample opportunity to be heard” means any meaningful opportunity (verbal


or written) given to the employee to answer the charges against him and submit
evidence in support of his defense, whether in a hearing, conference or some
other fair, just and reasonable way.

(b) a formal hearing or conference becomes mandatory only when requested by


the employee in writing or substantial evidentiary disputes exist or a company rule
or practice requires it, or when similar circumstances justify it.

(c) the “ample opportunity to be heard” standard in the Labor Code prevails over
the “hearing or conference” requirement in the implementing rules and
regulations.

On the other hand, an employee may be validly suspended by the employer for
just cause provided by law. Such suspension shall only be for a period of 30 days,
after which the employee shall either be reinstated or paid his wages during the
extended period.

Where the dismissal was without just or authorized cause and there was no due
process, Article 279 of the Labor Code mandates that the employee is entitled to
reinstatement without loss of seniority rights and other privileges and full
backwages, inclusive of allowances, and other benefits or their monetary
equivalent computed from the time the compensation was not paid up to the
time of actual reinstatement. In this case, however, reinstatement is no longer
possible because of the length of time that has passed from the date of the
incident to final resolution. 14 years have transpired from the time petitioners were
wrongfully dismissed. To order reinstatement at this juncture will no longer serve
any prudent or practical purpose. So petitioners will just be paid their separation
pay.

Petition is hereby GRANTED.

Bernardo vs NLRC

GR 122917 07/03/99

Facts:

Petitioners numbering 43 are deaf–mutes who were hired on various periods from
1988 to 1993 by respondent Far East Bank and Trust Co. as Money Sorters and
Counters through a uniformly worded agreement called ‘Employment Contract
for Handicapped Workers. Subsequently, they are dismissed.

Petitioners maintain that they should be considered regular employees, because


their task as money sorters and counters was necessary and desirable to the
business of respondent bank. They further allege that their contracts served
merely to preclude the application of Article 280 and to bar them from becoming
regular employees.

Private respondent, on the other hand, submits that petitioners were hired only as
“special workers and should not in any way be considered as part of the regular
complement of the Bank.”[12] Rather, they were “special” workers under Article
80 of the Labor Code.

Issue: WON petitioners have become regular employees.

Held:

The uniform employment contracts of the petitioners stipulated that they shall be
trained for a period of one month, after which the employer shall determine
whether or not they should be allowed to finish the 6-month term of the
contract. Furthermore, the employer may terminate the contract at any time for
a just and reasonable cause. Unless renewed in writing by the employer, the
contract shall automatically expire at the end of the term.

Respondent bank entered into the aforesaid contract with a total of 56


handicapped workers and renewed the contracts of 37 of them. In fact, two of
them worked from 1988 to 1993. Verily, the renewal of the contracts of the
handicapped workers and the hiring of others lead to the conclusion that their
tasks were beneficial and necessary to the bank. More important, these facts
show that they were qualified to perform the responsibilities of their positions. In
other words, their disability did not render them unqualified or unfit for the tasks
assigned to them.

In this light, the Magna Carta for Disabled Persons mandates that
a qualified disabled employee should be given the same terms and conditions of
employment as a qualified able-bodied person. Section 5 of the Magna Carta
provides:

“Section 5. Equal Opportunity for Employment.—No disabled person shall be


denied access to opportunities for suitable employment. A qualified disabled
employee shall be subject to the same terms and conditions of employment and
the same compensation, privileges, benefits, fringe benefits, incentives or
allowances as a qualified able bodied person.”

The fact that the employees were qualified disabled persons necessarily removes
the employment contracts from the ambit of Article 80. Since the Magna Carta
accords them the rights of qualified able-bodied persons, they are thus covered
by Article 280 of the Labor Code, which provides:

“ART. 280. Regular and Casual Employment. — The provisions of written


agreement to the contrary notwithstanding and regardless of the oral agreement
of the parties, an employment shall be deemed to be regular where the
employee has been engaged to perform activities which are usually necessary or
desirable in the usual business or trade of the employer, x x x”

“The primary standard, therefore, of determining regular employment is the


reasonable connection between the particular activity performed by the
employee in relation to the usual trade or business of the employer. The test is
whether the former is usually necessary or desirable in the usual business or trade
of the employer. The connection can be determined by considering the nature
of the work performed and its relation to the scheme of the particular business or
trade in its entirety. Also if the employee has been performing the job for at least
one year, even if the performance is not continuous and merely intermittent, the
law deems repeated and continuing need for its performance as sufficient
evidence of the necessity if not indispensability of that activity to the
business. Hence, the employment is considered regular, but only with respect to
such activity, and while such activity exists.”

Respondent bank entered into the aforesaid contract with a total of 56


handicapped workers and renewed the contracts of 37 of them. In fact, two of
them worked from 1988 to 1993. Verily, the renewal of the contracts of the
handicapped workers and the hiring of others lead to the conclusion that their
tasks were beneficial and necessary to the bank. More important, these facts
show that they were qualified to perform the responsibilities of their positions. In
other words, their disability did not render them unqualified or unfit for the tasks
assigned to them.

Without a doubt, the task of counting and sorting bills is necessary and desirable
to the business of respondent bank. With the exception of sixteen of them,
petitioners performed these tasks for more than six months.

Petition granted
QUIRICO LOPEZ v. ALTURAS GROUP OF COMPANIES and/or MARLITO UY

G.R. No. 191008 , April 11, 2011

FACTS: Quirico Lopez (petitioner) was hired by respondent Alturas Group of


Companies in 1997 as truck driver. Ten years later or sometime in November 2007,
he was dismissed after he was allegedly caught by respondent’s security guard in
the act of attempting to smuggle out of the company premises 60 kilos of scrap
iron worth P840 aboard respondents’ Isuzu Cargo Aluminum Van with Plate
Number PHP 271 that was then assigned to him. When questioned, petitioner
allegedly admitted to the security guard that he was taking out the scrap iron
consisting of lift springs out of which he would make axes.

Petitioner, in compliance with the Show Cause Notice dated December 5, 2007
issued by respondent company’s Human Resource Department Manager, denied
the allegations by a handwritten explanation written in the Visayan dialect.

Finding petitioner’s explanation unsatisfactory, respondent company terminated


his employment by Notice of Termination effective December 14, 2007 on the
grounds of loss of trust and confidence, and of violation of company rules and
regulations. In issuing the Notice, respondent company also took into account
the result of an investigation showing that petitioner had been smuggling out its
cartons which he had sold, in conspiracy with one Maritess Alaba, for his own
benefit to thus prompt it to file a criminal case for Qualified Theft against him
before the Regional Trial Court (RTC) of Bohol. It had in fact earlier filed another
criminal case for Qualified Theft against petitioner arising from the theft of the
scrap iron.

ISSUE: Whether or not petitioner was not afforded procedural due process.

RULING: This Court has held that there is no violation of due process even if no
hearing was conducted, where the party was given a chance to explain his side
of the controversy. What is frowned upon is the denial of the opportunity to be
heard.

Petitioner was given the opportunity to explain his side when he was informed of
the charge against him and required to submit his written explanation with which
he complied.

The above rulings are a clear recognition that the employer may provide an
employee with ample opportunity to be heard and defend himself with the
assistance of a representative or counsel in ways other than a formal hearing. The
employee can be fully afforded a chance to respond to the charges against him,
adduce his evidence or rebut the evidence against him through a wide array of
methods, verbal or written.
After receiving the first notice apprising him of the charges against him, the
employee may submit a written explanation (which may be in the form of a letter,
memorandum, affidavit or position paper) and offer evidence in support thereof,
like relevant company records (such as his 201 file and daily time records) and the
sworn statements of his witnesses. For this purpose, he may prepare his
explanation personally or with the assistance of a representative or counsel. He
may also ask the employer to provide him copy of records material to his defense.
His written explanation may also include a request that a formal hearing or
conference be held. In such a case, the conduct of a formal hearing or
conference becomes mandatory, just as it is where there exist substantial
evidentiary disputes or where company rules or practice requires an actual
hearing as part of employment pretermination procedure.

The right to counsel and the assistance of one in investigations involving


termination cases is neither indispensable nor mandatory, except when the
employee himself requests for one or that he manifests that he wants a formal
hearing on the charges against him.
June 3, 2013

93. UNILEVER PHILIPPINES, INC.,Petitioner, vs. MARIA RUBY M. RIVERA, Respondent.


G.R. No. 201701

FACTS:

Maria Ruby M. Rivera was the Area Activation Executive of Unilever Philippines, Inc
for the cities of Cotabato and Davao for 14 years. She was dismissed pursuant to
company policy after she was found responsible for the deviation of funds by
Ventureslink, Unilever’s third party service provider for the company’s activation
projects. Her retirement benefits were forfeited as a legal consequence of her
dismissal from work. Rivera filed a case of illegal dismissal and money claims
against Unilever.
The LA dismissed her case for lack of merit and denied her monetary claim for lack
of basis. The NLRC however, partly grant her appeal by granting her nominal
damages for violation of her right to procedural due process, and retirement
benefits. CA affirmed the NLRC decision with modification by deleting the award
on retirement benefits and awarded separation pay in favor of Rivera as measure
of social justice.

ISSUE(S):

(1)

Whether or not a validly dismissed employee, like Rivera, is entitled to an award

of separation pay.
(2)

Whether or not the award for nominal damage to Rivera was proper.

HELD:

(1)

No. As a general rule, an employee who has been dismissed for any of the just
causes enumerated under Article 282 of the Labor Code is not entitled to a
separation pay, pursuant to Section 7, Rule I, Book VI of the Omnibus Rules
Implementing the Labor Code. In exceptional cases, however, the Court has
granted separation pay to a legally dismissed employee as an act of "social
justice" or on "equitable grounds." In both instances, it is required that the dismissal
(1) was not for serious misconduct; and (2) did not reflect on the moral character
of the employee as in the case of Philippine Long Distance Telephone Co. vs.
NLRC.
In this case, Rivera was dismissed from work because she intentionally
circumvented a strict company policy, manipulated another entity to carry out
her instructions without the company’s knowledge and approval, and directed
the diversion of funds, which she even admitted doing under the guise of
shortening the laborious process of securing funds for promotional activities from
the head office. These transgressions were serious offenses that warranted her
dismissal from employment and proved that her termination from work was for a
just cause. Hence, she is not entitled to a separation pay.

(2)

Yes.

In all cases of termination of employment, due process shall be substantially


observed as provided in Section 2, Rule XXIII, Book V of the Rules Implementing the
Labor Code. In this case, Unilever was not direct and specific in its first notice to
Rivera.The words it used were couched in general terms and were in no way
informative of the charges against her that may result in her dismissal from
employment. Evidently, there was a violation of her right to statutory due process
warranting the payment of indemnity in the form of nominal damages.
G.R. No. 173012 June 13, 2012

DOLORES T. ESGUERRA, Petitioner,


vs.
VALLE VERDE COUNTRY CLUB, INC. and ERNESTO VILLALUNA, Respondents.

FACTS:

 Petitioner Dolores Esguerra was first hired by Valle Verde Country Club as
Head Food Checker but was then promoted to Cost Control Supervisor.

 Esguerra was tasked to oversee the seminar held in the two function rooms –
the Ballroom and the Tanay Room. The arrangement was that the food shall
be served in the form of pre-paid buffet, while the drinks shall be paid in a
"pay as you order" basis.5

 However, the following day, the Valle Verde Management found out that
only the proceeds from the Tanay Room had been remitted to the
accounting department. Furthermore, there were also unauthorized
charges of food on the account of Judge Rodolfo Bonifacio, one of the
participants.

 An investigation was then conducted wherein the employees who were


assigned in the two function rooms were summoned and made to explain,
in writing, what had transpired.

 Thereafter, Valle Verde sent a memorandum to Esguerra requiring her to


show cause as to why no disciplinary action should be taken against her for
the non-remittance of the Ballroom’s sales. Esguerra was also placed under
preventive suspension with pay, pending investigation.

 In her letter-response, Esguerra denied having committed any


misappropriation. She explained that it had been her daughter who was
assigned as a food checker who lost the money. To settle the matter,
Esguerra paid the unaccounted amount as soon as her daughter informed
her about it. She also alleged that Judge Bonifacio took pity on her and told
her to take home some food and to charge it on his account. However,
Valle Verde found Esguerra’s explanation unsatisfactory and issued a
second memorandum terminating Esguerra’s employment.

 Esguerra then filed a complaint with the National Labor Relations


Commission (NLRC) for illegal dismissal which was dismissed for lack of merit.
Petitioner appealed the case to the NLRC. In its December 27, 2002
decision, it modified the decision and only awarded ₱143,000.00 as
separation pay, equivalent to one-half (½) month for every year of
service, after taking into account Esguerra’s long years of service and
absence of previous derogatory records.

 A partial motion for reconsideration was filed by Esguerra while Valle Verde
filed its own motion for reconsideration. In its resolution, the NLRC denied
Esguerra’s motion, but granted Valle Verde’s motion for reconsideration by
affirming the Labor Arbiter’s decision.

 Esguerra then elevated her case to the CA through a Rule 65 petition for
certiorari which was however denied. The CA found that the NLRC did not
commit any grave abuse of discretion in finding that Esguerra was validly
dismissed from employment for loss of trust and confidence, and that her
length of service cannot be counted in her favor. Hence, Esguerra filed the
present petition after the CA denied her motion for reconsideration.

 Esguerra argues that the appellate court erred in ruling that she had been
validly dismissed on the ground of loss of trust and confidence. She alleges
that she was only a regular employee and did not occupy a supervisory
position vested with trust and confidence. Esguerra also questions the
manner of dismissal since Valle Verde failed to comply with procedural
requirements.

ISSUES:

(1) Whether or not Esguerra had been validly dismissed?

(2) Whether or not Esguerra occupied a position of trust and confidence?

RULING:

(1) YES. In the case, Valle Verde complied with the two-fold procedural
requirements of notice and hearing.

The Court failed to find any irregularities in the service of notice to Esguerra.
Esguerra’s allegation that the notice was insufficient since it failed to contain any
intention to terminate her is incorrect.

Contrary to Esguerra’s allegation, the law does not require that an intention to
terminate one’s employment should be included in the first notice. It is enough
that employees are properly apprised of the charges brought against them so
they can properly prepare their defenses. It is only during the second notice that
the intention to terminate one’s employment should be explicitly stated.

There is also no basis to question the absence of a proper hearing. The existence
of an actual, formal "trial-type" hearing, although preferred, is not absolutely
necessary to satisfy the employee's right to be heard. Esguerra was able to
present her defenses; and only upon proper consideration of it did Valle Verde
send the second memorandum terminating her employment. Since Valle Verde
complied with the two-notice requirement, no procedural defect exists in
Esguerra’s termination.

(2) YES. Esguerra occupied a position of trust and confidence. There are two (2)
classes of positions of trust – the first class consists of managerial employees, or
those vested with the power to lay down management policies; and the second
class consists of cashiers, auditors, property custodians or those who, in the normal
and routine exercise of their functions, regularly handle significant amounts of
money or property.

In the case, Esguerra holds a position of trust of the second class. As a Cost Control
Supervisor, she had the duty to remit to the accounting department the cash sales
proceeds from every transaction she was assigned to. For this reason, Esguerra
occupies a position of trust and confidence. Any breach of the trust imposed
upon her can be a valid cause for dismissal.

In Jardine Davies, Inc. v. National Labor Relations Commission, the Supreme Court
held that loss of confidence as a just cause for termination of employment can be
invoked when an employee holds a position of responsibility, trust and
confidence. In order to constitute a just cause for dismissal, the act complained of
must be related to the performance of the duties of the dismissed employee and
must show that he or she is unfit to continue working for the employer for violation
of the trust reposed in him or her.

The Court also found no merit in the allegation that it was Esguerra’s daughter
who should be held liable since it was her responsibility to account for the cash
proceeds. In case of problems, she should have reported it. Thus, Esguerra’s failure
to make the proper report reflects on her irresponsibility in the custody of cash for
which she was accountable.

As to Esguerra’s explanation on the unauthorized charging on Judge Bonifacio’s


account, the Court found it self-serving and without merit. It held that it was highly
unethical for an employee to bring home food intended to be sold to customers.

Hence, the petition was denied.


G.R. No. 192571 July 23, 2013

ABBOTT LABORATORIES, PHILIPPINES, CECILLE A. TERRIBLE, EDWIN D. FEIST, MARIA


OLIVIA T. YABUTMISA, TERESITA C. BERNARDO, AND ALLAN G. ALMAZAR, Petitioners,
vs.
PEARLIE ANN F. ALCARAZ, Respondent.

FACTS:

 On June 27, 2004, Abbott caused the publication in a major broadsheet


newspaper of its need for a Regulatory Affairs Manager, indicating therein
the job description as well as the duties and responsibilities attendant to the
aforesaid position. Respondent Pearlie Ann Alcaraz, who worked with
another pharmaceutical company, applied for such position. Petitioner
Abbott then hired respondent Alcaraz as Regulatory Affairs Manager of the
company’s Hospira Affiliate Local Surveillance Unit (ALSU) department. The
employment contract stated that she was to be placed on probation for a
period of six (6) months beginning February 15, 2005 to August 14, 2005 and
that unless renewed, her probationary appointment expires on the date
indicated.

 Thereafter, Alcaraz received a pre-employment orientation meeting with


Almazar and received from Misa, Abbott’s Human Resources (HR) Director,
an e-mail which contained an explanation of the procedure for evaluating
the performance of probationary employees.

 During the course of her employment, Alcaraz noticed that some of the
staff had disciplinary problems. Thus, she would reprimand them for their
unprofessional behavior such as non-observance of the dress code,
moonlighting, and disrespect of Abbott officers. However, Alcaraz’s method
of management was considered by Walsh, her supervisor, to be "too
strict."14 Alcaraz approached Misa to discuss these concerns and was told
to "lie low" and let Walsh handle the matter. Misa even assured her that
Abbott’s HRD would support her in all her management decisions. 15

 On April 12, 2005, Alcaraz received an e-mail from Misa requesting


immediate action on the staff’s performance evaluation as their
probationary periods were about to end. This Alcaraz eventually submitted.

 On April 20, 2005, Alcaraz had a meeting with petitioner Cecille Terrible
(Terrible), Abbott’s former HR Director, wherein Alcaraz accidentally saw a
printed copy of an e-mail sent by Walsh to some staff members which
contained queries regarding the former’s job performance. Alcaraz asked if
Walsh’s action was the normal process of evaluation. Terrible, however, said
that it was not.
 On May 16, 2005, Alcaraz was called to a meeting with Walsh and Terrible
where she was informed that she failed to meet the regularization standards
for her position. Thereafter, she was requested to tender her resignation, else
they be forced to terminate her services. She was also told that, regardless
of her choice, she should no longer report for work and was asked to
surrender her office identification cards.

 The following day, Alcaraz learned that Walsh and Terrible had already
announced to the whole staff that Alcaraz already resigned due to health
reasons.

 On May 23, 2005, Walsh, Almazar, and Bernardo personally handed to


Alcaraz a letter stating that her services had been terminated effective May
19, 2005.21 The letter detailed the reasons for Alcaraz’s termination –
particularly, that Alcaraz: (a) did not manage her time effectively; (b) failed
to gain the trust of her staff and to build an effective rapport with them; (c)
failed to train her staff effectively; and (d) was not able to obtain the
knowledge and ability to make sound judgments on case processing and
article review which were necessary for the proper performance of her
duties.22 On May 27, 2005, Alcaraz received another copy of the said
termination letter via registered mail.23

 Alcaraz then filed a complaint for illegal dismissal and damages against
Abbott and its officers. She claimed that she should have already been
considered as a regular and not a probationary employee given Abbott’s
failure to inform her of the reasonable standards for her regularization upon
her engagement as required under Article 295 of the Labor Code. She
contended that while her employment contract stated that she was to be
engaged on a probationary status, the same did not indicate the standards
on which her regularization would be based.26

 On the contrary, petitioners maintained that Alcaraz was validly terminated


from her probationary employment given her failure to satisfy the prescribed
standards for her regularization which were made known to her at the time
of her engagement.

 In a Decision, the Labor Arbiter dismissed Alcaraz’s complaint for lack of


merit. The LA found that there was no evidence to conclude that Abbott’s
officers and employees acted in bad faith in terminating Alcaraz’s
employment.

 Alcaraz filed an appeal with the National Labor Relations Commission which
set aside the Labor Arbiter’s ruling. It held that Abbot had committed illegal
dismissal and was ordered to immediately reinstate complainant to her
former position and to pay backwages. It held that Alcaraz’s receipt of her
job description and Abbott’s Code of Conduct and Performance Modules
was not equivalent to her being actually informed of the performance
standards upon which she should have been evaluated on.
 Petitioners then filed a motion for reconsideration which was denied by the
NLRC. Hence, they filed with the CA a Petition for Certiorari. The CA
however affirmed the ruling of the NLRC and held that the latter did not
commit any grave abuse of discretion in finding that Alcaraz was illegally
dismissed.

 Since Abbott’s motion for reconsideration was denied, the petitioners filed
the instant petition with the Supreme Court.

ISSUES:

(1) Whether or not Alcaraz was sufficiently informed of the reasonable standards
to qualify her as a regular employee?

(2) Whether or not Alcaraz was validly terminated from her employment?

RULINGS:

(1) YES. The Court held that Abbott had complied with the requirements of
communicating the regularization standards to the probationary employee at the
time of the employee’s engagement. This conclusion is largely supported by
several instances which include: the publication in a newspaper of its need for a
Regulatory Affairs Manager, indicating therein the job description as well as the
duties and responsibilities to which Alcaraz applied for; the employment contract
signed by Alcaraz also specifically stated that she was to be placed on probation
for a period of six (6) months; Alcaraz received copies of Abbott’s organizational
structure and her job description through e-mail and was made to undergo a pre-
employment orientation; Alcaraz was also required to undergo a training as part
of her orientation; and received copies of Abbott’s Code of Conduct and
Performance Modules from Misa who explained to her the same.

Hence, the Court held that it cannot, therefore, be doubted that Alcaraz was
well-aware that her regularization would depend on her ability and capacity to
fulfill the requirements of her position as Regulatory Affairs Manager and that her
failure to perform such would give Abbott a valid cause to terminate her
probationary employment. Thus, the Court ruled that Alcaraz’s status as a
probationary employee and her consequent dismissal must stand.

Consequently, in holding that Alcaraz was illegally dismissed due to her status as a
regular and not a probationary employee, the Court finds that the NLRC
committed a grave abuse of discretion. Alcaraz’s receipt of her job description
and Abbott’s Code of Conduct and Performance Modules was not equivalent to
being actually informed of the performance standards upon which she should
have been evaluated on.64 It, however, overlooked the legal implication of the
other attendant circumstances which should have warranted a contrary finding
that Alcaraz well-aware of her duties and responsibilities and that her failure to
adequately perform the same would lead to her non-regularization and
eventually, her termination.

(2) NO. Alcaraz was not validly terminated since Abbott violated its own
procedure in dismissing a probationary employee.

While there lies due cause to terminate Alcaraz’s probationary employment for
her failure to meet the standards required for her regularization, and while it must
be further pointed out that Abbott had satisfied its statutory duty to serve a written
notice of termination, the fact that it violated its own company procedure renders
the termination of Alcaraz’s employment procedurally infirm, warranting the
payment of nominal damages.

Records show that Abbott’s PPSE procedure mandates, that the job performance
of a probationary employee should be formally reviewed and discussed with the
employee at least twice: first on the third month and second on the fifth month
from the date of employment. Abbott is also required to come up with a
Performance Improvement Plan during the third month review to bridge the gap
between the employee’s performance and the standards set, if any. In addition,
a signed copy of the PPSE form should be submitted to Abbott’s HRD as the same
would serve as basis for recommending the confirmation or termination of the
probationary employment.

In this case, it is apparent that Abbott failed to follow the above-stated procedure
in evaluating Alcaraz. For one, there lies a hiatus of evidence that a signed copy
of Alcaraz’s PPSE form was submitted to the HRD. It was not even shown that a
PPSE form was completed to formally assess her performance. Neither was the
performance evaluation discussed with her during the third and fifth months of her
employment. Nor did Abbott come up with the necessary Performance
Improvement Plan to properly gauge Alcaraz’s performance with the set
company standards.

While it is Abbott’s management prerogative to promulgate its own company


rules and even subsequently amend them, this right equally demands that when it
does create its own policies and thereafter notify its employee of the same, it
accords upon itself the obligation to faithfully implement them.

In this light, case law has settled that an employer who terminates an employee
for a valid cause but does so through invalid procedure is liable to pay the latter
nominal damages. As held in the case of Agabon v. NLRC (Agabon), the lack of
statutory due process should not nullify the dismissal, or render it illegal, or
ineffectual. However, the employer should indemnify the employee for the
violation of his statutory rights.
Anent the proper amount of damages to be awarded, the Court observes that
Alcaraz’s dismissal proceeded from her failure to comply with the standards
required for her regularization. As such, it is undeniable that the dismissal process
was, in effect, initiated by an act imputable to the employee or upon a just
cause. Therefore, the Court deems it appropriate to fix the amount of nominal
damages at the amount of ₱30,000.00.

G.R. No. 202996 June 18, 2014

MARLO A. DEOFERIO, Petitioner,


vs.
INTEL TECHNOLOGY PHILIPPINES, INC. and/or MIKE WENTLING, Respondents.

FACTS:

 On February 1, 1996, respondent Intel Technology Philippines, Inc. (Intel)


employed petitioner Deoferio as a product quality and reliability engineer.
In July 2001, Intel assigned him to the United States as a validation engineer
for an agreed period of two years. However, Deoferio was repatriated to
the Philippines after being confined at Providence St. Vincent Medical
Center for major depression with psychosis. In the Philippines, he worked as
a product engineer.

 Deoferio underwent a series of medical and psychiatric treatment at Intel’s


expense after his confinement in the United States. On August 8, 2005, Dr.
Paul Lee, a consultant psychiatrist of the Philippine General Hospital,
concluded that Deoferio was suffering from schizophrenia. Thereafter, Dr.
Lee issued a psychiatric report concluding and stating that Deoferio’s
psychotic symptoms are not curable within a period of six months and "will
negatively affect his work and social relation with his co-workers. Pursuant
to these findings, Intel issued Deoferio a notice of termination on March 10,
2006.

 Deoferio responded to his termination of employment by filing a complaint


for illegal dismissal with prayer for money claims against respondents Intel
and Mike Wentling. He denied that he never had mental illness and insisted
that he satisfactorily performed his duties as a product engineer. He argued
that Intel violated his statutory right to procedural due process when it
summarily issued a notice of termination. Deoferio also prayed for
backwages, separation pay, moral and exemplary damages, as well as
attorney’s fees.

 Meanwhile, the respondents argued that Deoferio’s dismissal was based on


Dr. Lee’s certification. They also insisted that Deoferio’s presence at Intel’s
premises would pose an actual harm to his co-employees as shown by his
previous acts which included stepping on the keyboards and disarranging
the desks of his co-employees. They further asserted that the twin-notice
requirement in dismissals does not apply to terminations under Article 284 of
the Labor Code. They emphasized that the Labor Code’s implementing
rules (IRR) only requires a competent public health authority’s certification
to effectively terminate the services of an employee. They insisted that
Deoferio’s separation and retirement payments were also offset by his
company car loan.

 The Labor Arbiter (LA) ruled that Deoferio had been validly dismissed. It
gave weight to Dr. Lee’s certification that Deoferio had been suffering from
schizophrenia and was not fit for employment. The LA further held that the
Labor Code and its IRR do not require the employer to comply with the twin-
notice requirement in dismissals due to disease. On appeal by Deoferio, the
National Labor Relations Commission (NLRC) wholly affirmed the LA’s ruling
and denied the petitioner’s motion for reconsideration.

 The CA likewise affirmed the NLRC decision. It agreed with the lower
tribunals’ findings that Deoferio was suffering from schizophrenia and that
his continued employment at Intel would be prejudicial to his health and to
those of his co-employees. It ruled that the only procedural requirement
under the IRR is the certification by a competent public health authority on
the non-curability of the disease within a period of six months even with
proper medical treatment. Hence, Deoferio filed the present petition after
the CA denied his motion for reconsideration.

 In the present petition, Deoferio argues that the uniform finding that he was
suffering from schizophrenia is belied by his subsequent employment which
both offered him higher compensations. He also asserts that the Labor
Code does not exempt the employer from complying with the twin-notice
requirement in terminations due to disease.

 On the contrary, the respondents posit that Deoferio’s subsequent


employment in other corporations is irrelevant in determining the validity of
his dismissal since the law merely requires the non-curability of the disease
within a period of six months even with proper medical treatment.

ISSUES:

(1) Whether or not Intel had an authorized cause to dismiss Deoferio from
employment?
(2) Whether or not the twin-notice requirement in dismissals applies to terminations
due to disease?

(3) Whether or not Deoferio is entitled to nominal damages for violation of his right
to statutory procedural due process?

(4) Whether or not Deoferio is entitled to salary differential, backwages, separation


pay, moral and exemplary damages, as well as attorney's fees?

RULING:

(1) YES. Intel had an authorized cause to dismiss Deoferio.

The present case involves termination due to disease – an authorized cause for
dismissal under Article 284 of the Labor Code.

Art. 284. Disease as ground for termination. – An employer may


terminate the services of an employee who has been found to be
suffering from any disease and whose continued employment is
prohibited by law or is prejudicial to his health as well as to the health of
his co-employees: Provided, That he is paid separation pay equivalent
to at least one (1) month salary or to one-half (1/2) month salary for
every year of service, whichever is greater, a fraction of at least six (6)
months being considered as one (1) whole year.

Without the medical certificate, there can be no authorized cause for the
employee’s dismissal. The absence of this element thus renders the dismissal void
and illegal. This requirement is not merely a procedural requirement, but a
substantive one. The certification from a competent public health authority is
precisely the substantial evidence required by law to prove the existence of the
disease itself, its non-curability within a period of six months even with proper
medical treatment, and the prejudice that it would cause to the health of the sick
employee and to those of his co-employees.

In the case, the Court held that Dr. Lee’s psychiatric report substantially proves
that Deoferio was suffering from schizophrenia, that his disease was not curable
within a period of six months even with proper medical treatment, and that his
continued employment would be prejudicial to his mental health. This conclusion
is further substantiated by the unusual and bizarre acts that Deoferio committed
while at Intel’s employ.

(2) YES. The twin-notice requirement applies to terminations under Article 284 of
the Labor Code.

The Labor Code and its IRR are silent on the procedural due process required in
terminations due to disease. Despite the seeming gap in the law, Section 2, Rule 1,
Book VI of the IRR expressly states that the employee should be afforded
procedural due process in all cases of dismissals.

In Sy v. Court of Appeals, the Court held that the employer must furnish the
employee two written notices in terminations due to disease, namely: (1) the
notice to apprise the employee of the ground for which his dismissal is sought; and
(2) the notice informing the employee of his dismissal, to be issued after the
employee has been given reasonable opportunity to answer and to be heard on
his defense.

From these perspectives, the CA erred in not finding that the NLRC gravely
abused its discretion when it ruled that the twin-notice requirement does not
apply to Article 284 of the Labor Code.

(3) YES. Deoferio is entitled to nominal damages for violation of his right to
statutory procedural due process. Intel’s violation of Deoferio’s right to statutory
procedural due process warrants the payment of indemnity in the form of nominal
damages.

In Jaka Food Processing Corp. v. Pacot, the Court distinguished between


terminations based on Article 282 and Article 283 of the Labor Code. It pegged
the nominal damages at ₱30,000.00 if the dismissal is based on a just cause but
the employer failed to comply with the twin-notice requirement. On the other
hand, it fixed the nominal damages at ₱50,000.00 if the dismissal is due to an
authorized cause but the employer failed to comply with the notice requirement.

With respect to Article 284 of the Labor Code, terminations due to disease do not
entail any wrongdoing on the part of the employee. It also does not purely involve
the employer’s willful and voluntary exercise of management prerogative – a
function associated with the employer's inherent right to control and effectively
manage its enterprise. Rather, terminations due to disease are occasioned by
matters generally beyond the worker and the employer's control.

In fixing the amount of nominal damages whose determination is addressed to our


sound discretion, the Court should take into account several factors surrounding
the case, such as: (1) the employer’s financial, medical, and/or moral assistance
to the sick employee; (2) the flexibility and leeway that the employer allowed the
sick employee in performing his duties while attending to his medical needs; (3)
the employer’s grant of other termination benefits in favor of the employee; and
(4) whether there was a bona fide attempt on the part of the employer to comply
with the twin-notice requirement as opposed to giving no notice at all.

In the case, the Court awarded Deoferio the sum of ₱30,000.00 as nominal
damages for violation of his statutory right to procedural due process. In so ruling,
it took into account the fact that Intel financed Deoferio’s medical expenses for
more than four years and allowed the latter to take lengthy leave of absences to
attend to his medical needs.

(4) NO. Deoferio is not entitled to salary differential, backwages, separation pay,
moral and exemplary damages, as well as attorney's fees.

Deoferio's claim for salary differential is already barred by prescription. Under


Article 291 of the Labor Code, all money claims arising from employer-employee
relations shall be filed within three years from the time the cause of action
accrued. In the current case, more than four years have elapsed from the pre-
termination of his assignment to the United States until the filing of his complaint
against the respondents.

Meanwhile, his claim for backwages, separation pay, moral and exemplary
damages, as well as attorney's fees must also necessarily fail as a consequence of
our finding that his dismissal was for an authorized cause and that the respondents
acted in good faith when they terminated his services.

Hence, the petition was partially granted.

G.R. No. 197353 April 1, 2013

ALEXANDER B. BANARES, Petitioner,


vs.
TABACO WOMEN'S TRANSPORT SERVICE1 COOPERATIVE (T A WTRASCO),
represented by DIR. RENOL BARCEBAL, ET AL., Respondents.

FACTS:

 Petitioner Alexander Banares was for some time the general manager of
Tabaco Women's Transport Service Cooperative (TAWTRASCO) until its
management, on March 6, 2006, terminated his services. On March 7, 2006,
petitioner filed a complaint for illegal dismissal and payment of monetary
claims before the Labor Arbiter.

 The Labor Arbiter rendered a decision declaring complainant to have been


illegally dismissed from his employment. Respondent Tabaco Women’s
Transport Service Cooperative (TAWTRASCO) was then ordered to
immediately reinstate complainant to his former position and to pay the
amount of ₱119,600.00 as backwages and damages.
 Since TAWTRASCO opted not to appeal, the LA Decision became final and
executor and TAWTRASCO thereafter paid petitioner the amount of
P119,600. However, petitioner was not immediately reinstated. Owing to the
strained employer-employee relationship perceived to exist between them,
TAWTRASCO offered to pay petitioner separation pay of P172, 296 but
petitioner rejected the offer.

 Eventually, the two entered into a Compromise Agreement, in which


petitioner waived a portion of his monetary claim, specifically his
backwages for a certain period and agreed that the amount due shall be
payable in three (3) installments. In turn, TAWTRASCO undertook to reinstate
the petitioner effective February 6, 2007. Accordingly, the LA issued an
Order declaring the instant case closed and terminated.

 On February 24, 2007, petitioner received a copy of a Memorandum with a


copy of a board resolution requiring him to report at the company’s Virac,
Catanduanes terminal. A day after, petitioner went to see Oliva Barcebal
(Oliva), the BOD Chairman, to decry that the adverted return-to-work
memorandum and board resolution contravene the NLRC-approved
compromise agreement which called for his reinstatement as general
manager without loss of seniority rights.

 On March 20, 2007, TAWTRASCO served petitioner a copy of the


Memorandum which set forth his location assignment, as follows:
temporarily assigned at the Virac, Catanduanes terminal/office for two
months, after which he is to divide his time between the Virac Terminal and
the Araneta Center Bus Terminal (ACBT), three days (Monday to
Wednesday) in Virac and two days (Friday and Saturday) in Cubao, utilizing
Thursday as his travel day in between offices.

 As ordered, petitioner reported to the Virac terminal which purportedly


needed his attention due to its flagging operations and management
problems.

 However, barely a week into his new assignment, petitioner proposed the
construction/rehabilitation of the passenger lounge in the Virac terminal,
among other improvements. The proposal came with a request for a
monthly lodging accommodation allowance of P1,700 for the duration of
his stay in Virac. While the management approved the desired construction
projects, it denied petitioner’s plea for cash lodging allowance and instead
urged petitioner to use the Virac office for lodging purposes.

 On April 12, 2007, during an ocular inspection, Oliva discovered that


petitioner had not reported for work since March 31, 2007. Thus, a
memorandum was issued asking petitioner to explain his absence.

 In his letter-reply, petitioner argued that the reinstatement effected by the


respondent was fictitious and a sham kind of return to work order. Hence,
they cannot charge him for abandonment of work. He argued, among
others, that the manner and nature of work he was to perform was a
deviation from his original work and in effect a demotion in rank. He also
alleged that there are employees in his jurisdiction that were instructed not
to follow his lawful orders.

 On April 27, 2007, petitioner filed a complaint against TAWTRASCO for


nonpayment of salaries and withholding of privileges before the Labor
Arbiter. However, via a Manifestation with application for the issuance of an
alias writ of execution, petitioner prayed that his complaint be deemed
withdrawn "for the purpose of not confusing the essence of consolidation
and in order to give way to the smooth proceedings and fast adjudication
on the merits."

 Thereafter, the LA effectively issued the desired alias writ of execution. It


held that since there was no compliance of the reinstatement aspect of the
Decision, petitioner is entitled to his reinstatement salaries less the amount
he already received, reckoned from date of receipt by respondent of the
decision on October 11, 2006 to date of such order. Respondent was also
ordered to reinstate complainant to his former position as General
Manager, without loss of seniority right and pay petitioner the amount of
₱255,000.00, representing the latter’s reinstatement salaries and monthly
allowance. The latter was also ordered to show proof of compliance.

 TAWTRASCO appealed to the NLRC which dismissed such and also denied
its motion for reconsideration. It held that TAWTRASCO only partially
complied with the 2006 Decision of the LA by paying the backwages of
petitioner without complying with the reinstatement aspect. The NLRC
denied, through its November 18, 2009 Resolution,

 Upon appeal to the CA, the petition of TAWTRASCO


was granted. It found TAWTRASCO to have fully reinstated petitioner to his
former post and it was petitioner who abandoned his work by no longer
reporting. Since his motion for reconsideration was denied, petitioner then
filed a petition for review with the Supreme Court.

ISSUES:

(1) Whether or not petitioner’s refusal to report in the Virac terminal in early April
2007 constitutes abandonment?

(2) Whether or not there was a proper and genuine reinstatement of petitioner to
his former position of General Manager of TAWTRASCO without loss of seniority
rights and privileges?

(3) Whether or not reinstatement is still viable in the case?


RULING:

(1) NO. The Court held that petitioner’s refusal, during the period material, to
report for work at the Virac terminal does not, without more, translate to
abandonment. For abandonment to exist, it is essential (1) that the employee
must have failed to report for work or must have been absent without valid or
justifiable reason; and (2) that there must have been a clear intention to sever the
employer-employee relationship manifested by some overt acts. However, these
concurring elements of abandonment are not present in the instant case.

As discussed, the reinstatement order has not been faithfully complied with. And
varied but justifiable reasons obtain which made petitioner’s work at the Virac
terminal untenable which includes the absence of a proper work space, office
furniture and equipment as well as office supplies. Thus, it could not be said that
petitioner’s absence is without valid or justifiable cause.

Moreover, petitioner has not manifested, by overt acts, a clear intention to sever
his employment with TAWTRASCO. In fact, petitioner lost no time in filing a
complaint against respondent for nonpayment of salaries and forfeiture of
boarding house privilege. Thereafter, via a Manifestation, he sought the early
issuance of an alias writ of execution purposely for the full implementation of the
final and executory LA August 22, 2006 Decision. Thus, these twin actions clearly
argue against a finding of abandonment on petitioner’s part.

It is a settled doctrine that the filing of an illegal dismissal suit is inconsistent with
the charge of abandonment, for an employee who takes steps to protest his
dismissal cannot by logic be said to have abandoned his work.

(2) NO. The Court held that the "reinstatement" of petitioner as general manager
of TAWTRASCO effected pursuant to the compromise agreement was not a real,
bona fide reinstatement.

First, TAWTRASCO after the compromise agreement signing, directed petitioner to


report to the Virac terminal with duties and responsibilities not befitting a general
manager of a transport company. In fine, the assignment partook of the nature of
a demotion.

Petitioner was tasked to discharge menial duties, such as maintaining a record of


the "in" and "out" of freight loaded on all TAWTRASCO buses and signing the trip
records of the buses going out daily. The Court held that these tasks cannot be
classified as pertaining to the office of a general manager, but that of a checker.

Second, while Memorandum No. 10 was couched as if TAWTRASCO had in mind


the reinstatement of petitioner to his former position, TAWTRASCO withheld
petitioner’s customary boarding house privilege and did not provide him with a
formal office space but instead gave him an unsafe and dilapidated office.

Under Article 223 of the Labor Code, an employee entitled to reinstatement "shall
either be admitted back to work under the same terms and conditions prevailing
prior to his dismissal or separation x x x." Verily, an illegally dismissed employee is
entitled to reinstatement without loss of seniority rights and to other established
employment privileges, and to his full backwages. The boarding house privilege
being an established perk accorded to petitioner ought to have been granted
him if a real and authentic reinstatement to his former position as general
manager is to be posited.

In the case, the Court held that the embarrassing work arrangement provided by
respondent is what triggered petitioner’s refusal to work, which it considered as
very much justified.

The Court finds that petitioner was not truly reinstated by TAWTRASCO consistent
with the final and executory August 22, 2006 Decision of the LA and the February
5, 2007 Compromise Agreement inked by the parties in the presence of the
hearing LA.

(3) NO. Supervening events had already transpired which inexorably makes the
reinstatement infeasible. For one, the TAWTRASCO already appointed a new
general manager. As a matter of settled law, reinstatement and payment of
backwages, as the normal consequences of illegal dismissal, presuppose that the
previous position from which the employee has been removed is still in existence
or there is an unfilled position of a nature, more or less, similar to the one previously
occupied by said employee.

Furthermore, reinstatement is no longer viable where, among other things, the


relations between the employer and employee have been so severely strained,
that it is not in the best interest of the parties, nor is it advisable or practical to
order reinstatement.

Under the doctrine of strained relations, payment of separation pay is considered


an acceptable alternative to reinstatement when the latter option is no longer
desirable or viable. Separation pay is made an alternative relief in lieu of
reinstatement in certain circumstances, such as: (1) when reinstatement can no
longer be effected in view of the passage of a long period of time or because of
the realities of the situation; (2) reinstatement is inimical to the employer’s interest;
(3) reinstatement is no longer feasible; (4) reinstatement does not serve the best
interests of the parties involved; (5) the employer is prejudiced by the workers’
continued employment; (6) facts that make execution unjust or inequitable have
supervened; or (7) strained relations between the employer and the employee.34

Where reinstatement is no longer viable as an option, separation pay equivalent


to one (1) month salary for every year of service should be awarded as an
alternative.35 In lieu of reinstatement, petitioner is entitled to separation pay
equivalent to one (1) month salary for every year of service reckoned from the
time he commenced his employment with TAWTRASCO until finality of this
Decision.

In addition, petitioner is entitled to backwages and other emoluments due him


from the time he did not report for work on March 31, 2007 until the finality of this
Decision.

Hence, the instant petition was granted.

Bustamante vs. NLRC, 265 SCRA 61

FACTS :

Respondent company is engaged in the business of producing high grade


bananas in its plantation in Davao del Norte. Petitioners Paulino Bantayan,
Fernando Bustamante, Mario Sumonod and Osmalik Bustamante were employed
as laborers and harvesters while petitioner Sabu Lamaran was employed as a
laborer and sprayer in respondent company’s plantation.

All the petitioners signed contracts of employment for a period of six (6) months
from January to July 1990 but they had started working sometime in September
1989. Previously, they were hired to do the same work for periods lasting a month
or more, from 1985 to 1989. Before the contracts of employment expired on 2 July
1990, petitioners’ employments were terminated on 25 June 1990 on the ground of
poor performance on account of age, as not one of them was allegedly below
forty (40) years old.

Petitioners filed a complaint for illegal dismissal. NLRC find the dismissal illegal
,order respondent Evergreen Farms, Inc. to immediately reinstate complainants to
their former position with six (6) months backwages and claims for underpayment
of wages is hereby dismissed for lack of merit.

However, on 8 March 1993, public respondent dismissed the appeal of private


respondent company for lack of merit.Thereafter, issued a second resolution
affirming its earlier resolution on illegal dismissal but deleting the award of
backwages on the ground that the termination of petitioners' employments "was
the result of the latter's (private respondent) mistaken interpretation of the law and
that the same was therefore not necessarily attended by bad faith, nor
arbitrariness”.

In their present petition, petitioners argue that the public respondent gravely
abused its discretion in rendering the second resolution which removed the award
of backwages in their favor.
ISSUE :

Whether or not NLRC gravely abused its discretion in the removal of the award of
backwages.

HELD :

Yes. The Court did not sustain public respondent’s theory that private respondent
should not be made to compensate petitioners for backwages because its
termination of their employment was not made in bad faith. The act of hiring and
re-hiring the petitioners over a period of time without considering them as regular
employees evidences bad faith on the part of private respondent. The public
respondent made a finding to this effect when it stated that the subsequent
rehiring of petitioners on a probationary status “clearly appears to be a
convenient subterfuge on the part of management to prevent complainants
(petitioners) from becoming regular employees.”

In the case at bar, there is no valid cause for dismissal. The employees (petitioners)
have not performed any act to warrant termination of their employment.
Consequently, petitioners are entitled to their full backwages and other benefits
from the time their compensation was withheld from them up to the time of their
actual reinstatement.

WHEREFORE, the Resolution of the National Labor Relations Commission dated 3


May 1993 is modified in that its deletion of the award for backwages in favor of
petitioners, is SET ASIDE. The decision of the Labor Arbiter dated 26 April 1991 is
AFFIRMED with the modification that backwages shall be paid to petitioners from
the time of their illegal dismissal on 25 June 1990 up to the date of their
reinstatement. If reinstatement is no longer feasible, a one-month salary shall be
paid the petitioners as ordered in the labor arbiter's decision; in addition to the
adjudged backwages.

Conrado A. Lim Vs. HMR Philippines, Inc., Et Al; G.R. No. 201483, 04 August 2014

FACTS:

Petitioner Conrado A. Lim (Lim) filed a case for illegal dismissal and money claims
against respondents, HMR Philippines, Inc. (HMR)and its officers. The Labor Arbiter
(LA) dismissed the complaint for lack of merit. However, the National Labor
Relations Commission (NLRC), reversed the LA and declared Lim to have been
illegally dismissed. The respondent-appellee Company is hereby ordered to
reinstate immediately the said employee to his former position without loss of
seniority rights and other privileges. Furthermore, the respondent-appellee
Company is hereby ordered to pay the complainant-appellant his full
backwages, reckoned from his dismissal on February 3, 2001 up to the
promulgation of this Decision.

All other claims are hereby DISMISSED for lack of merit.

The Computation and Research Unit (CRU) of this Commission is hereby directed
to compute the backwages and the 10% annual increase from 1998 to 2000 and
computed the backwages from February 3, 2001, the date of the illegal dismissal,
up to October 31, 2007, the date of factual reinstatement.

HMR opposed the computation arguing that the backwages should be


computed until April 11, 2003 only, the date of promulgation of the NLRC decision,
as stated in the dispositive portion of the NLRC decision, which provided that
backwages shall be "reckoned from his dismissal on February 3, 2001 up to the
promulgation of this Decision." It also noted that the 10% annual increase was
computed from 1998 to 2007, instead of only from 1998 to 2000 as decreed.

Lim argued that the body of the NLRC decision explictly stated that he was
entitled to full backwages from the time he was illegally dismissed until his actual
reinstatement, which was also in accord with Article 279 of the Labor Code and
all prevailing jurisprudence.

Complainant also claims that he is entitled to 15 days sick leave pay, a perusal of
the personnel policy handbook on the grant of said benefit shows that sick leave
pay is availed of only upon notification of illness and conversion thereof to cash is
subject to the discretion of management.

ISSUES:

1. Whether the computation of backwages should be reckoned until the


promulgation of the NLRC Decision or until actual reinstatement?

2. Whether the petitioner is entitled to salary differentials?

3. Whether the petitioner is entitled to holiday pay?

HELD:

1. Back wages should be computed from the time the petitioner was illegally
dismissed up to his actual reinstatement.
Article 279 of the Labor Code is clear in providing that an illegally dismissed
employee is entitled to his full backwages computed from the time his
compensation was withheld up to the time of his actual reinstatement, to wit:

Art. 279. Security of tenure.In cases of regular employment, the employer shall not
terminate the services of an employee except for a just cause or when authorized
by this Title. An employee who is unjustly dismissed from work shall be entitled to
reinstatement without loss of seniority rights and other privileges and to his full
backwages, inclusive of allowances, and to his other benefits or their monetary
equivalent computed from the time his compensation was withheld from him up
to the time of his actual reinstatement. [Emphases and underscoring supplied]

In accordance with this provision, the body of the April 11, 2003 NLRC decision
expressly recognizes that Lim is entitled to his full backwages until his actual
reinstatement, as follows:

In fine, the act of complainant-appellant herein, do not constitute a serious


misconduct as tojustify his dismissal. As such, he is, thus, entitled to reinstatement to
his former position as Assistant Technical Manager, unless such position no longer
exists, in which case, he shall be given a substantially equivalent position without
loss of seniority rights. He is, likewise, entitled to his full backwages from the time he
was illegally dismissed until his actual reinstatement.20 [Emphasis and underscoring
supplied]

2. Petitioner is entitled to salary differentials.

The court see no reason, therefore, why complainant-appellant herein, being a


regular employee, should be deprived of what he is entitled to under Company
policy. As such, he should be paid his unpaid 10% annual increase for the years
1998, 1999 and 2000.

In Equitable Banking Corporation v. Sadac,41 the Court held that although Article
279 of the Labor Code mandates that an employee’s full backwages be inclusive
of allowances and other benefits, salary increases cannot be interpreted as either
an allowance or a benefit, as allowances and benefits are separate from salary,
while a salary increase is added to salary as an increment thereto.42 It was further
held therein that the base figure to be used in the computation of backwages
was pegged at the wage rate at the time of the employee’s dismissal, inclusive of
regular allowances that the employee had been receiving such as the
emergency living allowances and the 13th month pay mandated by law. The
award of salary differentials was not allowed, the rule being that upon
reinstatement, illegally dismissed employees were to be paid their backwages
without deduction and qualification as to any wage increases orother benefits
that might have been received by their co-workerswho were not dismissed.43

It must be noted that the NLRC did not err in awarding the unpaid salary increase
for the years 1998-2000 as such did not constitute backwages as a consequence
of the petitioner’s illegal dismissal, but was earned and owing to the petitioner
before he was illegally terminated.

3. Petitioner is entitled to Holiday pay as Labor Arbiter finds that such is not yet
included in the base pay.

The respondents insist that the base pay of Lim is already inclusive of holiday pay.
The records, however, are insufficient to determine whether holiday pay is indeed
included in the petitioner’s base pay.

Under Article 94 of the Labor Code, every worker shall be paid his regular daily
wage during regular holidays. Thus, an employee must receive his daily wage
even if he does not work on a regular holiday. The purpose of holiday pay is to
prevent diminution of the monthly income of workers on account of work
interruptions declared by the State.44

Whether or not holiday pay is included in the monthly salary of an employee, may
be gleaned from the divisors used by the company in the computation of
overtime pay and employees’ absences. To illustrate, if all nonworking days are
paid, the divisor of the monthly salary to obtain daily rate should be 365. If
nonworking days are not paid, the divisor is 251, which is a result of subtracting all
Saturdays, Sundays, and the ten legal holidays. 45 Hence, if the petitioner’s base
pay does not yet include holiday pay, it must be added tohis monetary award.

This matter is clearly for the LA to determine being the labor official charged with
the implementation of decision46 and concomitant computations.

ARMEN B. DY-DUMALASA v. DOMINGO SABADO S. FERNANDEZ, et al.

593 SCRA 656, (2009)

FACTS:

Domingo Fernandez, et al., former employees of Helios Manufacturing


Corporation (HELIOS), filed a complaint for illegal dismissal or illegal closure of
business, non-payment of salaries and other money claims against HELIOS. The
Labor Arbiter found that the closure of the Muntinlupa office/plant was a sham, as
HELIOS simply relocated its operations to a new plant in Carmona, Cavite under
the new name of ―Pat & Suzara,‖ in response to the newly-established local union.
HELIOS and it Board of Directors and stockholders were held liable.

The NLRC modified the Labor Arbiter’s Order, holding that Dumalasa is not jointly
and severally liable with HELIOS for Fernandez, et al.’s claim, there being no
showing that she acted in bad faith nor that HELIOS cannot pay its obligations.
Dumalasa moved for reconsideration, but this was denied, hence, she appealed
to the Court of Appeals.
The appellate court reversed and set aside the NLRC Resolution, holding that
what the NLRC, in effect, modified was not the Order denying the Motion to
Quash the Writ of Execution, but the Labor Arbiter’s Decision itself. This is an
impermissible act since the Decision has become final and executor; hence, it
could no longer be reversed or modified.

Respecting NLRC’s pronouncement that Dumalasa was not jointly and severally
liable, the appellate court held that the same is a superfluity since there was no
statement, either in the main case or in the Writ, that the liability is solidary.
Therefore, Dumalasa is merely jointly liable for the judgment award. Dumalasa
moved for reconsideration of the appellate court’s Decision, which was denied.
Hence, this petition.

ISSUES:

1.) Whether or not Dumalasa is solidarily liable with HELIOS for the judgment award

HELD:

On Carmen’s liability
A perusal of the Labor Arbiter’s Decision readily shows that, notwithstanding the
finding of bad faith on the part of the management, the dispositive portion did
not expressly mention the solidary liability of the officers and Board members,
including Dumalasa.

Ineluctably, absent a clear and convincing showing of the bad faith in effecting
the closure of HELIOS that can be individually attributed to petitioner as an officer
thereof, and without the pronouncement in the Decision that she is being held
solidarily liable, petitioner is only jointly liable.

The Court in fact finds that the present action is actually a last-ditch attempt on
the part of Dumalasa to wriggle its way out of her share in the judgment
obligation and to discuss the defenses which she failed to interpose when given
the opportunity. Even as Dumalasa avers that she is not questioning the final and
executory Decision of the Labor Arbiter and admits liability, albeit only joint, still,
she proceeds to interpose the defenses that jurisdiction was not acquired over her
person and that HELIOS has a separate juridical personality.

As for Dumalasa’s questioning the levy upon her house and lot, she conveniently
omits to mention that the same are actually conjugal property belonging to her
and her husband. Whether petitioner is jointly or solidarily liable for the judgment
obligation, the levied property is not fully absolved from any lien except if it be
shown that it is exempt from execution.
Park Hotel, et al. vs. Manolo Soriano, et al., G.R. No. 171118, 10 September 2012.

FACTS:
Petitioner Park Hotel 3 is a corporation engaged in the hotel business. Petitioners
Gregg Harbutt4 (Harbutt) and Bill Percy5 (Percy) are the General Manager and
owner, respectively, of Park Hotel. Percy, Harbutt and Atty. Roberto Enriquez are
also the officers and stockholders of Burgos Corporation (Burgos), 6 a sister
company of Park Hotel.

Respondents were dismissed from work for allegedly stealing company properties.
As a result, respondents filed complaints for illegal dismissal, unfair labor practice,
and payment of moral and exemplary damages and attorney's fees, before the
Labor Arbiter (LA). In their complaints, respondents alleged that the real reason for
their dismissal was that they were organizing a union for the company's
employees.

On the other hand, petitioners alleged that aside from the charge of theft, Soriano
and Gonzales have violated various company rules and regulations 8 contained in
several memoranda issued to them. After dismissing respondents, Burgos filed a
case for qualified theft against Soriano and Gonzales before the Makati City
Prosecutor's Office, but the case was dismissed for insufficiency of evidence

LA rendered a Decision11 finding that respondents were illegally


dismissed.Unsatisfied with the LA's decision, petitioners appealed to the National
Labor Relations Commission (NLRC).
On February 1, 2001, the NLRC affirmed the LA's decision and dismissed the
appeal for lack of merit. Petitioners filed a motion for reconsideration, but it was
denied for lack of merit.
Undaunted, Park Hotel, Percy, and Harbutt filed a petition for certiorari with the
CA ascribing grave abuse of discretion amounting to lack or excess of jurisdiction
on the part of the NLRC in holding Park Hotel, Harbutt and Percy jointly and
severally liable to respondents.
CA rendered a Decision dismissing the petition and affirming with modification the
ruling of the NLRC. Hence, the instant petition.

ISSUE:
1. If petitioners are liable, whether Park Hotel, Percy and Harbutt are jointly and
severally liable with Burgos for the dismissal of respondents.

HELD:

The Court rules that before a corporation can be held accountable for the
corporate liabilities of another, the veil of corporate fiction must first be pierced. 33
Thus, before Park Hotel can be held answerable for the obligations of Burgos to its
employees, it must be sufficiently established that the two companies are actually
a single corporate entity, such that the liability of one is the liability of the other.34
In the case at bar, respondents utterly failed to prove by competent evidence
that Park Hotel was a mere instrumentality, agency, conduit or adjunct of Burgos,
or that its separate corporate veil had been used to cover any fraud or illegality
committed by Burgos against the respondents. Accordingly, Park Hotel and
Burgos cannot be considered as one and the same entity, and Park Hotel cannot
be held solidary liable with Burgos.

Nonetheless, although the corporate veil between Park Hotel and Burgos cannot
be pierced, it does not necessarily mean that Percy and Harbutt are exempt from
liability towards respondents. Verily, a corporation, being a juridical entity, may
act only through its directors, officers and employees. Obligations incurred by
them, while acting as corporate agents, are not their personal liability but the
direct accountability of the corporation they represent.38 However, corporate
officers may be deemed solidarily liable with the corporation for the termination of
employees if they acted with malice or bad faith. 39 In the present case, the lower
tribunals unanimously found that Percy and Harbutt, in their capacity as corporate
officers of Burgos, acted maliciously in terminating the services of respondents
without any valid ground and in order to suppress their right to self-organization.

In view of the foregoing, respondents are entitled to the payment of full


backwages, inclusive of allowances, and other benefits or their monetary
equivalent, and separation pay in lieu of reinstatement equivalent to one month
salary for every year of service.43 The awards of separation pay and backwages
are not mutually exclusive, and both may be given to respondents.4

G.R. No. 147590 April 2, 2007


ANTONIO C. CARAG, Petitioner,
vs.
NATIONAL LABOR RELATIONS COMMISSION, ISABEL G. PANGANIBAN-ORTIGUERRA,
as Executive Labor Arbiter, NAFLU, and MARIVELES APPAREL CORPORATION LABOR
UNION, Respondents.

FACTS:

National Federation of Labor Unions (NAFLU) and Mariveles Apparel Corporation


Labor Union (MACLU) (collectively, complainants), on behalf of all of MAC's rank
and file employees, filed a complaint against MAC for illegal dismissal brought
about by its illegal closure of business. In their complaint dated 12 August 1993,
complainants alleged the following:

1. Complainant NAFLU is the sole and exclusive bargaining agent representing


all rank and file employees of [MAC]. That there is an existing valid
Collective Bargaining Agreement (CBA) executed by the parties and that
at the time of the cause of action herein below discussed happened there
was no labor dispute between the Union and Management except cases
pending in courts filed by one against the other.

2. That on July 8, 1993, without notice of any kind filed in accordance with
pertinent provisions of the Labor Code, [MAC], for reasons known only by
herself [sic] ceased operations with the intention of completely closing its
shop or factory. Such intentions [sic] was manifested in a letter, allegedly
claimed by [MAC] as its notice filed only on the same day that the
operations closed.

3. That at the time of closure, employees who have rendered one to two
weeks work were not paid their corresponding salaries/wages, which
remain unpaid until time [sic] of this writing.

4. That there are other benefits than those above-mentioned which have
been unpaid by [MAC] at the time it decided to cease operations, benefits
gained by the workers both by and under the CBA and by operations [sic]
of law.

5. That the closure made by [MAC] in the manner and style done is perce [sic]
illegal, and had caused tremendous prejudice to all of the employees, who
suffered both mental and financial anguish and who in view thereof merits
[sic] award of all damages (actual, exemplary and moral), [illegible] to set
[an] example to firms who in the future will [illegible] the idea of simply
prematurely closing without complying [with] the basic requirement of
Notice of Closure.

In their position paper dated 3 January 1994, complainants moved to implead


Carag and David, both being owners MAC Corporation to guarantee the
satisfaction of any judgment award on the basis of Article 212(c) of the Philippine
Labor Code. Atty. Joshua L. Pastores, as counsel for respondents, submitted a
position paper dated 21 February 1994 and stated that complainants should not
have impleaded Carag and David because MAC is actually owned by a
consortium of banks. Carag and David own shares in MAC only to qualify them to
serve as MAC's officers.

Without any further proceedings, Arbiter Ortiguerra rendered her Decision dated
17 June 1994 granting the motion to implead Carag and David. In the same
Decision, Arbiter Ortiguerra declared Carag and David solidarily liable with MAC
to complainants.
ISSUE:

Whether or not mere failure to comply with notice requirement on closure or


dismissal amount to illegal act?

RULING:

No.

Bad faith does not arise automatically just because a corporation fails to comply
with the notice requirement of labor laws on company closure or dismissal of
employees. The failure to give notice is not an unlawful act because the law does
not define such failure as unlawful. Such failure to give notice is a violation of
procedural due process but does not amount to an unlawful or criminal act. Such
procedural defect is called illegal dismissal because it fails to comply with
mandatory procedural requirements, but it is not illegal in the sense that it
constitutes an unlawful or criminal act.

For a wrongdoing to make a director personally liable for debts of the


corporation, the wrongdoing approved or assented to by the director must be a
patently unlawful act. Mere failure to comply with the notice requirement of labor
laws on company closure or dismissal of employees does not amount to a
patently unlawful act. Patently unlawful acts are those declared unlawful by law
which imposes penalties for commission of such unlawful acts. There must be a
law declaring the act unlawful and penalizing the act. An example of a patently
unlawful act is violation of Article 287 of the Labor Code, which states that
violation of this provision is hereby declared unlawful and subject to the penal
provisions provided under Article 288 of this Code.

G.R. No. 151378. March 28, 2005


JAKA FOOD PROCESSING CORPORATION, Petitioners,
vs.
DARWIN PACOT, ROBERT PAROHINOG, DAVID BISNAR, MARLON DOMINGO, RHOEL
LESCANO and JONATHAN CAGABCAB, Respondents.

FACTS:

Respondents Darwin Pacot, Robert Parohinog, David Bisnar, Marlon Domingo,


RhoelLescano and Jonathan Cagabcab were earlier hired by petitioner JAKA
Foods Processing Corporation (JAKA, for short) until the latter terminated their
employment on August 29, 1997 because the corporation was "in dire financial
straits". It is not disputed, however, that the termination was affected without JAKA
complying with the requirement under Article 283 of the Labor Code regarding
the service of a written notice upon the employees and the Department of Labor
and Employment at least one (1) month before the intended date of termination.

Respondents filed a complaint for illegal dismissal against JAKA. JAKA was
defeated on appeal in the lower court hence this petition.

ISSUE:

Whether or not full back wages and separation pay be awarded to respondents
when employers effected termination without complying with the two-notice rule.

RULING:

The dismissal of the respondents was for an authorized cause under Article 283. A
dismissal for authorized cause does not necessarily imply delinquency or
culpability on the part of the employee. Instead, the dismissal process is initiated
by the employer’s exercise of his management prerogative, i.e. when the
employer opts to install labor-saving devices, when he decides to cease business
operations or when he undertakes to implement a retrenchment program.

Accordingly, it is wise to hold that:

1. if the dismissal is based on a just cause but the employer failed to comply
with the notice requirement, the sanction to be imposed upon him should
be tempered because the dismissal was initiate by an act imputable to the
employee.

2. if the dismissal is based on an authorized cause but the employer fails to


comply with the notice requirement, the sanction should be stiffer because
the dismissal process was initiated by the employer’s exercise of his
management prerogative. Thus, dismissal was upheld but ordered JAKA to
pay each of the respondents the amount of PhP 50,000.00 representing
nominal damages for non-compliance with statutory due process.
G.R. No. 164518 March 30, 2006

INDUSTRIAL TIMBER CORPORATION, INDUSTRIAL PLYWOOD GROUP CORPORATION,


TOMAS TANGSOC, JR., LORENZO TANGSOC and TOMAS TAN, Petitioners,
vs.
VIRGILIO ABABON, IGNACIO ABACAJEN, ANGELINA ABAY-ABAY, EDITH ABREA,
SAMUEL ABREA, BIENVENIDO ACILO, RODRIGO ACILO, VICTOR ACILO, ARTURO
ADVINCULA, GERTRUDES AMPARO, VIRGILIO ANTONIO, MILA ARQUITA, PRUDENCIO
ARQUITA, ALBERT ATON, WARLITA AUTIDA, ALICIA AWITAN, LEOPOLDO AYATON,
ARTURO BALBOTEN, DANILO BANATE, LOLITA BATAN, RAMIL BUTALON, CARMILITA
CAINGLES, VICENTE CAHARIAN, BENEDICTA CAJIPE, FELIPE CALLANO, ALFREDO
CARILLO, NILA CARILLO, ALGER CORBETA, GREGORIO DABALOS, TERESITA
DABALOS, VENERANDO DALAUTA, RICARDO DANGCULOS, MONTANO DAPROSA,
LUISITO DIAZ, FELIZARDO DUMULAO, EDITHA DUMANON, ALFREDO FAELNAR, RAUL
FORTUN, MAXIMO GALLA, ANGELES GALUPO, PERFECTO GAMBE, VERGINITA
GANGCA, RUPERTO GORGONIO, ROMEO HERRERO, SERGIO HORO-HORO,
FRANCISCO IBARRA, ABRAHAM JALE, DANDY LABITAD, ANTONINA LAMBANG,
ERNESTO LAUSA, VICTORIA LOOD, NEMESIO LOPE, JR., ESCARLITO MADLOS,
MARCOS MAKINANO, REMEGIO MAKINANO, VICENTE MAKINANO, REYNALDO
MASUHAY, HELEN MARATAS, ELIZABETH MENDOZA, GUILBERTA MONTEROSO, GILDA
NAVALTA, PILAR NAVARRO, SIMPORIANO NUÑEZ, JR., ELISEO ORONGAN,
ARMANDO OROPA, ASUNCION OROPA, JOSE EDWIN OROPA, BALDEMAR
PAGALAN, BARTOLOME PAGALAN, DAMASO PALOMA, MANALO PLAZA, JEREMIAS
PELAEZ, FRANCISCO PICARDAL, HERMINIA PUBLICO, ROMULO QUINTOS, FIDEL
QUITA, FELICIANO RANADA, RODOLFO RARU, LEAN CILDRIC RODRIGUEZ, SAMUEL
SAROMINES, NATIVIDAD SIGNAR, CHERRIE SON, SAMUEL TAGUPA, VICTOR TAGUPA,
BRIGIDA TABANAO, PEDRO TABANAO, ROBERTO TABANAO, MARIA TAN, RONNIE
TAN, TOLENTINO TEE, ROGELIO TAMADA, MINDA TUMAOB and ROBERTO
TUTOR, Respondents.

FACTS:

Petitioner Industrial Timber Corporation (ITC) was leased a plywood plant located
at Butuan City for a period of 5 years by Industrial Plywood Group Corporation
(IPGC). Thereafter, ITC commenced operation of the plywood plant and hired 387
workers. Sometime after, ITC notified DOLE and its workers of the plant’s shutdown
due to the non-renewal of the anti-pollution permit and the alleged lack of logs
for milling constrained ITC to lay off all its workers until further notice. A final notice
of closure or cessation of business operations followed advising the workers to
collect the benefits due them under the law and CBA. Later, IPGC took over the
plywood plant and was issued a permit to operate coincidentally the same day
the ITC ceased operation of the plant. This prompted respondents to file a
complaint for illegal dismissal and unfair labor practice alleging that the cessation
of ITC’s operation was intended to bust the union and that both corporations are
one and the same entity. LA dismissed the complaint. On appeal, NLRC first
ordered the reinstatement of employees but later on, ruled to dismiss herein
respondent’s complaints. CA set aside the decision.

ISSUE:

Whether or not to delete or reduce the nominal damages awarded to each


employee.

RULING:

YES.

The court rule that it wise and just to reduce the amount of nominal damages to
be awarded for each employee to P10,000.00 each instead of P50,000.00 each.

In the determination of the amount of nominal damages which is addressed to


the sound discretion of the court, several factors are taken into account: (1) the
authorized cause invoked, whether it was a retrenchment or a closure or
cessation of operation of the establishment due to serious business losses or
financial reverses or otherwise; (2) the number of employees to be awarded; (3)
the capacity of the employers to satisfy the awards, taken into account their
prevailing financial status as borne by the records; (4) the employer’s grant of
other termination benefits in favor of the employees; and (5) whether there was a
bona fide attempt to comply with the notice requirements as opposed to giving
no notice at all.

In the case at bar, there was valid authorized cause considering the closure or
cessation of ITC’s business which was done in good faith and due to
circumstances beyond ITC’s control. Moreover, ITC had ceased to generate any
income since its closure on August 17, 1990. Several months prior to the closure,
ITC experienced diminished income due to high production costs, erratic supply of
raw materials, depressed prices, and poor market conditions for its wood
products. It appears that ITC had given its employees all benefits in accord with
the CBA upon their termination.
G.R. No. 101427 November 8, 1993

CONSUELO B. KUNTING, petitioner,


vs.
THE NATIONAL LABOR RELATIONS COMMISSION (Fifth Division), CAGAYAN DE ORO
CITY, ST. JOSEPH SCHOOL, FR. ALOYSIUS CHANG and/or JOSEFINA MANUEL,
respondents.

FACTS:

In 1969, Consuelo B. Kunting was employed as a teacher by respondent St. Joseph


School in Gov. Camins Avenue, Zamboanga City. She was paid a basic pay and
emergency cost of living allowance (ECOLA) except during summer period when
she was paid only the basic pay. Effective January, 1988, her monthly salary was
One Thousand Eight Hundred and Twenty Pesos (P1,820.00) including ECOLA
integrated into the basic wage. She was also paid the 13th month pay up to 1987
but not her service incentive leave pays.

Every year from 1969 until, the school year 1987-1988, Consuelo and St. Joseph
executed a Teacher's Contract. For the school year 1987-1988, her performance
rating was very satisfactory. In spite of this, St. Joseph School did not renew her
employment contract for the school year 1988-89, thereby terminating her
employment with the school.

ISSUE:

Whether or not the NLRC gravely abused its discretion in ordering the payment of
separation pay in lieu of reinstatementnotwithstanding its finding on the illegal
dismissal.

RULING:

Yes.

An illegally dismissed employee's right to reinstatement is not absolute. The Court


has a long line of decisions concerning non-reinstatement of illegally dismissed
employees on various grounds. One of these grounds is when there is a finding
that the relationship between the parties has become so strained and ruptured as
to preclude a harmonious working relationship.
The order to grant petitioner separation pay instead of reinstatement is
predicated on the following finding of strained relations by the Executive Labor
Arbiter which was sustained by the NLRC:

. . . . In the instant case, while the manner of dismissal was patently illegal,
still complainant failed to refute the charges or lapses in her conduct as a
teacher, i.e. disrespectful at time, acts of insubordination, non-improvement
in her teaching methods, etc. (Affidavit of Sister Josefina Manuel, O.P.,
Annex "7" respondent's position paper, p. 7, Record). As aptly put by the
Executive Labor Arbiter, reinstatement would bring the parties in close or
frequent contact in work that may only serve to further aggravate and
inflame the existing animosity and antagonism between them.

As shown by the above-quoted portion of the decision of the NLRC, conclusion on


the "strained relations" between petitioner and private respondents was merely
gathered from the latter's evidence on the former's less than ideal conduct and
nothing more. There is no proof that such conduct actually caused animosity
between her and private respondents. Besides, there is no clear showing that the
perceived "strained relations" between the parties is of so serious a nature or of
such a degree as to justify petitioner's dismissal.

The principle of "strained relations" cannot be applied indiscriminately. Otherwise


reinstatement can never be possible simply because some hostility is invariably
engendered between the parties as a result of litigation. That is human nature.

Besides, no strained relations should arise from a valid and legal act of asserting
one's right; otherwise an employee who shall assert his right could be easily
separated from the service, by merely paying his separation pay on the pretext
that his relationship with his employer had already been strained.Whatever
resentments had been harbored by petitioner upon her unceremonious dismissal
after having been employed by St. Joseph School for more than sixteen (16) years
is understandable. Such resentments, however, would not suffice to deny her
reemployment because to do so would render for naught her constitutional right
to security of tenure and her corollary right to reinstatement under Article 279 of
the Labor Code.
DOMINICO C. CONGSON
V.
NATIONAL LABOR RELATIONS COMMISSION, NOE BARGO, ROGER HIMENO,
RAYMUNDO BADAGOS, PATRICIO SALVADOR, SR., NEHIL BARGO, JOEL MENDOZA,
and EMMANUEL CALIXIHAN
G.R. No. 114250 April 5, 1995

CASE FACTS:
Petitioner, Congson is the registered owner of Southern Fishing Industry.
The private respondents here are hired by the petitioner as regular piece-
rate workers. They were uniformly paid at a rate of P1.00 per tuna weighing thirty
(30) to eighty (80) kilos per movement, that is — from the fishing boats down to
petitioner's storage plant at a load/unload cycle of work until the tuna catch
reached its final shipment/destination.
During the first week of June 1990, petitioner notified his workers of his
proposal to reduce the rate-per-tuna movement due to the scarcity of tuna
which the respondent resisted.
The next day, they were informed that they had been replaced by a new
set of workers. They requested for a dialogue with the management but instructed
to wait for further notice. They waited for the notice of dialogue for a full week but
in vain.
This prompted the respondents to file a case against petitioner before the
NLRC Sub-Regional Arbitration for underpayment of wages and non-payment of
overtime pay, 13th month pay, holiday pay, rest day pay, and five (5)-day service
incentive leave pay; and for constructive dismissal. They claimed that petitioner
refused to give them work assignments and replaced them with new workers
when they showed resistance to the petitioner's proposed reduction of the rate-
per-tuna movement.
The respondents further filed another case against petitioner,an additional
claim for separation pay should their complaint for constructive dismissal be
upheld.

Congson filed his position paper wherein he claimed that the only issue for
resolution was private respondents' monetary claims, and that there was no
constructive dismissal. He further argued that private respondents were not
dismissed but rather, they abandoned their work after learning of petitioner's
proposal to reduce tuna movement rates because of the scarcity of tuna, and
that, it took private respondents one (1) month to return to work, but they could
no longer be accommodated as petitioner had already hired theirreplacements
after private respondents failed to heed petitioner's repeated demands for them
to return to work. Thus, respondents were not entitled to separation pay.
The Labor Arbiter decided in favor of the private respondents that they
were (constructively) dismissed from employment without just or unauthorized
cause hence illegal.
The petitioner appealed to NLRC which they affirmed the decision of LA
that the petitioner is guilty of illegal dismissal. Subsequently, petitioner's motion for
reconsideration and supplemental motion for reconsideration were denied for
lack of merit.
Hence this petition.
ISSUE: Whether or not NLRC committed grave abuse of discretion in upholding
LA’s grant of private respondents' prayer for separation pay in lieu of
reinstatement?
RULING:
No. The NLRC is correct in upholding LA’s grant of private grant of private
respondents' prayer for separation pay in lieu of reinstatement.
The Supreme Court believed that there is the existence of strained
relationship between parties after careful scrutiny of the records of the case at
bench.

Firstly, petitioner consistently refused to re-admit private respondents in his


establishment. Petitioner even replaced private respondents with a new set of
workers to perform the tasks of private respondents. Furthermore, in the petitioner’s
supplemental motion for reconsideration they only prayed for the mere deletion
of the award of separation pay, but not reinstatement of private respondents.

And secondly, private respondents themselves, from the very start, had
already indicated their aversion to their continued employment in petitioner's
establishment. The very filing of their second case, specifically for separation pay is
conclusive of private respondents' intention to sever their working ties with
petitioner.

The SC even reiterated the jurisprudence in the case of Arturo Lagniton, Sr.
vs. National Labor Relations Commission, to wit: we ruled that the refusal of the
dismissed employee to be re-admitted is constitutive of strained relations.

LORENZO MA. D.G. AGUILAR V. BURGER MACHINE HOLDINGS CORPORATION,


OSCAR E. RODRIGUEZ and MELCHOR V. DE JESUS, JR. 516 SCRA 609

CASE FACTS:

Burger Machine hired Aguilar as a Strategic Business Unit Manager Trainee.

During his employment, after the audit of BMNC's operation it shows that petitioner
had not complied with the company's purchasing system policy manual and that
he made several purchases, the amounts of which were beyond his authority to
approve.

In reply, petitioner attributed the lapses in the approval of purchases to the lack of
information on the standard operating procedures of the company.

Eventually, De Jesus, the VP of BMNC, ordered Aguilar to focuson BMNC to resolve


faster all critical problems such as shortages, low ADS, low promo compliance,
etc. And further ordered him to reduce his gross sales shortages to 1% or less by
the end of November 2001. De Jesus was only able to reduce at to as low as
0.86%.

At the end of the year, Aguilar did not receive his 14th month pay bonus of
P35,000.00 while the amount of P15,291.00 representing the alleged unauthorized
expenses was deducted from his salary.

And also, De Jesus ordered petitioner to turn over BMNC to Ms. Gloria
Centino starting March 12 up to the end of March without any explanation on the
directive. With this, Aguilar wrote a letter to Rodriguez, the BMNC’s Chairperson
seeking an explanation for the actions of De Jesus.
Thereafter, the management appointed Aguilar as Profit Center Manager
of Tatyana Foods Corporation (TFC), a new project of Burger Machine to be
established in La Union, Ilocos Sur, Ilocos Norte, Cagayan and Isabela which he
accepted.
Then he was transferred to the National Capital Region (NCR) which an
accident happened while on his way to De Jesus' office in Metro Manila. He was
thus hospitalized and was constrained to go on leave. He requested for cash
advance and financial assistance from the company for his medical expenses but
was denied. When he returned for work, De Jesus issued a memorandum directing
him to report at the Epifanio de los Santos Avenue (EDSA) office.
This prompted Aguilar to file a complaint for constructive dismissal
contending that the totality of respondents' conduct constitutes harassment
aimed to pressure him to resign from his job.
The Labor Arbiter ruled that petitioner was constructively dismissed and that
respondent corporate officials of Burger Machine are solidarily liable with the latter
for petitioner's monetary awards.
In an appeal, NLRC affirmed with LA. However, reversed by CA.
Hence, this petition.

ISSUE: Whether or not reinstatement is proper for the instant case.

RULING:
The SC held that it would be best to award separation pay instead of
reinstatement, in view of the strained relations between petitioner and
respondents. In fact, while petitioner prayed for reinstatement, he also admitted
that there is a “strained relationship now prevailing between him and
respondents. Under the doctrine of strained relations, the payment of separation
pay has been considered an acceptable alternative to reinstatement when the
latter option is no longer desirable or viable.

In view of the illegal dismissal of petitioner, he is entitled to separation pay in


lieu of reinstatement for the reason above stated, computed from the date of
petitioner’s employment until finality of our decision; and backwages to be
computed from the date he was constructively dismissed, i.e., July 17, 2002, up to
the finality of this decision, less the amounts paid in accordance with his payroll
reinstatement. While the discretion to choose the mode of reinstatement lies with
the employer, the exercise thereof by respondents in the instant case was a
mockery of the true import of actual reinstatement, considering that petitioner
was reinstated as a Reserved Franchise Manager and was made to perform
demeaning jobs. Moreover, payroll reinstatement is proper in this case because
the physical presence of petitioner in the office might have worsened the already
strained relations between him and respondents, particularly, his immediate
superior respondent De Jesus, to whom he will directly report every day, as a
Manager Reserve.

Petition is partly granted.

MARILOU S. GENUINO
V.
NATIONAL LABOR RELATIONS COMMISSION, CITIBANK, N.A., WILLIAM FERGUSON,
and AZIZ RAJKOTWALA
G.R. Nos. 142732-33, December 4, 2007

TOPIC: On actual reinstatement vs. payroll reinstatement; effect where the original
decision finding for illegal termination was reversed on appeal.

CASE FACTS:

Genuino, the petitioner, was employed by Citibank as Treasury Sales Division


Head with the rank of Assistant Vice-President.
Citibank sent Genuino a letter charging her with "knowledge and/or
involvement" in transactions "which were irregular or even fraudulent” and was
under preventive suspension. She was given 3 days to explain herself.
Subsequently, her counsel replied through a letter demanding for a bill of
particulars regarding the charges against Genuino.
Genuino did not appear in the administrative investigation. It was found
that Genuino with Santos used "facilities of Genuino's family corporation, namely,
Global Pacific, personally and actively participated in the diversion of bank
clients' funds to products of other companies that yielded interests higher than
what Citibank products offered, and that Genuino and Santos realized substantial
financial gains, all in violation of existing company policy and the Corporation
Code, which for your information, carries a penal sanction.
Genuino's employment was terminated by Citibank on grounds of (1)
serious misconduct, (2) willful breach of the trust reposed upon her by the bank,
and (3) commission of a crime against the bank.
This prompted Genuino to file a complaint before the Labor Arbiter against
Citibank for illegal suspension and illegal dismissal with damages and prayer for
temporary restraining order and/or writ of preliminary injunction.
The LA found out that the dismissal to be without just cause and in violation
of her right to due process. The LA ordered to reinstate complainant immediately
to her former position as Treasury Sales Division Head or its equivalent without loss
of seniority rights and other benefits, with backwages from August 23, 1993 up to
April 30, 1994 in the amount of P493,800.00 (P60,000 x 8.23 mos.) subject to
adjustment until reinstated actually or in the payroll.
The NLRC by way of an appeal by CITIBANK set aside the decision of LA.
Subsequently, Genuino filed a petition for certioraribefore the CA which the CA
denied.And also, Citibank questioned before the CA the NLRC's order to pay
Genuino's salaries from the date of reinstatement until the date of the NLRC's
decision.

Hence it reached to the Supreme Court.

ISSUE:Whether or not Genuino, the petitioner whose dismissal was found out to be
with just cause but without the observance of due process, has the right to
payroll reinstatement.

RULING:
According to SC the dismissal was for just cause but lacked due
process.With respect to the issue mentioned above, the SCruled in
accordance toArt. 223, paragraph 3 of the Labor Code, which states:

In any event, the decision of the Labor Arbiter reinstating a dismissed


or separated employee, insofar as the reinstatement aspect is
concerned, shall immediately be executory, even pending appeal.
The employee shall either be admitted back to work under the same
terms and conditions prevailing prior to his dismissal or separation or,
at the option of the employer, merely reinstated in the payroll. The
posting of a bond by the employer shall not stay the execution for
reinstatement provided herein.

If the decision of the labor arbiter is later reversed on appeal upon the
finding that the ground for dismissal is valid, then the employer has the right to
require the dismissed employee on payroll reinstatement to refund the salaries
s/he received while the case was pending appeal, or it can be deducted from
the accrued benefits that the dismissed employee was entitled to receive from
his/her employer under existing laws, collective bargaining agreement provisions,
and company practices.

However, if the employee was reinstated to work during the pendency of


the appeal, then the employee is entitled to the compensation received for
actual services rendered without need of refund.

Considering that Genuino was not reinstated to work or placed on payroll


reinstatement, and her dismissal is based on a just cause, then she is not entitled to
be paid the salaries stated in the NLRC Decision.
JUANITO A. GARCIA and ALBERTO J. DUMAGO V. PHILIPPINE AIRLINES, INC

G.R. No. 164856, January 20, 2009

CASE FACTS:

Petitioners, Dumago and Garcia were employed as Aircraft Furnishers


Master "C" and Aircraft Inspector, respectively by respondent PAL.

A combined team of the PAL Security and National Bureau of


Investigation (NBI) Narcotics Operatives raided the Toolroom Section – Plant
Equipment Maintenance Division (PEMD) of the PAL Technical Center which
the petitioners were found with four others and found shabu paraphernalia
inside the company-issued locker of Ronaldo Broas who was also within the
vicinity. The six employees were later brought to the NBI for booking and
proper investigation.

Thereafter, they were allegedly "caught in the act of sniffing shabu


inside the Toolroom Section which the petitioners denied and sought for a
proof that were indeed caught on the act.

Dumago and Garcia were dismissed for violation of Chapter II,


Section 6, Article 46 (Violation of Law/Government Regulations) and
Chapter II, Section 6, Article 48 (Prohibited Drugs) of the PAL Code of
Discipline. Hence, they filed a a case for illegal dismissal and damages.

In the meantime, the Securities and Exchange Commission (SEC)


placed PAL under an Interim Rehabilitation Receiver due to severe financial
losses.

The LA rendered judgment in favor of Dumago and Garcia finding


them guilty of illegal suspension and illegal dismissal and ordering them to
reinstate complainants to their former position without loss of seniority rights
and other privileges.

However, since reinstatement is no longer feasible, PAL is hereby


ordered, in lieu, to pay unto the complainants their separation pay
computed at one month for every year of service.

On an appeal, the NLRC reversed the Labor Arbiter’s decision.


However, the LA still filed a Writ of Execution commanding the sheriff to execute its
decisions.

Thereafter, PAL appealed to the Court of Appeals. One of the issues


asserted by PAL is that there was no longer any legal or factual basis to reinstate
petitioners as a result of the reversal by the NLRC of the Labor Arbiter’s decision.
The petition was given due course by CA.
Hence, Dumago and Garcia reached to the Supreme Court.

ISSUE: Whether or not Dumago and Garciaentitled to execution of the Labor


Arbiter’s order of reinstatement and back wages even if PAL is under
receivership?

RULING:

No.

The SC emphasized that since petitioners’ claim against PAL is a


money claim for their wages during the pendency of PAL’s appeal to the
NLRC, the same should have been suspended pending the rehabilitation
proceedings. The Labor Arbiter, the NLRC, as well as the Court of Appeals
should have abstained from resolving petitioners’ case for illegal dismissal
and should instead have directed them to lodge their claim before PAL’s
receiver.

However, it does not bar Dumago and Garciato re-file their labor
claims against PAL on the premise that that their dismissal was eventually
held valid with only the matter of reinstatement pending appeal being the
issue.

MT. CARMEL COLLEGE VS. JOCELYN RESUENA, EDDIE VILLALON,SYLVIA


SEDAYON and ZONSAYDA EMNACE

DOCTRINES/PRINCIPLES:
1. Art. 223 of the Labor Code provides that reinstatement is immediately executory even
pending appeal only when the Labor Arbiter himself ordered the reinstatement.
2. Art. 224 of the Labor Code applies when the order of reinstatement was first decided upon
appeal to the NLRC. In other words, the Labor Arbiter himself did not order reinstatement.
3. Art. 279 of the Labor Code provides that backwages are to be computed from the time of
illegal dismissal until reinstatement or upon petitioner’s payment of separation pay to
respondents if reinstatement is not longer feasible.

FACTS:
PetitionerMt. Carmel College is a private educational institution and
respondents were its employees. Respondents were dismissed for joining the
protest action against the school administration. The Labor Arbiter (LA) found that
they were not illegally dismissed but ordered that they be awarded 13 th month
pay, separation pay and attorney’s fees. The NLRC reversed the findings of the LA
finding the termination of the respondents as illegal and ordering the payment of
backwages of respondents.It further directed the reinstatement of respondents by
way of payment of separation pay, with backwages. This was affirmed by the
Court of Appeals.
Petitioner is appealing not the judgment of the NLRC but the manner of
execution of the same. Petitioner argues that the CA erred in upholding the LA
and the NLRC that the award for backwages goes beyond the period May 15,
1998 to May 25, 1999 on the supposition that reinstatement is self-executory and
does not need a writ of execution for its enforcement.Petitioner avers that the LA
went beyond the terms of the NLRC Decision, as affirmed by the CA, and
erroneously used as bases inapplicable law and jurisprudence in the execution of
the same.Petitioner contends that the award of backwages subject to execution
is limited to the period prior to the appeal and does not include the period during
the pendency of the appeal, on the contention that reinstatement during appeal
is warranted only when the Labor Arbiter rules that the dismissed employee should
be reinstated.

ISSUES:
1.Whetherreinstatement in the case is self-executory and does not need a
writ of execution for its enforcement.
2. Whether the continuing award of backwages is proper.

RULING:
1.No(though the court sees no cogent reason as to the relevance of a
discussion of this issue only that petitioner raised it as an issue).The court states that
the above findings will not affect the award of backwages for the period beyond
May 25, 1999.
Article 224 applies in the given case since the order of reinstatement was
first decided upon appeal to the NLRC and affirmed with finality by the CA.
2. Yes. The court found out that there is a conflict between the dispositive
portion of the falloand the body of the decision. The fallo stated that respondents
were illegally dismissed and must therefore be ordered reinstated with payment of
backwages from the time were illegally dismissed up to the time of their actual
reinstatement. In view thereof, the court declared that the fallocontrols.
Applying Article 279 of the Labor Code, the court emphasized that
backwages are to be computed from the time of illegal dismissal until
reinstatement or upon petitioner’s payment of separation pay to respondents if
reinstatement is not longer feasible.

WENPHIL v. ABING

IN a complaint for illegal dismissal filed by respondents Almer R. Abing and


Anabelle M. Tuazon against petitioner Wenphil Corp., the former were awarded
backwages. But the period for the computation of the backwages set by the
Labor Arbiter (LA) was inconsistent with that of the Court of Appeals (CA).
According to the LA, whose ruling the National Labor Relations Commission
(NLRC) affirmed, the period for computation should be from Feb. 15, 2002, the day
when petitioner last paid respondents’ backwages, until Nov. 8, 2002 when the
NLRC’s decision became final and executory.

On the other hand, the CA, in setting aside the NLRC’s rulings, relied on the case
of Pfizer v. Velasco (G.R. No. 177467, March 9, 2011, 645 SCRA 135) where the
Supreme Court ruled that the backwages of the dismissed employee should be
granted during the period of appeal until reversal by a higher court. Since the first
CA decision that found the respondents had not been illegally dismissed was
promulgated on Aug. 27, 2003, then the reversal by the higher court was
effectively made on Aug. 27, 2003. Which computation is correct?

Ruling: That of CA.

Among these views, the commanding one is the rule in Pfizer, which merely echoes the
rulings the Supreme Court (SC) made in the cases of Roquero v. Philippine Airlines (G.R.
No. 152329, 449 Phil. 437 (2003)) and Garcia v. Philippine Airlines (G.R. No. 164856,
January 20, 2009, 576 SCRA 479) that the period for computing the backwages due to
the respondents during the period of appeal should end on the date that a higher court
reversed the labor arbitration ruling of illegal dismissal. In this case, the higher court that
first reversed the NLRC’s ruling was not the SC but rather the CA. In this light, the CA was
correct when it found that that the period of computation should end on Aug. 27, 2003.
The date when the SC’s decision became final and executory need not matter as the rule
in Roquero, Garcia and Pfizer merely referred to the date of reversal, not the date of the
ultimate finality of such reversal.

As a last minor detail, we do not agree with the CA that the date of computation should
start on Feb. 15, 2002. Rather, it should be on Feb. 16, 2002. The respondents
themselves admitted in their motion for computation and issuance of writ of execution that
the last date when they were paid their backwages was on Feb. 15, 2002. To start the
computation on the same date would result to a duplication of wages for this day; thus,
computation should start on the following date – Feb. 16, 2002. (Brion, J., SC Second
Division; Wenphil Corporation vs. Almer R. Abing and Anabelle M. Tuazon, G.R. No.
207983, April 7, 2014).

BERGONIO v. SOUTH EAST ASIAN AIRLINES, G.R. No. 195227, April 21, 2014

Labor law; Payment of accrued wages despite reversal of decision. An employer, who, despite the
Labor Arbiter’s order of reinstatement, did not reinstate the employee during the pendency of the
appeal up to the reversal by a higher tribunal may still be held liable for the accrued wages of the
employee, i.e., the unpaid salary accruing up to the time the higher tribunal reverses the
decision. The rule, therefore, is that an employee may still recover the accrued wages up to and
despite the reversal by the higher tribunal. This entitlement of the employee to the accrued wages
proceeds from the immediate and self-executory nature of the reinstatement aspect of the LA’s
decision.

Exception. By way of exception to the above rule, an employee may be barred from collecting the
accrued wages if shown that the delay in enforcing the reinstatement pending appeal was without
fault on the part of the employer. To determine whether an employee is thus barred, two tests
must be satisfied: (1) actual delay or the fact that the order of reinstatement pending appeal was
not executed prior to its reversal; and (2) the delay must not be due to the employer’s unjustified
act or omission. Note that under the second test, the delay must be without the employer’s fault. If
the delay is due to the employer’s unjustified refusal, the employer may still be required to pay the
salaries notwithstanding the reversal of the LA’s decision.
FACTS:

IN A decision dated May 31, 2005, the Labor Arbiter (LA) found the petitioners Froilan M.
Bergonio Jr. and nine others illegally dismissed and ordered respondents South East
Asian Airlines and Irene Dornier, among others, to immediately reinstate the petitioners
with full back wages. For failure of respondents to reinstate petitioners despite
respondents’ manifestation to reinstate them in the payroll, the petitioners filed before the
LA a manifestation for their immediate reinstatement. On Oct. 3, 2005, respondents filed
an opposition to the motion. On Jan. 31, 2008, the petitioners filed with the LA an Urgent
Ex-Parte Motion for the Immediate Release of the Garnished Amount, which was granted.
In its July 16, 2008 resolution, the National Labor Relations Commission (NLRC) affirmed
in toto the LA’s order. It denied the respondents’ motion for reconsideration for lack of
merit. The Court of Appeals (CA) reversed and set aside the decision and resolution of the
NLRC. It ruled further that the computation of petitioners’ accrued wages stopped when
they failed to report for work on Feb. 24, 2006 per respondents’ memorandum of Feb. 21,
2006. Did the CA err?

Ruling: Yes.

Our careful consideration of the facts and the circumstances that surrounded the case
convinced us that the delay in the reinstatement pending appeal was due to the
respondents’ fault. For one, the respondents filed several pleadings to suspend the
execution of the LA’s

reinstatement order, i.e., the opposition to the petitioners’ motion for execution filed on
Oct. 3, 2005; the motion to quash the Oct. 7, 2005 writ of execution with prayer to hold in
abeyance the implementation of the reinstatement order; and the motion to suspend the
order for the petitioners’ reinstatement filed on Feb. 28, 2006 after the LA issued the Feb.
16, 2006 alias writ of execution. These pleadings, to our mind, show a determined effort
on the respondents’ part to prevent or suspend the execution of the reinstatement pending
appeal.

All told, the delay was due to the acts of the respondents that we find were unjustified. We
reiterate and emphasize, Article 223, paragraph 3, of the Labor Code mandates the
employer to immediately reinstate the dismissed employee, either by actually reinstating
him/her under the conditions prevailing prior to the dismissal or, at the option of the
employer, in the payroll. The respondents’ failure to exercise either option rendered them
liable for the petitioners’ accrued salary until the LA decision was reversed by the CA on
Dec. 17, 2008. We find that the NLRC, in affirming the release of the garnished amount,
merely implemented the mandate of Article 223; it recognized as immediate and self-
executory the reinstatement aspect of the LA’s decision. Accordingly, we reverse for legal
errors the CA decision. We find no grave abuse of discretion attended the NLRC’s July
16, 2008 resolution that affirmed the March 13, 2008 decision of the LA granting the
release of the garnished amount. (Brion J., SC Second Division; Froilan M. Bergonio, Jr.,
et. al. vs. South East Asian Airlines and Irene Dornier, G.R. No. 195227, April 21, 2014).
Alcantara& Sons v. Court of Appeals, G.R. No. 155109, September 29, 2010

FACTS:

This case is about a) the consequences of an illegally staged strike upon the
employment status of the union officers and its ordinary members and b) the right
of reinstated union members to go back to work pending the company’s appeal
from the order reinstating them.

The Company and the Union entered into a Collective Bargaining Agreement
(CBA) that bound them to hold no strike and no lockout in the course of its life. At
some point the parties began negotiating the economic provisions of their CBA
but this ended in a deadlock, prompting the Union to file a notice of strike. After
efforts at conciliation by the Department of Labor and Employment (DOLE) failed,
the Union conducted a strike vote that resulted in an overwhelming majority of its
members favoring it. The Union reported the strike vote to the DOLE and, after the
observance of the mandatory cooling-off period, went on strike.

During the strike, the Company filed a petition for the issuance of a writ of
preliminary injunction with prayer for the issuance of a temporary restraining order
(TRO) Ex Parte3 with the National Labor Relations Commission (NLRC) to enjoin the
strikers from intimidating, threatening, molesting, and impeding by barricade the
entry of non-striking employees at the Company’s premises.

The NLRC first issued a 20-day TRO and, after hearing, a writ of preliminary
injunction, enjoining the Union and its officers and members from performing the
acts complained of. But several attempts to implement the writ failed. Only the
intervention of law enforcement units made such implementation possible.
Meantime, the Union filed a petition 4 with the Court of Appeals (CA), questioning
the preliminary injunction order. On February 8, 1999 the latter court dismissed the
petition. The Union did not appeal from such dismissal.

The Company, on the other hand, filed a petition with the Regional Arbitration
Board to declare the Union’s strike illegal,5 citing its violation of the no strike, no
lockout, provision of their CBA.

During the pendency of these cases, the affected Union members filed with the
Labor Arbiter a motion for reinstatement pending appeal by the parties and the
computation of their backwages based on the CA decision. After hearing, the
Labor Arbiter issued a resolution dated November 21, 2002,11 holding that due to
the delay in the resolution of the dispute and the impracticability of reinstatement
owing to the fact that the relations between the terminated Union members and
the Company had been severely strained by the prolonged litigation, payment of
separation pay to such Union members was in order. The Labor Arbiter thus
approved the computation and payment of their separation pay and denied all
their other claims.
ISSUE:
Whether or not the terminated Union members are entitled to the payment of
backwages on account of the Company’s refusal to reinstate them, pending
appeal by the parties, from the Labor Arbiter’s decision?

RULING:
The terminated Union members contend that, since the Company refused to
reinstate them after the Labor Arbiter rendered a decision in their favor, the
Company should be ordered to pay them their wages during the pendency of
the appeals from the Labor Arbiter’s decision.

The CA denied reinstatement for the reason that the reinstatement pending
appeal provided under Article 223 of the Labor Code contemplated illegal
dismissal or termination cases and not cases under Article 264. But this perceived
distinction does not find support in the provisions of the Labor Code.

The grounds for termination under Article 264 are based on prohibited acts that
employees could commit during a strike. On the other hand, the grounds for
termination under Articles 282 to 284 are based on the employee’s conduct in
connection with his assigned work. Still, Article 217, which defines the powers of
Labor Arbiters, vests in the latter jurisdiction over all termination cases, whatever
be the grounds given for the termination of employment. Consequently, Article
223, which provides that the decision of the Labor Arbiter reinstating a dismissed
employee shall immediately be executory pending appeal, cannot but apply to
all terminations irrespective of the grounds on which they are based.

Here, although the Labor Arbiter failed to act on the terminated Union
members’ motion for reinstatement pending appeal, the Company had the duty
under Article 223 to immediately reinstate the affected employees even if it
intended to appeal from the decision ordaining such reinstatement. The
Company’s failure to do so makes it liable for accrued backwages until the
eventual reversal of the order of reinstatement by the NLRC on November 8,
1999,24 a period of four months and nine days.

Lansangan v. Amkor Technology, G.R. No. 177026, January 30, 2009

FACTS:

An anonymous e-mail was sent to the General Manager of Amkor Technology


Philippines (respondent) detailing allegations of malfeasance on the part of its
supervisory employees LunesaLansangan and Rosita Cendaña (petitioners) for
"stealing company time."1 Respondent thus investigated the matter, requiring
petitioners to submit their written explanation. In handwritten letters,
petitioners admitted their wrongdoing.2 Respondent thereupon terminated
petitioners for "extremely serious offenses" as defined in its Code of
Discipline,3 prompting petitioners to file a complaint for illegal dismissal against it.4
The Arbiter, however, ordered the reinstatement of petitioners to their former
positions without backwages "as a measure of equitable and compassionate
relief" owing mainly to petitioners’ prior unblemished employment records, show of
remorse, harshness of the penalty and defective attendance monitoring system of
respondent.

Respondent assailed the reinstatement aspect of the Arbiter’s order before the
National Labor Relations Commission (NLRC).

In the meantime, petitioners, without appealing the Arbiter’s finding them guilty
of "dishonesty as a form of serious misconduct and fraud or breach of
trust," moved for the issuance of a "writ of reinstatement."

ISSUE:
Whether or not the petitioners are entitled to back wages?

RULING:
Roquero, as well as Article 22318 of the Labor Code on which the appellate
court also relied, finds no application in the present case. Article 223 concerns
itself with an interim relief, granted to a dismissed or separated employee while
the case for illegal dismissal is pending appeal, as what happened in Roquero. It
does not apply where there is no finding of illegal dismissal, as in the present case.
The Arbiter found petitioners’ dismissal to be valid. Such finding had, as stated
earlier, become final, petitioners not having appealed it. Following Article 279
which provides:
x xxx
In cases of regular employment, the employer shall not terminate the services
of an employee except for a just cause or when authorized by this Title. An
employee who is unjustly dismissed from work shall be entitled to reinstatement
without loss of seniority rights and other privileges and to his full backwages,
inclusive of allowances, and to his other benefits or their monetary equivalent
computed from the time his compensation was withheld from him up to the time
of his actual reinstatement (Emphasis, underscoring and italics supplied),
Petitioners are not entitled to full backwages as their dismissal was not found to
be illegal. Agabon v. NLRC19 so states –– payment of backwages and other
benefits is justified only if the employee was unjustly dismissed.

Pfizer v. Velasco, G.R. No. 177467, March 9, 2011

FACTS:

Private respondent Geraldine L. Velasco was employed with petitioner PFIZER,


INC. as Professional Health Care Representative since 1 August 1992. Sometime in
April 2003, Velasco had a medical work up for her high-risk pregnancy and was
subsequently advised bed rest which resulted in her extending her leave of
absence. Velasco filed her sick leave for the period from 26 March to 18 June
2003, her vacation leave from 19 June to 20 June 2003, and leave without pay
from 23 June to 14 July 2003.

On 26 June 2003, while Velasco was still on leave, PFIZER through its Area Sales
Manager, herein petitioner Ferdinand Cortez, personally served Velasco a "Show-
cause Notice" dated 25 June 2003. Aside from mentioning about an investigation
on her possible violations of company work rules regarding "unauthorized deals
and/or discounts in money or samples and unauthorized withdrawal and/or pull-
out of stocks".

That same day, Velasco filed a complaint for illegal suspension with money
claims before the Regional Arbitration Branch. Finally, on 29 July 2003, PFIZER
informed Velasco of its "Management Decision" terminating her employment.

On 5 December 2003, the Labor Arbiter rendered its decision declaring the
dismissal of Velasco illegal, ordering her reinstatement with backwages and
further awarding moral and exemplary damages with attorney’s fees. On appeal,
the NLRC affirmed the same but deleted the award of moral and exemplary
damages.

PFIZER filed with the Court of Appeals a special civil action for the issuance of a
writ of certiorari under Rule 65 of the Rules of Court to annul and set aside the
aforementioned NLRC issuances. In a Decision dated November 23, 2005, the
Court of Appeals upheld the validity of respondent’s dismissal from employment.

ISSUE:
Whether or not the Court of Appeals committed a serious but reversible
error when it ordered Pfizer to pay Velasco wages from the date of the Labor
Arbiter’s decision ordering her reinstatement until November 23, 2005, when the
Court of Appeals rendered its decision declaring Velasco’s dismissal valid.

RULING:
PFIZER argues that, contrary to the Court of Appeals’ pronouncement in its
assailed Decision dated November 23, 2005, the ruling in Roquero v. Philippine
Airlines, Inc.14 is not applicable in the case at bar, particularly with regard to the
nature and consequences of an order of reinstatement.
As far back as 1997 in the seminal case of Pioneer Texturizing Corporation v.
National Labor Relations Commission,21 the Court held that an award or order
of reinstatement is immediately self-executory without the need for the issuance
of a writ of execution in accordance with the third paragraph of Article 223 22 of
the Labor Code.
In the case at bar, PFIZER did not immediately admit respondent back to
work which, according to the law, should have been done as soon as an order
or award of reinstatement is handed down by the Labor Arbiter without need
for the issuance of a writ of execution. Thus, respondent was entitled to the
wages paid to her under the aforementioned writ of execution.

Emeritus Security v. Dailig, G.R. No. 204761, April 2, 2014


FACTS:
In August 2000, petitioner hired respondent as one of its security guards.
During his employment, respondent was assigned to petitioner's various clients, the
last of which was Panasonic in Calamba, Laguna starting 16 December 2004.
On 10 December 2005, respondent was relieved from his post.
On 27 January 2006, respondent filed a complaint for underpayment of wages,
non-payment of legal and special holiday pay, premium pay for rest day and
underpayment of ECOLA before the Department of Labor and Employment,
National Capital Region. The hearing officer recommended the dismissal of the
complaint since the claims were already paid.
On 16 June 2006, respondent filed a complaint for illegal dismissal and payment
of separation pay against petitioner before the Conciliation and Mediation
Center of the NLRC. On 14 July 2006, respondent filed another complaint for illegal
dismissal, underpayment of salaries and non-payment of full backwages before
the NLRC.
Respondent claimed that on various dates in December 2005 and from
January to May 2006,4 he went to petitioner’s office to follow-up his next
assignment. After more than six months since his last assignment, still respondent
was not given a new assignment. Respondent argued that if an employee is on
floating status for more than six months, such employee is deemed illegally
dismissed.
Petitioner denied dismissing respondent. Petitioner admitted that it relieved
respondent from his last assignment on 10 December 2005; however, petitioner
required respondent to report to the head office within 48 hours from receipt of
the order of relief.

ISSUE:
Whether respondent is entitled to separation pay, instead of reinstatement?
RULING:
Article 279 of the Labor Code of the Philippines mandates the reinstatement
of an illegally dismissed employee, to wit:
Security of Tenure. - x xx An employee who is unjustly dismissed from work shall be
entitled to reinstatement without loss of seniority rights and other privileges and to
his full back wages, inclusive of allowances, and to his other benefits or their
monetary equivalent computed from the time his compensation was withheld
from him up to the time of his actual reinstatement.
Thus, reinstatement is the general rule, while the award of separation pay is
the exception. The circumstances warranting the grant of separation pay, in lieu
of reinstatement.
Contrary to the Court of Appeals' ruling, there is nothing in the records
showing any strained relations between the parties to warrant the award of
separation pay. There is neither allegation nor proof that such animosity existed
between petitioner and respondent. In fact, petitioner complied with the Labor
Arbiter's reinstatement order.
Considering that (1) petitioner reinstated respondent in compliance with the
Labor Arbiter's decision, and (2) there is no ground, particularly strained relations
between the parties, to justify the grant of separation pay, the Court of Appeals
erred in ordering the payment thereof, in lieu of reinstatement.
TENAZAS V. VILLEGAS TAXI TRANSPORT

Facts:

Tenazas, Francisco, and Endraca filed a complaint for illegal dismissal against R.
Villegas Taxi Transport and/or Romualdo and Andy, both Villegas.

Tenazas alleged that on 2007, the taxi unit assigned to him was sideswiped by
another vehicle, causing a dent on the left fender near the driver seat.

Upon reporting the incident to the company, he was scolded by respondents...


and was told to leave the garage for he is already fired. He was even threatened
with physical harm should he ever be seen in the company's premises again.
Despite the... warning, Tenazas reported for work on the following day but was
told that he can no longer drive any of the company's units as he is already fired.

Francisco, on the other hand, averred that his dismissal was brought about by the
company's unfounded suspicion that he was organizing a labor union. He was
instantaneously terminated, without the benefit of procedural due process.

Endraca, for his part, alleged that his dismissal was instigated by an occasion
when he fell short of the required boundary for his taxi unit.

He related that before he was dismissed, he brought his taxi unit to an auto shop
for an urgent repair. He was charged the amount of Php700.00 for the repair
services and the replacement parts. As a result, he was not able to meet his
boundary for the day.

Respondents admitted that Tenazas and Endraca were employees of the


company, the former being a regular driver and the latter a spare driver. The
respondents, however, denied that Francisco was an employee of the company
or that he was able to drive one of the company's units at any point in time.
The Labor Arbiter (LA) dismissed for lack of merit.

Petitioners appealed the decision of the LA to the NLRC. The NLRC rendered a
Decision reversing the appealed decision of the LA. Respondents filed a motion
for reconsideration but the NLRC denied the same respondents filed a petition for
certiorari with the CA.

The award of Jaime Francisco's claims is DELETED. The separation pay granted in
favor of Bernard Tenazas and Isidro Endraca is, likewise, DELETED and their
reinstatement is ordered instead. Petitioners filed a motion for reconsideration but
the same was denied by the CA. Undeterred, the petitioners filed the instant
petition for review on certiorari.
Issues:Whether or not there was an illegal dismissal

Ruling:

The petition lacks merit.


No substantial evidence was presented to support the conclusion that Francisco
was an employee of the respondents and accordingly modified the NLRC
decision.

With the respondents' denial of employer-employee relationship, it behooved


Francisco to present substantial evidence to prove that he is an employee before
any question on the legality of his supposed dismissal becomes appropriate for
discussion. Francisco, however, did not offer evidence to substantiate his claim of
employment with the respondents. Short of the required quantum of proof, the
CA correctly ruled that the NLRC's finding of illegal dismissal and the monetary
awards which necessarily follow such ruling lacked factual and legal basis and
must therefore be deleted.

It is an oft-repeated rule that in labor cases, as in other administrative and quasi-


judicial proceedings, "the quantum of proof necessary is substantial evidence, or
such amount of relevant evidence which a reasonable mind might accept as
adequate to justify a conclusion."

"[T]he burden of proof rests upon the party who asserts the affirmative of an issue."

Corollary, as Francisco was claiming to be an employee of the respondents, it is


incumbent upon him to proffer evidence to prove the existence of said
relationship.

Francisco failed to present any proof substantial enough to establish his


relationship with the respondents. He failed to present documentary evidence like
attendance logbook, payroll, SSS record or any personnel file that could somehow
depict his status as an employee. Anent his claim that he was not issued with
employment records, he could have, at least, produced his social security records
which state his contributions, name and address of his employer, as his co-
petitioner Tenazas did. He could have also presented testimonial evidence
showing the respondents' exercise of control over the means and methods by
which he undertakes his work. This is imperative in light of the respondents' denial
of his employment and the claim of another taxi operator, Emmanuel Villegas,
that he was his employer.

The utter lack of evidence is fatal to Francisco's case especially in cases like his
present predicament when the law has been very lenient in not requiring any
particular form of evidence or manner of proving the presence of employer-
employee relationship.
Here, Francisco simply relied on his allegation that he was an employee of the
company without any other evidence supporting his claim. Unfortunately for him,
a mere allegation in the position paper is not tantamount to evidence.

Bereft of any evidence, the CA correctly ruled that Francisco could not be
considered an employee of the respondents.

A bare claim of strained relations by reason of termination is insufficient to warrant


the granting of separation pay. Likewise, the filing of the complaint by the
petitioners does not necessarily translate to strained relations between the parties.

BANI RURAL BANK V. GUZMAN


Facts:

The respondents were employees of Bani Rural Bank, Inc. and ENOC Theatre I and
II who filed a complaint for illegal dismissal against the petitioners. The complaint
was initially dismissed by the LA but the NLRC reversed LAs decision. The NLRC, in
its resolution dated March 17, 1995, ordered that respondents be reinstated with
payment of backwages from the time of their dismissal until their actual
reinstatement. Such decision has become final and executory. Computation of
backwages was referred to Labor Arbiter Gambito.

Petitioners appealed the computation of the backwages with the NLRC. In a


decision dated July 31, 1998, the NLRC modified the terms of the March 17, 1995
resolution insofar as it clarified the phrase less earnings elsewhere. The NLRC
additionally awarded the payment of separation pay, in lieu of reinstatement on
account of the strained relations between the parties.

As explained in the assailed Decision, what is controlling for purposes of the


backwages is the NLRC’s Resolution dated 17 March 1995 which decreed that
private respondents are entitled to backwages from the time of their dismissal
(constructive) until their actual reinstatement; and considering that the award of
reinstatement was set aside by the NLRC in its final and executory Decision dated
3 July 1998 which ordered the payment of separation pay in lieu of reinstatement
to be computed up to the finality on 29 January 1999 of said Decision dated 3 July
1998, then the computation of the backwages should also end on said date,
which is 29 January 1999.

Issue: Whether or not NLRC erred in ruling how the backwages are to be
computed

Ruling:
No. CA decision affirming NLRC ruling sustained.
Labor Law - The computation of backwages depends on the final awards
adjudged as a consequence of illegal dismissal.

First, when reinstatement is ordered, the general concept under Article 279 of the
Labor Code, as amended, computes the backwages from the time of dismissal
until the employees’ reinstatement. The computation of backwages (and similar
benefits considered part of the backwages) can even continue beyond the
decision of the labor arbiter or NLRC and ends only when the employee is actually
reinstated.

Second, when separation pay is ordered in lieu of reinstatement (in the event that
this aspect of the case is disputed) or reinstatement is waived by the employee (in
the event that the payment of separation pay, in lieu, is not disputed), backwages
is computed from the time of dismissal until the finality of the decision ordering
separation pay.

Third, when separation pay is ordered after the finality of the decision ordering the
reinstatement by reason of a supervening event that makes the award of
reinstatement no longer possible (as in the case), backwages is computed from
the time of dismissal until the finality of the decision ordering separation pay.

As the records show, the contending parties did not dispute the NLRC s order of
separation pay that replaced the award of reinstatement on the ground of the
supervening event arising from the newly-discovered strained relations between
the parties. The parties allowed the NLRC s July 31, 1998 decision to lapse into
finality and recognized, by their active participation in the second computation of
the awards, the validity and binding effect on them of the terms of the July 31,
1998 decision.

Under these circumstances, while there was no express modification on the period
for computing backwages stated in the dispositive portion of the July 31, 1998
decision of the NLRC, it is nevertheless clear that the award of reinstatement
under the March 17, 1995 resolution (to which the respondents backwages was
initially supposed to have been computed) was substituted by an award of
separation pay. As earlier stated, the awards of reinstatement and separation pay
are exclusive remedies; the change of awards (from reinstatement to separation
pay) under the NLRC s July 31, 1998 not only modified the awards granted, but
also changed the manner the respondent’s backwages is to be computed. The
respondent’s backwages can no longer be computed up to the point of
reinstatement as there is no longer any award of reinstatement to speak of.

Thus, the computation of the respondents' backwages must be from the time of
the illegal dismissal from employment until the finality of the decision ordering the
payment of separation pay. It is only when the NLRC rendered its July 31, 1998
decision ordering the payment of separation pay (which both parties no longer
questioned and which thereafter became final) that the issue of the respondents'
employment with the petitioners was decided with finality, effectively terminating
it. The respondents' backwages, therefore, must be computed from the time of
their illegal dismissal until January 29, 1999, the date of finality of the NLRC's July 31,
1998 Decision.

ESCARIO V. NLRC
Facts:

The petitioners were among the regular employees of respondent Pinakamasarap


Corporation (PINA), a corporation engaged in manufacturing and selling food
seasoning. They were members of petitioner MalayangSamahan ng
mgaManggagawasa Balanced Foods (Union).
At 8:30 in the morning of March 13, 1993, all the officers and some 200 members of
the Union walked out of PINA’s premises and proceeded to the barangay office
to show support for an employee and officer of the union who was charged with
oral defamation by a manager of the company. All officers and members of the
union went back to work afterwards.

As a result of the walkout, PINA preventively suspended all officers of the Union
because of the March 13, 1993 incident. PINA terminated the officers of the Union
after a month.

On April 14, 1993, PINA filed a complaint for unfair labor practice (ULP) and
damages. LA ruled that the incident was an illegal walkout constituting ULP; and
that all the Union’s officers, except Cañete, had thereby lost their employment.

Union filed a notice of strike, claiming that PINA was guilty of union busting through
the constructive dismissal of its officers. Union held a strike vote, at which a
majority of 190 members of the Union voted to strike.
PINA retaliated by charging the petitioners with ULP and abandonment of work,
stating that they had violated provisions on strike of the collective bargaining
agreement (CBA).

On September 30, 1994, the Third Division of the National Labor Relations
Commission (NLRC) issued a temporary restraining order (TRO). On November 29,
1994, the NLRC granted the writ of preliminary injunction.

The LA rendered decision declaring the strike as illegal.

NLRC sustained, but held that there was no abandonment on the part of the
employees.

CA sustained the NLRC and explained that they were not entitled to full back
wages as only instance under Article 264 when a dismissed employee would be
reinstated with full backwages was when he was dismissed by reason of an illegal
lockout; that Article 264 was silent on the award of backwages to employees
participating in a lawful strike; and that a reinstatement with full backwages would
be granted only when the dismissal of the petitioners was not done in
accordance with Article 282 (dismissals with just causes) and Article 283 (dismissals
with authorized causes) of the Labor Code.

Issue: Whether or not they are entitled to backwages during the illegal strike

Ruling:

Petitioners not entitled to backwages despite their reinstatement.

A fair day’s wage for a fair day’s labor.

Back-wages are not granted to employees participating in an illegal strike simply


accords with the reality that they do not render work for the employer during the
period of the illegal strike.

With respect to backwages, the principle of “fair day’s wage for a fair day’s
labor” remains as the basic factor in determining the award thereof. If there is no
work performed by the employee there can be no wage or pay unless, of course,
the laborer was able, willing and ready to work but was illegally locked out,
suspended or dismissed or otherwise illegally prevented from working.

Under the principle of a fair day’s wage for a fair day’s labor, the petitioners were
not entitled to the wages during the period of the strike (even if the strike might be
legal), because they performed no work during the strike. Verily, it was neither fair
nor just that the dismissed employees should litigate against their employer on the
latter’s time.

SARONA V. NLRC
Facts:
Petitioner, a security guard in Sceptre since April 1976, was asked
bySceptre’soperations manager on June 2003, tosubmit a resignation letter as a
requirement for an application in Royale and to fill up an employment
applicationform for the said company. He was then assigned at Highlight Metal
Craft Inc. from July 29 to August 8, 2003 and waslater transferred to Wide Wide
World Express Inc. On September 2003, he was informed that his assignment
atWWWE Inc. was withdrawn because Royale has been allegedly replaced by
another security agency which he laterdiscovered to be untrue. Nevertheless, he
was once again assigned at Highlight Metal sometime in September 2003and
when he reported at
Royale’soffice on October 1, 2003, he was informed that he would no longer be
given an assignment as instructed bySceptre’sgeneral manager.

He thus filed acomplaint for illegal dismissal. The LA ruled inpetitioner’s favor as he
found him illegally dismissed andwas not convinced by therespondent’sclaim
onpetitioner’sabandonment.
Respondents were ordered to pay back wages computed from the day he was
dismissed up to the promulgation of hisdecision on May 11, 2005.The LA also
ordered for the payment of separation pay but refused to
pierceRoyale’scorporate veil.

Respondents appealed to the NLRC claiming that the LA acted with grave abuse
of discretion upon ruling on theillegal dismissal of petitioner. NLRC partially
affirmed theLA’sdecision with regard topetitioner’sillegal dismissal andseparation
pay but modified the amount of backwages and limited it to only 3 months of his
last month salaryreducing P95, 600 to P15, 600 since he worked for Royale for only
1 month and 3 days.

Petitioner did not appeal to LA but raised the validity of LA’sfindings on


piercingRoyale’scorporate personality andcomputation of his separation pay and
such petition was dismissed by the NLRC. Petitioner elevatedNLRC’s decisionto the
CA on a petition for certiorari, and the CA disagreed with theNLRC’sdecision of
not proceeding to review theevidence for determining if Royale isSceptre’salter
ego that would warrant the piercing of its corporate veil.

Issue: Whether or not the petitioner’s back wages should be limited to his salary for
3 months

Ruling:

The Court rules in the negative.

With Aida’s control over Sceptre’s and Royale’s business affairs, it is patent that
Royale was a mere subterfuge for Aida. Since a sole proprietorship does not have
a separate and distinct personality from that of the owner of the enterprise, the
latter is personally liable.

The petitioner cannot be deemed to have changed employers as Royale and


Sceptre are one and the same. His separation pay should, thus, be computed
from the date he was hired by Sceptre in April 1976 until the finality of this decision.

The intervening period between the day an employee was illegally dismissed and
the day the decision finding him illegally dismissed becomes final and executory
shall be considered in the computation of his separation pay as a period of
"imputed" or "putative" service:

Separation pay, equivalent to one month's salary for every year of service, is
awarded as an alternative to reinstatement when the latter is no longer an option.
Separation pay is computed from the commencement of employment up to the
time of termination, including the imputed service for which the employee is
entitled to backwages, with the salary rate prevailing at the end of the period of
putative service being the basis for computation.
This Court holds Royale liable to pay the petitioner backwages to be computed
from his dismissal on October 1, 2003 until the finality of this decision. Nonetheless,
the amount received by the petitioner from the respondents in satisfaction of the
November 30, 2005 Decision shall be deducted accordingly.

Wenphil Corporation vs. Almer R. Abing and AnabelleTuazon


GR No. 207983 April 7, 2014

Facts: Respondents filed with the NLRC a case for illegal dismissal against
petitioner corporation. Labor Arbiter Bartolabac ruled in favor of respondents and
ordered petitioner to pay respondents their backwages for the period from
February 15, 2002 to November 8, 2002, pursuant to the rule that an order of
reinstatement is immediately executory even pending appeal. Petitioner
appealed to the NLRC on April 16, 2001. In the meantime, the respondents moved
for the immediate execution of the LA’s December 8, 2000 decision.On January
30, 2002, the NLRC issued a resolution affirming LA Bartolabac’s decision with
modifications. Instead of ordering the respondents’ reinstatement, the NLRC
directed Wenphil to pay the respondents their respective separation pay at the
rate of one (1) month salary for every year of service. Also, the NLRC found that
while the respondents had been illegally dismissed, they had not been illegally
suspended. Thus, the period from February 3 to February 28, 2000 during which the
respondents were on preventive suspension – was excluded by the NLRC in the
computation of the respondents’ backwages.
Petitioner filed a motion for reconsideration but was denied. On appeal to the CA,
the NLRC decision was reversed; there being said to be enough evidence to show
that the respondents had been guilty of serious misconduct; thus, their dismissal
was for a valid cause. The respondents moved for the reconsideration of the CA’s
decision. In a resolution dated February 23, 2004, the CA denied the respondents’
motion, and when brought to the Supreme Court, docketed as GR No. 162447,
the Court denied the respondents’ motion.
When the SC decision became final and executory, the respondents filed with LA
Bartolabac a motion for computation and issuance of writ of execution, asserting
that although the CA’s ruling on the absence of illegal dismissal (as affirmed by
the SC) was adverse to them, under the law and settled jurisprudence, they were
still entitled to backwages from the time of their dismissal until the NLRC’s decision
finding them to be illegally dismissed was reversed with finality. The LA agreed with
them, directing Wenphil to to pay each complainant their salaries on
reinstatement covering the period from February 15, 2002 (the date Wenphil last
paid the respondents’ respective salaries) to November 8, 2002 (since the NLRC’s
decision finding the respondents illegally dismissed became final and executory
on February 28, 2002).

Both parties appealed this decision to NLRC: Wenphil argued that the respondents
were no longer entitled to payment of backwages in view of the compromise
agreement they executed on October 29, 2001. According to Wenphil, the
compromise agreement provided that Wenphil’s obligation to pay the
respondents’ backwages should cease as soon as LA Bartolabac’s decision was
"modified, amended or reversed" by the NLRC, and respondents questioned in
their appeal the determined period for the computation of their backwages; they
posited that the period for payment should end, not on November 8, 2002, but on
February 14, 2007, since the SC’s decision which upheld the CA’s ruling became
final and executory on February 15, 2007. Both appeals were denied. In a 2012
ruling by the CA on appeal, it prescribed a different computation period.

The CA ruled that the NLRC committed grave abuse of discretion when it affirmed
the LA’s computed period which was from February 15, 2002 to November 8, 2002,
reasoning that it was a "higher court" than the NLRC when it reversed the NLRC’s
rulings; thus, the period for computation should end when it promulgated its
decision reversing that of the NLRC, and not on the date when the SC affirmed its
decision.

The CA also held that the compromise agreement did not contain any waiver of
rights for any award the respondents might have received when the NLRC
changed or modified the LA’s award.

Issue: Whether or not the respondents are no longer entitled to payment of


backwages in view of the modification of the LA’s ruling

Held: No.Under Article 223 of the Labor Code, "the decision of the Labor Arbiter
reinstating a dismissed or separated employee, insofar as the reinstatement
aspect is concerned, shall immediately be executory, even pending appeal. The
employee shall either be admitted back to work under the same terms and
conditions prevailing prior to his dismissal or separation, or at the option of the
employer, merely reinstated in the payroll. The posting of a bond by the employer
shall not stay the execution for reinstatement."

The Court discussed reason behind this legal policy in Aris v. NLRC, where it
explained:

In authorizing execution pending appeal of the reinstatement aspect of a


decision of the Labor Arbiter reinstating a dismissed or separated employee, the
law itself has laid down a compassionate policy which, once more, vivifies and
enhances the provisions of the 1987 Constitution on labor and the working-man.
These provisions are the quintessence of the aspirations of the workingman for
recognition of his role in the social and economic life of the nation, for the
protection of his rights, and the promotion of his welfare.

The commanding one is the rule in Pfizer, which echoes the rulings made in the
cases of Roquero v. Philippine Airlines and Garcia v. Philippine Airlines that the
period for computing the backwages due to the respondents during the period of
appeal should end on the date that a higher court reversed the labor arbitration
ruling of illegal dismissal. In this case, the higher court which first reversed the
NLRC’s ruling was not the SC but rather the CA. In this light, the CA was correct
when it found that that the period of computation should end on August 27, 2003.
The date when the SC’s decision became final and executory need not matter as
the rule in Roquero, Garcia and Pfizer merely referred to the date of reversal, not
the date of the ultimate finality of such reversal.

Dario Nacar vs. Gallery Frames and/or Felipe Bordey, Jr.


GR No. 189871 August 13, 2013

Facts: Petitioner Nacar filed with the Arbitration Branch of the NLRC a case of
constructive dismissal against respondent Gallery Frames and/or Felipe Bordey, Jr.
The LA rendered a decision in petitioner’s favor that he was unjustly dismissed and
awarded backwages in lieu of reinstatementin the amount of ₱158,919.92.. The
NLRC affirmed the ruling of the LA on appea, and so did CA. Respondents then
sought relief before the Supreme Court, docketed as G.R. No. 151332. Finding no
reversible error on the part of the CA, this Court denied the petition in the
Resolution dated April 17, 2002, and the decision became final and executory. On
November 5, 2002, petitioner filed a Motion for Correct Computation, praying that
his backwages be computed from the date of his dismissal on January 24, 1997 up
to the finality of the Resolution of the Supreme Court on May 27, 2002. Upon
recomputation, the Computation and Examination Unit of the NLRC arrived at an
updated amount in the sum of ₱471,320.31. Respondents filed a motion to quash,
arguing that the decision has become final and executory and thus the amount
cannot be changed anymore. The LA denied the motion, but the NLRC granted
their appeal and ordered a recomputation, coming up with the amount of
₱147,560.19, which was received by petitioner.
Petitioner then filed a Manifestation and Motion praying for the re-computation of
the monetary award to include the appropriate interests, which, according to
NLRC, was only up to ₱11,459.73. The Labor Arbiter reasoned that it is the October
15, 1998 Decision that should be enforced considering that it was the one that
became final and executory. However, the Labor Arbiter reasoned that since the
decision states that the separation pay and backwages are computed only up to
the promulgation of the said decision, it is the amount of ₱158,919.92 that should
be executed. Thus, since petitioner already received ₱147,560.19, he is only
entitled to the balance of ₱11,459.73. Petitioner appealed before the NLRC and
the CA, but his petitions were denied.
Issue: Whether or not the basis for the computation of the backwages should be
reckoned on the date of finality of the Supreme Court decision in GR No. 151332
Held: Yes. The computation the labor arbiter made shows that it was time-bound
as can be seen from the figures used in the computation. This part, being merely a
computation of what the first part of the decision established and declared, can,
by its nature, be re-computed. This is the part, too, that the petitioner now posits
should no longer be re-computed because the computation is already in the
labor arbiter's decision that the CA had affirmed. The public and private
respondents, on the other hand, posit that a re-computation is necessary because
the relief in an illegal dismissal decision goes all the way up to reinstatement if
reinstatement is to be made, or up to the finality of the decision, if separation pay
is to be given in lieu reinstatement.
That the labor arbiter's decision, at the same time that it found that an illegal
dismissal had taken place, also made a computation of the award, is
understandable in light of Section 3, Rule VIII of the then NLRC Rules of Procedure
which requires that a computation be made.
By the nature of an illegal dismissal case, the reliefs continue to add up until full
satisfaction, as expressed under Article 279 of the Labor Code. The recomputation
of the consequences of illegal dismissal upon execution of the decision does not
constitute an alteration or amendment of the final decision being implemented.
The illegal dismissal ruling stands; only the computation of monetary
consequences of this dismissal is affected, and this is not a violation of the
principle of immutability of final judgments.

[G.R. No. 178699 : September 21, 2011] BPI EMPLOYEES UNION - METRO MANILA
AND ZENAIDA UY, PETITIONERS, VS.BANK OF THE PHILIPPINE ISLANDS, RESPONDENT.
[G.R. NO. 178735]
BANK OF THE PHILIPPINE ISLANDS, PETITIONER, VS.BPI EMPLOYEES UNION - METRO
MANILA AND ZENAIDA UY, RESPONDENTS.

FACTS: On December 14, 1995, ZenaidaUy'sservices as a bank teller in BPI's Escolta


Branch was terminated on grounds of gross disrespect/discourtesy towards an
officer, insubordination and absence without leave. Uy, together with the Union,
thus filed a case for illegal dismissal. The Voluntary Arbitrator rendered a Decision
finding Uy's dismissal as illegal and ordering BPI to immediately reinstate Uy and to
pay her full back wages,including all her other benefits under the Collective
Bargaining Agreement (CBA)and attorney's fees. The CA affirmed with
modification the Decision of the Voluntary Arbitrator. Instead of reinstatement, the
CA ordered BPI to pay Uy her separation pay. Further, instead of full back wages,
the CA fixed Uy's back wages to three years. The case eventually reached the SC
when both parties separately filed petitions for review oncertiorari. While BPI's
petition which was docketed as G.R. No. 137856 was denied for failure to comply
with the requirements of a valid certification of non-forum shopping,Uy's and the
Union's petition which was docketed as G.R. No. 137863 was given due course
and directed the respondent BPI to pay petitioner UY backwages from the time of
her illegal dismissal until her actual reinstatement and ordered reinstatement.

After the Decision in G.R. No. 137863 became final and executory, Uy and the
Union filed with the Office of the Voluntary Arbitrator a Motion for the Issuance of
a Writ of Execution. In Uy's computation, she based the amount of her back
wages on thecurrentwage level and included all the increases in wages and
benefits under the CBA that were granted during the entire period of her illegal
dismissal. BPI disputed Uy's/Union's computation arguing that it contains items
which are not included in the term "back wages" and that no proof was
presented to show that Uy was receiving all the listed items therein before her
termination. It claimed that the basis for the computation of back wages should
be the employee's wage rateat the time of dismissal.

The Voluntary Arbitrator agreed with Uy's/Union's contention that full back wages
should include all wage and benefit increases, including new benefits granted
during the period of dismissal. Imputing grave abuse of discretion on the part of
the Voluntary Arbitrator, BPI filed with the CA. Uy and the Union alleged that BPI's
remedy is not acertioraripetition under Rule 65 of the Rules of Court but an appeal
from judgments, final orders and resolutions of voluntary arbitrators under Rule 43
of the Rules of Court. They also contended that BPI's petition is wanting in
substance. Meanwhile, the CA issued a TRO. The CA initially rendered a
Decisionupholding that BPI's resort tocertiorariwas proper and that the award of
CBA benefits and attorney's fees has legal basis. The CA however found that the
Voluntary Arbitrator erroneously computed Uy's back wages based on the current
rate. The CA also deleted the award of dental allowance since it was granted in
2002 or more than six years after Uy's dismissal. Both parties thereafter filed their
respective motions for reconsideration.

The CA amended its decision and upheld the propriety of BPI's resort tocertiorari.
The CA ruled that the computation of Uy's full back wages, as defined
underRepublic Act No. 6715, should be based on the basic salary at the time of
her dismissal plus the regular allowances that she had been receiving likewise at
the time of her dismissal. It held that any increase in the basic salary occurring
after Uy's dismissal as well as all benefits given after said dismissal should not be
awarded to her in consonance with settled jurisprudence on the matter. From the
foregoing Amended Decision, both parties separately filed petitions before this
Court. Uy's and the Union's petition is docketed as G.R. No178699, and that of BPI is
docketed as G.R. No. 178735. The Court resolved to consolidate both petitions in a
Resolution dated September 3, 2007.

ISSUE: Shoud the basis of the computation of backwages be the wage rate at the
time of dismissal?

HELD: The full backwages, as referred to in the body of the decision pertains to
"backwages"as defined in Republic Act No. 6715. Under said law, and as provided
in numerous jurisprudence, "full backwages" means backwages without any
deduction or qualification, including benefits or their monetary equivalent the
employee is enjoying at the time of his dismissal, that is, "unqualified by any wage
increases or other benefits that may have been received by co-workers who were
not dismissed." It is likewise settled that the base figure to be used in the
computation of back wages is pegged at the wage rate at the time of the
employee's dismissal unqualified by deductions, increases and/or modifications.
Hence, Petitioner Uy was granted full backwages computed from the time of her
dismissal on December 14, 1995 up to her reinstatement on August 1, 2006
including benefits which were proven to be received by her at the time of her
dismissal.

Section 1, Rule 41 of the Rules of Court explicitly provides that no appeal may be
taken from an order of execution, the remedy of an aggrieved party being an
appropriate special civil action under Rule 65 of the Rules of Court. Thus, BPI
correctly availed of the remedy of certiorari under Rule 65 of the Rules of Court
when it assailed the December 6, 2005 order of execution of the Voluntary
Arbitrator.

INTEGRATED MICROELECTRONICS, INC., Petitioner, v. ADONIS A.


PIONILLA, Respondent.

G.R. No. 200222, August 28, 2013

Facts: Respondent was hired by petitioner as its production worker. Respondent


received a notice from IMI asking him to explain an incident that occurred where
he was seen escorting a woman to board the company’s bus shuttle. The bus
marshal reported that the lady was wearing a company ID which serves as a free
pass for the shuttle passengers even if she was an applicant. Respondent
admitted that the woman was his relative and that he lent her his ID. Nevertheless,
he apologized for his actions.
IMI found Pionilla guilty of violating Article 6.12 of the Company Rules and
Regulations (CRR) which prohibits the lending of ones ID since the same is
considered a breach of its security rules and carries the penalty of dismissal.
Subsequently, Pionilla received a letter dated August 16, 2005 informing him of his
dismissal from service. Three days after, he filed a complaint for illegal dismissal
with damages against IMI.

LA ruled that Pionilla has been illegally dismissed. On appeal, the NLRC reversed
the LAs ruling, finding Pionillas dismissal to be valid. It pointed out that Pionillas act
of lending his temporary ID was willful and intentional. Dissatisfied, Pionilla filed a
petition for certiorari before the CA. The CA rendered a DecisiongrantingPionillas
petition. It found that while IMIs regulations on company IDs were reasonable, the
penalty of dismissal was too harsh and not commensurate to the misdeed
committed. In view of the CAs ruling, IMI filed a petition for review on certiorari
before the Supreme Court but the same was denied. Hence, motion for
reconsideration was filed.
ISSUE: Whether or not Pionilla may be entitled to full backwages
HELD: No. Full backwages are deleted.
As a general rule, an illegally dismissed employee is entitled to reinstatement (or
separation pay, if reinstatement is not viable) and payment of full backwages. In
certain cases, however, the Court has carved out an exception to the foregoing
rule and thereby ordered the reinstatement of the employee without backwages
on account of the following : (a) the fact that dismissal of the employee would be
too harsh of a penalty; and (b) that the employer was in good faith in terminating
the employee.
The Court is convinced that petitioner's guilt was substantially established.
Nevertheless, we agree with respondent Minister's order of reinstating petitioner
without backwages instead of dismissal which may be too drastic. Denial of
backwages would sufficiently penalize her for her infractions. The bank officials
acted in good faith. They should be exempt from the burden of paying
backwages. The good faith of the employer, when clear under the
circumstances, may preclude or diminish recovery of backwages. Only
employees discriminately dismissed are entitled to backpay.

Victory Liner, Inc. vs. Race, G.R. No. 164820, December 8, 2008

Facts:
Petitioner Victory Liner, Inc. Filed the present Motion for Reconsideration seeking
modification of the decision dated March 28, 2007. In the said Decision, the court
found out that respondent Pablo Race, employed as one of petitioner’s bus
drivers, was illegally dismissed by the petitioner since petitioner failed to comply
with both substantive and procedural due process in terminating respondent’s
employment. However, considering the leg injury sustained by the respondent in
an accident which already rendered him incapable of driving a bus, the court
ordered his separation pay instead of reinstatement.

Issue:

Petitioner impugns the Decision on two grounds:


1. Whether or not the award of full backwages inclusive of allowances and other
benefits or their monetary equivalent to respondent is not warranted; and

2. Whether or not the dismissal of respondent is authorized under Article 284 of the
Labor Code.
Ruling:

Petitioner’s motion is partly meritorious.

In the present Decision, respondent suffered leg injury after figuring in an accident
while driving the petitioner’s bus, for which he was operated and confined at the
hospital. The court is unable to sustain petitioner’s position that respondent
abandoned his job as early as 1994. For the next four years, respondent was
reporting to petitioner’s office twice a month and still receiving his salary and
medical assistance from the petitioner. It was only in January 1988 that
respondent was actually dismissed from employment when he was expressly
informed that he was considered resigned from his job. They further found out that
respondent was not afforded procedural due process prior to his dismissal in 1998.
The court ordered petitioner pay respondent (1) separation pay of one month for
every year of service, in lieu of reinstatement; and (2) full backwages inclusive of
allowances and other benefits of their monetary equivalent from January 1, 1998
up to the finality of the said Decision.

At the present motion, petitioner is asserting that it should be deemed to have


acted in good faith when it considered respondent as resigned from work
because the Court itself stated in the Decision that respondent’s reinstatement is
no longer feasible due to his leg injury, and that to allow the respondent to drive
petitioner’s bus in this present physical condition would place the petitioner in
jeopardy of violating its obligation as a common carrier to always exercise
extraordinary diligence. Thus, invoking good faith, petitioner denies any liability to
respondent for the payment of his backwages and allowances from January 1,
1998 to the date of finality of decision.

In attributing good faith to petitioner, the court give due regard to the following
circumstances:

(1) Respondent had been working for petitioner for only 15 months, from June
1993 to August 1994, when the accident occurred causing injury to his leg. Hence
he was able to render actual service to petitioner as a bus driver for the mere
period for over a year.

(2) In January 1998, when he went to petitioner’s office and was informed that he
was deemed resigned from work, he was still limping heavily. In fact, respondent’s
letter to petitioner’s Vice President , dated March 18, 1996 requesting that he be
transferred to position of dispatcher or conductor, is revealing of the fact that he
could no longer drive a bus because of his leg injury.

(3) Despite respondent’s inability to render actual service for four years following
the accident in 1994, petitioner still continued to pay him his salary and shoulder
his medical expenses. When the petitioner informed respondent that he was
deemed resigned in 1998, petitioner even offered respondent the amount of P
50,000.00 as financial assistance; and when respondent refused to receive the
said amount, petitioner raised its offer to P 100,000.00.

Finally, petitioner is a common carrier and, as such, is obliged to exercise


extraordinary diligence in transporting its passengers safely. Understandably,
petitioner feared that it would be exposing to danger the lives of its passengers if it
allowed the respondent to drive its bus despite the fact that his leg was injured.
Although the court could not depart its previous ruling that respondent was
illegally dismissed since petitioner was, at the beginning unable to identify with
certitude the basis for respondent’s termination, as well as the date of effectivity
thereof, the court is convinced that taking into account the foregoing
circumstances, that petitioner acted without malice and in good faith when it
formally informed respondent in 1998 that he was deemed resigned from work.

For reasons of fairness and equity, as well as the particular factual circumstances
attendant in this case, it dictates modification of the decision ordering the
petitioner to pay respondent limited backwages (inclusive of allowances and
other benefits or their monetary equivalent) for five years, from January 1998 to
December 2002, in addition to the separation pay of one month for every year of
service awarded in lieu of reinstatement. It must be clarified, however, that for
purposes of computing respondents separation pay, he must still be deemed in
petitioner’s employment until the finality of its decision since its termination remains
illegal, and is only mitigated by petitioner’s good faith.

Bluer Than Blue vs. Esteban, GR No. 192582, April 7, 2014

Facts:

Respondent Glyza Esteban was employed as a sales clerk, and assigned at Bluer
Than Blue Venture Company in SM city Marilao, Bulacan, beginning the year 2006.
Part of her tasks were attending to all customer needs, ensuring efficient inventory,
coordinating orders from clients, cashiering and reporting to the accounting
department.

In November 2006, the petitioner received a report that several employees have
access to its point-of-sale system through a universal password given by Elmer
Flores. Upon investigation, it was found out that Esteban was the one who gave
Flores the password. The petitioner sent a letter of memorandum to Esteban,
asking her to explain in writing why she should not be disciplinary dealt with for
tampering with the POS system through the use of unauthorized password. She
was also placed under preventive suspension for ten days.

After Esteban’s preventive suspension was lifted, a notice of termination was sent
to her, finding her explanation unsatisfactory and immediately terminates her
employment on the ground of loss of trust and confidence. She received her final
pay including benefits and bonuses, less inventory variances incurred by the store
accounting. Esteban signed a quitclaim and release in favor of the petitioner.

Esteban filed a complaint for illegal dismissal, illegal suspension, holiday pay, rest
day and separation pay. The labor arbiter ruled in favor of Esteban and dismissed
the case of illegal dismissal. The Court of appeals on the other hand, granted her
petition and reinstated the labor arbiter’s decision. Petitioner argues that it had
just cause to terminate Esteban for loss of trust and confidence.

Issue:

Whether or not Esteban’s acts constitutes just cause to terminate her employment
with the company on the ground of loss of trust and confidence.

Ruling:

No. Loss of trust and confidence is premised in the fact that the employee
concerned holds a position of responsibility, trust and confidence. The employee
must be vested with confidence on delicate matters, such as the custody of
handling, care and protection of the employer’s property and funds. Such cause
termination to rank and file employee requires proof of involvement in the alleged
events in question, and that mere uncorroborated assertions and accusations by
the employer will not be sufficient.

Esteban is, no doubt, a rank and file employee. She was a sales clerk. Her duties
were more than that of a sales clerk. Aside from attending to customers and
tending to the shop, Esteban also assumed cashiering duties. She does not deny
this fact but insists that the competency clause provided that her tasks were that
of a sales clerk and the cashiering function was labeled “to follow”. A perusal of
the competency clause, however, shows that it is merely an attestation on her
part that she is competent to meet the basic requirements needed for the
position. It does not define her actual duties. As consistently ruled, it is not the job
title but the actual work that the employee performs that determines whether he
or she occupies a position of trust and confidence. Given that she had in her care
and custody the store’s property and funds, she is considered occupying a
position of trust and confidence

However, the Court finds that the acts committed by Esteban do not amount to a
willful breach of trust. The facts on hand shows that the used of the password
“123456” by Esteban was not done intentionally, knowingly, and purposely. The
suspension would have sufficed as punishment, considering that the petitioner
had already been witht he company for more than two years, and the petitioner
apologized and readily admitted her mistake in her written explanation, and
considering that no clear and convincing evidence of loss or prejudice, which
was suffered by the petitioner from Esteban’s supposed infraction.

In this case, the petitioner was acting well within its rights when it imposed a 10-
day preventive suspension on Esteban. While it may be that the acts complained
of were committed by Esteban almost a year before the investigation was
conducted, still, it should be pointed out that Esteban was performing functions
that involve handling of the employer’s property and funds, and the petitioner
had every right to protect its assets and operations pending Esteban’s
investigation. Thus, the court partially granted the petition.

Artificio vs. NLRC, G.R. No. 172988, July 26, 2010

Facts:

Petitioner Jose P. Artificio was employed as security guard by respondent RP


Guardians Security Agency, Inc., a corporation duly organized and existing under
Philippine Laws and likewise duly licensed to engage in the security agency
business.

Sometime in June 2002, Artificio had a heated argument with a fellow security
guard, Merlino B. Edu (Edu). On 25 July 2002, Edu submitted a confidential report 5
to Antonio A. Andres (Andres), Administration & Operations Manager, requesting
that Artificio be investigated for maliciously machinating Edu's hasty relief from his
post and for leaving his post during night shift duty to see his girlfriend at a nearby
beerhouse.

On 29 July 2002, another security guard, Gutierrez Err (Err), sent a report 6 to
Andres stating that Artificio arrived at the office of RP Guardians Security Agency,
Inc. on 25 June 2002, under the influence of liquor. When Artificio learned that no
salaries would be given that day, he bad-mouthed the employees of RP
Guardians Security Agency, Inc. and threatened to "arson" their office.

On even date, Andres issued a Memorandum temporarily relieving Artificio from


his post and placing him under preventive suspension pending investigation for
conduct unbecoming a security guard, such as, abandonment of post during
night shift duty, light threats and irregularities in the observance of proper relieving
time. He also directed Artificio to report to the office of RP Guardians Security
Agency, Inc. and submit his written answer immediately upon receipt of the
memorandum.
In another memorandum, Andres informed Artificio that a hearing will be held on
12 August 2002.

Without waiting for the hearing to be held, Artificio filed on 5 August 2002, a
complaint for illegal dismissal, illegal suspension, non-payment of overtime pay,
holiday pay, premium pay for holiday and rest days, 13th month pay, and
damages. He also prayed for payment of separation pay in lieu of reinstatement.

Labor Arbiter rendered a decision dated 6 October 2003, finding respondents


guilty of illegal suspension and dismissal. It was also held that Artificio should have
been allowed to confront Edu and Err before he was preventively suspended.
Since the complainant does not seek reinstatement, he is entitled to limited
backwages and separation pay.
On appeal, the NLRC, set aside the decision of the Labor Arbiter ruling that the
Labor Arbiter erred in considering preventive suspension as a penalty. The motion
for reconsideration filed by Artificio was denied for lack of merit

Artificio next filed a petition for certiorari before the Court of Appealswhich
rendered a decision affirming the NLRC decision. Artificio filed a motion for
reconsideration which the Court of Appeals again denied.

Issues:

1. Whether or not Petitioner Artificio's preventive suspension was justified


2. Whether or not, he is entitled to backwages and separation pay

Rulings:

1. Yes. Sections 8 and 9 of Rule XXIII, Implementing Book V of the Omnibus Rules
Implementing the Labor Code provides that preventive suspension is justified
where the employee's continued employment poses a serious and imminent
threat to the life or property of the employer or of the employee's co-workers.
Without this kind of threat, preventive suspension is not proper.

In this case, Artificio's preventive suspension was justified since he was employed
as a security guard tasked precisely to safeguard respondents' client. His
continued presence in respondents' or its client's premises poses a serious threat to
respondents, its employees and client in light of the serious allegation of conduct
unbecoming a security guard such as abandonment of post during night shift
duty, light threats and irregularities in the observance of proper relieving time.

Besides, Management has the prerogative to discipline its employees and to


impose appropriate penalties on erring workers pursuant to company rules and
regulations.This Court has upheld a company's management prerogatives so long
as they are exercised in good faith for the advancement of the employer's interest
and not for the purpose of defeating or circumventing the rights of the employees
under special laws or under valid agreements.
Significantly, Artificio regrettably chose not to present his side at the administrative
hearing scheduled to look into the factual issues that accompanied the
accusation against him. In fact, he avoided the investigation into the charges by
filing his illegal dismissal complaint ahead of the scheduled investigation. He, on
his own decided that his preventive suspension was in fact illegal dismissal and
that he is entitled to backwages and separation pay. Indeed, Artificio would even
reject reinstatement revealing his bent to have his own way through his own
means. As aptly noted by the NLRC, Artificio preempted the investigation that
could have afforded him the due process of which he would then say he was
denied.

2. Yes for Backwages. No for separation pay.

Having determined that the imposition on Artificio of preventive suspension was


proper and that such suspension did not amount to illegal dismissal, we see no
basis for the grant of backwages.

Nonetheless, given the attendant circumstances in this case, namely, that Artificio
had been working with the company for a period of sixteen (16) years and
without any previous derogatory record, the ends of social and compassionate
justice would be served if Artificio be given some equitable relief in the form of
separation pay.

Artificio is entitled to separation pay considering that while reinstatement is an


option, Artificio himself has never, at anytime after the notice of preventive
suspension intended to remain in the employ of private respondents.

Mandapat vs. Add Force Personnel, G.R. No. 192582, July 6, 2010

Facts:

On 15 September 2003, petitioner Ma. Socorro Mandapat was hired as Sales and
Marketing Manager for respondent Add Force Personnel Services, Inc. As detailed
in her appointment letter, her duties include negotiation and consummation of
contracts with clients who wanted to avail of respondents services. She reported
directly to the Chief Executive Officer (CEO), Colwyn Ron C. Longstaff (Longstaff).

Respondent claims that during her five-month stint as sales manager, petitioner
failed to close a single deal or contract with any client. In addition, petitioner
issued several proposals to clients which were either grossly disadvantageous to
respondent or disregarded the clients budget ceiling. Petitioner also sent out
several communications to clients containing erroneous data and computations;
submitted fictitious daily activity reports and reimbursement slips; and consistently
failed to submit her reports, such as the daily activity report, expense report,
weekly sales call plan and internet-based calendar system on time.
These infractions were contained in a show-cause notice sent to petitioner on 23
February 2004, directing her to explain why she should not be disciplined for gross
and habitual neglect of duties and willful breach of trust. Petitioner was also
preventively suspended and was asked to turn over pending tasks and to leave
the office premises.

Mandapat gave Add Force her response to the show-cause memorandum along
with her resignation letter supposedly in protest of the preventive suspension.
Subsequently, she filed a complaint with the labor arbiter, claiming she was
constructively dismissed when she was placed on preventive suspension, her
access to the internet cut-off, and then pressured by Add Force to resign in
exchange for separation pay. She denied that she was negligent, and faulted the
Chief Executive Officer for his indecisiveness and the lack of support staff for the
sales department. She claimed that her preventive suspension was illegal for
being indefinite, since its duration was not stated in the show-cause
memorandum. She argued that she did pose any danger to the lives of Add
Force’s officers or its properties to warrant the preventive suspension.

Add Force insisted that Mandapat resigned and was not dismissed. It explained
that Mandapat was placed on preventive suspension because of the risk she
posed on its property and business. Add Force added that Mandapat’s
preventive suspension for 1 day can hardly be considered indefinite, given that
she immediately resigned 1 day after the suspension.

Issue:

Whether or not petitioner was constructively dismissed.

Ruling:

No. Constructive dismissal exists when an act of clear discrimination, insensibility or


disdain by an employer has become so unbearable to the employee leaving him
with no option but to forego his continued employment. There was no act of
discrimination committed against Mandapat that would render her employment
unbearable.

Preventive suspension may be legally imposed on employee whose alleged


violation is the subject of an investigation. The purpose of his suspension is to
prevent him from causing harm or injury to the company as well as to his fellow
employees (Section 8, Rule XXIII, Book V, Omnibus Rules Implementing the Labor
Code, as amended by Department Order No. 9, Series of 1997). No preventive
suspension shall last longer than 30 days and the employer shall thereafter
reinstate the worker in his former or in a substantially equivalent position or the
employer may extend the period of suspension provided that during the period of
extension, he pays the wages and other benefits due to the worker (Section 9,
Rule XXIII, Book V, Omnibus Rules Implementing the Labor Code, as amended by
Department Order No. 9, Series of 1997).
When preventive suspension exceeds the maximum period allowed without
reinstating the employee either by actual or payroll reinstatement or when
preventive suspension is for an indefinite period, only then will constructive
dismissal set in. While no period was mentioned in the show-cause memorandum,
the inclusion of the phrase “during the course of investigation” would lead to a
reasonable and logical presumption that said suspension in fact had a duration
which could very well be not more than 30 days as mandated by law. And, as the
CA correctly observed, the suspension was rendered moot by Mandapat’s
resignation tendered a day after the suspension was made effective.

The preventive suspension was also necessary to protect Add Force’s assets and
operations pending investigation of Mandapat. As Sales Manager, Mandapat
had the power to enter into contracts that would bind Add Force, regardless of
whether these contracts would prove to be beneficial or prejudicial to its interest.
The cutting-off of Mandapat’s internet access was not harassment but a
consequence of the investigation against her and was intended to prevent her
from having further access to the company’s network-based documents and
forms. Add Force’s acts were just measures to protect itself while the investigation
was ongoing. There was no coercion employed on Mandapat to resign. Mere
allegations of threat or force do not constitute evidence to support a finding of
forced resignation.

In order for intimidation to vitiate consent, the following requisites must concur: (1)
the intimidation caused the consent to be given; (2) the threatened act is unjust
or unlawful; (3) the threat is real or serious, there being evident disproportion
between the evil and the resistance which all men can offer, leading to the
choice of doing the act which is forced on the person to do as the lesser evil; and
(4) it produces a well-grounded fear from the fact that the person from whom it
comes has the necessary means or ability to inflict the threatened injury to his
person or property. None of these requisites was proven by Mandapat.

No demand was made on her to resign. At most, she was merely given the option
to either resign or face disciplinary investigation, which Add Force had every right
to conduct in light of her numerous infractions. There was nothing irregular in
providing an option to her. Ultimately, the final decision on whether to resign or
face disciplinary action rested on her alone.

G.R. No. 158637 April 12, 2006

MARICALUM MINING CORPORATION, Petitioner,


vs.
ANTONIO DECORION, Respondent.

Facts:
Decorion was a regular employee of Maricalum Mining who started out as a Mill
Mechanic assigned to the Concentrator MaintenanceDepartment and was later
promoted to Foreman I.On April 11, 1996, the Concentrator Maintenance
Supervisor called a meeting which Decorion failed to attend as he was then
supervising the workers under him. Because of his alleged insubordination for
failure to attend the meeting, he was placed under preventive suspension on the
same day. He was also not allowed to report for work the following day.

A month after or on May 12, 1996, Decorion was served a Notice of Infraction and
Proposed Dismissal to enable him to present his side. On May 15, 1996, he
submitted to the Personnel Department his written reply to the notice.

A grievance meeting was held upon Decorion’s request on June 5, 1996, during
which he manifested that he failed to attend the meeting on April 11, 1996
because he was then still assigning work to his men. He maintained that he has
not committed any offense and that his service record would show his efficiency.

On July 23, 1996, Decorion filed before the National Labor Relations Commission
(NLRC) Regional Arbitration Branch VI of Bacolod City a complaint for illegal
dismissal and payment of moral and exemplary damages and attorney’s fees.

In the meantime, the matter of Decorion’s suspension and proposed dismissal was
referred to Atty. Roman G. Pacia, Jr., Maricalum Mining’s Chief and Head of Legal
and Industrial Relations, who issued a memorandum on August 13, 1996,
recommending that Decorion’s indefinite suspension be made definite with a
warning that a repetition of the same conduct would be punished with dismissal.
Maricalum Mining’s Resident Manager issued a memorandum on August 28, 1996,
placing Decorion under definite disciplinary suspension of six (6) months which
would include the period of his preventive suspension which was made to take
effect retroactively from April 11, 1996 to October 9, 1996.

On September 4, 1996, Decorion was served a memorandum informing him of his


temporary lay-off due to Maricalum Mining’s temporary suspension of operations
and shut down of its mining operations for six (6) months, with the assurance that in
the event of resumption of operations, he would be reinstated to his former
position without loss of seniority rights.

Decorion, through counsel, wrote a letter to Maricalum Mining on October 8,


1996, requesting that he be reinstated to his former position. The request was
denied with the explanation that priority for retention and inclusion in the skeleton
force was given to employees who are efficient and whose services are necessary
during the shutdown.

Conciliation proceedings having failed to amicably settle the case, the labor
arbiter rendered a decision dated November 26, 1998, finding Decorion’s dismissal
illegal and ordering his reinstatement with payment of backwages and attorney’s
fees. According to the labor arbiter, Decorion’s failure to attend the meeting
called by his supervisor did not justify his preventive suspension. Further, no
preventive suspension should last longer than 30 days.

Issue:

Whether or not the preventive suspension of Decorion is proper.

Ruling: No.

Sections 8 and 9 of Rule XXIII, Book V of the Implementing Rules provide as follows:

Section 8. Preventive suspension. --- The employer may place the worker
concerned under preventive suspension if his continued employment poses a
serious and imminent threat to the life or property of the employer or his co-
workers.

Section 9. Period of Suspension --- No preventive suspension shall last longer than
thirty (30) days. The employer shall thereafter reinstate the worker in his former or in
a substantially equivalent position or the employer may extend the period of
suspension provided that during the period of extension, he pays the wages and
other benefits due to the worker. In such case, the worker shall not be bound to
reimburse the amount paid to him during the extension if the employer decides,
after completion of the hearing, to dismiss the worker.

The Rules are explicit that preventive suspension is justified where the employee’s
continued employment poses a serious and imminent threat to the life or property
of the employer or of the employee’s co-workers. Without this kind of threat,
preventive suspension is not proper.

In this case, Decorion was suspended only because he failed to attend a meeting
called by his supervisor. There is no evidence to indicate that his failure to attend
the meeting prejudiced his employer or that his presence in thecompany’s
premises posed a serious threat to his employer and co-workers. The preventive
suspension was clearly unjustified.

Decorion’s suspension persisted beyond the 30-day period allowed by the


Implementing Rules. In Premiere Development Bank v. NLRC, 11 private
respondent’s suspension lasted for more than 30 days counted from the time she
was placed on preventive suspension on March 13, 1986 up to the last day of
investigation on April 23, 1986. The Court ruled that preventive suspension which
lasts beyond the maximum period allowed by the Implementing Rules amounts to
constructive dismissal.

Maricalum Mining’s contention that there was as yet no illegal dismissal at the
time of the filing of the complaint is evidently unmeritorious. Decorion’s preventive
suspension had already ripened into constructive dismissal at that time. While
actual dismissal and constructive dismissal do take place in different fashion, the
legal consequences they generate are identical.

Decorion’s employment may not have been actually terminated in the sense that
he was not served walking papers but there is no doubt that he was constructively
dismissed as he wasforced to quit because continued employment was rendered
impossible, unreasonable or unlikelyby Maricalum Mining’s act of preventing him
from reporting for work.

G.R. No. 159730 February 11, 2008

NORKIS TRADING CO., INC. and/or MANUEL GASPAR E. ALBOS, JR., petitioner,
vs.
MELVIN GNILO, respondent.

Facts:

Melvin R. Gnilo (respondent) was initially hired by Norkis Trading Co., Inc.
(petitioner Norkis) as Norkis Installment Collector (NIC) in April 1988. Manuel
Gaspar E. Albos, Jr. (petitioner Albos) is the Senior Vice-President of petitioner
Norkis. Respondent held various positions in the company until he was appointed
as Credit and Collection Manager of Magna Financial Services Group, Inc.-
Legaspi Branch, petitioner Norkis’s sister company, in charge of the areas of Albay
and Catanduanes with travel and transportation allowances and a service car.

A special audit team was conducted in respondent's office in Legaspi, Albay from
March 13 to April 5, 2000 when it was found out that respondent forwarded the
monthly collection reports of the NICs under his supervision without checking the
veracity of the same. It appeared that the monthly collection highlights for the
months of April to September 1999 submitted by respondent to the top
management were all overstated particularly the account handled by NIC Dennis
Cadag, who made it appear that the collection efficiency was higher than it
actually was; and that the top management was misled into believing that
respondent’s area of responsibility obtained a favorable collection efficiency.
During the investigation, respondent admitted that he was negligent for failing to
regularly check the report of each NIC under his supervision; that he only checked
at random the NIC's monthly collection highlight reports; and that as a leader, he
is responsible for the actions of his subordinates. He however denied being lax in
supervising his subordinates, as he imposed discipline on them if the need arose.

On May 30, 2000, petitioner Norkis through its Human Resource Manager issued a
memorandum3 placing respondent under 15 days suspension without pay, travel
and transportation allowance, effective upon receipt thereof. Respondent filed a
letter protesting his suspension and seeking a review of the penalty imposed.

In a letter5 dated July 27, 2000, respondent requested petitioner Albos that he be
assigned as Sales Engineer or to any position commensurate with his qualifications.
However, on July 28, 2000, respondent was formally appointed as Marketing
Assistant to petitioner Albos, which position respondent subsequently assumed.

However, on October 4, 2000, respondent filed with the Labor Arbiter (LA) a
complaint for illegal suspension, constructive dismissal, non-payment of
allowance, vacation/sick leave, damages and attorney's fees against petitioners.

On March 30, 2001, the LA rendered his decision 6 dismissing the complaint for lack
of merit.

The NLRC ruled that respondent was constructively dismissed and therefore he
was entitled to reinstatement and payment of full backwages from the time he
quit working on October 19, 2000 due to his demotion up to the time of his actual
reinstatement.

Issue:

Whether or not the transfer of Gnilo from being a Collections Manager to a


Marketing Assistant constitutes a Constructive Dismissal.

Ruling:

Yes.

The employer bears the burden of showing that the transfer is not unreasonable,
inconvenient or prejudicial to the employee; and does not involve a demotion in
rank or a diminution of his salaries, privileges and other benefits. 18 Should the
employer fail to overcome this burden of proof, the employee’s transfer shall be
tantamount to constructive dismissal.19

Constructive dismissal is defined as a quitting because continued employment is


rendered impossible, unreasonable or unlikely; when there is a demotion in rank or
a diminution of pay.20 Likewise, constructive dismissal exists when an act of clear
discrimination, insensibility or disdain by an employer becomes unbearable to the
employee, leaving him with no option but to forego his continued employment.
A transfer is defined as a "movement from one position to another which is of
equivalent rank, level or salary, without break in service." 22 Promotion, on the other
hand, is the "advancement from one position to another with an increase in duties
and responsibilities as authorized by law, and usually accompanied by an
increase in salary."23 Conversely, demotion involves a situation in which an
employee is relegated to a subordinate or less important position constituting a
reduction to a lower grade or rank, with a corresponding decrease in duties and
responsibilities, and usually accompanied by a decrease in salary.24

In this case, while the transfer of respondent from Credit and Collection Manager
to Marketing Assistant did not result in the reduction of his salary, there was a
reduction in his duties and responsibilities which amounted to a demotion
tantamount to a constructive dismissal as correctly held by the NLRC and the CA.

There is constructive dismissal when an employee's functions, which were originally


supervisory in nature, were reduced; and such reduction is not grounded on valid
grounds such as genuine business necessity

G.R. No. 198261 October 16, 2013

HECHANOVA BUGAY VILCHEZ LAWYERS, HECHANOVA & CO., INC., ATTY. EDITHA R.
HECHANOVA, Petitioners,
vs.
ATTY. LENY O. MATORRE, Respondent.

Facts:

Atty. Matorre claimed that on August 1, 2008, she was employed by HBV Law Firm
as a Senior Associate Attorney. Due to her work experience, her probationary
period was waived and she was immediately employed as a regular employee of
the said law firm with a monthly salary of ₱40,000, consultancy fee of ₱5,000, and
an incentive pay equivalent to 8% of ₱1,500 per billable hour.

As the managing partner of HBV Law Firm, Atty. Hechanova was the one who
supervised Atty. Matorre and gave her work assignments.

On August 11, 2008, Atty. Matorre, orally or through e-mails, started to express her
feelings of being harassed by Atty. Hechanova. She also explained 8 that she
intended to improve her work and that she was not making excuses when she
could not accomplish assigned tasks on time.

During a meeting, Atty. Matorre told Atty. Hechanova that since she was not
satisfied with her work and because they were frequently arguing with each other,
it would be best if she resigns from the firm. 10 Atty. Matorre requested that her
resignation be made effective on September 30, 2008, but thinking that the said
date was too far off, Atty. Hechanova accepted the resignation, with the
condition that it be made effective on September 15, 2008.

On September 1, 2008, Atty. Matorre filed a complaint for constructive illegal


dismissal, nonpayment of separation pay, and for payment of moral and
exemplary damages and attorneys’ fees against HBV Law Firm.

On November 13, 2008, during the conciliation conference, upon previous order
of the Labor Arbiter, HBV Law Firm gave Atty. Matorre’s last pay, consultancy fee,
and incentive pay.

Labor Arbiter rendered judgment in favor of HBV Law Firm by dismissing Atty.
Matorre’s complaint for lack of merit. It held that Atty. Matorre voluntarily resigned
from her employment on August 19, 2008, and that Atty. Hechanova readily
accepted Atty. Matorre’s oral resignation. LA cited jurisprudence stating that
"once resignation is accepted, the employee no longer has any right to the job. It,
therefore, goes without saying that resignation terminates the employer-employee
relationship

On May 13, 2010, the NLRC reversed23 the Decision of the Labor Arbiter and
declared that Atty. Matorre was illegally dismissed

On March 14, 2011, the CA upheld the ruling of the NLRC and held that no
voluntary resignation took place.31 It ruled in favor of Atty. Matorre, saying that she
was illegally dismissed in light of the circumstances surrounding the supposed
resignation.

Issue:

Whether or not Atty. Matorre was constructively dismissed by petitioners.

Ruling:

No.

The resignation of Atty. Matorre was voluntary and she was not constructively
dismissed.Atty. Matorre failed to prove that her resignation was not voluntary, and
that Atty. Hechanova and other members of HBV Law Firm committed acts
against her that would constitute constructive dismissal.Atty. Matorre was not able
to prove her allegations of harassment, insults, and verbal abuse on the part of
Atty. Hechanova

The case of Vicente v. Court of Appeals (Former 17th Div.) 36 is instructive on this
matter. In the case at bar and in Vicente, the fact of resignation is not disputed,
but only the voluntariness thereof. In Vicente, the employee alleged that her
employer forced her to resign. The Court held that she voluntarily resigned and
was not constructively dismissed. The Court said, Hence, petitioner cannot take
refuge in the argument that it is the employer who bears the burden of proof that
the resignation is voluntary and not the product of coercion or intimidation.

Having submitted a resignation letter, it is then incumbent upon her to prove that
the resignation was not voluntary but was actually a case of constructive dismissal
with clear, positive, and convincing evidence. Petitioner failed to substantiate her
claim of constructive dismissal.

The Supreme Court agree with the Court of Appeals that it was grave error on the
part of the NLRC to rely on the allegation that Mr. Tecson threatened and forced
petitioner to resign. Other than being unsubstantiated and self-serving, the
allegation does not suffice to support the finding of force, intimidation, and
ultimately constructive dismissal.Bare allegations of constructive dismissal, when
uncorroborated by the evidence on record, cannot be given credence.

The 30-day notice requirement for an employee’s resignation is actually for the
benefit of the employer who has the discretion to waive such period. Its purpose is
to afford the employer enough time to hire another employee if needed and to
see to it that there is proper turn-over of the tasks which the resigning employee
may be handling. As one author42 puts it,

x x x The rule requiring an employee to stay or complete the 30-day period prior to
the effectivity of his resignation becomes discretionary on the part of
management as an employee who intends to resign may be allowed a shorter
period before his resignation becomes effective.

Moreover, the act of HBV Law Firm of moving the effectivity date of Atty.
Matorre’s resignation to an earlier date cannot be seen as a malicious decision on
the part of the firm in order to deprive Atty. Matorre of an opportunity to seek new
employment. This decision cannot be viewed as an act of harassment but rather
merely the exercise of the firm’s management prerogative. Surely, we cannot
expect employers to maintain in their employ employees who intend to resign, just
so the latter can have continuous work as they look for a new source of income.

In line with settled jurisprudence,43 since Atty. Matorre admittedly resigned, it was
incumbent upon her to prove that her resignation was not voluntary, but was
actually a case of constructive dismissal, with clear, positive, and convincing
evidence.

G.R. No. 198534 July 3, 2013

JENNY F. PECKSON, Petitioner,


vs.
ROBINSONS SUPERMARKET CORPORATION, JODY GADIA, ROENA SARTE, and RUBY
ALEX, Respondents
Facts:

The petitioner first joined the Robinsons Supermarket Corporation (RSC) as a Sales
Clerk on November 3, 1987. On October 26, 2006, she was holding the position of
Category Buyer when respondent Roena Sarte , RSCs Assistant Vice-President for
Merchandising, reassigned her to the position of Provincial Coordinator, effective
November 1, 2006. Claiming that her new assignment was a demotion because it
was non-supervisory and clerical in nature, the petitioner refused to turn over her
responsibilities to the new Category Buyer, or to accept her new responsibilities as
Provincial Coordinator. Jody Gadia and Ruby Alexwere impleaded because they
were corporate officers of the RSC.

Sarte demanded an explanation from petitioner for her refusal to accept her new
assignment despite written and verbal demands. Petitioner ignored the demand.
Sarte issued another memorandum reiterating her demand and warning her that
this could be her final chance to present her side or be deemed to have waived
her right to be heard. Petitioner then replied stating that she could not accept the
position of Provincial Coordinator since she saw it as a demotion. Sarte issued an
instruction to petitioner in preparation for the Christmas holidays but the petitioner
refused to heed.

The LA ruled that job reassignment or classification is a strict prerogative of the


employer, and that the petitioner cannot refuse her transfer since both positions
commanded the same salary structure. The LA also ruled that petitioners persistent
refusal to accept her new position amounted to insubordination, entitling RSC to
dismiss her from employment.

A month later, petitioner tendered her written forced resignation. The NLRC
sustained the findings of the LA. The CA sustained the findings of the NLRC.

Issue:

Whether or not Peckson was constructively dismissed.

Ruling:

No.

In Rural Bank of Cantilan, Inc. v. Julve,22 the Court had occasion to summarize the
general jurisprudential guidelines affecting the right of the employer to regulate
employment, including the transfer of its employees:

Under the doctrine of management prerogative, every employer has the inherent
right to regulate, according to his own discretion and judgment, all aspects of
employment, including hiring, work assignments, working methods, the time, place
and manner of work, work supervision, transfer of employees, lay-off of workers,
and discipline, dismissal, and recall of employees. The only limitations to the
exercise of this prerogative are those imposed by labor laws and the principles of
equity and substantial justice.

While the law imposes many obligations upon the employer, nonetheless, it also
protects the employer’s right to expect from its employees not only good
performance, adequate work, and diligence, but also good conduct and loyalty.
In fact, the Labor Code does not excuse employees from complying with valid
company policies and reasonable regulations for their governance and
guidance.

Concerning the transfer of employees, these are the following jurisprudential


guidelines: (a) a transfer is a movement from one position to another of equivalent
rank, level or salary without break in the service or a lateral movement from one
position to another of equivalent rank or salary; (b) the employer has the inherent
right to transfer or reassign an employee for legitimate business purposes; (c) a
transfer becomes unlawful where it is motivated by discrimination or bad faith or is
effected as a form of punishment or is a demotion without sufficient cause; (d) the
employer must be able to show that the transfer is not unreasonable,
inconvenient, or prejudicial to the employee.

As we have already noted, the respondents had the burden of proof that the
transfer of the petitioner was not tantamount to constructive dismissal, which as
defined in Blue Dairy Corporation v. NLRC,27 is a quitting because continued
employment is rendered impossible, unreasonable or unlikely, or an offer involving
a demotion in rank and diminution of pay:

The managerial prerogative to transfer personnel must be exercised without grave


abuse of discretion, bearing in mind the basic elements of justice and fair play.
Having the right should not be confused with the manner in which that right is
exercised. Thus, it cannot be used as a subterfuge by the employer to rid himself
of an undesirable worker. In particular, the employer must be able to show that
the transfer is not unreasonable, inconvenient or prejudicial to the employee; nor
does it involve a demotion in rank or a diminution of his salaries, privileges and
other benefits. Should the employer fail to overcome this burden of proof, the
employee’s transfer shall be tantamount to constructive dismissal, which has been
defined as a quitting because continued employment is rendered impossible,
unreasonable or unlikely; as an offer involving a demotion in rank and diminution
in pay. Likewise, constructive dismissal exists when an act of clear discrimination,
insensibility or disdain by an employer has become so unbearable to the
employee leaving him with no option but to forego with his continued
employment.

As further held in Philippine Japan Active Carbon Corporation,28 when the transfer
of an employee is not unreasonable, or inconvenient, or prejudicial to him, and it
does not involve a demotion in rank or a diminution of his salaries, benefits and
other privileges, the employee may not complain that it amounts to a
constructive dismissal.29

But like all other rights, there are limits to the exercise of managerial prerogative to
transfer personnel, and on the employer is laid the burden to show that the same
is without grave abuse of discretion, bearing in mind the basic elements of justice
and fair play.30 Indeed, management prerogative may not be used as a
subterfuge by the employer to rid himself of an undesirable worker.31

CANDIDO S. GEMINA JR. vs. BANKWISE INC.


FACTS:
On August 9, 2002, petitioner signed an employment contract with respondent
Bankwise, Inc. (Bankwise) as Marketing Officer with the rank of Senior Manager,
with an annual salary of ₱750,000.00 based on a fifteen-month scheme or
₱50,000.00 per month and a service vehicle for his field work. The same contract
stipulated for a fund level commitment of ₱100,000,000.00 for the first six (6)
months of employment. In his Memorandum, Gemina alleged that during his first
three (3) months at work, he had a satisfactory performance and was able to
bring in new and former clients to Bankwise. However, when Bankwise was
embroiled in a controversy involving the deposits of Foreign Retirees Association,
he started to experience difficulty in soliciting new depositors. To alleviate the
situation, he suggested innovations in Bankwise’s marketing strategies to his
immediate superiors, respondents, who then worked out promotional schemes
without his participation. The schemes, however, failed to materialize and he was
blamed for the failure. Thereafter, he was subjected to several forms of
harassment by some officers of Bankwise by forcing him to file an indefinite leave
of absence, demanding for the return of his service vehicle and intentionally
delaying the release of his salaries and allowances. Gemina filed a complaint for
constructive dismissal against Bankwise.
ISSUE:
Whether or not Gemina was constructively dismissed?
Ruling:
The court ruled that there was no constructive dismissal. There is constructive
dismissal when "there is cessation of work, because ‘continued employment is
rendered impossible, unreasonable or unlikely, as an offer involving a demotion in
rank or a diminution in pay’ and other benefits. Aptly called a dismissal in disguise
or an act amounting to dismissal but made to appear as if it were not,
constructive dismissal may, likewise, exist if an act of clear discrimination,
insensibility, or disdain by an employer becomes so unbearable on the part of the
employee that it could foreclose any choice by him except to forego his
continued employment." A close scrutiny of the facts of the case will bear out that
Gemina indeed failed to state circumstances substantiating his claim of
constructive dismissal. To begin with, he does not claim to have suffered a
demotion in rank or diminution in pay or other benefits. What he claims is that he
had been subjected to several acts of harassment by some of the officers of
Bankwise by way of (1) asking him to take a forced leave of absence, (2)
demanding for the return of his service vehicle, and (3) delaying the release of his
salaries and allowances in order to compel him to quit employment. Moreover,
Bankwise was able to address the allegations of harassment hurled against its
officers and offered a plausible justification for its actions. The Court also finds
Bankwise’s order to return the service vehicle assigned to Gemina inadequate to
warrant his claim of constructive dismissal. It bears noting that the service vehicle
was only temporarily assigned for Gemina’s use. Nonetheless, it remains the
property of the Bank and therefore may be disposed of or utilized by the
company in the manner that it deems more beneficial for its interests. This is plainly
an exercise of management prerogative. The employer’s right to conduct the
affairs of its business, according to its own discretion and judgment, is well-
recognized. An employer has a free reign and enjoys wide latitude of discretion to
regulate all aspects of employment and the only criterion to guide the exercise of
its management prerogative is that the policies, rules and regulations on work-
related activities of the employees must always be fair and reasonable.

VERDADERO VS BARNEY AUTOLINES


FACTS:
An altercation took place between Verdadero and respondent Atty. Gerardo
Gimenez (Gimenez), BALGCO’s Disciplinary Officer. Gimenez was on board
BALGCO Bus. No. 55455, together with his wife and four other companions,
travelling from Mulanay to Macalelon, Quezon. Verdadero was then the assigned
bus conductor. BALGCO has a company policy of granting free rides to company
employees and their wives. The story started when Verdadero began issuing fare
tickets to passengers, including the wife of Gimenez. The wife informed Verdadero
who she was6 and the incidents thereafter took two versions as both parties told a
different story. Gimenez filed an unverified complaint for serious misconduct
against Verdadero before the BALGCO Management. He requested Barney D.
Chito(Barney) and Rosela F. Chito (Rosela), owners of BALGCO, to preside over
the conciliation proceedings. Thereafter, Verdadero furtively reported for work for
fear of having another confrontation with Gimenez. Rosela sent Verdadero a
letter, requiring him to immediately report for work and finish the pending
disciplinary proceedings against him. On March 28, 2008, Verdadero submitted his
Letter-Reply, explaining that he had been receiving threats. He likewise believed
he was already illegally dismissed as he was not given any work assignment since
January 28, 2008. Rosela responded to Verdadero's letter and reminded him of
the letter of apology which he was yet to submit as compliance.
On April 15, 2008, however, Verdadero filed a complaint for illegal dismissal before
the Labor Arbiter (LA)Verdado claimed that he was not given any trip assignment
since the January 27, 2008 incident. He argues that when Gimenez committed the
verbal abuse against him in the presence of the bus passengers and threatened
him with physical harm, there was termination by the employee of his
employment under the doctrine of constructive dismissal.
ISSUE:
Whether or not Verdado was constructively dismissed?
RULING:
No. The court ruled that Constructive dismissal exists where there is cessation of
work, because "continued employment is rendered impossible, unreasonable or
unlikely, as an offer involving a demotion in rank or a diminution in pay" and other
benefits. Aptly called a dismissal in disguise or an act amounting to dismissal but
made to appear as if it were not, constructive dismissal may, likewise, exist if an
act of clear discrimination, insensibility, or disdain by an employer becomes so
unbearable on the part of the employee that it could foreclose any choice by
him except to forego his continued employment. In this case, Verdadero cannot
be deemed constructively dismissed. Records do not show any demotion in rank
or a diminution in pay made against him. Neither was there any act of clear
discrimination, insensibility or disdain committed by BALGCO against Verdadero
which would justify or force him to terminate his employment from the company.

LEUS VS ST. SCHOLASTICA’S COLLEGE WESTGROOVE


FACTS:
Cheryll Santos Leus was hired by St. Scholastica’s College Westgrove (SSCW) as an
Assistant to SSCW’s Director of the Lay Apostolate and Community Outreach
Directorate on May 2001. Sometime in 2003, the petitioner and her boyfriend
conceived a child out of wedlock. When SSCW learned of the petitioner’s
pregnancy, Sr. Edna Quiambao (Sr. Quiambao), SSCW’s Directress, advised her to
file a resignation letter effective June 1, 2003. In response, the petitioner informed
Sr. Quiambao that she would not resign from her employment just because she
got pregnant without the benefit of marriage.
On May 28, 2003, Sr. Quiambao formally directed the petitioner to explain in
writing why she should not be dismissed for engaging in pre-marital sexual relations
and getting pregnant as a result thereof, which amounts to serious misconduct
and conduct unbecoming of an employee of a Catholic school.
Cheryll replied stating that her pregnancy outside of wedlock does not amount to
serious misconduct. She thereafter requested a copy of SSCW’s policy so that she
can better respond to the charge against her. SSCW did not a have these
guidelines as the guidelines handbook was currently pending of its promulgation.
It instead stated that they follow the 1992 Manual of Regulations for Private School
(1992 MRPS), specifically, Sec.94, which cites “disgraceful or immoral conduct" as
a ground for dismissal, in addition to the just causes for termination of employment
under Art.282, Labor Code.
The Labor Arbiter in Quezon City decided in favor of SSCW, stating that Cheryll
being pregnant out of wedlock is considered “disgraceful and immoral conduct”
taking into account that she was employed in a Catholic institution which expect
its employees to live up to the Catholic values it teaches to the students. The NLRC
affirmed the decision of the Labor Arbiter.
The question now raised before the Supreme Court is this – is Cheryll’s pregnancy
out of wedlock constitutes a valid ground to terminate her employment?

ISSUE:
Whether or not Leus’ pregnancy out of wedlock constitutes a valid ground to
terminate her employment?
RULING:
The Supreme Court held that Cheryll was illegally dismissed by her employer. Her
pregnancy out of wedlock does not constitute a valid ground to terminate her
employment. Disgraceful conduct is viewed in two ways, the “public and secular
view” and “religious view”. Our laws concern the first view. Disgraceful conduct
per se will not amount to violation of the law – the conduct must affect or poses a
danger to the conditions of society, for example, the sanctity of marriage, right to
privacy and the like.
The Court cited Estrada vs. Escritur in the said case, stating the following relevant
explanation;

1. If the father of the child is himself unmarried, the woman is not ordinarily
administratively liable for disgraceful and immoral conduct. It may be a not-
so-ideal situation and may cause complications for both mother and child
but it does not give cause for administrative sanction. There is no law which
penalizes an unmarried mother under those circumstances by reason of her
sexual conduct or proscribes the consensual sexual activity between two
unmarried persons. Neither does the situation contravene any fundamental
state policy as expressed in the Constitution, a document that
accommodates various belief systems irrespective of dogmatic origins.

2. If the father of the child born out of wedlock is himself married to a


woman other than the mother, then there is a cause for administrative
sanction against either the father or the mother. In such a case, the
“disgraceful and immoral conduct” consists of having extramarital relations
with a married person. The sanctity of marriage is constitutionally recognized
and likewise affirmed by our statutes as a special contract of permanent
union. Accordingly, judicial employees have been sanctioned for their
dalliances with married persons or for their own betrayals of the marital vow
of fidelity. In this case, it was not disputed that, like respondent, the father of
her child was unmarried. Therefore, respondent cannot be held liable for
disgraceful and immoral conduct simply because she gave birth to the
child Christian Jeon out of wedlock.
Furthermore, there was no substantial evidence to prove that Cheryll’s pregnancy
out of wedlock caused grave scandal to SSCW and its students. Mere allegation
of such will not render a judgment in favor of the one making the allegation. It is
the burden of the employer to prove by substantial evidence that the termination
of the employment of the employee was made and failure to discharge that duty
would mean that the dismissal is not justified and therefore illegal.
The Court ordered SSCW to reinstate Cheryll. But because this is not possible
anymore due to constrained relations with SSCW, the Court ordered the employer
to pay Cheryll separation pay, full backwages and attorney’s fees.

JENNY F. PECKSON, Petitioner, v. ROBINSONS SUPERMARKET CORPORATION, JODY


GADIA, ROENA SARTE, and RUBY ALEX, Respondents.

FACTS:
The petitioner first joined the Robinsons Supermarket Corporation (RSC) as a Sales
Clerk on November 3, 1987. On October 26, 2006, she was holding the position of
Category Buyer when respondent Roena Sarte (Sarte), RSCs Assistant Vice-
President for Merchandising, reassigned her to the position of Provincial
Coordinator, effective November 1, 2006. Claiming that her new assignment was
a demotion because it was non-supervisory and clerical in nature, the petitioner
refused to turn over her responsibilities to the new Category Buyer, or to accept
her new responsibilities as Provincial Coordinator. Jody Gadia (Gadia) and Ruby
Alex (Alex) were impleaded because they were corporate officers of the RSC.

Sarte demanded an explanation from petitioner for her refusal to accept her new
assignment despite written and verbal demands. Petitioner ignored the demand.
Sarte issued another memorandum reiterating her demand and warning her that
this could be her final chance to present her side or be deemed to have waived
her right to be heard. Petitioner then replied stating that she could not accept the
position of Provincial Coordinator since she saw it as a demotion. Sarte issued an
instruction to petitioner in preparation for the Christmas holidays but the petitioner
refused to heed.

The LA ruled that job reassignment or classification is a strict prerogative of the


employer, and that the petitioner cannot refuse her transfer since both positions
commanded the same salary structure. The LA also ruled that petitioners persistent
refusal to accept her new position amounted to insubordination, entitling RSC to
dismiss her from employment.

A month later, petitioner tendered her written forced resignation. The NLRC
sustained the findings of the LA. The CA sustained the findings of the NLRC.

ISSUE:
Was petitioner's transfer a demotion?

HELD:
In Philippine Japan Active Carbon Corporation v. NLRC, held that the exercise of
managements prerogative concerning the employee's work assignments is based
on its assessment of the qualifications, aptitudes and competence of its
employees, and by moving them around in the various areas of its business
operations it can ascertain where they will function with maximum benefit to the
company.

Under the doctrine of management prerogative, every employer has the inherent
right to regulate, according to his own discretion and judgment, all aspects of
employment, including hiring, work assignments, working methods, the time, place
and manner of work, work supervision, transfer of employees, lay-off of workers,
and discipline, dismissal, and recall of employees. The only limitations to the
exercise of this prerogative are those imposed by labor laws and the principles of
equity and substantial justice.

Concerning the transfer of employees, these are the following jurisprudential


guidelines: (a) a transfer is a movement from one position to another of equivalent
rank, level or salary without break in the service or a lateral movement from one
position to another of equivalent rank or salary; (b) the employer has the inherent
right to transfer or reassign an employee for legitimate business purposes; (c) a
transfer becomes unlawful where it is motivated by discrimination or bad faith or is
effected as a form of punishment or is a demotion without sufficient cause; (d) the
employer must be able to show that the transfer is not unreasonable,
inconvenient, or prejudicial to the employee. (Rural Bank of Cantilan, Inc. v. Julve)

As a privilege inherent in the employers right to control and manage its enterprise
effectively, its freedom to conduct its business operations to achieve its purpose
cannot be denied. We agree with the appellate court that the respondents are
justified in moving the petitioner to another equivalent position, which presumably
would be less affected by her habitual tardiness or inconsistent attendance than if
she continued as a Category Buyer, a frontline position in the day-to-day business
operations of a supermarket such as Robinsons.

In Philippine Japan Active Carbon Corporation, when the transfer of an employee


is not unreasonable, or inconvenient, or prejudicial to him, and it does not involve
a demotion in rank or a diminution of his salaries, benefits and other privileges, the
employee may not complain that it amounts to a constructive dismissal. DENIED

JONATHAN V. MORALES, Petitioner,


vs.
HARBOUR CENTRE PORT TERMINAL, INC. Respondent.

FACTS:
On 16 May 2000, petitioner Jonathan V. Morales (Morales) was hired by
respondent Harbour Centre Port Terminal, Inc. (HCPTI) as an Accountant and
Acting Finance Officer with a monthly salary of P18,000.00. Regularized on 17
November 2000, Morales was promoted to Division Manager of the Accounting
Department, for which he was compensated a monthly salary of P33,700.00, plus
allowances starting 1 July 2002.

Subsequent to HCPTI’s transfer to its new offices at Vitas, Tondo, Manila on 2


January 2003, Morales received an inter-office memorandum dated 27 March
2003, reassigning him to Operations Cost Accounting, tasked with the duty of
“monitoring and evaluating all consumables requests, gears and equipment”
related to the corporation’s operations and of interacting with its sub-contractor,
Bulk Fleet Marine Corporation.

Morales wrote Singson (admin manager), protesting that his reassignment was a
clear demotion since the position to which he was transferred was not even
included in HCPTI’s plantilla. For the whole of the ensuing month Morales was
absent from work and/or tardy. Singson issued to Morales a 29 April 2003 inter-
office memorandum denominated as a First Warning.
In view of the absences Morales continued to incur, HCPTI issued a Second
Warning dated 6 May 2003 and a Notice to Report for Work and Final Warning
dated 22 May 2003.

LABOR ARBITER: Morales was not constructively dismissed

NLRC: Morales’ reassignment was a clear demotion despite lack of showing of


diminution of salaries and benefits.

CA rendered the herein assailed decision, reversing the NLRC’s 29 July 2005
Decision, upon the following findings and conclusions: (a) Morales’ reassignment
to Operations Cost Accounting was a valid exercise of HCPTI’s prerogative to
transfer its employees as the exigencies of the business may require; (b) the
transfer cannot be construed as constructive dismissal since it entailed no
demotion in rank, salaries and benefits; and, (c) rather than being terminated,
Morales refused his new assignment by taking a leave of absence from 4 to 17
April 2003 and disregarding HCPTI’s warnings and directives to report back for
work.

ISSUE:
WON Morales was constructively dismissed

HELD:
YES.

Constructive dismissal exists where there is cessation of work because “continued


employment is rendered impossible, unreasonable or unlikely, as an offer involving
a demotion in rank or a diminution in pay” and other benefits. Aptly called a
dismissal in disguise or an act amounting to dismissal but made to appear as if it
were not, constructive dismissal may, likewise, exist if an act of clear discrimination,
insensibility, or disdain by an employer becomes so unbearable on the part of the
employee that it could foreclose any choice by him except to forego his
continued employment. In cases of a transfer of an employee, the rule is settled
that the employer is charged with the burden of proving that its conduct and
action are for valid and legitimate grounds such as genuine business necessity
and that the transfer is not unreasonable, inconvenient or prejudicial to the
employee. If the employer cannot overcome this burden of proof, the employee’s
transfer shall be tantamount to unlawful constructive dismissal.

Morales was subsequently reassigned by HCPTI “from managerial accounting to


Operations Cost Accounting” on 27 March 2003, without any mention of the
position to which he was actually being transferred. That the reassignment was a
demotion is, however, evident from Morales’ new duties which, far from being
managerial in nature, were very simply and vaguely described as inclusive of
“monitoring and evaluating all consumables requests, gears and equipments
related to [HCPTI’s] operations” as well as “close interaction with [its] sub-
contractor Bulk Fleet Marine Corporation.”
Morales’ demotion is evident from the fact that his reassignment entailed a
transfer from a managerial position to one which was not even included in the
corporation’s plantilla.

BEST WEAR GARMENTS and/or WARREN PARDILLA, Petitioners, v. ADELAIDA B. DE


LEMOS and CECILE M. OCUBILLO, Respondents.

FACTS:
Respondents Adelaida De Lemos and Cecile Ocubillo were employees of Best
Wear Garments (Best Wear) owned by Warren Pardilla. In 2004, De Lemos and
Ocubillo filed a case for illegal dismissal. Both alleged that they were arbitrarily
transferred to other areas of operation of Pardilla’s garments company, which
they said amounted to constructive dismissal as it resulted in less earnings for
them. They also claimed that the reason for their transfer is their refusal to render
overtime work until 7:00 p.m.

Best wear countered that De Lemos and Ocubillo are piece-rate workers and
hence they are not paid according to the number of hours worked. Best Wear
also averred that the two were not illegally terminated; rather, they were the ones
who resigned.

The Labor Arbiter ruled that De Lemos and Ocubillo were constructively dismissed
from employment. On appeal, the NLRC found no basis for the charge of
constructive dismissal. Aggrieved, De Lemos and Ocubillo appealed to the Court
of Appeals. The CA reinstated the LA’s decision. Hence, this instant petition.

ISSUE:
Whether or not the Court of Appeals erred in ruling that De Lemos and Ocubillo
were constructively dismissed?

HELD:
De Lemos and Ocubillo were not constructively dismissed.

LABOR LAW: transfer; management prerogative; piece-rate workers

The right of employees to security of tenure does not give them vested rights to
their positions to the extent of depriving management of its prerogative to
change their assignments or to transfer them. Thus, an employer may transfer or
assign employees from one office or area of operation to another, provided there
is no demotion in rank or diminution of salary, benefits, and other privileges, and
the action is not motivated by discrimination, made in bad faith, or effected as a
form of punishment or demotion without sufficient cause.

Being piece-rate workers assigned to individual sewing machines, their earnings


depended on the quality and quantity of finished products. That their work output
might have been affected by the change in their specific work assignments does
not necessarily imply that any resulting reduction in pay is tantamount to
constructive dismissal. Workers under piece-rate employment have no fixed
salaries and their compensation is computed on the basis of accomplished tasks.
The constitutional policy of providing full protection to labor is not intended to
oppress or destroy management. While the Constitution is committed to the policy
of social justice and the protection of the working class, it should not be supposed
that every labor dispute will be automatically decided in favor of labor.
Management also has its rights which are entitled to respect and enforcement in
the interest of simple fair play. Thus, where management prerogative to transfer
employees is validly exercised, as in this case, courts will decline to interfere.

Petition is GRANTED.

ARMANDO ALILING,
petitioner
,
vs.
JOSE B. FELICIANO, MANUEL F. SAN MATEOIII, JOSEPH R. LARIOSA, and WIDE WIDE
WORLD EXPRESS CORPORATION,
respondents.

Nature of the Case:


This Petition for Review on Certiorari under Rule 45 assails andseeks to set aside the
July 3, 2008 Decision and December 15, 2008 Resolution of the Court of Appeals
(CA), in CA-G.R. SP No. 101309, entitled Armando Aliling v. National Labor
Relations Commission, Wide Wide World Express Corporation, Jose B.
Feliciano,Manuel F. San Mateo III and Joseph R. Lariosa.

The assailed issuances modified theResolutions dated May 31, 2007 3and August
31, 2007 rendered by the National Labor Relations Commission (NLRC) in NLRC
NCR Case No. 00-10-11166-2004, affirmingthe Decision dated April 25, 2006 of the
Labor Arbiter.

Facts:
Respondent Wide Wide World Express Corporation (WWWEC) offered to employ
petitioner Armando Aliling (Aliling) on June 2, 2004 as “Account Executive
(Seafreight Sales),” with a compensation package of a monthly salary of PhP
13,000, transportationallowance of PhP 3,000, clothing allowance of PhP 800, cost
of living allowance of PhP500, each payable on a per month basis and a 14th
month pay depending on theprofitability and availability of financial resources of
the company. The offer came with asix (6)-month probation period condition with
this express caveat: “Performance during probationary period shall be made as
basis for confirmation to Regular or PermanentStatus.”

On June 11, 2004, Aliling and WWWEC inked an Employment Contract under
theterms of conversion to regular status shall be determined on the basis of
workperformance; and employment services may, at any time, be terminated for
just causeor in accordance with the standards defined at the time of
engagement.

However, instead of a Seafreight Sale assignment, WWWEC asked Aliling tohandle


Ground Express (GX), a new company product launched on June 18,
2004involving domestic cargo forwarding service for Luzon. Marketing this product
and finding daily contracts for it formed the core of Aliling’s new assignment.

A month after, Manuel F. San Mateo III (San Mateo), WWWEC Sales and Marketing
Director, emailed Aliling to express dissatisfaction with the latter’s performance.On
September 25, 2004, Joseph R. Lariosa (Lariosa), Human ResourcesManager of
WWWEC, asked Aliling to report to the Human Resources Department toexplain his
absence taken without leave from September 20, 2004. Aliling responded two
days later. He denied being absent on the days inquestion, attaching to his reply-
letter a copy of his timesheet which showed that he worked from September 20 to
24, 2004. Aliling’s explanation came with a query regarding the withholding of his
salary corresponding to September 11 to 25, 2004.On October 15, 2004, Aliling
tendered his resignation to San Mateo. WhileWWWEC took no action on his
tender, Aliling nonetheless demanded reinstatement anda written apology,
claiming in a subsequent letter dated October 1, 2004 tomanagement that San
Mateo had forced him to resign.

Lariosa’s response-letter of October 1, 2004, informed Aliling that his case was still
in the process of being evaluated. On October 6, 2004, Lariosa again wrote,this
time to advise Aliling of the termination of his services effective as of that date
owing to his “non-satisfactory performance” during his probationary period.
Records show that Aliling, for the period indicated, was paid his outstanding
salary.However, or on October 4, 2004, Aliling filed a Complaint for illegal dismissal
dueto forced resignation, nonpayment of salaries as well as damages with the
NLRC against WWWEC. Appended to the complaint was Aliling’s Affidavit dated
November 12, 2004,in which he stated: “At the time of my engagement,
respondents did not make known to me the standards under which I will qualify as
a regular employee.”

Refuting Aliling’s basic posture, WWWEC stated that in the letter offer and
employment contract adverted to, WWWEC and Aliling have signed a letter of
appointment on June 11, 2004 containing the terms of engagement.WWWEC also
attached to its Position Paper a memo dated September 20,2004 in which San
Mateo asked Aliling to explain why he should not be terminated for failure to meet
the expected job performance, considering that the load factor for the
GXShuttles for the period July to September was only 0.18% as opposed to the
allegedlyagreed upon load of 80% targeted for August 5, 2004. According to
WWWEC, Aliling,instead of explaining himself, simply submitted a resignation
letter.On April 25, 2006, the Labor Arbiter issued a decision declaring that the
grounds upon which complainant’s dismissal was based did not conform not only
the standardbut also the compliance required under Article 281 of the Labor
Code, Necessarily, complainant’s termination is not justified for failure to comply
with the mandate the law requires. Respondents should be ordered to pay salaries
corresponding to theunexpired portion of the contract of employment and all
other benefits amounting to atotal of P35,811.00 covering the period from
October 6 to December 7, 2004.The Labor Arbiter explained that Aliling cannot be
validly terminated for non-compliance with thw quota threshold absent a prior
advisory of the reasonablestandards upon which his performance would be
evaluated.Both parties appealed the decision to the NLRC, which affirmed the
decision of the Labor Arbiter. The separate motions for reconsideration were also
denied by theNLRC.The CA anchored its assailed action on the strength of the
following premises:
(a) respondents failed to prove that Aliling’s dismal performance constituted gross
and
habitual neglect necessary to justify his dismissal;
(b) not having been informed at thetime of his engagement of the reasonable
standards under which he will qualify as a regular employee, Aliling was deemed
to have been hired from day one as a regular employee; and
(c) the strained relationship existing between the parties argues againstthe
propriety of reinstatement.Hence, the instant petition.

Issue:
What is the effect once a decision was assailed for appeal?

Held:
It is axiomatic that an appeal, once accepted by this Court, throws the entire
caseopen to review, and that this Court has the authority to review matters not
specificallyraised or assigned as error by the parties, if their consideration is
necessary in arrivingat a just resolution of the case.

Settled is the rule that the findings of the Labor Arbiter, when affirmed by theNLRC
and the Court of Appeals, are binding on the Supreme Court, unless
patentlyerroneous. It is not the function of the Supreme Court to analyze or weigh
all over againthe evidence already considered in the proceedings below. The
jurisdiction of this Courtin a petition for review on
certiorari is limited to reviewing only errors of law, not of fact,unless the factual
findings being assailed are not supported by evidence on record or the impugned
judgment is based on a misapprehension of facts. The more recent Peñafrancia
Tours and Travel Transport, Inc., v. Sarmiento , 634 SCRA 279(2010), has reaffirmed
the above ruling, to wit: Finally, the CA affirmed the ruling of the NLRC and
adopted as its own the latter’s factual findings. Long-established is thedoctrine
that findings of fact of quasi-judicial bodies are accorded respect, even finality,if
supported by substantial evidence. When passed upon and upheld by the CA,
theyare binding and conclusive upon this Court and will not normally be
disturbed. Thoughthis doctrine is not without exceptions, the Court finds that none
are applicable to thepresent case.

SAMEER OVERSEAS PLACEMENT AGENCY, INC., vs. JOY C. CABILES, G.R. No.
170139, August 5, 2014

FACTS:

Petitioner, Sameer Overseas Placement Agency, Inc., is a recruitment and


placement agency.

Respondent Joy Cabiles was hired thus signed a one-year employment contract
for a monthly salary of NT$15,360.00. Joy was deployed to work for Taiwan
Wacoal, Co. Ltd. (Wacoal) on June 26, 1997. She alleged that in her
employment contract, she agreed to work as quality control for one year. In
Taiwan, she was asked to work as a cutter.

Sameer claims that on July 14, 1997, a certain Mr. Huwang from Wacoal
informed Joy, without prior notice, that she was terminated and that “she should
immediately report to their office to get her salary and passport.” She was asked
to “prepare for immediate repatriation.” Joy claims
that she was told that from June 26 to July 14, 1997, she only earned a total of
NT$9,000.15 According to her, Wacoal deducted NT$3,000 to cover her plane
ticket to Manila.

On October 15, 1997, Joy filed a complaint for illegal dismissal with the NLRC
against petitioner and Wacoal. LA dismissed the complaint. NLRC reversed
LA’s decision. CA affirmed the ruling of the National Labor Relations
Commission finding respondent illegally dismissed and awarding her three
months’ worth of salary, the reimbursement of the cost of her repatriation, and
attorney’s fees

ISSUE:

Whether or not Cabiles was entitled to the unexpired portion of her salary due
to illegal dismissal.

HELD:

YES. The Court held that the award of the three-month equivalent of
respondent’s salary should
be increased to the amount equivalent to the unexpired term of the
employment contract.

In Serrano v. Gallant Maritime Services, Inc. and Marlow Navigation Co., Inc.,
this court ruled that the clause “or for three (3) months for every year of the
unexpired term, whichever is less” is unconstitutional for violating the equal
protection clause and substantive due process.

A statute or provision which was declared unconstitutional is not a law. It


“confers no rights; it imposes no duties; it affords no protection; it creates
no office; it is inoperative as if it has not been passed at all.”

The Court said that they are aware that the clause “or for three (3) months for
every year of the unexpired term, whichever is less” was reinstated in Republic
Act No. 8042 upon promulgation of Republic Act No. 10022 in 2010.

Ruling on the constitutional issue

In the hierarchy of laws, the Constitution is supreme. No branch or office of the


government may exercise its powers in any manner inconsistent with the
Constitution, regardless of the existence of any law that supports such exercise.
The Constitution cannot be trumped by any other law. All laws must be read in
light of the Constitution. Any law that is inconsistent with it is a nullity.
Thus, when a law or a provision of law is null because it is inconsistent with the
Constitution, the nullity cannot be cured by reincorporation or reenactment of
the same or a similar law or provision.

A law or provision of law that was already declared unconstitutional remains


as such unless circumstances have so changed as to warrant a reverse
conclusion.

The Court observed that the reinstated clause, this time as provided in
Republic Act. No. 10022, violates the constitutional rights to equal protection
and due process.96 Petitioner as well as the Solicitor General have failed to
show any compelling change in the circumstances that would warrant us to
revisit the precedent.

The Court declared, once again, the clause, “or for three (3) months for every
year of the unexpired term, whichever is less” in Section 7 of Republic Act No.
10022 amending Section 10 of Republic Act No. 8042 is declared
unconstitutional and, therefore, null and void.

ETPI VS. ETEU


G.R. No. 185665

FACTS:
Eastern Telecommunications Phils., Inc. (ETPI) is a corporation engaged in the
business of providing telecommunications facilities. Eastern Telecoms Employees
Union (ETEU) is the certified exclusive bargaining agent of the company’s rank
and file employees. It has an existing CBA with the company to expire in the
year 2004 with a Side Agreement signed on September 3, 2001.In essence, the
labor dispute was a spin-off of the company’s plan to defer payment of the
2003 14th, 15th and 16th month bonuses sometime in April 2004. The company’s
main ground in postponing the payment of bonuses is due to allege continuing
deterioration of company’s financial position which started in the year 2000.
However, ETPI while postponing payment of bonuses sometime in April 2004,
such payment would also be subject to availability of funds.Invoking the Side
Agreement of the existing CBA for the period 2001-2004 between ETPI and ETEU,
the union strongly opposed the deferment in payment of the bonuses by filing a
preventive mediation complaint with the NCMB.Later, the company made a
sudden turnaround in its position by declaring that they will no longer pay the
bonuses until the issue is resolved through compulsory arbitration.Thus ETEU filed
a Notice of Strike on the ground of unfair labor practice for failure of ETPI to pay
the bonuses in gross violation of the economic provision of the existing CBA.ETPI
insists that it is under no legal compulsion to pay 14th, 15th and 16th month
bonuses for the year 2003 and 14th month bonus for the year 2004 contending
that they are not part of the demandable wage or salary and that their grant
is conditional based on successful business performance and the availability of
company profits from which to source the same. To thwart ETEU’s monetary
claims, it insists that the distribution of the subject bonuses falls well within
the company’s prerogative, being an act of pure gratuity and generosity on its
part. Thus, it can withhold the grant thereof especially since it is currently
plagued with economic difficulties and financial losses.ETPI further avers that the
act of giving the subject bonuses did not ripen into a company practice arguing
that it has always been a contingent one dependent on the realization of profits
and, hence, the workers are not entitled to bonuses if the company does not
make profits for a given year. It asseverates that the 1998 and 2001 CBA Side
Agreements did not contractually afford ETEU a vested property right to a
perennial payment of the bonuses. It opines that the bonus provision in the Side
Agreement allows the giving of benefits only at the time of its execution. For this
reason, it cannot be said that the grant has ripened into a company practice.

ISSUE: Is ETPI is liable to pay 14th, 15th and 16th month bonuses for the year 2003
and 14th month bonus for the year 2004 to the members of respondent union?
HELD:
From a legal point of view, a bonus is a gratuity or act of liberality of the giver
which the recipient has no right to demand as a matter of right. The grant of a
bonus is basically a management prerogative which cannot be forced upon
the employer who may not be obliged to assume the onerous burden of
granting bonuses or other benefits aside from the employee’s basic salaries or
wages.A bonus, however, becomes a demandable or enforceable obligation
when it is made part of the wage or salary or compensation of the employee.

Particularly instructive is the ruling of the Court in Metro Transit Organization, Inc.
v. NLRC, where it was written:
Whether or not a bonus forms part of wages depends upon the circumstances
and conditions for its payment. If it is additional compensation which the
employer promised and agreed to give without any conditions imposed for its
payment, such as success of business or greater production or output, then it
is part of the wage. But if it is paid only if profits are realized or if a certain level of
productivity is achieved, it cannot be considered part of the wage. Where it is
not payable to all but only to some employees and only when their labor
becomes more efficient or more productive, it is only an inducement for
efficiency, a prize therefore, not a part of the wage.

In the case at bench, it is indubitable that ETPI and ETEU agreed on the inclusion
of a provision for the grant of 14th, 15th and 16th month bonuses in the 1998-
2001 CBA Side Agreement, as well as in the 2001-2004 CBA Side Agreement,
which was signed on September 3, 2001.

The provision, which was similarly worded, states:

Employment-Related Bonuses
The Company confirms that the 14th, 15th and 16th month bonuses (other than
the 13th month pay) are granted.

A reading of the above provision reveals that the same provides for the giving of
14th, 15th and 16th month bonuses without qualification. The wording of the
provision does not allow any other interpretation. There were no conditions
specified in the CBA Side Agreements for the grant of the benefits contrary to
the claim of ETPI that the same is justified only when there are profits earned by
the company. Terse and clear, the said provision does not state that the subject
bonuses shall be made to depend on the ETPI’s financial standing or that their
payment was contingent upon the realization of profits. Neither does it state that
if the company derives no profits, no bonuses are to be given to the
employees. In fine, the payment of these bonuses was not related to the
profitability of business operations.

The records are also bereft of any showing that the ETPI made it clear before or
during the execution of the Side Agreements that the bonuses shall be subject
to any condition. Indeed, if ETPI and ETEU intended that the subject bonuses
would be dependent on the company earnings, such intention should have
been expressly declared in the Side Agreements or the bonus provision should
have been deleted altogether. Verily, by virtue of its incorporation in the CBA
Side Agreements, the grant of 14th, 15th and 16th month bonuses has become
more than just an act of generosity on the part of ETPI but a contractual
obligation it has undertaken. Moreover, the continuous conferment of bonuses
by ETPI to the union members from 1998 to 2002 by virtue of the Side
Agreements evidently negates its argument that the giving of the subject
bonuses is a management prerogative.

Granting arguendo that the CBA Side Agreement does not contractually bind
petitioner ETPI to give the subject bonuses, nevertheless, the Court finds that its
act of granting the same has become an established company practice such
that it has virtually become part of the employees’ salary or wage. A bonus may
be granted on equitable consideration when the giving of such bonus has been
the company’s long and regular practice.

In Philippine Appliance Corporation v. CA, it was pronounced:


To be considered a “regular practice,” however, the giving of the bonus should
have been done over a long period of time, and must be shown to have been
consistent and deliberate. The test or rationale of this rule on long practice
requires an indubitable showing that the employer agreed to continue giving
the benefits knowing fully well that said employees are not covered by the law
requiring payment thereof.

The records show that ETPI, aside from complying with the regular 13th month
bonus, has been further giving its employees 14th month bonus every April as
well as 15th and 16th month bonuses every December of the year, without fail,
from 1975 to 2002 or for 27 years whether it earned profits or not. The
considerable length of time ETPI has been giving the special grants to its
employees indicates a unilateral and voluntary act on its part to continue giving
said benefits knowing that such act was not required by law. Accordingly, a
company practice in favor of the employees has been established and the
payments made by ETPI pursuant thereto ripened into benefits enjoyed by the
employees.

The rule is settled that any benefit and supplement being enjoyed by the
employees cannot be reduced, diminished, discontinued or eliminated by the
employer. The principle of non-diminution of benefits is founded on the
constitutional mandate to protect the rights of workers and to promote their
welfare and to afford labor full protection.

MANILA JOCKEY CLUB EMPLOYEES LABOR UNION-PTGWO vs. MANILA JOCKEY


CLUB, INC.
FACTS:
Petitioner Manila Jockey Club Employees Labor Union-PTGWO and respondent
Manila Jockey Club, Inc., a corporation with a legislative franchise to conduct,
operate and maintain horse races, entered into a Collective Bargaining
Agreement (CBA) effective January 1, 1996 to December 31, 2000. The CBA
governed the economic rights and obligations of respondent’s regular monthly
paid rank-and-file employees.3 In the CBA, the parties agreed to a 7-hour work
schedule from 9:00 a.m. to 12:00 noon and from 1:00 p.m. to 5:00 p.m. on a work
week of Monday to Saturday, as contained under Section 1, Article IV,4 of the
same CBA. The CBA likewise reserved in respondent certain management
prerogatives, including the determination of the work schedule. On April 3, 1999,
respondent issued an inter-office memorandum declaring that, effective April
20, 1999, the hours of work of regular monthly-paid employees shall be from 1:00
p.m. to 8:00 p.m. when horse races are held, that is, every Tuesday and
Thursday. The memorandum, however, maintained the 9:00 a.m. to 5:00 p.m.
schedule for non-race days. Petitioner and respondent entered into an
Amended and Supplemental CBA retaining Section 1 of Article IV and Section 2
of Article XI, supra, and clarified that any conflict arising therefrom shall be
referred to a voluntary arbitrator for resolution.
Subsequently, before a panel of voluntary arbitrators of the NCMB, petitioner
questioned the above office memorandum as violative of the prohibition
against non-diminution of wages and benefits guaranteed under Section 1,
Article IV, of the CBA which specified the work schedule of respondent's
employees to be from 9:00 a.m. to 5:00 p.m. Petitioner claimed that as a result of
the memorandum, the employees are precluded from rendering their usual
overtime work from 5:00 p.m. to 9:00 p.m.
ISSUE:
Whether or not it was a management prerogative to change the working hours
of the petitioner?
RULING:
Yes. The court ruled that it was avalid Exercise of management prerogative
When the races were moved to 2:00 p.m., there was no other choice for
management but to change the employees' work schedule as there was no
work to be done in the morning. Evidently, the adjustment in the work schedule
of the employees is justified. While the CBA provided for a schedule, it also
reserved expressly to management the right to change existing methods or
facilities to change the schedules of work. The CBA also grants respondent the
prerogative to relieve employees from duty because of lack of work. No
diminution of benefits The CBA does not guarantee overtime work for all the
employees but merely provides that "all work performed in excess of seven (7)
hours work schedule and on days not included within the work week shall be
considered overtime and paid as such." Respondent was not obliged to allow all
its employees to render overtime work everyday for the whole year, but only
those employees whose services were needed after their regular working hours
and only upon the instructions of management. The overtime pay was not given
to each employee consistently, deliberately and unconditionally, but as a
compensation for additional services rendered. Thus, overtime pay does not fall
within the definition of benefits under Article 100 of the Labor Code on
prohibition against elimination or diminution of benefits

PHIL. TELEGRAPH VS NLRC, G.R. no. 118978, MAY 23, 1997

FACTS:Seeking relief through the extraordinary writ of certiorari, petitioner


Philippine Telegraph and Telephone Company (hereafter, PT & T) invokes the
alleged concealment of civil status and defalcation of company funds as
grounds to terminate the services of an employee. That employee, herein
private respondent Grace de Guzman, contrarily argues that what really
motivated PT & T to terminate her services was her having contracted marriage
during her employment, which is prohibited by petitioner in its company policies.
She thus claims that she was discriminated against in gross violation of law, such
a proscription by an employer being outlawed by Article 136 of the Labor Code.

Grace de Guzman was initially hired by petitioner as a reliever for a fixed period
from November 21, 1990 until April 20, 1991 vice one C.F. Tenorio who went on
maternity leave.1 Under the Reliever Agreement which she signed with petitioner
company, her employment was to be immediately terminated upon expiration
of the agreed period.On September 2, 1991, private respondent was once more
asked to join petitioner company as a probationary employee, the probationary
period to cover 150 days. In the job application form that was furnished her to
be filled up for the purpose, she indicated in the portion for civil status therein
that she was single although she had contracted marriage a few months earlier,
that is, on May 26, 1991.3

It now appears that private respondent had made the same representation in
the two successive reliever agreements which she signed on June 10, 1991 and
July 8, 1991. When petitioner supposedly learned about the same later, its
branch supervisor in Baguio City, Delia M. Oficial, sent to private respondent a
memorandum dated January 15, 1992 requiring her to explain the discrepancy.
In that memorandum, she was reminded about the company's policy of not
accepting married women for employment.4
In her reply letter dated January 17, 1992, private respondent stated that she
was not aware of PT&T's policy regarding married women at the time, and that
all along she had not deliberately hidden her true civil status. 5 Petitioner
nonetheless remained unconvinced by her explanations. Private respondent
was dismissed from the company effective January 29, 1992,6 which she readily
contested by initiating a complaint for illegal dismissal, coupled with a claim for
non-payment of cost of living allowances (COLA), before the Regional
Arbitration Branch of the National Labor Relations Commission in Baguio City.

On November 23, 1993, Labor Arbiter Irenarco R. Rimando handed down a


decision declaring that private respondent, who had already gained the status
of a regular employee, was illegally dismissed by petitioner.On appeal to the
National Labor Relations Commission (NLRC), said public respondent upheld the
labor arbiter and, in its decision dated April 29, 1994, it ruled that private
respondent had indeed been the subject of an unjust and unlawful
discrimination by her employer, PT & T. However, the decision of the labor arbiter
was modified with the qualification that Grace de Guzman deserved to be
suspended for three months in view of the dishonest nature of her acts which
should not be condoned. In all other respects, the NLRC affirmed the decision of
the labor arbiter, including the order for the reinstatement of private respondent
in her employment with PT & T.

ISSUE: Whether or not the post-employment ban of employees being married is


a valid management prerogative

RULING: No

The government, to repeat, abhors any stipulation or policy in the nature of that
adopted by petitioner PT & T. The Labor Code state, in no uncertain terms, as
follows:

Art. 136. Stipulation against marriage. — It shall be unlawful for an employer to


require as a condition of employment or continuation of employment that a
woman shall not get married, or to stipulate expressly or tacitly that upon getting
married, a woman employee shall be deemed resigned or separated, or to
actually dismiss, discharge, discriminate or otherwise prejudice a woman
employee merely by reason of marriage.

Article 136 is not intended to apply only to women employed in ordinary


occupations, or it should have categorically expressed so. The sweeping
intendment of the law, be it on special or ordinary occupations, is reflected in
the whole text and supported by Article 135 that speaks of non-discrimination on
the employment of women.

The judgment of the Court of Appeals in Gualberto, et al. vs. Marinduque Mining
& Industrial Corporation 34 considered as void a policy of the same nature. In
said case, respondent, in dismissing from the service the complainant, invoked a
policy of the firm to consider female employees in the project it was undertaking
as separated the moment they get married due to lack of facilities for married
women. Respondent further claimed that complainant was employed in the
project with an oral understanding that her services would be terminated when
she gets married. Branding the policy of the employer as an example of
"discriminatory chauvinism" tantamount to denying equal employment
opportunities to women simply on account of their sex, the appellate court
struck down said employer policy as unlawful in view of its repugnance to the
Civil Code, Presidential Decree No. 148 and the Constitution.

Under American jurisprudence, job requirements which establish employer


preference or conditions relating to the marital status of an employee are
categorized as a "sex-plus" discrimination where it is imposed on one sex and not
on the other. Further, the same should be evenly applied and must not inflict
adverse effects on a racial or sexual group which is protected by federal job
discrimination laws. Employment rules that forbid or restrict the employment of
married women, but do not apply to married men, have been held to violate
Title VII of the United States Civil Rights Act of 1964, the main federal statute
prohibiting job discrimination against employees and applicants on the basis of,
among other things, sex. 35

Further, it is not relevant that the rule is not directed against all women but just
against married women. And, where the employer discriminates against married
women, but not against married men, the variable is sex and the discrimination
is unlawful. 36 Upon the other hand, a requirement that a woman employee
must remain unmarried could be justified as a "bona fide occupational
qualification," or BFOQ, where the particular requirements of the job would justify
the same, but not on the ground of a general principle, such as the desirability
of spreading work in the workplace. A requirement of that nature would be valid
provided it reflects an inherent quality reasonably necessary for satisfactory job
performance. Thus, in one case, a no-marriage rule applicable to both male
and female flight attendants, was regarded as unlawful since the restriction was
not related to the job performance of the flight attendants. 37

Petitioner's policy is not only in derogation of the provisions of Article 136 of the
Labor Code on the right of a woman to be free from any kind of stipulation
against marriage in connection with her employment, but it likewise assaults
good morals and public policy, tending as it does to deprive a woman of the
freedom to choose her status, a privilege that by all accounts inheres in the
individual as an intangible and inalienable right. 38 Hence, while it is true that the
parties to a contract may establish any agreements, terms, and conditions that
they may deem convenient, the same should not be contrary to law, morals,
good customs, public order, or public policy. 39 Carried to its logical
consequences, it may even be said that petitioner's policy against legitimate
marital bonds would encourage illicit or common-law relations and subvert the
sacrament of marriage.

Parenthetically, the Civil Code provisions on the contract of labor state that the
relations between the parties, that is, of capital and labor, are not merely
contractual, impressed as they are with so much public interest that the same
should yield to the common good. 40 It goes on to intone that neither capital nor
labor should visit acts of oppression against the other, nor impair the interest or
convenience of the public. 41 In the final reckoning, the danger of just such a
policy against marriage followed by petitioner PT & T is that it strikes at the very
essence, ideals and purpose of marriage as an inviolable social institution and,
ultimately, of the family as the foundation of the nation. 42 That it must be
effectively interdicted here in all its indirect, disguised or dissembled forms as
discriminatory conduct derogatory of the laws of the land is not only in order but
imperatively required.

DUNCAN ASSOCIATION OF DETAILMAN-PTGWO vs. GLAXOWellcomePhils., Inc.


G.R. No. 162994 September 17, 2004

Principle: Glaxo’s policy prohibiting an employee from having a relationship with


an employee of a competitior company is a valid exercise of management
prerogative. While our laws endeavor to give life to the constitutional policy on
social justice and the protection of labor, it does not mean that every labor
dispute will be decided in favor of the workers; The law also recognizes that
management has rights which are also entitled to respect and enforcement in
the interest of fair play.

FACTS: Tecson was hired by Glaxo as a medical representative on Oct. 24, 1995.
Contract of employment signed by Tecson stipulates, among others, that he
agrees to study and abide by the existing company rules; to disclose to
management any existing future relationship by consanguinity or affinity with co-
employees or employees with competing drug companies and should
management find that such relationship poses a prossible conflict of interest, to
resign from the company. Company's Code of Employee Conduct provides the
same with stipulation that management may transfer the employee to another
department in a non-counterchecking position or preparation for employment
outside of the company after 6 months.

Tecson was initially assigned to market Glaxo's products in the Camarines


Sur-CamarinesNorte area and entered into a romantic relationship with Betsy,
an employee of Astra, Glaxo's competition. Before getting married, Tecson's
District Manager reminded him several times of the conflict of interest but
marriage took place in Sept. 1998. In Jan. 1999, Tecson's superiors informed him
of conflict of intrest. Tecson asked for time to comply with the condition (that
either he or Betsy resign from their respective positions).

Unable to comply with condition, Glaxo transferred Tecson to the Butuan-


Surigao City-Agusan del Sur sales area. After his request against transfer was
denied, Tecson brought the matter to Glaxo's Grievance Committee and while
pending, he continued to act as medical representative in the Camarines Sur-
CamarinesNorte sales area. On Nov. 15, 2000, the National Conciliation and
Mediation Board ruled that Glaxo's policy was valid...

ISSUE:Whether or not the Reasonable Business Necessity Rule was proved in this
case.

RULING: Yes, the Supreme Court ruled that the actuations of the company were
justified and the dismissal of Tecson was legally done.

Glaxo has a right to guard its trade secrets, manufacturing formulas,


marketing strategies, and other confidential programs and information from
competitors. The prohibition against pesonal or marital relationships with
employees of competitor companies upon Glaxo's employees is reasonable
under the circumstances because relationships of that nature might
compromise the interests of the company. That Glaxo possesses the right to
protect its economic interest cannot be denied.

It is the settled principle that the commands of the equal protection


clause are addressed only to the state or those acting under color of its
authority. Corollarily, it has been held in a long array of US Supreme Court
decisions that the equal protection clause erects to shield against merely
privately conduct, however, discriminatory or wrongful.

The company actually enforced the policy after repeated requests to the
employee to comply with the policy. Indeed the application of the policy was
made in an impartial and even-handed manner, with due regard for the lot of
the employee.
Constructive dismissal is defined as a quitting, an involuntary resignation
resorted to when continued employment becomes impossible, unreasonable or
unlikely; when there is demotion in rank, or diminution in pay; or when a clear
discrimination, insensibility, or disdain by an employer becomes unbearable to
the employee. None of these conditions are present in the instant case.

Dispositive Portion:
WHEREFORE, the Petition is DENIED for lack of merit. Costs against petitioners.

Star Paper Corporation, Josephine Ongsitco& Sebastian Chua vs Ronaldo D.


Simbol, Wilfreda N. Comia& Lorna E. Estrella
G.R. No. 164774 (April 12, 2006)

FACTS:
Respondent Simbol was employed by Star Paper Corporation. There, he
met Ama Dayrit, a co-employee, whom he married. Prior to the marriage,
Ongsitco, the Personnel Manager, advised the couple that should they decide
to get married, one of them should resign pursuant to a company policy. Simbol
resigned pursuant to the company policy. Comia and Estrella were on the same
situation of Simbol.
The respondents each signed an agreement stating that they had no
money and property accountabilities in the company and that they released
the latter of any claim or demand of whatever nature.
However, the respondents offered a different version of their dismissal.
Simbol and Comia alleged that they did not resign voluntarilybut they were
compelled to resign. Estrella, on the other hand, alleged that she was denied
entry of the company after 21-day recuperation from an accident and that she
was being dismissed for immoral conduct for having a relationship and
impregnated by his co-worker. Due to the urgent need of money, she later
submitted a letter of resignation in exchange for her 13 th month pay.
They filed a complaint but the Labor Arbiter dismissed it for lack of merit
perceiving the actuations of the corporation as management prerogative. On
appeal to the NLRC, the Commission affirmed the former’s decision. They
appealed to the CA after the denial of the Motion for Reconsideration. In its
decision, the CA reversed and set aside the NLRC decision declaring the
dismissal as illegal. Hence this petition.

ISSUE:
Whether or not such company policy is a valid exercise of management
prerogartive

HELD:
No.
The case at bar involves Article 136 of the Labor Code which provides:
Art. 136. It shall be unlawful for an employer to require as a
condition of employment or continuation of employment that a woman
employee shall not get married, or to stipulate expressly or tacitly that
upon getting married a woman employee shall be deemed resigned or
separated, or to actually dismiss, discharge, discriminate or otherwise
prejudice a woman employee merely by reason of her marriage.

The courts that have broadlyconstrued the term "marital status" rule that it
encompassed the identity, occupation and employment of one's spouse. They
strike down the no-spouse employment policies based on the broad legislative
intent of thestate statute. They reason that the no-spouse employment policy
violate the marital status provision because it arbitrarily discriminates against all
spouses of present employees without regard to the actual effect on the
individual's qualifications or work performance.These courts also find the no-
spouse employment policy invalid for failure of the employer to present any
evidence of business necessity other than the general perception that spouses
in the same workplace might adversely affect the business.They hold that the
absence of such a bona fide occupational qualificationinvalidates a rule
denying employment to one spouse due to the current employment of the
other spouse in the same office.Thus, they rule that unless the employer can
prove that the reasonable demands of the business require a distinction based
on marital status and there is no better available or acceptable policy which
would better accomplish the business purpose, an employer may not
discriminate against an employee based on the identity of the employee's
spouse. This is known as the bona fide occupational qualification exception.
Since the finding of a bona fide occupational qualification justifies an
employer's no-spouse rule, the exception is interpreted strictly and narrowly by
these state courts. There must be a compelling business necessity for which no
alternative exists other than the discriminatory practice. To justify a bona fide
occupational qualification, the employer must prove two factors: (1) that the
employment qualification is reasonably related to the essential operation of the
job involved; and, (2) that there is a factual basis for believing that all or
substantially all persons meeting the qualification would be unable to properly
perform the duties of the job.
We do not find a reasonable business necessity in the case at bar.
Petitioners' sole contention that "the company did not just want to have
two (2) or more of its employees related between the third degree by affinity
and/or consanguinity"is lame. That the second paragraph was meant to give
teeth to the first paragraph of the questioned rule is evidently not the valid
reasonable business necessity required by the law.
Petitioners failed to show how the marriage of Simbol, then a Sheeting
Machine Operator, to Alma Dayrit, then an employee of the Repacking Section,
could be detrimental to its business operations. Neither did petitioners explain
how this detriment will happen in the case of WilfredaComia, then a Production
Helper in the Selecting Department, who married Howard Comia, then a helper
in the cutter machine. The policy is premised on the mere fear that employees
married to each other will be less efficient. If we uphold the questioned rule
without valid justification, the employer can create policies based on an
unproven presumption of a perceived danger at the expense of an employee's
right to security of tenure.
Petitioners contend that their policy will apply only when one employee
marries a co-employee, but they are free to marry persons other than co-
employees. The questioned policy may not facially violate Article 136 of the
Labor Code but it creates a disproportionate effect and under the disparate
impact theory, the only way it could pass judicial scrutiny is a showing that it is
reasonable despite the discriminatory, albeit disproportionate, effect. The failure
of petitioners to prove a legitimate business concern in imposing the questioned
policy cannot prejudice the employee's right to be free from arbitrary
discrimination based upon stereotypes of married persons working together in
one company.
Lastly, the absence of a statute expressly prohibiting marital discrimination
in our jurisdiction cannot benefit the petitioners. The protection given to labor in
our jurisdiction is vast and extensive that we cannot prudently draw inferences
from the legislature's silencethat married persons are not protected under our
Constitution and declare valid a policy based on a prejudice or stereotype.
Thus, for failure of petitioners to present undisputed proof of a reasonable
business necessity, we rule that the questioned policy is an invalid exercise of
management prerogative.

G.R. No. 163512 February 28, 2007

DAISY B. TIU, Petitioner


vs.
PLATINUM PLANS PHIL., INC., Respondent.

FACTS:

 Respondent Platinum Plans Philippines, Inc. is a domestic corporation


engaged in the pre-need industry with petitioner Daisy B. Tiu as its Division
Marketing Director. On January 1, 1993, respondent re-hired petitioner as
Senior Assistant Vice-President and Territorial Operations Head in charge
of its Hongkong and Asean operations which was to last for five years. On
September 16, 1995, petitioner stopped reporting for work. Thereafter, on
November 1995, she became the Vice-President for Sales of Professional
Pension Plans, Inc., a corporation engaged also in the pre-need industry.

 Consequently, respondent sued petitioner for damages before the RTC of


Pasig City. It alleged, among others, that petitioner’s employment with
Professional Pension Plans, Inc. violated the non-involvement clause in her
contract of employment. Under such provision, the employee in case of
separation from the company, whether voluntary or for cause, shall not for
the next TWO (2) years thereafter, engage in or be involved with any
corporation, association or entity, whether directly or indirectly, engaged
in the same business or belonging to the same pre-need industry as the
employer. For any breach thereof, the employee is liable for the amount
of P100,000.00.

 Respondent counters that the validity of a non-involvement clause has


been sustained by the Supreme Court in a long line of cases. It contends
that the inclusion of the two-year non-involvement clause in petitioner’s
contract of employment was reasonable and needed since her job gave
her access to the company’s confidential marketing strategies.
Respondent adds that the non-involvement clause merely enjoined her
from engaging in pre-need business akin to respondent’s within two years
from petitioner’s separation from respondent. She had not been
prohibited from marketing other service plans.

 Petitioner countered that the non-involvement clause was unenforceable


for being against public order or public policy: First, the restraint imposed
was much greater than what was necessary to afford respondent a fair
and reasonable protection. Petitioner contended that the transfer to a
rival company was an accepted practice in the pre-need industry. Since
the products sold by the companies were more or less the same, there
was nothing peculiar or unique to protect. Second, respondent did not
invest in petitioner’s training or improvement. At the time petitioner was
recruited, she already possessed the knowledge and expertise required in
the pre-need industry and respondent benefited tremendously from it.
Third, a strict application of the non-involvement clause would amount to
a deprivation of petitioner’s right to engage in the only work she knew.

 The trial court upheld the validity of the non-involvement clause. It ruled
that a contract in restraint of trade is valid provided that there is a
limitation upon either time or place. In the case of the pre-need industry,
the trial court found the two-year restriction to be valid and reasonable.
Hence, judgment was rendered in favor of plaintiff in the amount of
P100,000.00 and as to the costs of suit.

 On appeal, the Court of Appeals affirmed the trial court’s ruling. It


reasoned that petitioner entered into the contract on her own will and
volition. Thus, she bound herself to fulfill not only what was expressly
stipulated in the contract, but also all its consequences that were not
against good faith, usage, and law. The appellate court also ruled that
the stipulation prohibiting non-employment for two years was valid and
enforceable considering the nature of respondent’s business.

 Petitioner moved for reconsideration but was denied. Hence, petitioner


filed the present appeal by certiorari.

ISSUE: Whether or not the non-involvement clause is valid?

RULING:

YES. The Court held that a non-involvement clause is not necessarily void for
being in restraint of trade as long as there are reasonable limitations as to time,
trade, and place.

In this case, the non-involvement clause is valid since it had a time limit: two
years from the time petitioner’s employment with respondent ends. It is also
limited as to trade, since it only prohibits petitioner from engaging in any pre-
need business akin to respondent’s.

More significantly, since petitioner was the Senior Assistant Vice-President and
Territorial Operations Head in charge of respondent’s Hongkong and Asean
operations, she had been privy to confidential and highly sensitive marketing
strategies of respondent’s business. To allow her to engage in a rival business
soon after she leaves would make respondent’s trade secrets vulnerable
especially in a highly competitive marketing environment. In sum, we find the
non-involvement clause not contrary to public welfare and not greater than is
necessary to afford a fair and reasonable protection to respondent.

Article 1306 of the Civil Code provides that parties to a contract may establish
such stipulations, clauses, terms and conditions as they may deem convenient,
provided they are not contrary to law, morals, good customs, public order, or
public policy. Article 1159 of the same Code also provides that obligations
arising from contracts have the force of law between the contracting parties
and should be complied with in good faith. Courts cannot stipulate for the
parties nor amend their agreement where the same does not contravene law,
morals, good customs, public order or public policy, for to do so would be to
alter the real intent of the parties, and would run contrary to the function of the
courts to give force and effect thereto.

Not being contrary to public policy, the non-involvement clause, which


petitioner and respondent freely agreed upon, has the force of law between
them, and thus, should be complied with in good faith. Thus, as held by the trial
court and the Court of Appeals, petitioner is bound to pay respondent ₱100,000
as liquidated damages.

Hence, the petition is denied for lack of merit.

G.R. No. 163269 April 19, 2006

ROLANDO C. RIVERA, Petitioner,


vs.
SOLIDBANK CORPORATION, Respondent.

Facts:

Petitioner Rolando Rivera had been working for Solidbank Corporation since July
1977. He was initially employed as an Audit Clerk, then as Credit Investigator,
Senior Clerk, Assistant Accountant, and Assistant Manager. Prior to his retirement,
he became the Manager of the Credit Investigation and Appraisal Division of
the Consumer’s Banking Group. In the meantime, Rivera and his brother-in-law
put up a poultry business in Cavite.

In December 1994, Solidbank offered two retirement programs to its employees:


(a) the Ordinary Retirement Program (ORP), under which an employee would
receive 85% of his monthly basic salary multiplied by the number of years in
service; and (b) the Special Retirement Program (SRP), under which a retiring
employee would receive 250% of the gross monthly salary multiplied by the
number of years in service. Since Rivera was only 45 years old, he was not
qualified for retirement under the ORP. Under the SRP, he was entitled to receive
P1,045,258.95 by way of benefits.
Deciding to devote his time and attention to his poultry business in Cavite,
Rivera applied for retirement under the SRP. Subsequently, Solidbank required
Rivera to sign an undated Release, Waiver and Quitclaim, which was notarized
on March 1, 1995. Rivera acknowledged receipt of the net proceeds of his
separation and retirement benefits and promised that “[he] would not, at any
time, in any manner whatsoever, directly or indirectly engage in any unlawful
activity prejudicial to the interest of Solidbank, its parent, affiliate or subsidiary
companies, their stockholders, officers, directors, agents or employees, and their
successors-in-interest and will not disclose any information concerning the
business of Solidbank, its manner or operation, its plans, processes, or data of
any kind.”

On May 1995, the Equitable Banking Corporation employed Rivera as Manager


of its Credit Investigation and Appraisal Division of its Consumers’ Banking Group.
Upon discovering this, Solidbank First Vice-President for Human Resources
Division (HRD) wrote a letter informing Rivera that he had violated the
Undertaking and demanded the return of all the monetary benefits he received
in consideration of the SRP within five days from receipt; otherwise, appropriate
legal action would be taken against him.

When Rivera refused to return the amount demanded within the given period,
Solidbank filed a complaint. Petitioner avers that the prohibition incorporated in
the Release, Waiver and Quitclaim barring him as retiree from engaging directly
or indirectly in any unlawful activity and disclosing any information concerning
the business of respondent bank, as well as the employment ban contained in
the Undertaking he executed, is oppressive, unreasonable, cruel and inhuman
because of its overbreath.

Issue:

Whether or not the post-retirement competitive employment ban is


unreasonable and oppressive.

Ruling:

The petition is granted. This case is REMANDED to the Regional Trial Court of
Manila for further proceedings. Unreasonableness still needs to be determined.

In this case, there is no dispute between the parties that, in consideration for his
availment of the Special Retirement Program, petitioner executed the Release,
Waiver and Quitclaim, and the Undertaking as supplement thereto, and that he
received retirement pay amounting to P963,619.28 from respondent. On May 1,
1995, within the one-year ban and without prior knowledge of respondent,
petitioner was employed by Equitable as Manager of its Credit Investigation and
Appraisal Division, Consumers’ Banking Group. Despite demands, petitioner
failed to return the P963,619.28 to respondent on the latter’s allegation that he
had breached the one-year ban by accepting employment from Equitable,
which according to respondent was a competitor bank.

Supreme Court agree with petitioner’s contention that the issue as to whether
the post-retirement competitive employment ban incorporated in the
Undertaking is against public policy is a genuine issue of fact, requiring the
parties to present evidence to support their respective claims.

Article 1306 of the New Civil Code provides that the contracting parties may
establish such stipulations, clauses, terms and conditions as they may deem
convenient, provided they are not contrary to law, morals, good customs,
public order or public policy. The freedom of contract is both a constitutional
and statutory right.A contract is the law between the parties and courts have no
choice but to enforce such contract as long as it is not contrary to law, morals,
good customs and against public policy.

The well-entrenched doctrine is that the law does not relieve a party from the
effects of an unwise, foolish or disastrous contract, entered into with full
awareness of what he was doing and entered into and carried out in good
faith. Such a contract will not be discarded even if there was a mistake of law or
fact. Courts have no jurisdiction to look into the wisdom of the contract entered
into by and between the parties or to render a decision different therefrom.
They have no power to relieve parties from obligation voluntarily assailed, simply
because their contracts turned out to be disastrous deals.

In cases where an employee assails a contract containing a provision


prohibiting him or her from accepting competitive employment as against
public policy, the employer has to adduce evidence to prove that the restriction
is reasonable and not greater than necessary to protect the employer’s
legitimate business interests.54 The restraint may not be unduly harsh or
oppressive in curtailing the employee’s legitimate efforts to earn a livelihood
and must be reasonable in light of sound public policy.55

Courts should carefully scrutinize all contracts limiting a man’s natural right to
follow any trade or profession anywhere he pleases and in any lawful manner.
But it is just as important to protect the enjoyment of an establishment in trade or
profession, which its employer has built up by his own honest application to
every day duty and the faithful performance of the tasks which every day
imposes upon the ordinary man. What one creates by his own labor is his. Public
policy does not intend that another than the producer shall reap the fruits of
labor; rather, it gives to him who labors the right by every legitimate means to
protect the fruits of his labor and secure the enjoyment of them to
himself.56 Freedom to contract must not be unreasonably abridged. Neither
must the right to protect by reasonable restrictions that which a man by industry,
skill and good judgment has built up, be denied.

S-ar putea să vă placă și